Sunteți pe pagina 1din 522

A Level Preliminary Examination Paper

2006
CHEMISTRY
PART2.

Raffles Junior. College


Anderson Junior College
National Junior College
Serangoon Junior College
Yishun Junior College
Temasek Junior College
Meridian Junior College
Catholic Junior College
Millennia Institute-

l
l~

[
RAFFLES JUNIOR COLLEGE
2006 PRELIMINARY EXAMINATION

CHEMISTRY

9251/01

Paper 1 Multiple Choice


18 September 2006

[
f~

1 hour
t

Additional Materials: Multiple Choice Answer Sheet


Data Booklet

READ THESE INSTRUCTIONS FIRST

Write your name and civics tutorial group in the spaces provided on the answer sheet

[
[

c
['

Do NOT open this question booklet until you are told to do so.

Do not use correction fluid on-the answer-sheet


There are forty questions on this paper. Answer all questions.
For each question there are four possible answers A, B, C and D. Choose the one you
consider correct and record your choice in soft pencil on the separate answer sheet

provided.
Each correct answer will score one mark. A mark will not be deducted for a wrong answer.
Do not write anything on the Data Booklet.
Any rough working should be done in this booklet.
You may use a calculator.

r.

L
L
l
L
[

L
[

This document consists of 14 printed pages.

\'
[
[

Section A
For each question there are four possible answers, A, B, C and D. Choose the one you
consider to be correct.

Methane was burned in an incorrectly adjusted burner. The methane was converted
into a mixture of carbon dioxide and carbon monoxide in the ratio of 98:2, together
with water vapour.'

What will be the volume of oxygen consumed when y dm 3 of methane is burned?

(2y- 0.01 y) dm3

(2y- 0.01y) dm3

(y.;:... 0.01 y) dm3

(y- 0.01y) dm3

A sample of a gaseous element X Wa$ qnalysed U$ing a mass spectrometer and the
complete spectrum is given below.
signal
intensity

M3 M4Ms

M1 M2

mle

Which of the following statements about element X is correct?

Its fragment ion has a relative mass which is always numerically equal to M1.
Its molecular ion has a relative mass which is always numerically equal to Ms.

.
.
1 R3 xM 3 +R 4 xM 4 +R 5 xM 5
Its relative atomic mass is equal to - x(
R R R
)] -

. R 1 xM 1 +R 2 xM 2
Its relative molecular mass is numerically equal to 2 x(
R +R
)J.

3+

4 +

RJ:~:Rs

R1: R2

r~

L
r~

The use of the Data Booklet is relevant to this question.

An ion Y2... contains 29 protons. Which of the following statements is correct?


A
B
C

The enthalpy change for the reaction Y(g) ~ Y2...(g) + 2e- is 1960 kJ mol1 .
The first ionisation energy ofY2... is 745 kJ mol1_
The electronic configuration of Y is 1s2 2s2 2p6 3s2 3p6 3d 9 4s2 . .
The electronic configuration of Y 2... is 1s2 2s2 2p6 3s2 3p6 3d9

L
r
L

r..

L
{.

r
[

Figure 1 shows the first six ionisation energies of an element Z while figure 2
shows the second ionisation energies of eight consecutive elements (including Z).

Figure 2

Figure 1

L~

0
E
"Q)

0
E
._
(I)
a.

c..
~

...,
""'

>-

~
Q)

>.

c:

.2

E'
<1>

Q)

.."'
Q)

1ii
II)

c
0

<U

.2

c:

"c

.2

0
0

Q)

en

L
L

L
l'

L
I

L
L
(I

Atomic number

Number of ionisation

With reference to Figure 2, which of the options, A, B, C or D, corresponds to the


second ionisation energy of element Z?

The boiling points (b.p.) of ammonia, hydrogen chloride and carbon dioxide are given
below.

gas

formula

ammonia
hydrogen chloride
carbon dioxide
------

b.p. /C

NH3

-33

HC/

-85

C02
- - - - - - - - - - - L ...

-78
----

Which of the following statements best explains the above data?


A

. NH3 has a- higher b.p. than HC/ because the hydrogen bonding in NH3 is
stronger than the permanent dipole-permanent dipole interactions in HC/.

The strength of the instantaneous dipole-induced dipole interactions increases


in the order HC/ < C02 < NH3.

The strength of the permanent dipole-permanent dipole interactions increases


in the order HC/ < C02 < NH3.

C02 has a higher b.p. than HC/ because the permanent dipole-permanent
dipole interactions in C0 2 are stronger than the permanent dipole-permanent
dipole interactions in HC/.

rc
6

The diagram shows the structure of boron


nitride, which is similar to that of a particular
allotrope of carbon.

key
boron

Which of the following is not a property of this


compound?
A
8
C
D

nitrogen

It is soft.
It is transparent when pure.
It is a solid with a high melting point.
It is a lubricant.

f~
I.-

[
[
[

Which of the following statements concerning the reaction A


correct?
A
8
C
D

28 is always

The overall order of reaction is one.


Increasing the concentration of A increases the rate of the reaction.
Removing 8 as it is formed helps to increase the rate of reaction.
Adding a suitable catalyst increases the rate constant of the reaction.

[
[
[

The stoichiometry of a catalysed reaction is shown by the equation below.


P (g)+ Q (g) "'

": R (g)+ S (!)

Two experiments were carried out in which the rates of this reaction were measured.
The results are shown in the diagram below.

Experiment I

pressure of R
in the reaction
vessel

[
[

Experiment IT

[
time
Which of the following independent changes in the conditions from Experiment I to
Experiment II might explain the results shown?
A
8
C
D

Product S was continuously removed from the reaction vessel.


An inert gas was added to the reaction vessel at constant volume.
More of P was used.
Less of Q was used.

. 4

.;

L
L
L
L
\

r
[

The following data was obtained from the studies of the reaction between 0 2 and NO
in a vessel at constant temperature.

Initial total pressure of 0 2 and NO/ atm

1.00

1.60

2.00

Initial partial pressure of Oi atm

0.60

1.20

1.20

Initial rate of reaction/ atm s-1

1.08

2.16

8.64

Experiment

Which of the following statements is correct regarding the above system?

I
I
I

B
C

10

c
[
.,

The reaction is first order with respect to NO.


The overall order of the reaction is three.
The rate equation is rate = k [NOf
The rate constant k has units of atm-1 s-1

Which of the following statements helps to explain why magnesium and oxygen form
MgO instead of Mg 2 0?
A

Less energy is required to remove one electron from the magnesium atom than
to remove two electrons.

When Mg 2 0 is formed from its elements, more energy is released than when
MgO is formed.

The lattice energy of MgO is more exothermic than that of Mg 2 0.

More energy is released in forming the oxide ions from oxygen molecules in the
form~tion of MgO 'than in the formation of Mg 2 0.

.~

11

/s,

[
[
r,

The solubility product of a substance is S. If the concentration of the cation in a


95
saturated solution is found to be (
then the substance could be
4

A
B
12

Ca3(P04)2

Ah(S04)3

The same quantity of electricity that caused the deposition of 5.DO g of silver from
silver nitrate solution liberated 3.02 g of an element X from a solution containing X"+.
If the relative atomic mass of X is 197, what is the value of n?

L
L
r

CaC/2
Al(OH)J

..

13

Given these standard electrode potentials


Ag+(aq) + e- ~ Ag(s)

+0.80V

Mn04-(aq) + 8H+(aq) + 5e- ~ Mn2+(aq) + 4H20(l)

+1.52

what would be the standard e.m.f. of the cell involving these two half-reactions and
the polarity of the silver electrode?
9

Polarity of the silver electrode

E cenN
A
8

c
0

14

c
0

Which of the following properties shows a decreasing trend from left to rightof the
elements or ions specified?

r~

+
+

2.48
0.72
0.72
2.48

r
[

bond length of Clz, Br2, h


2
ionic radius of
-, p3atomic radius of C, Si, Ge
metallic radius of Na, Mg, Al

[
i~-

cz-. s

r
c

15

The figure below shows the stable oxidation states of 5 consecutive elements, P to
T, in the Periodic Table plotted against their atomic numbers.

-~

oxidation number
f"'

+4
+3

+2
+11

+5

I
Q

I
R

, atomic number

I
T

Which of the following are likely to be the atomic numbers of the consecutive
elements P to T?

1 to 5

3to 7

11 to 15

19 to 23

L
L
L
L
{

l
[

16

r,

17

Which of the following statements regarding Group II elements or their compounds is


correct?
A

Beryllium chloride has a higher melting point than magnesium chloride.

Magnesium is a stronger reducing agent than strontium.

Calcium hydroxide is less soluble in water than barium hydroxide_

Strontium,reacts more readily with oxygen than radium.

The graphs given below show the trends in some properties related to the halogens
X2 (fluorine, chlorine, bromine and iodine) and their compounds down the group.

r,

Which of the following properties is not among the four illustrated in the graphs
above?

pH of 0.1 mol dm-3 aqueous HX


X-X bond dissociation energy
H-X bond dissociation energy
boiling point of HX

A
B
C
D

[
--,

18

The -colours of various vanadium ions in aqueous solution are given in the table
below.

vo2+

Colour

yellow

blue

19

Ion

IV
vo 2+

yellow

blue

v3+

II
y2+

green

violet

green

violet

Cu 2\aq)

The use of the Data Booklet is relevant to this question.


The salt, Na3Fe(CN)5, ca_n be prepared by oxidising N<4Fe(CN)6.
Which reagent will carry out this oxidation?

cr(aq)

Mn 2+(aq)

[
L~

III

What is the final colour of the solution when zinc powder is added to an acidified
- -- --- solution containing excess V02+ ions?
-

L
L

Oxidation state

['

The use of the Data Booklet is relevant to this question.

Fe3+(aq)

['

[
20

The use of the Data Booklet is relevant to this question.

The reaction between I- and S2ol- is catalysed by Fe3+ as shown below:


Step 1: 2Fe 3+(aq) + 2qaq) -)-2Fe2+(aq) + h(aq)
Step 2: 2Fe2+(aq) + S2ol-(aq) -)-2Fe3\aq) + 2Sol-(aq)

Which of the following statements is not correct?


A

B
C
D

21

22

The reaction.can be catalysed by Fe2+ as welL


The E9 cell for Step 1 is less than that for Step 2.
The E9 cell of the uncatalysed reaction is less than zero volt.
The overall catalysed and uncatalysed reactions have the same L1H value.

0.10 mol of an organic compound L gives 0.30 mol of carbon dioxide and 0.40 mol of
water on complete combustion. 3.00 g of L when vapourised, was found to occupy
1.12 dm 3 at standard temperature and pressure. Which of the following could be the .
formula of L?
A
B

CH3CH2CH3
CH3CH2CH20H

c
[
[

CH3COOH
CH3CH2CHO

r,

How many chiral carbons are present in the following molecule?

i,

CHC/CH3

[
OH

-23

10

-One mole of.an organic compound V is treated with an excess of thionyl chloride
(SOC/ 2) and two moles of hydrogen chloride are formed.

CH3CH=CHCH3

HO-o-OH

[
[

Which of the following compounds could V be?


A

HOCHzCOzH

Ho-@-cooH

L
[
8

L,
~-

l_

L
L

24

Which of the followjng statements is correct with regards to the conversion of


compound P to compound Q?

Br

c
[
25

CH3
o2N-v-?-ooccH3
CH 3
LJI~

0 2 N:o-yH3
C-Br

Br

CH 3

The conversion involves, in sequential order, an elimination reaction, an


electrophilic substitution reaction and esterification.

The conversion involves, in sequential order, a nucleophilic substitution


reaction, acidic hydrolysis and esterification.

The first step must consist of the conversion of compound P into an alkene.

The conversion can be carried out in less than three steps.

The following compound is a flavouring agent in food.

HO--Q-O-~CH3

CHO

Which of the following reagents does not react with the above compound?

A
B

C
D

26

Hot acidified potassium dichromate(VI) solution


Fehling's solution
Neutral iron(III) chloride solution
2,4-Dinitrophenylhydrazine

In which of the following sequence is the value of pKa decreasing?

A
B

(CH3)3COH > CH3CHzCHzCHzOH > CH3CHC/CHC/CH 20H > CsH50H

CH3CHzCHzCHzOH > (CH3)3COH > CH3CHC/CHCZCH 20H > CsH50H

CH3CHC/CHC/CHzOH >: (CH 3)3COH > CH3CH 2 CHzCH 20H > CsH50H

[
[

L
L
L
f

27

Csl::I50H >:..CH3CHC/CHClCHzOH > CH3CHzCHzCHzOH > '(CH3)3COH

In which sequence are the compounds listed in order of increasing ease of


hydrolysis?

A
B

CH3COC/ < CsH5CHzC/ < CsH5C/

CH3CHzBr < CH3CH 2C/ < CH 3CH2F


CsH 5Br < CH 3CH2Br < CH 3COBr

CH 3Br < CH3CHzBr < CsH5Br

c
28

Which of the following reactions would not produce benzene-1 ,2-dicarboxylic acid?

CCOCH3

alkaline
aqueous iodine
warm

COCH3

aqueous
diamminesilver(I) ions

C:::
CCH,CH3

dilute
hydrochloric acid

warm
acidified aqueous
potassium manganate(VII)
reflux

CH2 0H

dilute
hydrochloric acid

CCN

concentraled

OOCCH3

sulphuric acid

reflux

c
[

[
[
[

[
r-

29

Which of the following correctly represents a repeat unit of a protein?


(Note: R and R' are organic groups)

R
A

-C~C~C~

R'

NH-CH-NH-

R'

-N~CH- NH-CO-CH-CO-

R
D

-C~C~NH-C~CH-~-

R'

-CG-CH-NH-CQ-0-CH-0-NH-

[
['
L.

10

l
L
L
r

l_

L
[

30

L
[

Which of the following statements about Terylene is not correct?


A

Complete hydrolysis of Terylene with boiling aqueous sodium hydroxide results


in the formation of benzene-1 ,4-carboxylic acid and ethane-1 ,2-diol.

Terylene can be produced by the polymerisation of the monomer,

Hboc--Q--coocHzCH20H.

Tery/ene can be produced by the co-polymerisation of ethane-1,2-diol and


benzene-1 ,4-dicarboxylic acid.

The polymerisation process involved in the production of Terylene proceeds via


condensation.

L
[
[

Section 8
For each of the questions in this section one or more of the three numbered statements 1
to 3 may be correct.
Decide whether each of the statements is or is not correct (you may find it helpful to put a
tick against the statements which you consider to be correct).

The responses A to D should be selected on the basis of


j

-,
l

1, 2 and 3
are
correct

1 and 2
only are
correct

2 and 3
only are
correct

1 only
is
correct

.'

c
--

No other combination of statements is used as a correct response.

31

_,

t_~

L"

32

Which of the following statements about real gases are correct?


1
2

The molar mass of any gas can be estimated by M = mRT!pV.


The compressibility factor, pVIRT, of one mole of any gas approaches 1.0
when p approaches 0 atm.

The volume of one mole of any gas at room temperature and pressure is
24.0 dm3 .

Which of the following statements about the following reaction are correct?

l'
L
l~

L
f

2103-(aq) + 5HSo3-(aq) ~ 3HSo4-(aq) + 2Soi-(aq) + Iz(aq) + H20(!)


1

The hydrogensulphite ion is a reducing agent.

The iodine liberated can be determined using standard sodium thiosulphate


solution.
20 cm3 of 0.1 mol dm-3 iodate(V) solution produces 0.001 mole of iodine.

11

l'
l_
A

1, 2 and 3
are
correct

1 and 2
only are
correct

2 and 3
only are
correct

1 only
is
correct

1I

f~

[
[

33

The standard redox potentials for some manganese species are shown below.

E9 cenN
2

Mn +(aq) + 2e- ,.: Mn(s)

-1.18
3

Mn02(s) + 4H+(aq) + e- ,.: Mn +(aq) + 2H 20(/)


2

Mn +(aq) + e- ,.: Mn +(aq)

+0.95
+1.49

Which of the following reactions under standard conditions could be deduced from
the given data?
1
2

3
34

Data: Ksp of BaS04 = 1.3 x 10-10 mol2 dm-6


Ksp of BaC0 3 5.5 x 10-10 mol 2 dm-6
Which of the following statements are correct regarding the 'barium meal'?

35

r-

Barium sulphate is used because it is insoluble in water and hence not


poisonous when ingested by mouth.
- Barium carbonate can also be used since it is more insoluble in water than
barium sulphate.
Barium hydroxide can also be used since it is thermally stable in the human
body.

Which of the following statements explain why silver chloride is soluble in aqueous
ammonia but silver iodide is not?
1

The equilibrium constant for the reaction, AgX(s) + 2NH3(aq) ,.: Ag(NH3)zX(aq),
is numerically greater for X Cl than for X I.

The solubility product of silver chloride is numerically larger than that of silver
iodide.

The lattice energy of silver chloride is numerically larger than that of silver
iodide.

12

[
[

Mn 3+ could disproportionate.
Mn could reduce Mn3+ to Mn2 +.
Mn02 could oxidise Mn 2+ to Mn 3+.

Barium atoms are opaque to X-rays as a consequence of the large number of


electrons they contain. In hospitals, before an X-ray examination is carried out,
patients with digestive tract problems are sometimes given a 'barium meal',
consisting of a suspension of barium sulphate in water.

l
-

l
L
L
I

['
( ~

1, 2 and 3
are
correct

1 and 2
only are
correct

2and 3
only are
correct

1 only
is
correct

[
[

------

36

Which of the following statements are incorrect regarding the above reaction?

[
[
[

A disproportionation reaction occurred and the final products obtained included


barium chlorate(!) and barium chlorate(V).

3y moles of barium hydroxide reacted and 5y/2 moles of barium chloride


produced.

y moles of barium hydroxide would react to produce y/2 moles of barium


chloride if the experiment were repeated using excess cold aqueous barium
hydroxide.

37

In which of the following pairs of compounds is the compound on the left more
volatile than that on the right?
1
2
3

[
[

In an experiment, y moles of chlorine gas were bubbled into excess hot aqueous
barium hydroxide.

38

propylamine and propan-1-ol


pentan-2-one and pentan-2-ol
cyclohexylamine and aminoethanoic acid

Which of the following statements are correct with regards to the compound X
shown below?
OH
CH3

L
[

[
['

L
l
L
r.

CH3

1 mole of X reacts with excess gaseous HBr to yield a major product with 7
chiral centres.

1 mole of X reacts with hot acidified KMn04 to give a tribasic carboxylic acid as
one of the products.

1 mole of X reacts with 4 moles of


a saturated. compound.

H2 in the presence of nickel catalyst to yield

13

[
r_
--

'

I
I

1, 2 and 3
are
correct

39

2 and 3
only are
correct

1 and 2
only are
correct

1 only
is
- correct

I
i

Pentaerythritol is an intermediate in the manufacture of paint. Its structure is shown


below.
HOH2 C~ /CH2 0H

HOH2 C /

c
[
r-

L_~

~CH2 0H

,-,

Which of the following statements about pentaerythritol are correct?

1 mole of pentaerythritol reacts with excess phosphorus(V) chloride to liberate


4 moles of hydrogen chloride.
It reacts with excess concentrated sulphuric acid at 170 C to give an alkene.

It reacts with warm alkaline aqueous iodine to give a yellow precipitate.

2
3
40

Which of the following terms may be applied to the formation of a polymer in the
reaction below?
n H02CCH2CH 2C02H + n HOCH2CH20H---+ --f-COCH2CH2C02CH2CH20}n- + (2n -1) H20

1
2
3

Esterification
Condensation
Neutralisation

L,
[
[
"

End of Paper

--

-l

_,

r~

L
L
l
14

L
I-

L
[
[

Raffles Junior College


2006 JC 2 Preliminary Examination

9251/02

CHEMISTRY

Paper 2
Monday

18 September 2006

0945 - 1115 hrs

Time: 1 hours 30 minutes

Additional Materials: Data booklet

[
Name: .................................................. (

f'

CT Group: 25 ................. .

READ THESE INSTRUCTIONS FIRST

Write your name and civics tutorial group in the spaces provided above.
Answer all questions.

Write your answers in the spaces provided on the question paper.

-,
l

The number of marks is given in brackets [ ] at the end of each question or part of a
question.

Write in dark blue or black pen on both sides of the paper.

--'

A Data Booklet is provided. Do not write anything on th.e Data Booklet

You may use a pencil for any diagrams, graphs, or rough working,

[
[

Do not use staples, paper clips, highlighters, glue or correction .fluid.


You may use a calculator.
__

L
L
L
I

You are reminded ofthe need for good English and clear presentation in your answers.

FOR EXAMINER'S USE


Q4
Q2
Q3

Q1

17

/10

/11

. /4

Q5
/6

Q7

Q6

/6

/4

Q8
/8

Q9

TOTAL

/4

This document consists of 15 printed pages, including one cover page.

/60

L
L

c
[
[

Question 1
Aluminium is the most abundant metal in the surface of the earth. It is
extracted from its oxide, bauxite, by electrolysis, and often forms substances
that exhibit both ionic and covalent characteristics. One common and useful
aluminium compound is aluminium chloride.
(a)

Briefly explain why aluminium chloride is a covalent compound


whereas aluminium fluoride is predominantly ionic.
[2]

[
[
[
[
[

[
[
[

In the Periodic Table, diagonal relationships can exist between elements in


different Groups if they resemble each other in their chemistry. For example,
beryllium resembles aluminium in that it can also form covalent compounds
which are said to be electron deficient

(b)

In the gaseous state, both beryllium chloride, BeCh and aluminium


chloride, AZC/3 can exist as simple, discrete molecules. Draw dot-andcross diagrams for these two gases and state their shapes.
[2]

BeC/z

AIC/3

Shape:

Shape:

[
[

L
l
L~

L
r-

t.
r~

L
(c)

At certain temperatures, both gaseous aluminium chloride and


beryllium chloride can dimerise. Suggest a value for the chlorine[1]
aluminum-chorine bond angle in NzC/5.

[
[

[
[
(d)

Beryllium chloride is a polymer in the solid state, with each chlorine


atom bonded to two beryllium atoms. Draw a portion of this polymer
[2]
and state the types of bonding between Be and C/ atoms.

[
[

r"
[,

[
[

L
f'

c
[

L
L
L
r

L
[
[
[

Question 2
Pyridine, C5H5N, is widely used as a solvent and an intermediate in the
synthesis of insecticides, herbicides and pharmaceuticals. Pyridine is a simple
heterocyclic aromatic organic compound that is structurally related to benzene
and is a liquid at room temperature with a boiling point of 115 oc_

[
[

(a)

Pyridine

(i)

Experimentally the enthalpy change of atomisation of pyridine


has been found to be +5130 kJ mor1 Write an equation to
represent this reaction.
(1]

(ii)

Using relevant bond energy data from the Data Booklet,


calculate another value for the enthalpy change of atomisation
of pyridine.
[2]

(iii)

Suggest a reason for the difference between the value given in


- (i) and that calculated in (ii).
[1]

[
[
[

L
[
[
[

[
['

l
l_

L
r-

l"

[-

l
(b)

(i)

Define standard enthalpy change of combustion of pyridine.

[1]

r[

(ii)

When 1.13 g of pyridine was combusted in a calorimeter, the


procedure heated 100 g of water from 25 oc to 87 oc. This
process was known to be only 65% efficient. Calculate the
[3]
enthalpy change of combustion of pyridine.
(Specific heat capacity of water: 420 J g-1 K-1)

[
[
[

[
r

L_~

[
(iii)

Use the value calculated in (ii) and the data below to calculate
the enthalpy change of hydrogenation of pyridine.
[2]

I
O<n+3H2(9l

O<n

Data:
Enthalpy change. of combustion ofC5H 11N =
Enthalpy change of combustion of H2 =

- 3356-kJ mor1
- 285.8 kJ mor1

L
[
[
[
[

L
L~
r-

l
[
[

L
[
[
[

Question 3
(a)
Morphine is a weak monoacidic alkali and is used in small quantities as
a pain relief. When 30 cm 3 of 0.30 mol dm-3 morphine solution was
titrated against a 0.20 mol dm-3 hydrochloric acid, the end point was
determined using a suitable indicator.
You' may use Mor and MorH+ to denote Morphine and its conjugate
acid respectively.

(Ka of MorH+ = 2.09 x 10-12 mol dm-3)


(i)

Determine the concentration


equivalence point

of salt that was formed

(ii)

Hence determine the pH at the equivalence point.

at
[2]

[
[
[

[2]

[
[

['
[

(iii) Sketch and label the graph of pH against volume of HCI used,

indicating the point of maximum buffer capacity.

[
[

L
[

L
r'

[2]

[.

[
(b)

The solubility product of Mg(OH)z is 1.00 x 10-11 mol 3 dm-9 . Calculate


the solubility of Mg(OH)2 in 5.00 x 1o-2 mol dm-3 aqueous NaOH. [3]

L
[
[
[
[
[
[

(c)

The normal pH of blood is 7.40. It is found that this pH is controlled by


several buffer systems in the body fluid. Among them is the
HzP04-/HPo/- buffer system. Briefly explain how this buffer system
[2]
works when pH of blood is momentarily decreased.

L
[

L
['

[
[
[

[
7

l
l"
L_
r.

~
[

"

l-

Question 4
Study the reaction scheme given below carefully. A is a sodium salt and it
contains an anion which is tetrahedral in shape.

B
~range-brown

AgN0 3 (aq)

cream
precipitate

""

AgN03 (aq)

colourless
solution

yellow

precipitate

yellow-brown solution

(a)

Suggest a suitable identity each for A, 8 and C.

lA

[
[

white
precipitate

Ba(N03 }z{aq)

colourless
solution

colourless solution

solution

(b)

~~mmm

F
m

[2]

Write a balanced ionic equation for the reaction between A and B.

(1]

[
[

(c)

Suggest a simple test which can be used to distinguish between Band


~]

C.

L
[
[

L
[

l
[

r-

[
[
~
Question 5
X, Y and Z are Group II metals (Mg to Ba). X(I03)2, Y(I03)z and Z(l03)z are
Group II iodates(V). Upon heating, each of these iodates(V) decomposes to
yield a metal oxide, a purpie gas and a colourless gas which rekindles glowing
splints.
(a)

Draw the dot-and-cross diagram of magnesium iodate(V).

l>

[1]

[
[
Write a balanced equation for the decomposition of Mg(I03)2.

(b)

[1]

L
The three graphs given below show the change in mass when 2.00 g
each of X(I0 3)z, Y(I0 3)z and Z(I03)z were heated separately at a
temperature T C.

(c)

mass/g

Y(IOah

2.00

2.00+--------

2.00

1.00

1.00

1.00

T----------0

(i)

mass/g

mass/g

X(IOah

'

time

[
-

time

Z(IOah

0.47 g
I

0.29g

time

With the aid of information from the graphs, identify the three
iodates(V) and write their identities in the spaces provided
below.
[1]

r~-- -~

Y(!Os)t:

.I

Z(!Osh:

l
l
L
r
I

1
l
[

(ii)

Arrange the three iodatesM in order of increasing thermal


stability and account for the trend.
(3]

[
[
[

['
[

C
[
[
[
[
[

L
L
[

10

[',

r
[

Question 6
Compound J can exhibit isomerism.

CH3

CH2-C-CH-CH==CH

J
(a)

C/

Draw all possible stereoisomers of J.

CH3

[
[4]

[
[

[\
[
(b)

Give the structural formulae of the organic products when J is refluxed


with hot acidified potassium manganate(VII) solution.
[2]

[
[

L
[
[
r-

l
11

l
L
r

[
[
~

Question 7
Ethanol reacts with Nal in phosphoric acid in situ to give an iodine-containing
organic product. However, it gives a different iodine-containing organic
product when it is heated with alkaline, aqueous iodine.
(a)

State the name of the mechanism when ethanol reacts with Nal in
phosphoric acid and draw the structural formula of the organic product
formed.
[2]

(b)

Write a balanced equation for the reaction between ethanol and


alkaline aqueous iodine. State the observation that accompanies this
reaction.
(2]

f~

[
[

[
[

r
L

[
rL

[
f'
L

c
l
L
r

12

-~-----c

[
[
Question 8
(a)
Compounds P and Q are structural isomers.
CH2CH2CH2NH2

CH3CH2CHNH2

=4.0}

p (pl<b = 3.5)

Q (PI<b

With reference to the p~ values, state the nature of X and explain its
[3]
effect on the basicity of the compounds.

c
[

[
[
(b)

Phenacetin is a pain relieving drug often prescribed as an alternative to


aspirin.

Phenacetin
_Explain why Phenacetin is not basic.

r-

CH,CH,O--NHCOCH 3

(i)

[;

[1]

l--

r
L-

l3

L
l
L
[
r-

(ii)

Phenacetin reacts with dilute potassium hydroxide when heated.


Write a balanced equation for this reaction.
[1]

[
[

r
[

(c)

Ethanoic acid is a stronger acid than phenol. Explain why this is so. [3]

c
'

[
[

[
[

L
l
L

14

L
[
[

r,_

Question 9
The structure of compound R is as follows:

HOOC-C-CH2NH2

C=CH 2

['

H
(a)

R is a solid with a high melting point Explain why this is so.

[1]

L
[~
(b)

Give the structural formulae of the organic products when R is reacted


with the following reagents.
[3]
(i)

gaseous HBr

(ii)

excess CH 3 Br

['
[!
[
r-~

(iii)

LiA/04 , in dry ether,


followed by 020

['

L
L----------

END OF PAPER

15

L
l

L
r-

Raffles Junior College


2006 JC 2 Preliminary Examination

CHEMISTRY

9251/03

Paper 3
11 September 2006

Monday

1330-1615 hrs

c
[

c
[

Time: 2 hours 45 minutes


Additional Materials: Data booklet I Paper I Graph Paper

READ THESE INSTRUCTIONS FIRST


Write your name and civics tutorial group on all the work you hand in.
Write in dark blue or black pen on both sides of the paper.
You may use a pencil for any diagrams, graphs, or rough working,
Do not use staples, paper clips, highlighters, glue or correction fluid.
Answer all questions in Sections A, B, C and D. The last question in each section is of
the form either/or except for Section D.
Begin each question on a fresh sheet of paper.

[~

At the end of the examination, detach page 13 and attach it to the top of your answer
scripts.

[,

The number of marks is given in brackets ( ] at the end of each question or part of a
question.
A Data Booklet is provided. Do not write anything on the Data Booklet

You are reminded of the need for good English and clear presentation in your answers.

L
L
l
[

This document consists of 13 printed pages, including one cover page.

L
~

SECTION A

Answer all questions in this section.


1

(a)

Chlorine reacts with iodine to form a compound T, lCh. When dissolved in


an excess of aqueous potassium iodide, T liberates iodine, h. which is the
only iodine-containing product in the reaction.

r
[

(i)

Write the equation for the reaction between T and potassium iodide.
[1]

(ii)

Calculate the number of moles of iodine liberated when 1.00 g of T


reacts with an excess of aqueous potassium iodide.
(1]

(iii)

What volume of 1.00 mol dm-3 sodium thiosulphate would be


(1]
required to react with all the iodine-liberated in (ii)?

(b) Cyanogen, a highly toxic gas, is composed of 46.2% carbon and 53.8%
nitrogen by mass. At 25C and 1 atm, 1.05 g of cyanogen occupies
0.500 dm 3 .

(i)

Determine the empirical formula of cyanogen.

[1]

[i
r

c
I

"

(ii)

Calculate the relative molecular mass of cyanogen, and hence,


(2]
write down its molecular formula.

(iii)

Draw the dot-and-cross diagram for the cyanogen molecule.

(iv)

Using the Valence Shell Electron Pair Repulsion (VSEPR) Theory,


state the shape of the cyanogen molecule.
[1]

(v)

"Cyanogen is soluble in ethanol." Explain this statement with the aid


of a suitable diagram.
[2]

[1]

[Total: 10]

c
c
[

[
[
lL
! '

L
2

L
r-

{J

(a)

A galvanic cell was set up with a zinc electrode dipped into 1.00 mol dm-3
zinc sulphate solution as one half-cell, and a gallium electrode dipped into
1.00 mol dm-3 gallium chloride (GaC/3 ) solution as the other. The e.m.f. of
the cell under standard conditions was measured to be +0.23 V. The zinc
electrode was found to be negatively charged.

(i)

Draw the cell set-up. Indicate on the diagram the direction of


electron flow in the external circuit.
[2]

[~

(ii)

Calculate the E9 value for Ga3 +(aq) + 3e- ~ Ga(s).

(iii)

Effervescence is observed when sodium carbonate is added to the


Ga3+/Ga half-cell. Explain.
[2J

'r
,.>

(b)

A possibility for the future is the use of electric cars. Instead of burning
methanol in an internal combustion engine, it could also be used to power
a fuel cell in the car. The fuel cell generates an electric current which
drives an electric motor. One possible design of a methanol fuel cell
under investigation is shown below. The fuel and oxygen are fed
continuously to the two electrodes. The electrodes are made of platinum
dispersed onto a porous support.

(-}

(+}

r---t-----t--=t-----lnsulation
CH3~

-Air
Cathode

Anode
Acidic electrolyte
solution

r
'--

(i)

Write half equations for the reactions which take place at the anode
and the cathode of the fuel cell, and combine these to give an
- overall equation for the cell reaction.
[2]
[1]

(ii)

Write the cell diagram.

(iii)

Give an advantage of the fuel cell over the internal combustion


[1]
engine with methanol as the fuel.

(iv)

One refinement of the fuel cell design is to replace the acidic


electrolyte solution with a film of solid H+ ion-conducting polymer.
[1]
Briefly explain why this would be an improvement.
[Total: 10]

L
3

H20
Insulation

L
L

[1]

r
'-'-<'

[,
3

Either

The combination of iodine with hydrogen and the dissociation of hydrogen


iodide were studied by Bodenstein from 1893 to 1899. This is a homogenous
equilibrium system.

H2 (g) + I2 (g)

2HI (g)

Known amounts of hydrogen and iodine were sealed in a glass bulb. The bulb
was kept at a temperature of 445C until equilibrium was reached. The glass
bulb was then rapidly cooled and broken under alkali.
(a)

State one essential characteristic of a system in chemical equilibrium. [1]

(b)

Calculate the value of Kc. using the following equilibrium concentrations


which were obtained by Bodenstein for the reaction of hydrogen with
iodine at 445 C.
(1]
Equilibrium concentrations I mol dm_::rH2 (g)
h (g)
HI (g)
2.06
13.40
36.98

Kc
1024

H2 (g) + F2 (g)

2 HF (g)

1.0 X

H2 (g) + C/2 (g)

2 HC/ (g)

2.0 X 108

H2 (g) + h (g) ~ 2 HI (g)

1.3x101

(i)

By comparing the values of Kc at 445 C and 1000 C, deduce


whether the reaction between hydrogen and iodine is endothermic
or exothermic.
[2)

(ii)

A change in pressure has no effect on these equilibrium systems.


Explain why this is so.
[1]

(iii)

Using the above values of


hydrogen-halogen bonds.

Kc.

comment on the strength of


[2]

(d) The rate of the dissociation of HI is found to increase when the


temperature is raised. Wrth the aid of a sketch of the Maxwell-Boltzmann
distribution curve, explain how a small. increase in temperature can lead
to a large increase in the rate of the dissociation.
(3]
[Total: 10]

c
[
[

(c) The values for the equilibrium constant Kc for the following reactions of the
halogens with hydrogen at 1000 C are shown in the table below.
Reaction

('

L
("

c
[

c
[
[

'

L-

L
L
[
r

Or
The kinetics of the hydrolysis of the ester, RCOOCH3, may be investigated by
determining the concentration of RCOOH produced.
RCOOCH 3 (aq) + H20 (!)

RCOOH (aq) + CH 30H (aq)

When 0.350 moles of the ester was hydrolysed by heating with water and using
hydrochloric acid as catalyst, the following results were obtained.
Time/ s
0
0.36 X 103
0.72 X 103
1.08 X 103
1.44 X 103

~-"~

('

L
[

r.c,

(a). (i)

(ii)

[RCOOH] I mol dm-3


0
0.105
0.185
0.243
0.278

Explain what is meant by the order of reaction.

[1]

By using a graphical method, determine the half-life of the reaction


[3]
and hence the order of the reaction with respect to the ester.

:)

(b)

It has been found that the hydrolysis reaction is first order with respect to
hydrochloric acid.

(i)

Write down the rate equation for this reaction and state the units of
the rate constant.
[1]

(ii)

What will the half life of the reaction be if the experiment is repeated
with the .concentration of hydrochloric acid doubled? Explain your
[2]
answer.

,,..

L
[
[

(c)

The above hydrolysis reaction is in fact reversible. In an experiment, the


equilibrium concentrations of the various species at T K are as follows:

= 0.188 mol dm-3


= 0.125 mol dm-3
= 0.376 mol dm-3

[RCOOCH3 (aq)]
[RCOOH (aq)]
[CH30H (aq)]

Solid RCOOH is added to the equilibrium mixture such that when the
equilibrium is re-established at T K, the concentration of RCOOH (aq)

becomes 0.200 mol dm-3.


Calculate the [CH30H (aq)] when equilibrium is re-established at T K.
Assume there is no change in the total volume of the mixture.
[3]

L
l~

t
r'

l[l

SECTION B
Answer all questions in this section.

4.

(a)

Explain why aqueous ions containing transition metals are coloured,


[3]
whereas aqueous ions of other metals are usually colourless.

(b)

With the use of relevant data from the Data Booklet, state and illustrate
how the relative stabilities of the Fe(II)/Fe(III) oxidation states are affected
by a change in

(i)

ligands;

(ii)

pH.

(Data: Fe(OHh + e- - ~
(c)

(4]
Fe(OH)2 + OW

E6 =-0.56V)

In the presence of manganese(IV) oxide, aqueous hydrogen peroxide


decomposes to oxygen and water rapidly.
Manganese(IV) oxide reacts with concentrated hydrochloric acid, forming
chlorine and aqueous manganese(II) chloride.
solution

contains

pale

pink

{
L
[

State three characteristics that are typical of transition metals, quoting


evidence from the following statements,

Manganese(II)
chloride
manganese(II) ions.

!'

hydrated

'"
l

.~

If manganese(IV) oxide is heated strongly with an oxidising agent and an


alkali, green manganate(VI) ions are first formed. The green solution
- subsequently turns purple.

(3]
[Total: 10]

r-

-.>

[
[

L
l
L
r

'L-

r
[

Either
(a}

[
[

The enthalpy change of vapourisation, .6.Hvap of elements in the third period


of the Periodic Table are shown in the table below
Element

Na

Mg

Al

Si

Cl

AHvap/ kJmor 1

101

132

284

300

52

63

10

Explain the variation in these data in terms of structure and bonding

[
(b)

c
[

(c)

c
[

In the electrolysis of brine, one of the products X is allowed to react with


chlorine to form bleach (NaOCl). Write equations for the formation of:
(i)

X at the cathode;

(ii)

bleach.

(i)

Determine the empirical formula of the salt.

(ii)

Construct an equation for the reaction.

(iii)

Calculate the change in oxidation number of phosphorus during this


[4]
reaction. Give the name for this type of reaction.
[Total: 10]

Or
(a)

L
l

L
L
\'

Sketch and explain with the use of equations, the variation of pH of the
resultant mixture when Period 3 elements ( Na to Cl) are added to water.

[4]
(b) When chromium is made to react with different halogens, themain product
of the reaction with fluorine is CrF4, with chlorine it is CrCl3 and with iodine
it is Crh
Us.ing Ofle chemical property.of the. halogens and relevant data from the
[3]
Data Booklet, explain the above observations.

[2]

Phosphorus exhibits allotropy. The two common allotropes of phosphorus.


are white phosphorus (consisting of P4 molecules) and red phosphorus
(consisting of macromolecules). When white phosphorus is boiled with
concentrated aqueous solution of sodium hydroxide, phosphine gas (PH3)
and a salt is produced which has the following composition by mass:
Na, 26.1 %; H, 2.3%; P, 35.2%; 0, 36.4%.

[
5

[4]

(c)

Phosphine PH 3 a gas at room temperature can be prepared by the action


of sodium hydroxide on phosphonium iodide, PH4I. When 1.00 g of
phosphonium iodide reacted with solid sodium hydroxide, 0.925 g of white
solid was formed, together with 150 cm3 of phosphine gas, volume
measured at roomlemperature and pressure.

[1]

(i)

Identify the white solid.

(ii)

Use the above data to deduce the equation for the preparation of
phosphine.
[2]
[Total: 10]
7

c
[

Section C
Answer all questions in this section.

a-Hydroxy Acids (AHA's) are naturally occurring carboxylic acids which are
well-known for their use in the cosmetics industry. They are often found in
products claiming to reduce wrinkles, the signs of aging, and improve the
overall look and feel of the skin. They are also used as chemical peels.
The reaction scheme below shows how one such AHA, compound E, can be
synthesised from an alkyl halide, A

@-cH,CH,Cl

!
CN

COOH

CH2~H
I

[:

OH

(a)

Give the reagents and conditions required in stages I, II and IV.

[3]

(b)

With the aid of equations, describe the mechanism in stage m.

[4]

(cr Give a simple chemical test to distinguish between compounds A and B.


~

(d)

-~

OH

liV

[
[

@-cH2 CHO

_,

Ill

@-cH,4H

\~

@-cH:oCH,OH

Give the structure of.the .. product formed when compound C reacts with
2,4-DNPH.
[1]
[Total: 10]

[
r~
'-'

r
L
L
l_

L
L
r

r
[

[
r

X is an alcohol with the formula C5 H1 20. Two isomeric compounds Y and Z are
produced upon heating X with excess concentrated H2 S04 at 170 oc. Both Y
and Z do not exhibit geometric isomerism. The 1H NMR spectrum of the
compound Y is provided below:
~

[
3

[
[
[
2

1 1

J~ -

I
9

I
6

I
3

I
I

T
0

ppm

(a)

From the NMR spectra, deduce the structural formula of Y, and hence
[5]
draw the displayed formula of alcohol X.

(b)

Draw the structural formula of Z.

(c)

The NMR spectrum of X has a signal at 5 = 1.52 ppm which disappears on


. the addition of 0 20. Explain.
[2]

(d)

Give a simple chemical test to distinguish compounds Y and Z. You need


NOT write equations to support your answer.
[2]

L---

c
_,

[1]

{Total: 10]

L:
[

L
L
{

c
8

Either

An optically pure sample of 2-chlorobutane reacts with ethanolic sodium


cyanide to yield an optically pure product, X.

[J

(a)

With the aid of equations, describe the mechanism of the reaction and give
the structural formula of X.
(4]

(b)

Explain how the rate of this reaction changes when 2-chlorobutane is


replaced by 2-iodobutane.
[2]

(c)

In exactly FOUR steps, show how 2-chlorobutane can be used to


synthesise ethyl ethanoate. Indicate the reagents, conditions and
[4]
intermediate products in your answer.
[Total: 10]

Or
(a)

Compound A, CsH6 0, was first used as a surgical antiseptic till it was later
found to be carcinogenic. Reaction of A with 3-chloropropene in the
presence of anhydrous aluminium chloride yields B, C9 H 100. Both A and
B are found to decolourise aqueous bromine to form white precipitates C
and D, respectively. Both C and D contain the same number of bromine
atoms per molecule and D is found to be optically active.

(i)

(ii)

(b)

[]

n
u
L

u
[
[

Identify A and write equations to illustrate the mechanism for the


reaction of A with 3-chloropropene and anhydrous aluminium
chloride.
[4]

[2]

Give the structural formulae of C and D.

Show how benzene and 2-chloroethanoic acid can be used to synthesise


compound E shown below. Indicate the reagents, conditions and all
intermediates products in your answer.
[4]

..

.'

CCHzCOO-Na+

NH2
[Total: 10]

L
L

l.
10

r
c
[

Section D
CHEMICAL THERMODYNAMICS
Answer all questions in this section.
9

(a)

Consider the following reaction:


\

2NO(g) + 02(g) ~ ZN02(g), . ~G = -70.5 kJ mor at 25C


0

Some data of the above gases are given in the following table:

,.
0

~Ht (kJ mor1) i


S 0 (J mor1 K-1) '

NO(g)
+90,3
-~210:f:l'

~-..

N02(g)
+33.2
.
24o.r

,.,~.

For the above reaction, calculate tne follo~ing:

\:-....

The standard entropy change, ~S0~-Account .for the sigh of 'your-

(i)

~~

(ii)

The value of the standard entropy change in the surroundings,


L'lSsurr
[1]

(iii)

The value of the standard molar entropy, S 0 , for 0 2.

[
j

c
[

(b)

The workdone on the systern.

(v)

The change in internal energy of the system, ~u. for per mole of
N02, at standard state conditions.
[1]

With the help of the Data Booklet and the data given below, calculate the
solubility product for Cu(OH)2 at 298 K.
Cu(OH)2 (s) + 2e- ~ Cu (s) + 20H- (aq),

=-0.224 V
[Total: 10)

[
r,

E9

[3]

[1]

(iv)

c
L,

[1]

For two moles of NO that reacted with one mole of 0 2 at 25C, determine
the following:

.,

11

c
10

{a)

An Ellingham diagram shows how the Gibbs free energy change for a
particular reaction varies with temperature, T.
The following shows a sketch of an Elling ham diagram for three reactions:
: "'

(II\

\
\

.... -400
0

E -600

~b = (-1116 + 0.21T) kJ mor

-200

L
[

(IIn . -AI(s) + Oz(g) -+-Aiz03(s)


\ \ 3
I \

-~ L1G = (-223 - 0.18T) kJ mor


,. .. -. i'lG_= (-525 + 0.13T) kJ mor1

2C(s) + Oz(g) ~ 2CO(g)


~Fe(s) + 0 2(g)-+ ;FeO(s)

(I)

'

----un-- ------

['

-~

.:.t.
......
<!)

-8001-

.'/

<l

-1000

'

"

1200

500

1000

1500

2000

T/K
(i)

State, in physical terms, why i'lG for reaction (I) becomes more
[1]
negative as the temperature increases.

(ii)

Calculate the free energy change of the following reaction at 1000 K:


FeO (s) + C (s) -+ Fe (s) + CO (g)
What is the minimum temperature at which the reduction of iron(ll)
oxide by carbon becomes thermodynamically feasible?
[3]

(iii)

(b)

Aluminium oxide and carbon are both high melting materials. With
reference to the above Ellingham diagram, explain why aluminium
oxide is used, in preference to carbon, to line vessels that are used
to contain molten steel saturated with oxygen. (The melting point of
steel is around 1800 K.)
[2]

"Chemical processes move in directions in which the products have lower


Gibbs free energies than the reactants. It is possible to establish the
relationship between the free energy change of the chemical reactions
with the relative contributions of the enthalpy and entropy changes
accompanying the reaction."
Comment on the following in the light of the above statement:

{i)

When aqueous sodium hydrogencarbonate is added to dilute


hydrochloric acid at room temperature, the temperature of the
reaction mixture drops. Despite this, the reaction is spontaneous.

(ii)

Camping gas (liquid butane) is used commonly for cooking. It


undergoes combustion spontaneously at room temperature.
[4]
[Total: 10]
END OF PAPER
12

[
".

-~

r-,
L

L
L
L
l
t~

L
1

l ~

c
[
r

Name: ................................. (

Civics Tutorial Group: .............

Raffles Junior College


2006 JC 2 Preliminary Examination

CHEMISTRY
Paper3

Attach this cover page to the top of your answer scripts.

For Examiner's use only

Section

Question

Marks /10

2
3 either I or

5 either I or

6
-~

[--

7
8 either I or

10

'

Total 1100

c
L
l
L
L
(

Grade

13

,....-,

,-.......,

'

r---'>
l

2006 RJC Prelim Chemistry Paper 3 Answers

2
(a)

(i)

1
(a)

(i)

!Ch (aq) + 7Kl (aq) ~4h (aq)+ 7KC/ (aq)

(li)

Amount of !Ch reacted

(b)

b (aq) + 2S2 ~ - (aq) ~ 2r (aq) + S4 Qt(aq)


2
Amount ofS20:! 2" reacted
2 x 1.07 x 10"
2.13 x 10"2 mol
Volume ofS20:! 2' reacted
2.13x 10'2 I 1.00
3
2.13x 10'2 dm
21.3cm3
2

=
=
=

(i)

N
%mass
46.2
53.8
Mole ratio
46.2/12.0 3.85 53.8/14.0 3.84
Simplest
1
1
mole ratio
.
'
The emp1ncal formula of cyanogen 1s CN.

(li)

electron flow

= 1.OO
=2.663 x 103 mol
(127 + 7x35.5)
=4 x 2.663 x 1o =1.07 x 1o2 mol

Amount of 12 liberated
(Iii)

P = 1 atm
T = 2 5C

voltmeter

,.-----( v >-----,

PV

(iii)

Ga3 has a high charge density that polarizes the electron cloud of the water
molecules surrounding it, thus weakening the 0-H bond which leads to the
3
formation of H3o. Therefore the solution consisting of Ga (aq) is acidic and upon
addition of sodium carbonate, C~ (g) is evolved.
[Ga(H20);f+..,.. (Ga(H20}.;(0H)f + W
C03 2- + 2H ~ H20 + C02

(i)

Cathode:
02(g) + 4W(aq) + 4e _, 2H20(/)
Anode:
CH30H(I) + H20(/) _, C~(g) + 6W(aq) + 6e'
Overall:
2CH30H(/) + 302(g)....., 2C02(g) + 4H20(/)
Pt(s) I CH30H(/) I C~(g) I H(aq) I ~(g) I H20(I)I Pt(s)
There is no C nor CO produced in the fuel cell Or there is complete combustion
Or NOx products are not formed.
Eradicate the possibility of acid leakage from the fuel cell.

=nRT

1.01 X 1if X 0.5 X 10' 3


M

=52 g mol' 1

Or

= 1 ~ 5 X 8.31 X 298

M, of cyanogen

=52.0

=0.5/24 = 0.0208 mol


=1.05/0.0208
=50.5
= 52.0
=52.0
=2

(ii)
(iii)

Amount of cyanogen
Mr of cyanogen
n(Mr of CN)
n(12.0 + 14.0)

Molecular formula of cyanogen is C2N2

(iii)

(b)

(iv)

3
(a)

: N i~ C x. C :~ N ~

Either
Any of the following characteristics can be accepted:
There are no more changes to the concentration or pressure of any species even
thpugh both the forward and backward reactions still occur.
The rate of the forward reaction equals that of the backward reaction.
K,

(b)

(iv)

Linear

(v)

Cyanogen can form hydrogen bonds with ethanol as it has a lone pair of electrons
on nitrogen, thus soluble in ethanol.

(c)

=__Q:!.f__
[H,][I2]

=> K,

36.982
2.06 X 13.40

=49.5

(I)

The Kc value decreases as temperature increases is an indication that amount of


product formed is lesser at higher temperature. This is thus an exothermic
reaction since the position of equilibrium has shifted backward to remove the
excess heat energy.

(ii)

Both forward and backward reactions produce the same number of gaseous
molecules, hence an increase or decrease in pressure will not affect the position
of the equilibrium.

(iii)

Kc indicates the extent of formation of HX, i.e. higher Kc higher concentration of


products formed. Since K, of H F > HCI >Hi, the tendency for HX to form is H F >
HCI >HI and thus the strength of HX bond is in the order: H F > HC/ > H 1.
2

---'

L_

(._____,

(__)

LJ

c.__:,

L.J

L_j

LJ

~/

(___.:.

L_)

The distribution of molecular energies at two different temperatures.

K=

(c)

Fraction of molecules

total number ofmolecules with


energy greater than or equal to Ea atT, K

[ill]

total number of molecules wnh


enerb'Y greater than or equal toEa atr, K

Ea (activation energy)

OR
(i)

(a)

Or

4
(a)

Graph of [ester] /moldm' vs time/10 s


0.4

0.35
'E

0.3 .

:E0

o. 25 I

0.2

:" 0.15 .r----------';;,.,.._

(b)

IJJ

.!!. 0.1
0.05

t" = 630s
3

t"" 630s

time/ x 10 s

(b)

(i)
(ii)

Rate= k[RCOOCH3)[HC/]
Half life is halved = 315s

0.188

= 0.200(0.376- X)
0.188+X
: 0.200(0.376- X)
0.188+x
0.0627

= 0.376- 0.0627 = 0.313 mol dn1J

[Fe(H20)s)'+ + e
Fe3 + e
"""'"
[Fe(CN)s)'" + e
""""

Units of k: mor1 drn>s1

[Fe(H20lsf Ee
0
E
[Fe(CN)sf'
E0

""'Fe
"" 2 +

=+0.77 V
=+0.77V
=+0.36 V

Fe3 is stabilized with respect to Fe2 when complexed with eN, hence, less
positive reduction potential.

1.5

Time /s
[ RCOOH]/ mol dm""
[RCOOCH3_]/ mol dm..o
0
0
0.350
0.105
0.245
0.36 X 10
0.165
0. 72 X 100
0.185
0.243
0.107
1.08x100
1.44 X Hi
0.278
0.072
t\<1 =0.63 X 10 s = 630s
Since t" =constant, reaction is 1' 1 orderwrt ester

(I)

~
0.5

'~

Aqueous ions of transition metals are aqua complexes with six water ligands coordinated to a central metal ion, eg in [Cr(H 20);f+.
The resence of these six li ands s lit the d-orbitals of C
into two sets of orbitals of
different energies. C + with the electronic configuration of [A r] 3cf has partially tilled dsubshells. The electrons in the lower set can absorb energy of certain wavelengths of the
visible light and are excited to the upper set. Such d.<.f transitions are responsible for
colours in transition metal complexes and the colour of the complex observed is the
complement of the colour absorbed.
However aqueous ions of non-transition metals such as [AI(H 20)sf+ show no colours
since the metal ions do not have partially tilled d-subshells so that no d-d transitions
occur . The excitation of electrons from a lower energy orbital to one of higher energy
(e.g. from 3p to 4s) requires wavelengths of light which fall out of the visible region of the
electromagnetic spectrum. [AI(H 20);jl+ allows all the incident visible light to be
transmitted and is colourless. Hence no light from the visible spectrum is absorbed.

Or

0
0

The order of a reaction with respect to a reactant is defined as the power to which
the concentration of the reactant is raised to in the rate equation.
The overall order of reaction is the sum of the powers of the concentration terms
in the experimentally determined rate f.quation.

(ii)

,___)

=0.125 x 0.376 =0.25 mol dm3

Thus [CH30H (aq)J = 0.376- x

As the temperature is increased, there is higher proportion of reactant particles


with kinetic enemy greater than the activatjon energy (Ei!L Hence tlie dissociation
rate increases.

RCOOCHJ (aq) + H20 (/) _.. RCOOH (aq) + CH30H (aq)


0.188 +X
0.200
0.376- X

0.25

kinetic energy

[RCOOHJ[CHpH]
[RCOOCH 3 )

Eqm cone.
lmoldm 3

;...___;

Let rate= k'[RCOOCH3) where k' = k[HCI]


Since HCI is a catalyst and [HCI] is constant.
tv. ln2/ k':;::} when [HC/] Is doubled, k' is doubled, thus tv. is halved.

(d)

with given energy

'---'

"---"

Or

It is more difficult to reduce [Fe(CN)st(aq) as that would require the addition of


an electron (i.e. negatively charged) to the negatively charged [Fe(CN)s)'-(aq).)
(b)

(ii)

Or

[Fe(H20);f+e.....,.
[Fe(H20);f+
E 0 =+0.77V
3
2
Fe + e """"
Fe
E 0 +0.77 V
Or
Fe(OH)J + e
""""'
Fe(OH}l + OH'
E 8 = -0.56 V
Fe3 is stabilized with respect to Fe2 by an alkaline medium hence, less positive
reduction potential.

o:ar

It is easier to reduce [Fe(H2


as that would require the addition CJf an electron
to the positively charged [Fe(H 20)sf+.
4

,-------,
(

'

/.,

(c)

Variable oxidation states: +2 in MnCI 2, +4 in Mn02 , +6 in Mn04 2, +7 in Mno4


2
Coloured transition metal ions: Pink Mn , green Mno/, purple Mn04
Catalytic behaviour: Mn02 acted as heterogeneous catalyst for decomposition of
hydrogen peroxide

OR
alternative

(a)

. __ ;answer

--

Look out for 7 points.


Missing or wrong point -1/2
Cap at 1/2

Either

(a}

The value of liH,ap of an element depends very much on the strength of the bonds being
broken or the strength of the intermolecular forces being overcome in converting from
liquid to vapour.
Na, Mg and AI have giant metallic structures. Converting these metals from liquid to
vapour involves supplying heat energy to break metallic bonds. Since metallic bond
strength increases from Nato AI due to increasing number of valence electrons for
delocalisation (1 for Na, 2 for Mg and 3 for AI}, the value of liH,ap increases from Nato

Na

Mg

AI

Si

Cl

Na reacts vigorously with water to form a strongly alkaline solution/ solution of pH > 7

AI.
Si has a giant covalent structure. It has the highest liH,.p among the given Period 3
elements due to a lot of heat energy needed to break the relatively strong Si...Si covalent
bonds in liquid Si.
For the elements P to Cl, they exist as P4, Sa and Cl2 molecules respectively. They have
simple molecular structures with weak instantaneous dipole -induced dipole attractive
forces (or Van der Waals' forces) between molecules to be overcome when converting
from liquid to vapour. Hence they have low liHvap values.
The strength of instantaneous dipole-induced dipole interactions depends on the
polarisability of the electron cloud and hence on the number of electrons per molecule.
Since the latter increases in the order Cl2 < P4 <Sa, the strength of the instantaneous
dipole-induced dipole interactions and hence the value of liH,ap increases in the order
Cb <P4 <Sa.

(b)

(i)

Reaction at the cathode: 2Na(aq) + 2H 2 0(1} + 2e ~ 2Na0H(aq)

2Na (s) + 2H 20(1)

2Na0H(aq)

+ H2 (g)

Mg and AI only react with steam, hence pH of this solution remains at 7


Si, P4, Sa no reaction with water, pH remains 7
C/2 dissolve in water to form acidic solution.
C/2 (g)+ H20(/) HCI(aq) + HOCI(aq)

(b)

Oxidising power of halogen decreases down the group


9
It can also be shown by less positive E value on descending the group.

E9

F2 (g) + 2e- """' 2F (aq},


Cl2 (g) + 2e-...,.... 2CI- (aq),
12 (g) + 2e- _. 21- (aq),

+ H2 (g)

(c)

(il)

C/2 (g) + 2Na0H(aq) -----t NaCI (aq) + NaOC/(aq) + H20(/)

(I}

Molar ratio of Nato H toP to 0 = 26.1/23.0: 2.3/1.0: 35.2/31.0: 36.4/16.0


= 1.13:2.30: 1.14:2.28

=+2.87 V

E9 = +1.36 V
E9 = +0.54 V

Ruorine being the strongest oxidising agent, oxidise Cr to CrF4 when the oxidation state
ofCris +4.
Chlorine less oxidising than fluorine, oxidise Cr to CrCh when the oxidation state of Cr is
+3.
Iodine weakest oxidising agent, oxidise Cr to Crh when the oxidation state of C r is +2.

=1 : 2: 1: 2

Hence the empirical empirical of the salt is NaH2P~


(II)

(iii)

P4 (s} + 3NaOH(aq) + 3H20(1) -----t 3NaH2P~(aq) + PH3(g)


Suggested method to balance the equation:
Determine the balanced oxidation half-equation and the reduction half-equation
and then combine the two half-equations to get the above overall balanced
equation.
Oxidation number of P changes from 0 in P4 to +1 in NaH2P~ and -3 in PHJ.
Hence the changes in oxidation number of P in this reaction are +1 and -3.
Type of reaction: disproportionation reaction

(c)

(i}

The white solid is Nal

(ii}

Molar mass ofPH4l


31.0 + 4(1.0) + 127 162.0 g mol'
Amount of PH4l
1.00/162.0
0.00617 mol
Amount of Nai
0.925/(23.0 + 127)= 0.00617 mol
= 150/24000
= 0. 00625 mol
A mount of P H3
Mole ratio ofPH41: Nal: PH3 = 1:1:1

=
=

PH41 (s) + NaOH(s} - - +

Nal (s} + PH3(g} + H20(g)

-'

(__,

L_.l

6
(a)

l_.;

L.J

L_j

L_l

L...J

'1

'----'""'

l..-...!

(.____j

;_!

l,___.i

,----------,

"---'~

(a)

__}

&'+;H~

\=:~:JF~CH2-c,,

Products expected: alkene (dehydration of alcohol)

~ a -CHzCH3 unit present


2 doublets at around 8 = 4.7 ppm~ 2 vinylic protons
Last singlet at 8 = 1. 7 ppm due to -CH;~ unit that has no adjacent protons

The mechanism is nucleophilic addition. The required reagents are HCN and a small
amount of N aOH IN aCN as catalyst.
Step 1: Generation of the nucleophile. CN"
Since HCN Is a weak acid, concentration of CN" from the dissociation of HCN is very
low. As such, NaOH is added:
HCN _,.. W + CN"
The addition of OW removes H, hence driving the equilibrium to the right and increases
the concentration of CN". This increases the rate of reaction.

Or
-
chemical
shift /ppm

1.0
1.7
2.0

:C""'N

fAn anionic intermediate is obtained.

--

splitting
pattern
triplet

No. of adjacent relative


peak area
protons
2
3

singlet
quartet

4.7
4.8

~-

<Q)-cH 2 i~CN

Y: 2-methylbut-1-ene

IGIH3G

\=/
(!JI

<Q)-cH 2?-oH

(Since X can form two alkenes on dehydration, -OH group is on position:< rather than
position 1 -+ X is 2-methylbutan-2-ol.)

+ CN-

CN

Add NaOH to separate portions of both compounds and heat. Cool. Acidify with dil HN03
and add aqueous AgN~. A white precipitate obseNed for A but no white precipitate is
obseNed for 8.
CsHsCH2CHzCI + OH' -7 CsHsCHzCHzOH + C(
Ag + C( -7 AgCI (s)
Or
Add PC Is to separate portions of A and 8. White fumes of HCI obseNed for 8 but white
fumes not obseNed for A.
CaHsCHzCH20H + PC/s -7 CaHsCHzCH2C/ + POC/3 + HC/
Or
Use N a metal. 8 will give effeNescence of H2 and A will not have any effeNescence.
CaHsCH2CH20H + Na -7 CsHsCHzCHzO"Na + Y:zHz

o,,-d-~--~~
7

\H <AI

(8 I

H
I
H-C-H

A cyanohydrin is obtained and the catalyst is regenerated.

NOz

-----~-

.H(Al

CH.-C(

-CH3
-(adjacent to a -CHz-2_
-CH3
-CHz(adiacent to a -CH;u__

slow

Step 3: Attack on proton by anionic intermediate

IR\_cH2Lo-~
r
I
H-C""'N
ON

group responsible

1
doublet
1
=CHz
doublet
1
1
----- Focus on no. of adjacent protons and group responsible.

Step 2: Attack of eN- on electron deficient carbon atom

(d)

'.____,)

5 signals~ 5 types of chemically non-equivalent protons


Presence of qLiartet at 8 = 2.0 ppm and triplet at 8 = 1.0 ppm

Or
The complete dissociation of NaCN provides the initial CN' ions required.
NaCN -7 Na +eN

(c)

----i

\.=...J

L--._..

1: Aq. NaOH, heat


II: KzCrz~ I HzS04, immediate distillation
IV: Dilute HCI or dil H2S04, heat

(b)

H H
H
I I
I
H-c-c-c--c-H
I I I
I
H H 0-H H
(b)

Z is 2-methylbut-2-ene.

<DI H3 C

CH 3 r8J

\=/

IAI

CH 3 rCJ

(c)

The peak is due to a labile proton of -0H and hence when DzO is added, there is rapid
chemical (proton) exchange I (equation).

(d)

Heat separate portions of Y and Z with KMn04/HzS04.


COz is evolved which gave white precipitate with Ca(OH)2(aq) present for Y but no COz
evolved for Z.
8

i (---.,

Either

(a)

Nucleophilic substitution.

Step 2: ElectrophilicAttack

Step 3: Loss of proton


(X)

--

The nucleophile (CN*) attacks the chlorine bearing 6 carbon from the side opposite the
chlorine atom. This is known as the backside attack.

fast

Bond making and bond breaking occur simultaneously. The C-CN bond is forming while
the C-CI bond is breaking.
A pentavalent transition, state/activated complex which cannot be isolated is formed.
(ii)
This results in inversion of the configuration. (Walden Inversion)

Markovnikov's Rule! j
(b)

Rate will increase when 2-iodobutane is used. C-1 bond is weaker than the C-CI bond
due to less effective overlap in the former. Rate of reaction is dependent on the ease of
breaking of the carbon-halogen bond.
e1hanolic KOH

(c)

CH 3 CHCICH 2 CH 3

heat

CH 3 CH=CHCH 3

(b)
CH 3 CH=CHCH 3

CH 3 COOH

H 2 S0 4 KMn0 4

heat

LiAIH 4 in dry ether

2 CH 3 COOH

CH 3 CH20H

(D)

(C)

HCI

CH 2 CICOOH
AICI 3

~CH 2 COOH cone HN0 3 cone H 2 S0 4

r A f C H 2 COOH

~N0 2

heat

OH 3 CH 20H + CH 3 COOH

OR

CH 2COOH ( l)

_..:..:._S:.;n;:._,.:..co:.:.n.:..c...:H..:.C:.:.I...

(a)

(i)

NO,

( 2)

NaOH( a~)

(A)

Electrophilic Substitution
Can also undergo the following sequence:

Step 1: Generation of electrophile

1. Nitration

2. Reduction

3. Friedel Craft Alkylation

10

4. Salt formation

L._;

LJ

L.____;

(__J

r...__......\

l__;

::-L_j

L.._....,

9
(a)

(i)

ilH 0 "2(33.2)- 2(90.3) = -114.2 kJ mor 1


0
6G "-70.5 = -114.2- 298(6$ 0 )
Hence, .!lS 0 "-146.6 J mol- 1 K- 1 (-147 J mor 1 K- 1) (3sf)
]
0
6S is negative, as there is a decrease in the number of moles of gases going
from reactants to products.

.____

L-.:,

<----!'

Or

(iii)

(IV)

(i)

(iii)
+2.42kJ

Wori< is done on the gaseous system is 2.42kJ.


6H =-114.2kJ mol-1
w = 2.48 kJ (or 2.42)

NO(g) + '!.Oz (g)_, NOz (g)

6H = -57, 1 kJ mol-1
w = 1.24 kJ (or 1.21)

(b)

(i)

By inspection of the Ellingham diagram, at T = 1800 K, reaction (I) is still


thermodynamically feasible, i.e. oxygen can react with carbon to form CO since
the line'(!) reveals the process to have a negative .6.G0 value. Thus carbon is not
suitable to be used at a lining material at such a high temperature in the presence
of oxygen gas.

The reaction is
HC03 -(i:lq) + H+ (aq) _, HzO(I) + CO:I (g)
Although this is an endothermic process as the temperature is noted to drop on
addition of sodium hydrogen carbonate to HCI, the reaction proceeds
spontaneously i.e. 6G < 0 since it Is entropy-driven OR T6S more positive than

Solubility product of: Cu(OH)z(s) ..- CLf+(aq) + 20W(aq)


CLi(OH},(s) + 2e-""""' Cu(s) + 20W(aq)
E0 =-0.224 V
cJ (aq) + 2e =-"' Cu(s)
E0 = +0.34 V
Ecen ;: -0.224- 0.34 = -0.564 V
6G0 29BK "-nFE 0 ..
= -(2)(96500)( -0. 564) " 108852 J mol-1
0
6G 29BK
-R Tin Keqm(296K)
.
" -(8.31)(298) In Keqm(298K! = 108852 J mor1
20
3
9
Hence, Ksp(298K!
Koqm(29BKJ "8.13 x 10- mol dm-

At T = 1000 K,
(I)
2C (s) + 0 2 (g)-+ 2CO(g) 6G = -223- 0.18(1 000) "-403 kJ mol-1
1
(IT)
2Fe(s) + 02 (g)-. 2Fe0(s) 6G "-525 + 0.13(1 000) -395 kJ mor

Aluminium, from line (Ill), will also react with oxygen to form A12 0z, but this latter
material is highly stable and resistant to further oxidation, and it has very high
melting point.

6U "ilH - PV = 6H + w =-57, 1 + 1.24 = -55.9 kJ


(b)

The reaction: 2C(s) + Oz(g) ..... 2CO(g) shows an increase in the number of moles
of gas so that 6S 0 > 0 for the process.
0
Since 6G0 6H 0 - T6S ,
6G0 becomes more negative as T Increases.

6G0 =-223- 0.18T = -525 + 0.13T


Solving, 0.13T" 302, hence T = 974 K

2NO(g) + 02 (g)...., 2NOz (g)6n 2- (1 + 2) = -1


w -PilV "-(6n)RT -(-1 )(8.31 )(298) = +2.48 kJ

2NO(g) + 02 (g)...., 2NOz (g)

_1

For the minimum temperature in which the extraction can occur, this happens at
the intersection point:

Or

(v)

__j

Adding;
1
2Fe0 + 2C -. 2Fe + 2CO t;G= -403 -(-395) = -8 kJ mor
For the reaction: FeO(s) + C(s)-> Fe(s) + CO(g)
li.G= -4 kJ mor 1

6S 0 = 2(240.1)- 2(21 0.8)- S 0 (02 ) = -146.6 J mol-1 K- 1


Hence, S0 (Oz) =+205.2 J mor 1 K- 1 ( +205 J mor 1 K- 1 ) (3sf)
Note: Standard molar entropy of an element must be> 0.

w "-PeN" -(1.01x1tr)(-1x24x10- 3 ) =

;___}

'...._.)

6Ssurr = -6H 0 /T = -(-114.2)/298 = +383.2 J moi- 1 K- 1


( +3B3 J mor 1 K- 1 ) (3sf)
0
6Suniv -6G fT" 236.6
6Ssurr = 6Suniv - 6S,~, = 236.6- (-146.6) = +383.2 J mol-1 K- 1

;~

10
(a)

(ii)
(li)

'
1....--.)

\_~

C.!i.
There is an increase in entropy (of the system) in the reaction since it involves two
solution (liquid) phase species forming gaseous CQ.. (and water).
(li)

The reaction is
13

C4 H1o Q) + - Oz (g) ..... 4C0:2 (g) + 6H2 0(1)


2
The reaction is spontaneous i.e. 6G < 0
This is evidently exothermic given the use of liquid butane in the camping gas
context.
0
However, 6S is highly negative and unfavourable for the reaction as It involves
the conversion of 6~ moles of oxygen into 4 moles of COz.
It can be seen that 6H 0 must outweigh the T6S 0 term, at least at ambient

temperature.

11

12

"

2006 RJC Prelim Chemistry Paper 2 Answers

(b)

(i)

Question 1
AP' is small and highly charged i.e. has a high charge density
(a)
and hence it has a high polarizing power
is a larger anion (greater number of electrons) and
hence has a greater polarisablllty than than F-

CsHsN (/) + 29/402 (g)~ 5C02 (g) + 5/2H20

cr

(ii)

(b)
A/C/3

: Cl ~Be~ Cl :

.. x.

Molar mass of pyridine = 79.0 g mor'


Amount of pyridine burnt= 1.13/79.0 = 0.01430 mol
Heat evolved= 100 x 4.2 x (87-25) = 26.04 kJ
Total heat evolved = 26.04 x 100/65 = 40.06 kJ
Enthalpy change of combustion =- 40.06 kJ/ 0.0143
=- 2800 kJ mor'
Note: Try not to use Q. Q = heat change (-ve for combustion)

AI ~ Cl :

Cl

(0 + N02 (g)

Note: Equation is not a definition/

The bond between AI and C! is polarised to the extent that A/C/3 is covalent
but A/F3 is predominantly ionic.

BeC/2

Standard enthalpy of combustion of pyridine is the enthalpy change


when one mole of pyridine is completely burnt in oxygen at 298K and 1
atmospheric pressure (or standard conditions).

(iii)

Cl

'

Shape:

linear

Shape:

trigonal planar

-109.5

(c)

(d)
dative covalent I co-ordinate bonds and covalent bonds

Question 2
(a)
(i)
CsHsN (/)--+ 5 C (g) + 5 H (g) + N (g)
Enthalpy change of atomisation of pyridine
(ii)
= 2xE(C-C) + 2xE(C=C) + E(C-N) +E(C=N) + 5xE(C-H)
= 2(350) + 2(610) + 305 + 610 + 5(410)
= + 4885 kJ mo/" 1

Enthalpy change of combustion of CsH,,N


Enthalpy change of combustion of CsHsN
Enthalpy change of combustion of H2

By Hess' Law,
ilH, = ilHc" + 36Hc- 6Hc' =- 301 kJ mor 1 (3 s.f.)
Question 3
(a)

(iii)

Any of the following reasons:

1)

The value calculated in (ii) does not take into account the enthalpy
change of vapourisation of pyridine.
The bond energy data from the Data Booklet are average values which
may not agree with the actual values unique to the pyridine structure.
Pyridine is stabilised due to resonance (like benzene). It is more stable
than expected due to the delocalisation of the n electrons over the 5
carbons and the nitrogen atom. More energy is then required to atomise
pyridine.

2)
3)

t~Hc'

ilHc..
ilHc

(i)

_]Q_x 0.30
Amount of salt formed =
1000
= 9 x 1 3 mol
3
+ 0.20 = 0.045 dm' 3
Volume of the HCI added= 9 x 1

Total volume at equivalence point= 0.030 + 0.045 = 0.075 dm 3


Concentration of salt at equivalence point
= 9 x 10'3 + 0.075 = 0.12 mol dm' 3

(ii)

2 = [H30.J 2 + (o.12- [H3on


2.09 x
[H3o) = 5.01 x 10' 7 moldm' 3

w'

pH = -log [H3o] = 6.3

kJ mol- 1
- 3356
- 2800
- 285.8

__I

L-

L_,

L..J

[___..)

'------1

(__J

LJ

LJ

L_j

(iii)

L_l

r
.
..___.

,______,I

Question 4
(a)
A: Na 2 S2 0 3 (aq)

pH

t'!:S

n.c

-7- Vo I H c \

L---.J

B: Br 2(aq)

__

.),

C: h(aq)

S203 2'(aq) + 4Br 2(aq) + 5H 20(1)---" 2SO/'(aq) + 8Br'(aq) + 1OH'(aq)

(c)

Add starch solution to each sample in a test-tube.


For C, a blue-black coloura!ion would be observed due to the formation of the
starch-iodine complex.
ForB, there would not be any blue-black colouration,
Or

I~YV\3

Add CCI; (or any suitable organic solvent) to each sample and shake the
mixture.
ForB, a r!;lddish-brown organic layer would be observed.
For C, a violet organic layer would be observed.

Let the solubility of Mg(OH)z be x mol dm' 3,


Initial cone. I mol dm'3
change in cone. I mol dm' 3
eqm cone. I mol dm' 3

Mg(OH)z (s) ~ Mg 2' (aq) + 20H' (aq)


x
0
0.05
-x
+x
+2x
0
x
0.05 + 2x

Question 5
(a)
Dot-and-cr~ss diagram of the iodate(V) ion:

1.0 X 10'

11

[Mg]

XX

XX

xQx
X

=X(2x+0,05)

oe
XX

Since a common ion is present, assume that solubility is reduced and x is so


small such that 2x + 0.05 "'0.05, thus
1.ox 1o" x(0.05)'

So x

a .. r a
Xo

K,, = [Mg 2'][0H'] 2

(c)

____J

\____)

(b)

6.3

(b)

l_..]

(b)

2Mg(l03)2(s)---" 2MgO(s) + 2I,(g) + 50 2(g)

(c)

(I)

X(T03)z: Sr(I03):z

Y(T03)2: 8a(l03)z

Z(T03)2: Ca(l03)2

=4 x 1 o moldm'

H,POi(aq) ~ HPO/'(aq) + H'(aq)


When H' is added, the position of equilibrium shifts to the left. There is a large
concentration of HPO.'' ions to cope with addition of H' and the pH is kept
approximately constant.

OR
When pH of blood is momentarily decreased ([H'] increases), the following
reaction occurs.
HPO/'(aq) + H\aq)

->

H,POdaq)

The additional H' ions are removed by the large concentration of HPO/' ions
and the pH Is kept approximately constant.

(ii)

The thermal stability of the Group II iodates(V) increases in the


following order:
Z(!03)2 < X(I03)z < Y(I03)2 or .
Ca(!03)2 < Sr(!03)z < Ba(IOJ)z
The cationic size increases from Z 2 ' to X 2 ' to Y2 '.
This results in decreasing charge density and hence decreasing
polarising power from z2' to x2' to y2',
Thus there is decreasing distortion of the electron cloud of the iodate(V)
anion and hence decreasing weakening effect of the covalent bonds in
the iodate(V) anion leading to decreasing
ease of thermal
decomposition in going from Z(I03)z to X(I03)2 to Y(I0 3)2 Increasingly
more heat energy is needed to decompose the iodates(V) in going from
Z(I03)2 to X(!03)z to Y(!03)z.
Consequently the thermal stability of the Group II iodates(V) increases
in the order as sho:fn above.

__j

~
l
j

Question 6
(II)

(a)

H,C._C::;:;CH,

I
I c('''"H
"c=c
c1
I
\

CH,

(c)

CH,COOH + H20
CsHsOH + H,O

CH,COO .. + H,o
CsHso- + H,o

(Eqm 1)
{Eqm 2)

The acidity of an acid is determined by the stability of-its conjugate base/anion.


In phenoxide ion, the negative charge on oxygen is delocalised into the benzene ring,
hence stabilising the conjugate base.

m rror

In CH 3 coo ion, the negative charge on oxygen is delocalised into the -C=O group
and it results in two equivalent resonance structures.
The ethanoate ion is~ resonance-stabilised than the phenoxide ion.
H

Hence the position of eqm 1 shifts more to the right and produces more H,o ions,
ethanoic acid is a stronger acid.

I II
J-t--C-C-0-H

(b)

Question 9
(a)

R, being an amino acid, exists primarily as zwitterions which are dipolar. Large
amounts of energy are required to overcome strong ionic bonds between the
zwitterions during melting.

Question 7
(b)

(a)

Nucleophilic substitution

(b)

CH,CH,OH + 41, + 60H' ~ HCOO + CHI,+ 5!" + 5H,O


HOOC ---cH---CH2NH3+Br

Yellow ppt.
Br--- bH---CH3

Question 8
{a)

X= electron withdrawing group


.
X nearer to amino group in Q ::::>electron density of N atom Is more decreased ::::>
lone pair of electrons on N atom less available for donation to the proton.
Thus, Q is a weaker base ::::> pKb of Q higher

DOCD2---CH---CH2NH2
bn-=cH2

OR

'

When the -X group is closer to the -NH 2 group, pKb Increases, indicating that Q is
less basic. Since basicity of the amine depends on the availability of the lone pair on
N atom to form a dative covalent bond with a proton, Q has less availability of the lone
pair than P. Thus it can be deduced that -X has a base weakening effect on Q as it
withdraws electrons from the N atom. Thus X is electron withdrawing.
{b)

(I)

Lone pair of electrons on N atom interacts with the n-electrons of the adjacent
C=O bond (and the benzene ring), thus not available for donation to a proton.

l
]

..

-l

2006 RJC Chemistry Preliminary Examination Paper 1 Answers

1.
2.
3.
4.
5.
6.
7.
8.
9.
10.

']

'J
]

8
D
D

A
B
D
D
B

11. A
12. c
13. c
14. D
15. D
16. c
17. A
18. c
19. c
20. c

21.
22.
23.
24.
25.
26.
27.
28.
29.
30.

]
~-1

'J
]
]

J
J
J
]
]

J
-l

8
B

D
B
B

D
A
A

31.
32.
33.
34.
35.
36.
37.
38.
39.
40.

8
A
8
D
8
B

D
8

[
[
[

ANDERSON JUNIOR COLLEGE


PRELIMINARY EXAMINATION 2006
CHEMISTRY (9258/01)

20 SEP 2006

1 hour
Additional Materials:

[
[

Data Booklet
MCQ Optical Mark Sheet

READ THESE INSTRUCTIONS FIRST

Write your name, PDG and NRIC no. on the MCQ optical mark sheet.
Shade the last 5 digits of your NRIC no. only as index number. Do NOT shade any letter.

There are forty questions in this paper. Answer all questions.


f=or each.question there are four possible answers A, B, C and D. Choose the one you consider
correct and record your choice in soft pencil on the MCQ optical mark sheet.

Each correct answer Will score one mark. A mark will not be deducted for a wrong answer.

[
[

[
[
[

L
L
L
[

This document consists of 12 printed pages.

L
;T

Which of the following solutions contains 6.02 x 1023 solute particles?

0.25 dm 3 of 1.0 mol dm-~ potassium hexacyanoferrate(lll) solution

0.50 dm 3 of 2.0 mol dm-3 sodium nitrite solution

0.50 dm3 of 2.0 mol dm-3 potassium dichromate solution

1.00 dm 3 of 1.0 mol dm-3 potassium bromate(V) solution

-1

-2

+2

[
[

In an experiment, 100 cm 3 of 0.020 mol dm-3 acidified iron(ll) sulphate is


quantitatively oxidised by 50 cm 3 of a solution containing 0.020 mol dm-3 of an
oxidising agent, X0 3-. What is the final oxidation state of X?

[
[

+3

[
3

To identify an oxide of nitrogen, 0.10 mol of the oxide was mixed with 10 dm 3 of
hydrogen gas and passed over a heated catalyst. At the end of the reaction,
0.4dm3 of hydrogen gas remained. The ammonia produced required 125 cm 3 of
1.6 mol din-3 HCI for neutralisation. All gaseous volumes were measured at room
temperature and pressure.
. What is the formula of the oxide of nitrogen?
A

NO

N20

N02

N204

L
[
f~

The diagram below shows the mass spectrum of an amine.

,J

71

%abundance
15

29

43

m/e
20

40

60

80

100

Which of the following molecules will produce this spectrum?


A

CH3(CH2)4NH2

CH3CH (NH2)( CH2hCH3

CH3CH2NH(CH2)2CH3

CNH

[
[~

['

L
L
I

l'

[
[
6

Which of the following is the correct arrangement of the ions in terms of increasing
ionic radii?

p3--

<

sz-

<

cr

cr

<

K+

<

Ca 2 +

Ca 2..

<

sr-

<

As 3-

Mgz+

<

p3-

<

Na+

Which of the following alkanes is the most volatile?

[
[

The shapes of three species X, Y and Z are trigonal pyramidal, square planar and
bent respectively.
Which of the following represents X, Y and Z?

_
_,
l
-

NH3

IFs

HzS

PCI 5

BCb

ICI 2-

SbF 3

IC14-

SnCh

NCI3

CH4

COz

Some standard redox potentials are given below.

[
[

It is expected that

L
L

Brz I srFe3+ I Fe 2 +
lz I r
Co 2 + I Co

v
v
v
-0.28 v
+1.07
+0.77
+0.54

Cobalt will be formed when iron(lll) chloride is added to cobalt(ll) nitrate


solution.

Cobalt will be formed when potassium iodide is added to cobalt(ll) nitrate


solution.

Bromine will be formed when iron(! II) chloride is added to aqueous


potassium bromide.

Iodine will be formed when iron(! II) chloride is added to aqueous potassium
iodide.

L~

l-

..i~

I.

4
9

Use of the Data Booklet is relevant to this question.


Under high pressure and in the presence of a catalyst, benzene undergoes
hydrogenation to form cyclohexane in the reaction shown.

A
+
~(g)

3H2(g)

(g)

c
[

What is the enthalpy change of hydrogenation of benzene in kJ mol- ?

A
10

-132

-1152

3 Y(g)

the numerical value of the equilibrium constant Kp when the partial pressures are
measured in kPa is 1.50 x 106 at 500 K and is 1.60 x 107 at 600 K.

Which of the following statement is incorrect?

The forward reaction is endothermic.

The proportion of X in the equilibrium mixture is decreased by decreasing


the pressure.

The proportion of Yin the equilibrium mixture is increased by adding a


catalyst.

The units of Kp is kPa.

[
r.-
L

The solubility product of aluminium bromide is x. Which of the following represent


the numerical value of solubility of aluminium bromide?

12

-408

For the reaction


2 X(g)

11

-209

i><
1J27

lx

1JB1

Initial total pressure I atm

1.00

1.60

2.00

Initial pressure of NO I atm

0.40

0.40

0.80

Initial rate of reaction I atm s- 1

1.08

2.16

8.64

Experiment

Which of the following statements is true regarding the above system?

The reaction is first-order with respect to NO

The overall order of the reaction is 3


2

The rate equation is rate = k[N0]

The rate constant k has units of atm s- 1

<~lx
~27

The following data were obtained from studies of the reaction between NO and 0
in a vessel at constant temperature.

[
[
[

L
L
[

l.
l

5
13

A simple decomposition reaction is shown below.


AB(g) -7 A(g} + B(g)

The rate equation was found to be as follows.

Estimate the time taken for AB(g) to reach one third of its initial concentration of
1.5 mol dm- 3 .

rate

14

3.0 s

=k[AB]

where rate constant, k

3.5s

=0.2 s-1

5.5 s

7.0s

In which one of the following crystalline solids does the element exist as small
molecules?
A

sulphur

silicon

carbon

sodium

[
15

[
[

Which of the following oxides is best classified as a basic oxide?

aluminium oxide

beryllium oxide

carbon dioxide

magnesium oxide

"-"

-~'

16

['

How do the following trends change as Group II is descended?

reducing power

solubility of hydroxides

" magnitude of 6.Hhydration of cations

L
l
[

L
r

Reducing power

Solubility of
hydroxides

Magnitude of 6.Hhydration of
cations

decreases

increases

increases

increases

decreases

decreases

decreases

decreases

increases

increases

increases

decreases

l
[
6

17

When a white solid X is strongly heated, a solid residue Y is formed, alongwith


brown fumes and a colourless gas. When water is added to Y, the pH of the
solution formed is approximately 7.
Which of the following could be the correct identity of solids X and Y?

18

19

Solid X

Residue Y

MgC03

MgO

Mg(N03)2

MgO

SrC03

SrO

Sr(N03)2

SrO

phosphorus

potassium iodide

sodium hydroxide

ethene

[
[

Which one of the following substances reacts with chlorine to give two products in
which chlorine has different oxidation numbers?
A

[
[

An aqueous solution containing three transition metal ions, P 3+, Q 2+ and R2+ was
treated as follows:
I

excess NaOH,
filter

residue
mixed precipitates from Q 2 ... and R2+

soluble light green


complex from P 3+

excess NH 3(aq)
filter
residue

filtrate
soluble deep blue
. complex from Q 2+

green precipitate from R2+

on standing

red-brown precipitate
Which of the following correctly identifies P3... , Q 2+ and R2...?
p3+

Q2+

R2+

Cr3 +

Cu 2+

Fe2+

Cr3+

Cu 2+

Ni 2+

Fe 3 +

Ni 2+

Fe2 +

Fe 3 +

Ni2 +

Cu 2 +.

Aqueous solution of P3+, Q 2+and R2+

filtrate

[
[

u
u
u
u
L:

L
I

7
20

How many isomers (including both structural isomers and stereoisomers) are
, possible for a straight chain hydroGarbon that contains 85.7% carbon and has a
relative molecular mass of 70?

[
21

11

The reaction of benzene with iodine chloride, ICI in the presence of a suitable
catalyst gives

<Q)-c1
<Q)-1

a mixture of

a mixture of

g-el

Q-cl
.

<Q)-1

and

I~CI

and

[
22

Acarol is used as an insecticide for fruits and vegetables.

OHO

---~-1 II
Br~C-C-O-C(CH3)3

6
V

[
[
[

Which of the following reagents could bring about a nucleophilic substitution


reaction in Acarol?

[
[

L
L
L
L
L

Acarol

Br

23

aqueous bromine

phosphorus and iodine

potassium cyanide

aqueous sodium hydroxide

When aqueous bromine is added to phenol, a white precipitate is obtained.


What does this reaction show?

Phenol is acidic.

Phenol is unsaturated.

2-bromophenol is insoluble in water.

A hydroxyl group makes the benzene ring more susceptible to electrophilic


attack.

t"

[
8
24

The three formulae P, Q and R shown below are compounds which are important
flavours in citrus fruits. These compounds are much used in the food and perfume
industries.
CH 3

CH3

H2c--" ~CH
1
I
H2c_...._
CHO
CH2

CH~ ~CH2

dH
H2<Y 'CH

I 2

CHO
CH2

H2G....._

I 2
CH20H
CH2

CH~ ~CH2

CH3

dH
'CH

<Y.

21
H2G....._

CH~ ~CH2
R

[
Which of the following statements concerning P, Q and Ris true?

25

Q and R exhibit stereoisomerism but P does not.

Reaction of P with hot, acidified KMn0 4 gives two organic products.

R can be distinguished from P and Q by using phosphorus pentachloride.

P can be converted to R by using lithium aluminium hydride in dry ether.

The table shows the results of simple tests on two compounds X and Y.

result

reagent

X
positive
positive
positive

2,4-dinitrop]1enylhydrazine
Tollens' reagent
aqueous sodium carbonate
'-alkaline aqueous iodine

_ _

n_eg~tiv)

positive
negative
negative
QOSitive

-----

[
f~

[
[

From the results of the tests, X and Y could be

26

A CH3COCH2C0 2H

CH3COCeHs

H02CCH2CH2C02H

CeHsCOCsHs

H02CCH2CH2CHO

CH3COCH3

CH3CH2C02H

CH3CH2CHO

[
[

Consider.the four compounds below:


CH3C02H

CH3COCI

II

CH2CIC02H

CHCI2C02H

Ill

IV

Which of the following sequence arrange the compounds in increasing pKa?

I, IV, Ill, II

IV, Ill, I, II

II, IV, Ill, I

I, Ill, IV, II

L
L

L
l_~

L_

r-.

27

Which procedure gives the best yield of ethyl butanoate?

[
[

28

[
[

[
29

[
L.

L.
[-

reacting CH3CH 20H with SOCI2 , then adding CH3(CH 2)zC0 2 H

refluxing CHs(CH 2 )zC0 2 H with aqueous NaOH, then adding CH 3CH 2 0H

re~luxing

CH 3CHzOH with concentrated H2S04, then adding CHs(CH 2)zC0 2H

Which of the following compounds gives


(i)

fumes of HCI with PCis

(ii)

NHs when heated with NaOH(aq)?

HOCaH4CONHz

CHz(OH)C 6 H4CONHz

HzNCaH4COzH

HzNCeH4CHzOH

Partial hydrolysis of a tetrapeptide (containing four amino acid residues) produces


the following three dipeptides, as well as the individual amino acids.

1st

reacting CH3(CH 2)zC0 2 H with SOCb, then adding CH3CH 2 0H

Which of the following is the correct order in which the amino acids are bonded
together in the tetrapeptide?

NHzCHzCONHCH(CH 3 )COzH
NHzCH 2 CONHCH 2 C0 2 H
NH 2 CH(CH 3)CONHCH(CH 3)C0 2 H

Esters of carboxylic acids are often used as components of synthetic fruit flavours.
Ethyl butanoate is a major ,component of strawberry and pineapple flavourings.

30

2nd

NHzCHzC0 2H

4th

NHzCHzCOzH

NHzCH(CH3)COzH

NHzCH(CH3)COzH NHzCHzCOzH

NHzCH(CH3)COzH

NHzCHzC02H

NHzCH(CH3)COzH NHzCH(CH3)COzH

NHzCHzCOzH

NHzCHzCOzH

NHzCHzCOzH

NH2CH(CH3)COzH

NHzCH(CHs)COzH

Epoxyethane

-H

NHzCH(CH3)COzH

3rd

NHzCHzCOzH

2 ~CH 2

can undergo addition polymerization.

What is the repeat unit of the polymer formed?

o
A

I\
-HC-CH--

-CH2-G-CH2-o-

-CH2-CH2-o-

- . CH-CH1
I
OH OH

l'
[
10
For questions 31 to 40, one or more of the three numbered statements 1 to 3 may be
correct.
Decide whether each of the statements is or is not correct (you may find it helpful to put a
tick against the statements you consider correct).
The responses A to 0 should be selected on the basis of

[
[

31

The following table gives the axes of graphs plotted for one mole of an ideal gas.
Which of the graphs will give a straight line that passes through the origin?
x-axis

y-axis

condition

pressure

temperature (K)

at constant volume

volume

temperature (C)

at constant pressure

pressure

volume

at constant temperature

32

33

[
!

When an ionic solid dissolves in water,

energy is released when the ionic solid dissolves completely.

energy is absorbed to separate the ions in the solid.

energy is released when the ions are hydrated.

When copper is electroplated with silver, a solution containing both silver nitrate
and potassium cyanide, KCN, is used. The process involves the sequence shown
below.
stage 1
stage 2
Ag(CN)z~
Ag+(aq)
Ag
Which of the following statements are true of this sequence?

34

35

The cyanide ions reduce the concentration of aqueous silver ions.

The copper object will be the cathode.

Both stages 1 and 2 involve a change of oxidation number.

Which of the following statements is/are most likely to be true for astatine, the
element below iodine in Group VII of the Periodic Table?

Silver astatide is insoluble in concentrated aqueous ammonia.

Astatine oxidises potassium chloride to chlorine.

Astatine is more electronegative than iodine.

Which of the following trends across Period 3 (Na to Cl) is/are always true?
1

The electronegativity of the element increases.

The bonding in the oxides changes from ionic to covalent.

The first ionisation energy cif the element increases.

[
[

[
[
[

[
[
[

L
['

L_

11

[
[

36

The table below shows selected information about two metals, Wand Y.
Which of the following properties can best sho\1\/ that W is a transition metal and Y
is not?

Property

ElementW

Element Y

Colour of +2 complexes

Violet

Colourless

Possible oxidation states

+2,+3,+4,+5

+2,+4

E" (M(II)/M) IV

-1.20

-0.13

37

[
[
[

38

Which statements regarding the NMR spectra of isomeric esters with molecular
formula C4H80z are correct?

Each spectrum shows at least three signals.

Each spectrum has a singlet signal.

Only one spectrum has a doublet signal.

Dopamine is a neurotransmitter: its absence from the human brain might lead to
Parkinson's disease.

/7-P-

yOzH
HO\\,~) -CHzyH
HO

NHz
Dopamine

Which of the following could be obtained- by the reaction of dopamine with


chlorine?

HO

[
[

HOYCH,CCI(NH,)CO,H

H~CH,CH(CI)CO,H
HO

r-

L
[

HYCH,CH(NH,)CO,H
Cl

l
['
12

'[
[
39

2,4-dinitrophenylhydrazine, which has the following structure would be expected to


react with

02~
H2N-N~N02

40

benzaldehyde

dilute hydrochloric acid

tin and concentrated hydrochloric acid

Which of the following compounds could be formed by the reaction of HOCH2C02H


with itself?

0
II
HO-CH2-C-O-CH2-C02H

[
[

H2~~0

OYO'CH2

oII

oII

C-Q-CH2-o-C-CH2

l_~

[
[

u
lJ

L
I

I.

l: .:
[
[

ANDERSON JUNIOR COLLEGE


PRELIMINARY EXAMINATION 2006
20 SEP 2006

CHEMISTRY (9258/02)

1 hour 30 minutes
Additional Materials:

Data Booklet

[
[
[

READ THESE INSTRUCTIONS FIRST


Write your name and PDG on the cover page.
Answer all questions in the space provided.
You are reminded of the need for good English and clear presentation in your answers.
The number of marks is given in brackets [ ] at the end of each question or part question.

PDG:

Name:

[
[
[
[

L
L

'.-. ';tr,ti~:~~~iHi~F~ !J~~-- :~- :\':' :{ _': J


9m

7m

ill
~

8m

7~3m

Total
(60 marks)

I-

L
\L

L
L

This document consists of 10 printed pages.

I
I

)
2

Carbon dioxide and methane are two greenhouse gases. They trap infrared radiation
emitted from the Earth's surface and as a result warm up the planet.
(a) (i) Two gas cylinders containing 5 dm 3 of carbon dioxide at a pressure of 8 atm
and 10 dm 3 of methane at a pressure of 2 atm were transported separately to a
chemical plant for use in some industrial syntheses. Upon arrival, it was found
that the valve connecting the two gas cylinders was opened and the two gases
had been mixed.
Assuming that there was no increase in temperature and no chemical reaction .
between the gases, calculate the final total pressure.

[
[
[
[
[
[

[
[2]
(ii) The plots of PV/RT against P for one mole of an ideal gas and one mole of
carbon dioxide at 200K are given below. Show, on the same axes, how one
mole of methane will behave at the same temperature of 200K.
_Label your graph clearly.

PV/RT

I
~o2 <2oo~)
I

l .

'!

. ____. _. . . . . . . _____1I_____________. . . . . .l1. _______-,/...______.............._. .________. . . . . . . . . . . .-----j1


1

I
I

I/

1.0 .................... ......... --- . ......................... . . .....

l
I
I
i
................. - .. -------- . .............. . . . . . ......... -------------1

'

Ideal gas (200K)

I
_______\ , _____________ -----1---.............._______Il,._______________,_ -----------------
\

I
\

', ;/
0.0

_____. _____,.__T_______. . ..,------------------T----------------.. . . ______. ______1


I

p
[1]

(iii) Explain the difference in behaviour, if any, between methane and carbon
dioxide at 200K.

[
L.

[
_ _ _ _ _ _ _ _ _ _ ___;__ _ _ _ _ _~_[1]

L
I

3
(iv) What happens to the graph for carbon dioxide if the gas is heated to 300K?
[1]
lllustrate your answer clearly on the same axes in a(ii).

(v) Explain the difference .in the behaviour of carbon dioxide at the two
temperatures.

[
[

_ _ _ _ _ _ _ _ _ _ _ _ _ _ _ _ _ _ _ ,[2]

(b) It is estimated that doubling the carbon dioxide concentration will cause a 3C rise
in global temperature.

Assuming the estimated annual rate of increase in atmospheric carbon dioxide


continues at 0.4% for another two years, calculate the increase in global
temperature due to carbon dioxide after two years.

[
[

[2]
[Total: 9]

[
2

(a) The nitrogen molecule is isoelectronic with the carbon monoxide molecule.

(i) Compare and explain the bond energy of the nitrogen-nitrogen bond with that
of the carbon-oxygen bond in the respective molecules.

L
_ _ _ _ _ _ _ _ _ _ _ _ _ _ _ _ _ _ _ [2]

l
l
L
L

(ii) Suggest which substance, nitrogen or carbon monoxide, has a higher boiling
point. Explain your reasoning.

_ _ _ _ _ _ _ _ _ _ _ _ _ _ _ _ _ _ ____:__ [2]

L
I

4
(b) Oxides of nitrogen contribute towards air pollution. For example, nitrogen dioxide
is known to cause acid rain.
(i) Draw a dot-and-cross diagram to show the electronic structure of a molecule of
nitrogen dioxide.

['
[1]

(ii) Under completely different conditions, the nitronium ion, NO/, and the nitrite
ion, NOz- can be formed. Draw and state the shapes of NO/ and N0 2-.

f
[
[

c
[2]
[Total: 7]

.3

Consider the following equilibrium at 80C, -PH38CI3(s) ~ PH 3 (g) + BCI 3 (g),

.1.H

=x kJ mor

(a) (i) Describe the types of bonding present in a molecule of PH 3 BCb.

_ _ _ _ _ _ _ _ _ _ _ _ _ _ _ _ _ _ _ _ [1]

0
n
u
l

__ j

(ii) Comment on the sign and magnitude of x for the equilibrium.

_ _ _ _ _ _ _ _ _ _ _ _ _ _ _ _ _ _ _ _ _ [2]

L
L
[

L
l

t
r~
5

(iii) Explain how the concentrations of PH3 and BCb will be affected if some
PH 3 8CI 3 solid is a"dded to an equilibrium mixture of the three compounds at
80C.
"

[
[

[
------------------------------~-------[2]

(b) (i) Given that the numerical value of the equilibrium constant Kc for the above
eqUilibriUm iS 1.87 X 10-3 at 8QC, CalCUlate the equilibriUm COncentratiOnS Of
PH 3 and BCI 3 if a solid sample of PH 3 BCI 3 is placed in" a closed vessel and
decomposes until equilibrium is established.

[
[

r
"

[2]

(ii) If the flask has a volume of 500 cm 3 , what is the minimum mass of PH 3BCI 3
solid that must be added to the flask in order to achieve the equilibrium at
80C.

c
r
""

L
L
r-

L
L
L

[ 1]
[Total: 8]

Group II elements and their compounds find many uses in industries. When coal is
burnt in power stations, a Group II carbonate, XC0 3, is added to remove atmospheric
pollutants like gaseous oxides of c~rbon and sulphur. The high temperature of burning
coal causes the decomposition of XC0 3 at 900C.
(a) (i) Write a balanced equation for the thermal decomposition of XC03.

[1]

J'

6
(ii) When a 1Og sample of XC03 is heated strongly, it loses 44% of its mass.
Identify the metal X, showing your working clearly.

L
L
[
[
[

[2]
(b) Another Group II carbonate, YC0 3 , decomposes at 1290C. Predict the position of
element Y relative to element X in Group II and explain the difference in the
decomposition temperature of their carbonates.

[
[
[

_ _ _ _ _ _ _ _ _ _ _ _ _ _ _ _ _ _ _ _ [3]

(c) Write a balanced equation to show the reaction between XC0 3 and sulphur
dioxide.
[1]
(Total: 7]

(a) Chromium oxide, Cr0 2 , is a synthetic magnetic material once widely used in
magnetic cassette tapes. It produces a solution containing Cr3-+and Cr2 0/- ions
when dilute sulphuric acid is added to it.
(i) State and justify the type of reaction undergone.

[
['
-~

[J

u
f

[2]

(ii) A solution containing Cr2ol- ions is prepared and sodium hydroxide is added
to it. Describe, with the aid of an equation, what would be observed.

[2]

!:

.I

L
L

L
I

7
(b) Various chromiulll complexes can be obtained by combining Cr3+ ions with

different ligands_, .

(i) What do you understand by the term ligand?

[
[

----'-------------------_,[1]

(ii) Three unknown chromium (Ill) complexes, A-C, in which the coordination
number of chromium is 6, have been prepared. Different proportions of
chloride are precipitated when each of the complexes is treated separately with
aqueous silver nitrate.

[
[

c
.)

Empirical formula

Colour

Number of moles of AgCI


precjQitated_Qer mole of complex

CrCI 3(H20)e

Violet

CrCis(H20)5

Pale green

CrCis(H20)4

Dark green

1
---

--

----

Giving your reasoning, suggest a structure for each of the complexes A-C.

[
[
[
[3]
[Total: 8]

The refining of crude oil produces hydrocarbons which are used to manufacture a wide
range of products. Ethene, propene and butene are examples of such hydrocarbons.
The diagram below shows how propene and other hydrocarbons can be obtained form
crude oil.

L
L
L
L
L

Other Products

}'

l
[

(a) Process A produces propene and two other organic products.


What is process A and what are the conditions required? Write the structural
formulae of the other two products formed in the process.

Process A: ________________________________________________

Conditions:--------------------------Structural, formulae of the other two organic products:

[
[

[4]
(b) Propene is an important monomer used in the manufacturing of poly(propene).
One of the uses of poly(propene) is for making mesh used in surgery. The mesh is
inserted below the muscle tissue so that on healing the wound is less likely to reopen and the repair is stronger.

r
[

(i) Draw the displayed formula of poly(propene), giving two repeat units.

(ii) To which homologous series does poly(propene) belong?

Homologous s e r i e s : - - - - - - - - - - - - - - - - - - - - - -

(iii} Suggest two reasons why poly(propene) is used for surgical mesh rather than
polyester or polyamide.

[
[

[
[4]
7

[Total: 8]

(a) The psychoactive drug in cannabis is soluble in non-polar solvents. It can


accumulate in the fatty tissues of the body and be released into the blood stream
over a period of seven days. It has the structure shown below:

HO

t~
CH2CH2CH2CH2CH3
\.

HO
C-CH3

I;

H2C

-v-

(R)

_/

L
L
l
I

r:

(i) .Name two functional groups present in the drug.


(1) _ _ _ _ _ _ _ __

[2]

(2) - - - - - - - - - -

(ii) Explain why the drug is able to dissolve in fatty tissues.

[
_ _ _ _ _ _ _ _ _ _ _ _ _ _ _ _ _ _ _ _ [1]

[
(iii) By means of an asterisk (*), identify any chiral centre(s) in the structure of the
drug above.
[1]

(iv) In the boxes below, draw the structure of the organic compound that is formed
when the drug reacts with each of the following reagents:

You may useR to represent the -CH2CH 2CH 2CH 2 CH 3 group in your answer.

c
[
"

Bromine in CCI 4

[
[
[

Aqueous NaOH

r
[
[
f

l
[

Cold alkaline KMn04

"c

10

Dilute HN03

[
(b) Suggest methods by which the following pairs of compounds could be
distinguished from each other by chemical tests. The distinguishing of some of
these pairs may rely on a preliminary breaking-up of the compounds, and
subsequent testing of the reaction products. Do not use the same test or
spectroscopic techniques for parts (i) and (ii).

0
(i) W:

_)l .

ouCH3

and

X:

H,C'(yo

Test and observation:

[J
[j
_ _ _ _ _ _ _ _ _ _ _ _ _ _ _ _ _ _ _ _ _ [3]

NH2

(ii) Y:

-CONHCH

c
r:
-~

and

Z:

6COCH,

Test and observation:

_ _ _ _ _ _ _ _ _ _ _ _ _ _ _ _ _ _ _ _ _,[2]
[Total: 13]

L
L
[
l.

[
[
ANDERSON JUNiOR COLLEGE
PRELIMINARY EXAMINATION 2006

CHEMISTRY (9258/03)

12 SEP 2006

2 hours 45 minutes
Additional Materials:

[
[

Data Booklet
Writing Paper
Graph Paper

READ THESE INSTRUCTIONS FIRST


Write your name and PDG on the cover page and on all the work you hand in.

Answer all questions in Sections A, 8 and C. The last question in each section is of the form either/or.
In Section D, you must answer both questions from the Chemical Thermodynamics Option.

c
[
[

Begin your answer to each question on a fresh sheet of paper.


You are reminded of the need for good English and clear presentation in your answers.
At the end of the examination, detach this cover page from the question paper and staple your
answers, arranged in order, behind the cover page.
The number of marks is given in brackets [ ].at the end of each question or part question.

-~

Name:

PDG:

FQr,~x:am~i~~r'ir'u~~ T~\

Section A I

2
3 either I or

-4

Section 8

5 either I or

SectionC

~
8 either I or

-9

[
[

..
l

SectionD~
10

Total
(100 marks)

This document consists of 13 printed pages and 1 blank page.

~'

[
~~

OJ

)>

.,..,
"0

.m

'

,...._._..,
I
I
'
''

r--1
I

f)

[
[

SECTION A

Answer all questions in this section.

1 (a) One type of rechargeable battery used in electric cars makes use of the nickel-zinc
cell, in which the nickel and zinc electrodes, coated with their respective hydroxides
are immersed in a suitable electrolyte.

The relevant half cell potentials are:

[
[

"'

['

EG (Zn(OHh(s) I Zn(s))
EG (NiO(OH)(s) I Ni(OHh(s))

-1.24V

+0.49 v

{i)

Construct half-equations for the reactions at the anode and cathode and hence
write a balanced equation for the overall reaction that occurs during discharge.
Calculate the value of E9 ceu for this cell reaction.

{ii)

Suggest a suitable substance to use as an electrolyte.

[4]
(b) A student attempts to carry out an experiment to coat a flower petal with copper in
the laboratory. The petal has to be coated with a thin layer of graphite paste, before
placing it in a bath of aqueous copper(! I) sulphate.
(i)

Suggest why the article is first coated with graphite.

(ii)

Draw a labelled diagram of the electrical circuit for the electroplating, showing
clearly the polarity of the electrodes, and the direction of electron flow in the
external circuit.

(iii)

A surface area of 5 cm 2 is to be electroplated. How many coulombs of


electricity will be required to increase its copper layer by 0.1 mm in thickness?
(Density of copper is 8.92 g cm- 3)

(iv)

How may the thickness of copper be increased without changing the duration
of the electroplating process?
[6]

"-'

[
[
[
[

L
L
['

L
[
[

[Total: 10]

t
'"
l

4
2

An engineering consultation firm is engaged to design a chemical plant to produce a


top selling colour-treating solution, Wondersol.
The production of Wondersol
involves the acid buffered oxidation of iodide to iodine by hydrogen peroxide in one
of the many steps in the synthesis process.

1JX + H202 + 2H+

~ l2 + 2H 2 0

In order to set the optimum operating parameters and size of the reactor for the
production plant, two experiments were carried out to determine the rate of the
oxidation reaction. The results are as follows:

c
[

[
[

Time/ sec

f>")~~--~~-::0.1:::'~

::.:

i'}_":c:':".~j!'~:;..z. '-!;c:;,~.a;":l::,::~:.--,~~-

,;:. .:..~-".:.-,",;;..;:-; :v,-, -:--;"''=''"'-.-;._..:;,::-;.;,::-;!'_,-,..:..;-;:;-,o;._.:!.":,-,.c;!.,"'"'d:i.:l:::-"""

[KI] I mol dm-3

[KI] I mol dm-3

0.200

0.200

25

0.149

0.103

50

0.112

0.054

75

0.085

0.031

100

0.063

0.015

[~

125

0.047

0.003

~~

(a) Suggest how you could conduct the experiment to obtain these sets of data in the
laboratory.
[2]

L
c

. (b) Using the same axes, plot the graphs of [KI] against time for the two experiments. [2]

(c) Assuming that the acid buffer does not affect the rate of oxidation, use the graphs to
determine the order of reaction with respect to hydrogen peroxide and to iodide ion.
Show your working clearly.
Hence write the rate equation for the oxidation reaction.

[
[

[4]

(d) Based on your answers in (c), state what conclusions can be made regarding the
reaction mechanism of the oxidation of iodide to iodine by hydrogen peroxide.
Equations for the mechanism are not required.
[2]

[Total: 10]

L
L
L
l~

L
\

3 either
(a) The graph below shows the variation of pH when 0.1 mol dm-3 HCI is added to 25.0
cm 3 of 0.1 mol dm-3 aqueous ammonia.

arlation of pH when 0.1

Initial pH

= 11.13

M of HCIIs added to 25 cm3


of 0.1M NH 3 (aq)

"l . - - - __

10

89

- - - -

------~----'----!---'--

r~=-~=:=~~==

-~~~~~~~-;~~~~~~~---~~~~~~--~~:~=~~
--===-=--------

:t=~~----------f-

-------

--------"---

------'\----

5
4

W'
-'
-'
--""7"--1
~-----~-----~--tr-1
=~~~=::-:.:=====~~---; . : !
--~=~~-:

1o

20

30

Volume of HCI added (om

40

so

(i)

Calculate the base dissociation constant, Kb, of ammonia.

(ii)

Calculate the pH of the mixture when 12.5 cm 3 of HCI is added to 25.0 cm 3 of


0.1 mol dm-3 NH 3 (aq).

(iii)

From the graph, estimate the pH of the mixture at the equivalence point.
Justify its value with the aid of an equation.

(iv)

Suggest a suitable indicator for the above titration.

Jt

____________,____

-1---------

lf

+---------- .....:....----+-

[7]

:,;r;::.-~'

(b) The major biological buffer to regulate pH in our blood is the H2COs I HC0 3- mixture .
. The pH of normal blood ranges from 7.3 to 7.4.
(i) ,

Write an expression for the acid dissociation constant, Ka, of H2C0 3.


In a buffer of pH= 7.35, calculate the value of [H 2 C0 3] / [HC0 3-] if Ka is 10-6 37 .

(ii)

Wl'lat does your answer in b(i) suggest about the buffer capacity of blood?

[3]
\

[Total: 10]

\\
\

'.:.

3 or
(a) Conversion of S0 2(g) to 80 3(g) may be effected by a two-step sequence involving

oxides of nitrogen.
2 NO(g) + 02(g)
N02(g) + 802(g)
(i)

~
~

2 N02(g)
NO(g} + 803(g)

Find a value for the enthalpy change of the following reaction:


802(g) + Y2 02(g)

(ii)

L'l.H =- 114.4 kJ mor 1


L'l.H -41.7 kJ mor1

S03(g)

Hence, with the aid of an energy level diagram, deduce whether the formation
of 80 2(g) or the formation of S03 (g) is more exothermic.
[3]

(b) 803(g) dissolves exothermically in water to form sulphuric acid, H2804 (aq). The
standard enthalpy change of neutralisation of sulphuric acid can be determined
experimentally using a simple insulated calorimeter.
When 50 cm 3 of sulphuric acid and 100 cm 3 of NaOH, each of concentration 1.0 mol
dm-3 , are mixed in a polystyrene cup, the temperature rises by 9.7C.

(i)
(ii)

What do you
neutralisation?

understand

by the term standard enthalpy change

of

Assuming that all the solutions have a density of 1.0 g cm-3 and a specific heat
capacity of 4.2 J g-1 K- 1 , calculate the standard enthalpy change of
neutralisation of sulphuric acid.

[3]

(c) The enthalpy changes of neutralisation of 1 mol of sodium hydroxide by various


acids are as follows:

hydrochloric acid
nitric acid
ethanoic acid

-'57.3 kJ mol- 1
-57.3 kJ mol-1
-55.2 kJ mor 1

Explain the above observation as fully as you can.


Comment on the value you obtained for sulphuric acid in b{ii).

'

-,

[4]
(; [Total: 10]

......

SECTION B

Answerall questions in this section.

The following table lists some cobalt complexes together with their colours and
stability constants.

Complex

Kstab

Colour

[Co(H20)s] 2+(aq)
[Co(NH3) ] 2+(aq)

pink

[Co(NH3)s]3+(aq)

(Co(edta)f (aq)
(Co(edta)r(aq)

7.94 X 104

pale brown

33

. blue

2.00 X 10

16

r:::~Jr'\~'-,_- .,.--=.iF' -~"'~:=:~.;~Fe

1.00 X 10

36

5.01

10

---

[edta = COzCCH2)2NCH2CH2N(CHzCOz~h]

(a) What do you understand by the term stability constant,

Kstab.

of [Co(NHa)sf+?

[1]

(b) Using the data in the table, predict and explain

c
-"

[~
-'

(i)

the change in colour on adding excess aqueous NH 3 to aqueous Co(ll)


soiution.

(ii)

the effect on E0 value for the Co(III)/Co(ll) equilibrium on adding edta.


[4]

(c) State a feature of transition metal that makes Co 2 ... suitable as a catalyst but not
Ca 2+. Hence, using suitable E0 values from the Data Booklet, explain how Co 2+
catalyzes the oxidation of r by 8 2 0 82 -.
[3]

(d) It is known that Co(lll) ions are potentially unstable in water. Calculate the E

[
[

L
L
L
[

L
L

write a balanced equation for the reaction you predict will occur.

ceu

and
[2]

[Total: 10]

L
...
8
5 either

The table below shows some properties of certain Period 3 chlorides.


NaCI

MgCb

AI Cia

PC Ia

melting point/ C

801

700

178

-92

pH of a 1.0 mol, dm-3 solution in water

7.0

6.5

3.0

(a) Suggest how NaCI and PCI 3 differ in their bonding and give a reason for this.

(b) (i)

(ii)

[
[

[
[
{2]

By quoting suitable data from the Data Bookle.t, explain why aqueous solutions
of NaCI and AICI3 differ so greatly in pH. Write balanced equations where
appropriate.
Describe and explain what you would observe when water is gradually added
to PCI3, until in excess. Suggest the pH of the resulting solution.

[
[
[-

[5]
(c) Sulphur and chlorine can react together to form S2 CI 2
When S2CI2 is reacted with water, a yellow precipitate is formed, together with a
solution containing a mixture of sulphurous acid, H2 S0 3, and hydrochloric acid.

['
['

(i)

Write a balanced equation for the reaction between S2CI 2 and water.

(ii)

How many moles of sodium hydroxide would be needed to neutralise the


solution formed by adding 1 mole of S2CI 2 to an excess of water?

(iii)

Draw a 'dot-and-cross' diagram to show the electronic structure of a molecule


of S2CI2.
[3]

[]

[J

[Total: 10]

lJ

~J

L
[
[

l
L
[

[
[

5 or
(a) Use relevant data from the bata Booklet to describe and explain how the thermal
stability of the hydrogen halides varies down the group.
[3]

(b) (i)

Hydrogen chloride may be prepared by the action of concentrated sulphuric


acid on potassium chloride, but a corresponding method may not be used for
hydrogen iodide.
How do you account for the difference?
appropriate.

[
(ii)

Write balanced equations where

Hydrogen iodide is however commonly prepared in the laboratory by the action


of phosphoric(V) acid, H3P0 4, on potassium iodide.
In the light of your answer to (b)(i), suggest why phosphoric(V) acid can be
used to produce hydrogen iodide from potassium iodide. Write a balanced
equation for the reaction in which potassium dihydrogen phosphate(V) is one of
the products.

[5]

[
[
[
''""
__,

[
[

[
[

l
[
[,

L
r.~
~~

(c) When an excess of liquid chlorine is added to iodine a reaction occurs. When the
unreacted chlorine is allowed to evaporate an orange solid of formula ICix remains.
One mole of ICix reacts with excess potassium iodide solution to liberate two moles
of iodine.

(i)

Write an equation for the reaction between ICix and iodide ions.

(ii}

Deduce the value of x.


[2]

[Total: 10]

L
<;
10

Answer all questions in this section.

6 (a) The cyanide ion can react with both bromoethane and ethanal under different
conditions.
(i)

Describe the reagents and conditions needed for each of these reactions.

(ii)

Name and outline the mechanism of the reaction between the cyanide ion and
ethanal. Explain why the product does not exhibit optical activity.

(iii) Unlike bromoethane, 2-bromobutane contains a chiral carbon atom. Comment


on the optical activity of the product formed when 2-bromobutane is reacted
with the cyanide ion instead.

[7]
(b) Show how propanoic acid can be synthesized from ethanal. State the reagents and
[3]
conditions required and show the structure of all the intermediate compounds.

[Total: 101
Procaine was the first injectable local anesthetic used during surgery and other
medical and dental procedures.

o~C
H2

A molecule of procaine contains two nitrogen atoms, both of which can act as a
base by accepting a proton. Copy the structure of procaine and circle the
nitrogen atom which you think is the stronger base? Explain your reasoning.

(ii)

Draw the structural formula of the salt formed by treating one mole of procaine
with one mole of HCI.
[3]

(b) Procaine can be made by the following reaction scheme:

N02

C02CH2CH2CI

Ill

N02

r
c

c
ll
[]

[]

procaine

(i)

N02

[]

(a) Procaine is usually marketed as the hydrochloride salt.

HI '11

SECTION C

procaine

l'
_,

NH2

State what reagents and conditions are used for each of the stages I to Ill.

C02CH2CH2CI

[3]

I_,

[
[
[
[
[
[

[
[
[
[
[

L
[
[

[
[
[

L
l

11
(c) Procaine has a half-life of about 40 to 84 seconds. It rapidly hydrolyses in the body
to form two products.

(i)

Draw the structural formulae of the two hydrolysis products.

(ii)

Draw the structural formula of the major species formed when the aromatic
product in (c)(i) enters the small intestines where the pH is about 8.
[3]

(d) ProcainE;! is rarely used today since more effective alternatives such as lidocaine
exist. Lidocaine has a half-life of about 1.5 to 2 hours.
Give a reason why lidocaine is more effective than procaine.

[1]
.. ,(Total: 10]

Seither
(a) P, C4H7CIO, is optically active and gives a positive test with 2,4dinltrophenylhydrazine. Q, C4Ha0 2 , is formed when P is refluxed with aqueous
potassium hydroxide. Q, upon oxidation with hot acidified potassium manganate
(VII), produces R, C4Ha03 . R reacts with aqueous sodium carbonate to give brisk
effervescence.
Both Q and R give a yellow precipitate when heated with a mixture of sodium
hydroxide and iodine. On acidification of the resulting mixture, both give S, C3H404.
Deduce the structural formulae of P to S, explaining the chemistry of the reactions
involved.
[8]

(b) Compound U has the following molecular structure.

~--

~CH2-bH-cH 3
OH

Compound U does not react with aqueous bromine while an isomer of it, T,
decolourises aqueous bromine rapidly.
The NMR spectrum of compound T consists of five signals which include a 3-proton
triplet at o 0.9 and a 2-proton triplet at o 2:5.
Suggest the structural formula ofT and explain the splitting pattern at
respectively.

o 0.9 and o 2.5


[2]
(Total: 10]

I
t

12

~
[

8 or
(a) Cinnamald13hye occurs naturally in the bark of cinnamon trees and its essential oil is
used widely in the flavouring industry.

~CH=CH~CHO

[
[

cirmamaldehyde
A proposed laboratory synthesis of cinnamaldehyde starts from compound E,
C9H11CIO. Compound E can be converted into F, C 9H100 in a one-step reaction.
Compound F further reacts with acidified potassium dichromate (VI) with immediate
distillation to form cinnamaldehye.
When refluxed with aqueous potassium hydroxide followed by adding aqueous silver
nitrate, compound E gives a white precipitate.
The NMR spectrum of compound E consists of five signals which include a 2-proton
doublet at o 2.7 and another 2-proton doublet at o 3.6~

(i)

Using the above information, suggest the structural formulae of E and F,


explaining the chemistry of the reactions involved.
Explain the splitting pattern in the proton NMR spectrum of E at o 2. 7 and
respectively.

o 3.6

[
[
[

[6]

(b) Compound G shares the same molecular formula C9 H80 as cinnamaldehyde.


Compound G has the structure below.

(ii)

State what reagents and conditions are used for the conversion of E to F?

H2=cH-Q-CHO

L~

With reference to compotlrid G and cinnamaldehyde,


(i)

suggest one reagent (and conditions, if any) which will react with both to give
the same observations.

(ii)

suggest another reagent (and conditions, if any) which will react with both to
give different observations.

Describe the observations when compound G and cinnamaldehyde are subjected to


the reagents you mentioned in each case.
[4]
(Total: 10]

[
[

l
L
i
[

L.

13
SECTION 0

Answer all questions in this section.

9 (a) Consider the equilibrium between solid silver chloride and its aqueous ions:
AgCl(s)

Ag+(aq) + Cr(aq)

The enthalpy changes of formation and entropies of the substances at 298 K are
given in the table below:

Substance
AgCI (s)
Ag+(aq)
cr (aq)

AWe/ kJ mol
-127.0
105.9
-167.2

S9 I J mor 1 K
96.1
73.9
56.5

I
'

(i)

Calculate the value of llHesorn. ~Sesorn and ~G sorn for the above equilibrium.

(ii)

"Silver chloride is only sparingly soluble in water at 25 C."


Determine the validity of this statement by calculating the solubility product of
silver chloride at 25 C.

{iii)

In terms of free energy, show that the solubility of silver chloride increases with
tern perature.
[5]

[
[

(b) The Ellingham diagram below shows how tiG changes between 0 K and 2000 K for
the formation of some oxides, where the value of ~G relates to the reaction with 1
mole of 0 2 .
i

100

-100

l----+--

""

-600 ~

-?oo

[
[
[

L
l

--+-==-!

I'; I

- $-H~-,-------+--

-800 -1----+--+-_.;..;.j!:..:...:.:..:j::.:.._j__j__j____;~-J-_j

'i

'

!::E~

l
:

1~

1~

1~

1a

1a

Terrperature I K

Use the diagram to


(i)

explain why a reductant is not necessary to extract silver from its oxide at a
temperature above 220 K;

(ii)

state, with a reason, whether hydrogen or carbon is a better reductant at 200K;

(iii)

decide if zinc inetal could be extracted by heating ZnO (s) with carbon and
state the conditions required.
[4]

(c) In the extraction of zinc from zinc oxide, what problem do you foresee in collecting
[1]
the metal as a solid?
[Total: 10]

L
14

10 (a) Petrol is composed primarily of hydrocarbons consisting mainly of molecules with


eight carbon atoms called octanes. One clean burning octane is a compound
called 2,3,4-trimethylpentane, C8 H1s- Its standard enthalpy change of combustion
is -5069 kJ mor1 .

{i)

Write a balanced equation for the complete combustion of 2,3,4trimethylpentane. Include state symbols that refer to standard conditions.

(ii)

Calculate the standard


trimethylpentane.

(iii)

Calculate the standard internal energy change of formation of 2,3,4trimethylpentane.

(iv)

Explain why the values calculated in (ii) and (Hi) differ.

enthalpy

change

of

formation

of

2,3,4-

[7]
[dH9 t H20(I) = -285.9 kJ mot1 ; dH9 t C0 2 = -393.5 kJ mol-1;
R = 8.310 J K-1 mol-1]

(b) (i)

(ii)

~
[

[
[
[
[

Trouton's rule states that the molar entropy change of vaporization is


approximately 88 J mor 1 K-1.
At 399K, 2,3,4-trimethylpentane liquefies. Assume that it obeys Trouton's
rule, calculate its molar enthalpy change of vaporization.
Explain whether propan-1-ol, which has a boiling point of 371 K, will obey
Trouton's -rule.
[3]

[
[

[Total: 10]

u
lJ

Ll
[J
[
~~
I~

L
{

,___
I

I~

,;h
.

ANDERSON JUNIOR COLLEGE


PRELIMINARY EXAMINTATION 2006
CHEMISTRY 9258

2a(i)

Bond energy of NsN bond is smaller than that of C=O bond.


C and 0 has very different electronegativities,
hence the bond is polar while the NsN bond is non-polar.

a(ii)

CO has a higher boiling point lhan N2


Dipole-dipole attractions between polar CO molecules are stronger than the van
der Waals' forces between the non-polar N2 molecules.

PAPER 1: MULTIPLE CHOICE (40 marks)


1

:6

. 11

16

21

26

sC.

31

36

!8

. 13

18

23

28

33

38

i 10

i 15

20

25

30

35

40

b(i)

2____..

b(ii)

NO/: linear

NO,-: bent

[:Y~oY

[O=N=O]'
PAPER 2 : STRUCTURED (60 marks)

[Total: 7]

P2.= 2.67 atm


CH 4:

Pr = 4.00 atm
a(li)

3a(l)

Dative bond between P and B atoms;


covalent bonds belween P and H atoms and belween B and Cl atoms

P, = 1.33 atm

- show that CH 4 deviated less than CO,

PV/RT -

a(li)

x has a small positive value.

a( iii)

Equilibrium composition is not affected,


as the (vapour) pressure exerted by a solid is constant at constant temperature.

b(i)

c = 0.0432 mol dm"3

b(i)

Min. mass of PH 3 BCI 3 required = 3.27 g


[Total: 8]

4a(i)

XC0 3 4 XO + C0 2

a(ii)

A, of X= 40.0

1.0 k---f--~PT---11--~t-----r. Ideal gas (200K)

+-----<--~

i
0.0 L---.....1..------......J----'---~'

a(lii)

Methane has . weaker intermolecular forces of attraction

a(lv)

Show C0 2 graph becomes closer to ideal gas graph

a(v)

(b)

X is calcium.

(b)

Element Y is below element X in Group II.


The size of
is larger than
The polarizing power of Y2 Is lower than X 2.
The electron cloud of Co,'- in YC03 is less distorted I C-0 bond of co,'- in YCO,
is less weakened.
YCO, is more stable than XC0 3

(c)

caco, + so, 4 caso, + co,

y'

x'.

p
[Total: 7]
Sa(i)

It has undergone disproportionation.


Cr changes from +4 oxidation state to +3 and +6.

At higher temperature (300K),


intermolecular attractive forces becomes less significant

a(ii)

Cr,ol- + 2 OH"

~ 2 CrO/" + H,O

The solution turns from orange to yellow.

Increase in global temperature after 2 years= 0.0240 'C


[Total: 9]
(b)(i)

A ligand can be a (neutral) molecule or anion which possesses at least one


unshared pair of electrons.

L_;

L._;

L__l

L_j

L..J

L__;

L_j

L._j

l__5

L,_]

1--....J

L_j

'-----'

i...-...,J

:__]

-,

.._____)

r---

--~

'
3
(ii)

The number of moles of free chloride ions is equal to the number of moles of AgCI
precipitated per mole of complex.

b(i)

Add dil H,so, (or dil HCI) and heat the samples.
Add aqueous iodine and NaOH(aq) with warming.
or Add aqueous iodine and excess NaOH(aq) with warming.

Thus, the structures are as follows:

X gives a yellow-precipitate of CHI 3; no precipitate

[Cr(H 20) 6] 3'.

A3CI"
B- [CrCI(H 20)sf'. zcrC- [CrCI,(H,O),r. Cl-

b(il)
[Total; 8]

6(a)

b(i)

or

zeolite, 400- 600'C (catalytic)

CHiCH 2

&

CH 3CH 2CH 2CH2CH 3

PAPER 3 : FREE RESPONSE (100 marks)

-c--c--c--c-

anode: Zn(s) + 2 OW(aq)-+ Zn(OH),(s) + 2ecathode: NiO(OH)(s) + H20(1) + e--+ NI(OH)2(s) + OH-(aq)
2Ni0(0H){s) + 2H 20(1) + Zn(s) -+ Zn(OH),(s) + 2Ni(OH),(s)
overall:

H-C-H

E0 ,. 11 = +0.49- (-1.24) = + 1,73 V

H
I

H
I

H
I

H
I

I
H

I
H-C-H

I
H

b(il)

1a(l)

alkane

Poly( propene) Is fully saturated and Is an alkane.


It has no reactive groups (or Is inert), unlike amides or esters which can be
hydrolysed.
[Total: 8]

7a(l)

phenol and alkene {do not accept other ans.)

a(ii)

The drug contains large alkyl groups (and/or alkanes group I aryl group /large
hydrocarbon group) that are non-polar in nature.

a(llf)

Z givesa orange precipitate; no precipitate observed for Y.


[Total: 13]

800 - 1OOO'C (thermal)

b(llf)

A_dd aqueous bromine to both samples. Z turns red-brown Br2 colourless and
gives a white precipitate and but not Y.

or Add 2,4-DNPH.

Thermal 1catalytic cracking

a(il)

aqueous KOH I NaOH

b(l)

Coating with graphite makes the petal an electrical conductor.

b(ii)

Jttt-: e; flow

-I
CHs

HO

h _ f i _ C H2CH2 CH 2CH2 CH 3

\Hr
C-CH 3

Hl/
a(lv)

ob~erved for W.

Br, in CCI,

CH,~HO
Br

OR

Br

HO
C-CH,

Br

BrH 2C

Br

NaOH(aq)

CHqp-'
Na'o

0:~,

=1.405 x 1o-2 x 96500 =1356 C

b(iv)

The student can Increase the current to increase the thickness.

2(a)

Titrimetric analysis

(c)

Look for working;_ value oft~ and gradients; order of reaction


Rate = k[KI][H 20,]

(d)

Slow I r;lte-determining step Involves 1 Kl and 1 H20 2


Mechanism involves more than one step.

either

a(i)

Kb =1.82 x 10-s niol dm-3

a(ii)

pH= 9.26

I;

H,C
d ilute HNO,(aq)

NO
2

CHrJn\_

HOQNO~

C-CHs
HOH,c{ 'oH

amount of charge used

C-CHJ

'ar

cold alkaline KMnO,

b(lll)

c-cH,

I;

H,C

a(iii)

pH= 5.5
NH:+ H20

'-----"
..-

NH 3 + H3 0'

5
b(ii)
a(iv)
b(i)

K _ [H+][HC03-]
'-

(H2C03j

(c)

[H2C03] = 0105
[HC03-]

b(ii)

The E value will decrease /less positive.


The K..ab value of [Co(edta)r is greater than that of [Co(edta)]2- which means that
edta stabilises Co(lll) more than Co(ll).

methyl orange I methyl red

2 Co2' + S20 62- ~ 2 Co''+ 2 so;2 Co'' + 2 1 ~ 2 Co2' + 1z

The buffer system has a high capacity to remove W ions (or low capacity to
remove oH ions).

or

a(i)

~H 0 ,eactlon = -98.9 kJ

a(li)

Co2' is.capable of having variable oxidation states but Ca2' only has a fixed
oxidation state of +2.

E,.u = +0.19V
E,.u = +1.28V

Both paths require lower activation energy than the uncatalysed reaction.

(d)

mer'

E0 cell= (+1.82)- (+1.23)

+0.59 V

S(s) + 3/2 O,(g)

~H, ofSOz

1
1

i-"=-S0
2z~(g'-'-)-+...;.Y,::...20.:..:2,_,(g'T)--'---98.9 kJ mar'

The more acidic solution with AICh is due to (partial) hydrolysis.


From data booklet, ionic radius (Na') = 0.095 nm: ionic radius (AI'')= 0.050 nm
The smaller Al 3' (or higher charge density Al 3')
polarises water molecules, releasing H'

'

Standard enthalpy change of neutralisation Is the energy (or heat) released when
one mole of water is formed as a result of the reaction between an acid and an
alkali (under standard condition).

AICI 3 + 6H 2 0~1(H20)sJ'' + 3CI'

=-61.1 kJ mar'

[Al(HzO)sJ'' ....--- [AI(H20)s(OH)f' + H'

b(ii)

"'H"

(C)

~H.for strong base- strong acid (e.g. HCI, HNO,) is constant as they are fully
dissociated and the reaction is essentially H' + OW ~ HzO

b(ii)

Steamy fumes of HCI are formed when a little water is added initially.
Undergoes complete hydrolysis to give solution pH 2.

For CH 3C0 2H, t.Hn is less exothermic as CH 3C0 2 H is a weak acid.


Some energy released is used to complete the dissociation of CH,C02H.

c(i)

2S2CI, + 3H 20

For H2S0 4, t.Hn is more exothermic as H2SO, is a strong acid.

c(ii)

3 moles NaOH (show working)

The exceptionally high value is because sulphate ions hydrate strongly In water.
This heat of hydration adds to the heat released during neutralisation.

c(iii)

: Cl !S ~ S! Cl:

K.., 0 is

an equilibrium constant that reflects the stability of the [Co(NH,)sf' complex


formed.

K.

tab-

b(i)

b(l)

803 {g)

'

4(a)

either
NaCI Is ionic whereas PCI, is simple covalent.
Na and Cl differ greatly in electronegativity (giving rise to ionic bonding) whereas P
and Cl do not (hence giving rise to covalent bonding).

t>H 1 of SO,

As shown; formation of SO,(g) is more exothermic than that of so, (g).


b(i)

5
(a)

[Co(NH3)6

+J

[NII3] 6[Co(II20)6 2+J

The solution will turn from pink to pale brown.


The Kstab value of [Co(NH 3 ) 5f' is high, suggesting that stronger NH, ligands
replaces H20 ligands.

3S + H2SO, + 4HCI

XM

XX

XX

XX

or

(a)

From data booklet


BE(H-CI) = 431

BE(H-Br) = 366

BE(H-1) = 299 (units in kJ mol-')

Thermal stability of HX decreases down Group VII


as the H-X bond gets weaker
due to increasing bond length from HCt to Hi.

___:

L-.:

L__.

l__l

L..J

'

L-...-1

L....J

'

L,____j

L..:

L_j

L.J

I
'
:.-...J

L-J

[__]

'

:.___.;

L--J

;_____)

_j

'_j

..

::~

~t

7 ''
b(l)

1- + HzS04 -> HI + HSo,8HI + H2SO, -> 41> + H2S + 4H>O

"

(b)

HI produced is a strong reducing agent and is easily oxidised by' concentrated


H2SO, to 12
.
:

Stage 1:

. PCis.

Stage II:

CHz(OH)CH 2CI;room temperature

Stage IIi: H2; Ni catalyst, heat


c(i)

H2~C02H

HCI is a weak reducing agent and is not oxidised to Cl 2


b(il)

c(l)

c(ll)

Phosphoric(V) acid is a weaker oxidizing agent than cone H2S0 4 and no oxidation
of HI occurs.
Kl + H,PO, "-> HI + KH2 PO,

(d)

Lidocaine has a longer half-life and will stay in the body for a longer time.

either

(a)

P Is optically active => P contains a chiral carbon

:::ox=3

Ga(i)

CH 3CH 2Br:
CH 3CHO:

a(il)

The mechanism is nucleophilic addition.

KCN in ethanol, reflux


HCN, KCN, 1oc

P, C,H7CIO gives a positive test with 2,4-dinitrophenylhydrazine


=> carbonyl compound present
With aqueous KOH; P undergoes nucleophilic substitution to form Q, C,Ha0 2

'\an. slow
C-00 CH() .~

a, C,H.o 2 contains
an -OH group
li
.

I CH:r-C(--0

. CN

Q is oxidised toR, C,H.o, which.glves effervescence with NazCO,


=> R conta.ins, a -COzH group which gives C02 gas with NazC03

:CN
H

CH

H z *co;

~ 12

ICI, + x 1 -> x Cl- +

~,;, 2

c(il)

I
:r-y-o
H'
CN

CH

a yellow precipitate of CHI 3


and S, C,H,O, oil acidification

=> Q and R contains either CH,CH(OH) group or CH,CO group

With NaOH and iodine, both Q and R give

6-oH

:r-1
CN

A racemic mixture is formed.


a(lli)

The optical isomerism of the product would be reversed.

(b)

CH,CHO LIAIH, in dry ether CH,CH 20H


oc

~cone.

H2SO,, reflux

CH 3CH2Br

reflux

CH,CH2CN

~N
in ethanol,
reflux
(b)

a(ll)

~~~
,,~~~

This nitrogen is attached to 2 ethyl


groups which are electron donating.
Hence, Its lone pair is more available
to accept a proton.

=> R is

II
II
CH3-C-CH2-C'-OH

=> Ql S

CH 3-et+-CH2-C-H

::::>Pis

Cl
o
J
II
CHr-CH-CHz--G--H

. OH

7a(l)

HO~~

0
11

and

SIS

Tis. HO-<Q>-CH2_:CHz-CH 3

o0.9- duet~ -CH, group next to -CH2 group


o2.5 -due to -CH 2group next to -CH 2 group

0
0
II
II
HO-C-CH2-C-OH

.-

--.

..l

rl

10

9
8

or

a(i)

F is @-cH=CH-CH20H

b(iii)

Yes

T;;,. 1050 K
(c)

On cooling Zn(g) to Zn(s). some ZnO (s) will be formed.

10a(i) C8 H18 (I}+ 2512 02 (g) -7 8 C02 (g)+ 9 H20 (I)


!:; undergoes elimination to form F.

a(ii)
C 8 H 18 (I) + 2512 02 (g)

Fundergoes oxidation with dichromate to form cinnamaldehyde.


With aqueous KOH, E undergoes nucleophilic substituti?n.

AH\

The resulting mixture has free c1 ions that give a white AgCI precipitate with
AgN0 3 solution.

8 C (s) + 8 02 (g)

5 2.7- due. to -CH 2 group next to -CH group


5 3.6 -due ta -CH 2 group next to -CH group and 0 atom
a(ll)

ethanolic KOH (or NaOH), reflux

b(i)

Br2 in CCI
Both compounds tum red-brown Br2 colourless

b(ll)

Fehling's reagent, warm


Cinnamaldehyde gives a red ppt of Cu2 0; G does not give any ppt

9a(i)

LIH",,n = 105.9 + (-167.2}- (-127.0)


LIS"''" = 73.9 + 56.5- 96.1
6.G 0 ~o1n

a(iii)

(298)(34.3)

= + 55478.6 J mol"'

= +55.5 kJ mar'
a(ii)

L\G'"'" = - 3RT In K"'

+55.5 x 10 = - (8.31 )(298) In K,p


In K,p = -22.412
Ksp = 1.85 x 10-10 mol 2 dm~
Since Ksp is very small, silver chloride is considered as an only sparingly soluble at
298K.

a(iii)

L\G,1n = 6Hsoln - TAS,.,n


Since both 1\H,.," and AS,01 n are positive, as T increases, the TLIS,.,, term outstrips
the 6Hso1n term and ll.Gso1n changes from positive to negative.

b(i)

b(ii)

t.u, = c.H",- c.nRT


= -652- (-9)(8.31)(298)11 000
= -630 kJ mol'
a(iv)

There is a decrease in gaseous volume.


This represents an Input of energy as the atmosphere does work on the mixture.

b(i)

LI.S,p = LI.Hvap IT 0 = +88 J mal' K- 1


to.Hvap = 88 x 399 J mal' = +35.1 kJ mol'

b(ii)

No
Because hydrogen bonding between molecules
causes extra degree of order in liquid propan-1-ol.
Hence change from liquid to vapour during boiling I vaporization
causes greater increase in entropy (or gives a more positive \S)

= +65.7 kJ mor'

=+34.3 J mor' K-

8 C (s) + 9 H2 (g) -7 C,H,. (I)

M = 0-9 = -9

=6.Hso1n - T3ASsoln
= 65.7 X 10

t.H 0 1 + (-5069) = 8 (-393.5) + 9 (-285.9)


t.H, = -652 kJ mar'

Above 220 K, silver oxide decomposes spontaneously into its elements, silver and
oxygen, since e.G (Ag 2 0 -> Ag) < 0 (or e.G (Ag -> Ag20) > 0)
Hydrogen

e.G for H2 -7 H20 is more negative than for C -7 CO.

..

c
[

[
NATIONAL JUNIOR COLLEGE
PRELIMINARY EXAMINATIONS 2006

[
CHEMISTRY

9258/01

Paper 1 Multiple Choice


14 September 2006

1 hour
Additional Materials: Multiple Choice Answer Sheet
Data Booklet

c
r

READ THESE INSTRUCTIONS FIRST


Write in soft pencil.
Do not use staples, paper clips, highlighters, glue or correction fluid.
Write your name and index number on the answer sheet in the spaces provided
unless this has been done for you.

'

There are forty questions on this paper. Answer all questions.


For each question there are. four possible answers A, B, C and D. Choose the one
you consider correct and record your choice in soft pencil on the separate answer
sheet.

Read the instructions on the answer sheet very carefully.

Each correct answer will score one mark. A mark will not be deducted for the
wrong answer.
Any rough working should be done in this booklet.
You may use a calculator.

~'

[
[
[

L
L
L
L
L

This question paper consists of 17 printed pages (including this page).

l
..

[
[

[
Section A
For each question there are four possible answers, A, 8; C, and D. Choose the one you
consider to be correct.

1.

Particle

Neutrons

Nucleons

16
18
16
17
16

33
35
32
34
31

vs2T2+

o3-

Which one of the following sets consists of species that are isoelectronic?
A

U, S2-, T2+

u. s 2-. 0 3c v-. s2-. o3v-. T2+, 0 30

[
[I

Which graph correctly describes the behaviour of a fixed mass of an ideal gas?
(Tis measured in C and d denotes density.]

piT

constantp

constantp

[
[

D
pV

constant T

..
p

L
[~

[j
2

rl

c
[

[
8

Oxygen reacts with nitrogen mohOxipe in the equation shown.

02(g)

2NO(g)

2N02(g)

In an experiment to investigate the effects of concentrations on the rate of reaction,


the following results were obtained.

r
'

Expt

[02] I mol dm-3

[NO] I mol dm-3

Rate I mol dm-3s-1

1.0

1.0

0.0007

1.0

2.0

0.0028

1.0

3.0

2.0

2.0

0.0056

The value of X is

[
[

0.0007

0.0021

0.0063

0.0189

Methylhydrazine, CH3NHNH 2, and dinitrogen tetraoxide, N204, are used as


propellants in the lunar module which landed the first man on the Moon in 1969.
The equation of the reaction is:
4CH 3NHN H2 (1) + 5N 20 4 (1)

--+

4C02(g) + 9N 2(g} + 12H 20(1) l\ H

=-4592 kJ mor1

What is the standard enthaply change of formation of CH3NHNH2(I)?

Compound
N204(I)
C02(g)
H20(1)

f'

-~

L
l
L-

+53 kJ mor1

+33 kJ mor1

-53 kJ mor1

-33 kJ mol-1

l\ Htf kJ mor

-20
-394
-242

L
['
-

L
[

[
10

A current is passed through two cells connected in series. Cell 1 contains


Cu(N0 3h(aq) and copper electrodes while Cell 2 contains HzS04 (aq) and platinum
electrodes as shown.

I
I

[
[

[
r-'"-

'-

'-

....__

'---

'---

r-

.___

Cell2

Cell1

[
[

Which one of the following describes correctly the reactions for the cells?

11

Ceii1-

Cell2

Anode dissolves

Oz evolves at the anode

Cathode dissolves

H2 evolves at the cathode

Cathode increases in mass

S02 evolves at the anode

Electrolyte becomes paler

02 evolves at the anode

r~
[
[~,

Electroplating of a plastic crown coated with graphite powder is carried out in an


aqueous solution of copper(ll) sulphate with a current of 100 A using pure copper
as one of the electrodes.

.Given that the surface area of: the crown is 50Qccm2and the density of copper is
8.96 g cm-3 , which one of the following expressions indicates the time required to
plate 0.025 em thickness of copper on the crown surface?

2 X 63.5 X 96500
5 X 8.96 X 50 X 0.025

5.

2 X 63.5 X 96500
5 X 8.96 X 500 X 0.025

-------5

8;96x500x0.025x96500
.
s
63.5x 100
500 X 0.025 X 8.96 X 2 X 96500

------------------5
63.5xl00

L
L

u
6

L
r

L
[

[
3

A tube was filled with a mixture 10 cm 3 of methane, 30 cm 3 of ethane and 160 cm 3


of oxygen at room temperature. The open end of the tube was placed in a beaker
of KOH (aq) as shown. The gas mixture was sparked.
What would the final level of the liquid be in the tube after it cooled to room
temperature?

20

40

60

80

100
120

140

160

180

200

[
[

KOH(aq)

Which one of the following shows increasing magnitudes of angles X, Y and Z?

H-b
I

L
l
L
l
t'

H H 0 H
I
I II
I
C-C-C-C-H
I
I
z'l
-- H H
H

Y<Z<X

Z<X<Y

X<Z<Y

Z<Y<X

Silicon carbide, SiC, is a major industrial abrasive and a refractory materiai. Which
type of structure explains these properties?

A simple covalent structure with covalent bonds between silicon and


carbon atoms.

A layer structure with covalent bonds between silicon and carbon atoms
and weak van der Waals': between the layers.

A giant covalent structure with strong covalent bonds between silicon


and carbon atoms forming a 3 dimensional network.

A giant ionic lattice.

r
r
[
6

Which one of the following pairs consists of a planar molecule and a polar
molecule?
A

CsHsCH3 and C02

CsH1 2 and HaP04

CH 2=CHCHO and CH3 0CHa

C6 H6 and CCI4

c
[

Compound V is converted into compound Y in three steps:


,~

Step 1:
Step 2:
Step 3:
Overall:

V-+W
W-+X
X -+Y

ilH1
ilH2
ilH3
ilH,

v---+ y

and the energy profile diagram is as shown.

[
Energy

v
y

Reaction pathway

Which one of the following is correct?

ilH1

il H,

rate determining step

Endothermic

Exothermic

Step 3

Endothermic

Endothermic

Step 3

Endothermic

Exothermic

Step2

Exothermic

Endothermic

Step 2

l
l

L
4

l.
L

c
[

[
12

N204

:;;;==:=

2N02

What is the value of the equilibrium constant,

I
A -atm

c
[

At a total pressure of 1.0 atm, dinitrogen tetraoxide is 50% dissociated at a


temperature of 60C, according to the following equation.

13

Kp. for this reaction at 60C?

-atm
3

-atm
3

D 2 atm

Which one of the statements below 'concerning the Period 3 elements (Na to Ar) is
correct?

-,
f

The elements become less electronegative from Na to Ar.

The hydrides of the elements become more acidic from Na to Cl.

The m;:Jximum oxidation state is shown by silicon.

Aluminium oxide is the only oxide that is insoluble in water.

-)

14

[
[

Which one of the following statements of the Group II elements and their
compounds is correct?
A

The pH of aqueous magnesium chloride is greater than that of aqueous


calcium chloride.

Calcium nitrate is more stable to heat than strontium nitrate.

Hydrogen is liberated when an excess of magnesium is added to an


aqueous solution of iron(lll) nitrate.

Both beryllium oxide and aluminium oxide are amphoteric due to similar
sizes of the cations.

[
15

c:
[

In agriculture, lime, CaO. is used to reduce the acidity of soil and ammonium
sulphate is used as a fertiliz.er. What is the best reason for not packing them in a
mixed form for sale to farmers?

The dry mixture is explosive.

When dampened, ammonia is given off.

CaS04 formed will affect the hardness of water.

Dilute sulphuric acid is formed.

[
L.

r
[

[
16

Aqueous chlorine is added to aqueous sodium iodide and the mixture is shaken
with an equal volume of trichloroethane.
Which one of the following observations would be seen?

red-brown

brown

colourless
orange

[
[

purple

[
17

18

Silver chloride but not silver iodide is soluble in aqueous ammonia. However. both
silver chloride and silver iodide are soluble in aqueous sodium thiosulphate. All of
the following statements are true except

The solubility product of silver chloride is numerically larger than that of the
silver iodide.

Silver iodide does not form [Ag(NH 3ht in excess aqueous ammonia.

Thiosulphate ion is a stronger ligand than ammonia.

The lattice energy of silver chloride is numerically greater than that of silver
iodide.

Use of Data Booklet is required for this question.


Which one of the following is true about vanadium and its compounds?

19

The maximum oxidation state of vanadium is found in the oxo-anion V0 2+.

Vanadium(V} oxide is used as a catalyst in the manufacture of ammonia.

Lead is able to

The density of vanadium is smailer than that of calcium.

re~duce

vanadium from the (V} to the (Ill) oxidation state.

Which one of the following conversions involves a copper species reduced?

8
C

CuC03 .Cu(OH)2 ~ 2Cu0 + C02 + H20


Cu2+ + 4Cr ~ (CuCI4] 2CuCI + 2NH3 ~ [Cu(NH3ht + cr
Cu2+ + SCW ~ [Cu(CN)4] 3- + %(CNh

i
c

L..

[~

l-

[
[
[

L
L
L
\

[
[

20

What is the total number of stereoisomers (cis-trans and optical) for the molecule
shown below?

CHzCH3

16

21

[
[
22

Which one of the following pairs of reagents reacts to form an organic product with
only 2 singlets in its nmr spectrum?
A

ethene and hydrogen bromide

ethanal and hydrogen cyanide

propan-2-ol and acidified sodium dichromate(VI)

propanone and hydrogen cyanide

The compound 2-ethylhexyl-p-methoxycinnamate {MOC) is used as a sunscreen.

~~\
CH 30~))-CH=CHC02CaH 17

L
[

OCH3

D 32

OCH 3

'..!

CH3-<;-CH 2CH 2 CH=CHCH=CHOOC--CH 3

CH 2CH 3

Which one of the following statements regarding this compound is correct?

It is slightly soluble in water.


It reacts w.lth cold aqueous NaOH to yield

A brown precipitate is formed with cold alkaline KMn0 4

A racemic mixture is produced when it is boiled with aqueous hydrochloric


acid.

L'

~1

L
L
L

0\_
=CHC"--Na+
)
CH
v-.<

CH3D-:-\ !\

23

Which one of the following methods would you choose for the synthesis of the
compound shown below starting from benzene?

8
-NH,

Ste~

24

Ste~

Ste~

II.

Ill

nitration

reduction

bromination

bromination

reduction

nitration

bromination

nitration

reduction

nitration

bromination

reduction

Hot ethanolic KOH is added to the following two compounds, CsH 5CH 2 CH2CI and
CsHsCH2CI. separately to give compound X and compound Y respectively. What
could compound X and Y be?

CH=CH2

6
6
6

CH=CH2

CH20H

c6
c

CH2CH20H

CH20H

c6
D

CH2CI

lVJ
8

X
A

6CI

CH2CH20H

6
u

L
L
[

L
[

10

c
[

25

[
(

Which one of the following will be formed when butan-1,3-diol is distilled with hot

acidified potassium dichromate(VI)?

CH3COCH 2CHO

CH3COCH 2 COOH

C02 and H20

CH3CH(OH)CH 2COOH

~,

26

CH3

c
c
[

L
L
l
['

L
r

soi-

X
,..

(intermediate)

..

CH3
I
HO-C-S03-

OH-

CH3

What is the mechanism involved ih the above reaction?

c=o
cH{

Propanone can react with Na 2 S0 3 (aq) to form hydroxysulphanate ions according to


the following route.

'27

electrophilic addition

electrophilic substitution

nucleophilic addition

nucleophilic substitution

4.40g of a monocarboxylic acid was found to react with excess sodium metal,
producing 0.56 dm 3 of gas (measured at s.t.p.). What is the relative molecular
mass of the acid? (1 mole of gas occupies 22.4 dm 3 at s.t.p.)

22.4x 4.40
0.56

22.4x 4.40
2x0.56

0.56x 4.40
22.4

2 X 0.56 X 4.40
11.2

II

l_

r
[
28

Which one of the following reactions is incorrect?


~

2 CH3CH 2 C02H + CaO

Ca(CH3CH2C0 2)2 + HzO

CH 3COC02H + 4[H] ~ CH 3 COCH 20H + H 2 0

HO-@-coOH

CeHsC02CsHs

+ PCI, ---l> HO-@-COCI + POCI, + HCI

NaOH

29

~H,

aOO

hat

c5

CsHsC02-Na+ + CsHsO-Na+

r
[

[
[
[
[

are both white solids.

r,

Which one of the following tests could be used to distinguish between these two
compounds?

Heat each compound with acidified potassium manganate(VII}.

Shake each compound with aqueous bromine.

Boil each compound with dilute sodium hydroxide.

Add PCis into each compound.

[
[

L
[

r.
12

l
l
L
L

L.

[
[
[
30

The following three amino acids exist as the following structures at neutral pH.

coo-

l
, (CH2h

I
H3N-C-COO

H3N-C-COO

glutamic acid
(Giu)

H3N-C-COO

H3c......._ /CH 3
CH

glycine
(Giy)

valine
{Val)

A mixture containing the three amino acids was placed in a solution of pH 3 and
subjected to electrophoresis as follows.

c
[
mixture of amino acids

[
[

Which one of the following filter papers shows a possible result of the separation
of the amino acid mixture at pH 3?

? .J
Giu

[
[

filter paper

,,9-

Val Gly

Giu

i'! ., !9Val Glu

0 J'

Gly

Gly

?-

Val

f-,-LfJ}Giy

Val Glu

L
L
L
[

t'
t

13

r
[
Section B
For each of the questions in this section one or more of the three numbered statements
1 to 3 may be correct.

Decide whether each of the statements is or is not correct (you may find it helpful to put
a tick against the statements which you consider to be correct).

The responses A to 0 should be selected on the basis of

1, 2 and 3 are
correct

1 and 2 only are


correct

2 and 3 only are


correct

1 only-is correct

---

------

----

No other combination of statements is used as a correct response.

[
31

In which of the following pairs of compounds does the first one have a higher
boiling point than the second one?
1

MgCI 2 and AICh

nylon 6,6 and Terylene

CH3CH2CH(CH3)CH2CH3 and (CH3)3CCH2CH3

L
-~

l
[

_/

32

Ions ol the two-most-common isotopes of the transition metal nickel are shown
below.
S8 Ni2+
60 Ni2+
28

28

Which of the following statements are true?

Both Ni2+ ions have the same number of electrons, but different number of
neutrons.

~he ~~ Ni 2+ ion will be deflected more thim the ~! Ni 2+ ion when a magnetic field

of same magnitude is applied.

The electron arrangement of both of these Ni 2+ ions is_1s22s2 2p6 3s2 3p6 3d64s2

14

L
L
L
L
{

[
[
[
The responses A to 0 should be selected on the basis of

[
[
33

1, 2 and 3 are
correct

1 and 2 only are


correct

2 and 3 only are


correct

1 only is correct

The diagram shows how the equilibrium constant Kp varies with temperature for the
reaction.
A(g) + B(g)

-~

C(l) + D(g)

Kp

[
[

Which of the following statements are correct?

The reaction is exothermic in the forward reaction.

The equilibrium mixture contains a smaller proportion of D at higher


pressure.
Kp is dimensionless.

34

[
[

Use of Data Booklet is relevant to this question.


Which of the following statements are true?
1

The reactivity of the Group II elements increases on descending the group.

The decomposition temperature of the 3 carbonates is of the order:


PbC03 > CaC03 > ZnC03

[
,
r
.

35

Calcium metal is extracted from its ore by reduction of calcium chloride with
carbon monoxide.

Nitrogen and bismuth are in Group V of the Periodic Table and their oxides have
the formulae N 2 0 3 and Bb03 respectively.

L
L
[

Bi203 is expected to be

less ionic

less volatile

more basic

15

l
[

L
[
The responses A to D should be selected on the basis of

--

36

37

1, 2 and 3'are
correct

1 and 2 only are


correct

2 and 3 only are


correct

1 only is correct

Which of the following are propagation steps in the reaction between methane and
chlorine under UV light?

CH3 + Cb

CH 2CI 2 + Cl

CH 4 + Cl

CH3CI + Cl

CHCI2 + HCI

CH 3CI + H

QCH,

'H

t(c:H,

'I Br
H

CH~

/C7C""Cl

CH~
/Br
--- C=C
H/

38

'I Br
H
/Br

"'

Cl

CH~

/C=C""Br

CH~

/H
/C=C""Br
Cl

Which compounds would be formed in the reaction between ethene and aqueous
bromine in the presence of sodium ethoxide?

CH2BrCH20H

CH2BrCH20CH2CH3

CH3CH 20CH2 CH 20CH2CH3

[
['

[
r

L_~

r-,

/CI

Which of the following illustrate cis-trans isomerism?


1

u
u
u

L
'
l
l
l
L
-'

16

c
[
[
The responses A to D should be selected on the basis of

39

[
[
[

40

1,2and3are
correct

1 and 2 only are


correct

2 and 3 only are

1 only is correct

correct

Which statements are true about the hydrolysis of propanoyl chloride?


1

Water is a nucleophile.

Two acids are formed as products.

A white precipitate is formed immediately with AgN0 3 (aq).

Which statements about phenylamine are true?


1

It has a larger pKb value than ethylamine.

The organic salt obtained from its reaction with dilute sulphuric acid
dissolves in water to give an acidic .solution.

It gives a violet coloration with neutral iron(lll) chloride ..

[
[
-,

~"

[
[
[~

L
L
[

!__
[

17

r---:
'

,..----,
J

-----

1._

:~

l
[-

..

MINISTRY OF EDUCATION

RUB OUT ERRORS THOROUGHLY

1.

2.

3.

[
[
r

~,

---'

[
[

4.

---

--

-[' [ -L[ -L-LL

t _-

- ------- -------------

Enier your NAME {as in. NRIC ).

fr.e_h_Q'l_

2 00.6

CheMis+rt~--------------

Enter the SUBJECT TITLE.

@
-----

1012
i

c::::::J

c=J

li:lll

C=:J

c:::J

c:::J

56
r=:::J

=0 ..... = =3 =4 =5 =
I .... = = = = = =
2

Enter l:he ClASS.

L_

Enter your CLASS HUMBER or

=0

Now SHADE the corresponding


lozenge in the grid for

EACH DIGIT or LETTER

=
0

E
R

SHADE APPROPRIATE BOXES

--+~--o-

D
E
X

INDEX NUMBER.

7.

Enter the TEST NAME.

~-- 1
N

6.

r----------\ USE PENCIL ONLY


i FOR ALL ENTRIES ON THIS SHEET

\012

~-\.IV RlTE
5.

(NJ'c)

= =2
=
=
=

=
B
=

=2
=2
=2
=c
=

=
4
=
4
=
4
=

=3
=3
=

::::1

=
F
=

. c::::J

!l

=5

c::J

--------------------1
5
6
I
I
7
9
=
!
= = = c
=
5
6
9
I
I
7

= =

=
6
=

=
7
=

c ::::1
8

::::1

L: :=!

c:::J
=
g
9
=
=!
CJ
=

I
j

I
1

'I

= = =
i
H
= = =
----------------'
G

INSTRUCTIONS FOR RECORDING M~SWERS


Suggested answers to each question are given in the question paper. Choose an answer and shade !he corresponding
lozenge_ if there are oniy four suggested answers, A, B, C, 0, ignore E on this sheet Don't worry if the question paper
has less than the 60 questions allowed ior below.
A

=
A

Bilii

=
B
=
B
=
B

=B

=
=E
=E
=E

T---;-21
n

2<.
23

24

~-------------~

c
=
c
c
=
c
=
c
=

.,.0; .- =E
i A B
= =
=0 =E
i A B
i = =
-0 =E
'
A
B
=
= =
0
E
A
B
-A =B C =0 =E
==--==
A
B
C
0
E
=- = = =
A
B
C
D
E
=A -B =C =!l =
E
-=A =B -C =0 =E
= = = =
C
0
_E
A
B
=
=
-A =B C 0 =
E
j

Wi!!

=A -=B
=A
=
A
=

==C

-!l

=E

=E
=
=E
-B =
C
0
=B -C =0 =E
=C =0 =E
=C =
=E
0
C

.-t

t:::l

r::::::J

c::l

c::J

r:::::J

c::J

41

44

!l

I=====
42 I A
B
C
0
E
J. = =
= =- =
DE
43 1 A B C
= = = = =
45
46

47
48
49
50
51

52
53
54

=
A
=

=
A
=
A
=

=
B
=
B
=
B
=
B

=B

=
=C
C

r=J

c=l

c::J

c:J

CJ

=
A
=

=
B
=

=
A
=
A
=

57

58

59

56

=
E
=

= q
=E
A
C
0
=-====

=
B
=
B
=
B
=

55

=C =D

A
A
A

=c

=
c
=
G
=
G
=
c
=c
=c
=c
=

=E
=
E
=

=
D
=

=
E
=

=0

=
D
=

SCAN-FORM SYSTEMS TEL: 9618 1848

'

,_
I

:'""'""l

l_

c
[

Registration Number

ST group

~-

Name

NATIONAL JUNIOR COLLEGE


Preliminary Examinations 2006

9258/02

CHEMISTRY

PAPER 2

14 September 2006
1 hour 30 minutes

[ -

Candidates answer on the Question paper.


Additional Materials: Data Booklet

READ THESE INSTRUCTIONS FIRST

~,

[
[
[

[
[

Do not open this booklet until you are told to do so.


Write your name, class and registration number in the spaces provided at the top of this
page.
Write in dark blue or black pen.
Do not use staples, paper clips, highlighters, glue or correction fluid.
Answer ALL questions.
You are advised to show all working in calculations.
You may use a calculator.
The number of marks is given Jn bracket& [ ] atthe end of each question or part of the
question.

For Examiner's Use

2
3

[,

4
5

L
l

L
L

Total
--

This question paper consists of 12 printed pages (including this page)

L
1-

.\

[
1(a)

Draw a displayed formula for a molecule of ethanoic acid, indicating clearly


the bond angles.

-.,

{2]
(b)

Three experiments were performed where different masses of ethanoic acid


were expanded into a vessel of volume 15 dm3 at different pressures and at a
temperature of 100 C.
Mass of ethanoic
acid /g

Pressure
I Nm-2

0.002

6.346

0.015

31.011

0.050

86.128

Experiment

(i)

-Relative

[
[

molecular mass

'

[
[
['

Assuming ideal behaviour, calculate the relative molecular mass of


ethanoic acid for each experiment (A to C) and enter their values into
the table.

[2]
(ii)

Explain the trend in relative molecular mass of ethanoic acid with


increasing pressure.

L
[
[

[
[

L
[2]

u
11

[
[

(iii)

If experiment B was repeated at 150 C, keeping all other conditions


constant, predict qualitatively what change in relative molecular mass
you would expect for the system.

[
[

[
[

L
[

[2]
(iv)

Draw the structure of the predominant speCies of ethanoic acid at high


pressure.

.,

_,

[
[1]

[Total:9]

["
[
[

l'
L.

L
[
[

L
. I
:l

2(a)

l
f

Acidified potassium manganate(VII}, KMn04, oxidizes ethanedioic acid,


H2C204, to form a colourless !;las. The cell reaction, if operated under standard
conditions, gives an e.m.f. of 2.01V.
Write ion-electron equations for the two half-reactions taking place, and
calculate the E0 value for the oxidation reaction.

f
[

'

[
[
[3]
(b)

~-~

1.27g of iron(ll) ethanedioate were made up to 250 cm of acidified aqueous


solution. 25.0 cm 3 of this solution reacted completely with 26.50 cm 3 of
0.0200 mol dm-3 KMn04 solution.

Calculate the mole ratio of KMn0 4 to FeC 20 4 taking part in thls


reaction.

(i)

['.

ll

u
(ii)

Hence write an ionic equation for the reaction between KMn04 and
FeC204 in the presence of H2S04

[4]

u
u

L
L
L

[
[
[
[

(c)

The rate of reaction between potassium manganate(VII) and ethanedioic acid


in dilute sulphuric acid is shown in the figure below:

rate

[
[
[

time
Explain the shape of the curve as fully as you can.

[
[
[
[
[
[

{3]
[Total:10]

[
r-

L
l
l
5

L"
f

L
[
3(a)

(i)

[
On each of the grids below, sketch the general trends of the properties of
the elements or their compounds across the third period of the Periodic
Table.

pH of
chloride

[
[

II

Electrical
conductivity

[
Na Mg Al Si P S C!

Na Mg AI Sr P S Cl

L
(ii)

. Briefly explain the shapes of each of your '.sketches.


I

pH of chloride

[
[
II

r-

electrical conductivity

L:
[

[5]

L
L
I.
L_

l
r.

[
[

(b)

Bauxite is the raw material from which aluminium is obtained. The first step is
to purify the ore, separating aluminium oxide from iron and silicon oxides.
Suggest how aluminium oxide can be separated from iron and silicon
oxides, writing equations where appropriate.

(i}

[
[
[
[

[3]

How can aluminium metal be eventually obtained from its ore,


aluminium oxide?

(ii)

[
[
[
[

[1]
(c)

Aluminium hydroxide is a very sparingly soluble solid.


Ksp
4.6 X 10-33 at 25C.

(i)

Write an expression for the

Ksp

of aluminium hydroxide.

[
[1]

(ii)

Calculate its solubility at 25C.

L
L
L
L
L

[2]

'7

L:
:.'

L
[

(iii)

Predict what will happen when 25.0 cm of 0.003 mol dmaluminium nitrate is mixed with 30.0 cm 3 of 0.001 mol dm-3 sodium
hydroxide.

[
~

[1
[
[

{2]

(iv)

Explain the data given in the table.

Compound
Na 2 0
Alz03

Melting point /C
1132
2054

[
Electrical conductivity in
aqueous solution
High
Nil

'

[
[
[

u
{4]
[Total:18]

u
L
[

r
[

Poly(methyl methacrylate) or 'Perspex' is a synthetic polymer often used as an


alternative to glass as it is lighter and does not shatter. A section of Perspex is
shown below.
H

COOCH3

I
I

CH3

CH3

I
I

(a)

COOCH3 H

I
I

-'-C

COOCH3 H

CH3

Suggest with reasons, whether you would expect Perspex to be biodegradable.

[
[
c

t~

[21
(b)

Suggest the structure of the monomer from which Perspex is formed.

[
[

[1]
. (c)

The monomer of Perspex can be synthesised through the following


sequence of reactions.

OH

CH3COCH3

H2S04 (aq)
.....
/

H3C-C-CH3

heat

AI203

I
COOH

.....

heat

II

[
[

l
L
L
r

Monomer of Perspex

(i) .

Suggest reagents and conditions for:

1: _ _ _ _ _ _ _ _ _ _ _ _ _ _ _ _ _ ___
II:_ _ _ _ _ _ _ _ _ _ _ _ _ _ _ _ __

.9

~
_...

~
.....
~

........

!"'"

3
0'
...,

c;r

::J

::r
II)

CD

CD

.....
::r

CD

0'

.,0

en

CD

c.

--

OJ

0.

:J

II)

>

!!'

0.

::J

"0
0

0
0

Ci'

:J

II)

ca

CD
0

0
-.
.....
::r

en

CD

c
$l
c...,

...,~

CD

:E
.....
::r

0
...,

II)

-:::

.::

I-

1.
[
5)

[
[

Phenylethanone, C6 H5COCH 3 , is used to create fragrances that resemble


almond, cherry, honeysuckle, jasmine, and strawberry. It is used in chewing
gum.
Phenylamine, CeH 5NH 2 , on the other hand, is used industrially to make
polyurethane and dyes.

(a)

Phenylethanone may be prepared in the laboratory by refluxing a mixture of


dry benzene and organic compound D in the presence anhydrous aluminium
chloride.

(i)

Write the structural formula of D.

[
[1]

[
(ii)

Explain why the reaction requires the use of catalyst.

[
[

[2]

: (iii)

c
>'

Why is it necessary for the benzene to be dry?

[1]
(iv)

Suggest a structural isomer of phenylethanone which reacts positively


with complexed copper(II) ions in alkaline solution.
Name the type of reaction.

l
L
[

[2]

ll

c
r
(v)

Upon reduction of phenyletharione, a new organic compound E will be


formed. Suggest a simple chemical test where both phenylethanone
and compound E will give the same positive results.

[
[
-~

1--

[
[
[3]

(b)

Phenylamine reacts with 0 to form an organic product F.

(i)

Give the structural formula of F and name the functional group in it.

[
[2]

(ii)

Write a balanced equation for the reaction between phenylamine and


aqueous bromine, stating the observations.

[
L:

[
r1

t
[2]
[Total: 13]

L
L

12

L
r

c:::.

~ ~
'-- '-...:

~ ;:=-

<::.

(j.J

-.l
w

;,. . i

'I

Pi

~~~:

s
........,

C>

{}J:::r:

?2-

"

"'
"'
"' :s.

s:-""'

~ ~:

\
l=l

r----

z:..

z:.

\;."""

J:.

0 - (\- 0

l!

' z.

(0

:r::

J)

:- 0I -o.. ,

tN

.....

....~

'?

::+-

~-

~-

('"

....,

l7' r
::.>

Ji

"'

l
(1-(\-0
l
J

z_

z:

(..

(j/

(\

....~ =o"' ~

z.
s
I

-L.

0'">

J:

2-0-

~.....

>
..

':.

';><-

---

-;,

.I.

-........
-z:
:,

f'\

:+-

. ., +

'-

~ ~s:

~
;t

"'

0-

.:;:-

\'

_;.

('-2

fJ

~-(l.-:I'

s
Q,.

~
..-

7'\:.

;;;.....$
C.

\S
"-'

,.....,

~~

'--'

.:I: )?
~

t=t

Y-

-c--f'-.r

I'

I-()-~

~.__

~-

4-

V'

<._s;

-4-

$:'

~-s;c.

-.s-

zG

.........

!::>

,--.

<::l

""

v.

'
<:>

(\,

<;:,

(l

;,:

to

,---,
l

Q-

'
I>
..,,

"'I

<"'>

).o

..
('

.,

1:::1

>v

_,.

N>

,1:>-1-

,...

_. .
_.,

iL

,..~")

';:'
~

1:;

r-

';>

s__t:_

~ 2J

--;p

~ ,..,
;:l...

("'>

<;:,

-.

"\:)

_j

"''

A,

::>--

-..)

z.

3
_..,

-....

'-'

;:z::
[;>

'i

v-1

;:,

<>

,]'.

~-

II-<

<:::,

"'

_,_

4-

('

......

'r;?
.....

c'

.:J.

,..j..

3"

...._

;l:
()

('.1
(">

:I..

(\

<
<-./

.:I.

5!
.._,

:z.

('.

<

.:,

<::>

s;
-~-~

i'

"

('.

- "'
('

~-~-

'> ,.
:::.

l'-..~
()

[21 Q

\.y

;;--

('.

(\

""<:)

~i

5...,

<::

[)

<-.,-::L

t_

s-

(\
0
(\

I'

..._;

(\

v-..

'--

;;;

(,

:;>
......

::t:
.-.......

<:::>

......,

>-

'--
'v

C)

""'

"'
t

-.(._

r,

s::

"-...

<:>

"'I

<::>-.,

;:::;<:'>

> "'

()

--....
.._

"

("

.__..

r"

'::r1.

_:,

(\

.l-.

r..,

~
..::.
(} ...;,
....
..::r:. ..::,
r_,

("

;.

\).

Cl

""

::+..

(\

0.

._;

"'

'-'

'-......J

..,
'"''
~

.:!:

1:::1

,......,

'--1

v-J
,..... ......

:1:::>

'I

II

"'

>

......

T\

'-

.....~

<r-:t

'w

~""'(

0",

'><:

I-'

0(1

)...)

'"'

IN

s:...

:s,

:sc

...C>

<:::>

'X

<:;--.

....:::;

)
'
co l/1
...j..
2"- ;s.....""' ~
'"

"'

'-

'"'

Vv

...__

<:::>

.......

::::,..

-\-..,

<>

.::;

~.

.,

...,_

..,_

~~ -::,

"' ;:;;""
~"""'
~ ~
......
w
..,..
::.

()

"

s:

5<..

'""'

"'

,..,

<::l:t'
.::,

:s

'

z.~

:s ;:"~
):>

~
"-.!'>

,,

~'<>

O::r-

~-

rl)

;::

t.

<""

:::(:

~ .

.::t

("

"?<>

.c;. ~
'-'

'h..

;;---.

f~ 8
~~

\::}-..1

":>

~
~

E. ~

"'

.:t

.1"'1.,

.;;::)

"'
~

(\

""'

('>

",,

I::
"""'
...,..

.:.

;:s~

"<: .:j- ....

"-'

-+

r----1
l

<:::)

"....

"-

-1- -

"
....-.

"'
'l.

('

<:>

"':sh

.:t:

C)

....::: cs-...
.,

1'0
~
-....
"' c1\
'+-.

--1-.

t'\

s:::

"

;;::: ~
::t:.
"'- .,. "" -"''

V\
Sl... "
'-'

""'

;:.

.......
c:,

&

L
[
[
NATIONAL JUNIOR COLLEGE

PRELIMINARY EXAMINATIONS 2006

[
[

CHEMISTRY
Paper3

9258/03

Monday 11th September 2006


2 hours 45 minutes

Additional Materials: Data Booklet


Writing. paper

[
READ THESE INSTRUCTIONS FIRST

Write your name, registration number in the spaces provided on the answer cover sheet

Write in dark blue or black pen.

-,

and on each piece of writing paper.

You may use a pencil for any diagrams, graphs or rough working.
Do not use staples, paper clips, high lighters, glue or correction fluid.

'

. Answer all questions in Sections A, B and C. The last question in each Section is of the

[
[

form either I or.


In Section D, you are required to answer both questions.

Start each question on a fresh piece of paper.


A Data Booklet is provided
You are reminded of the need for good English and clear presentation in your answers.
You should not attempt questions on option topics which you have not prepared.

L
L

L
[

You may use a calculator.


At the end of the examil)ation, fasten all your work securely together.
The number of marks is given in brackets [

] at the end of each question or part

question.

This question paper consists of 13 printed pages (including this page)

l
._

[
f

c
[

Section A
Answer all questions in this section.

(a) Lead, an element in Group IV, reacts with oxygen under different conditions to
form two oxides, lead (II} oxide, PbO, and lead (IV) oxide, Pb02~
Melting point I C

Species
PbO
Pb02

(i) Explain in terms of structure and bonding, the variation in the melting point of
the two oxides of lead as shown in the table above.

[2)
(ii) The two oxides of lead have different reactions with hydrochloric acid.

Pb02(s) + 4HCI(aq)

-7

-7

,-

890
290

PbO(s) + 2HCI(aq)

[
[
[

PbCI2(s) + H20(I)

PbCI2(s) + 2H20(I) + Cl2(g)

State the role of each oxide in the reactions described, giving your reasoning.
[2]

(b) The lead acid storage battery is probably the best known rechargeable battery. It
can produce a large initial current, a feature essential to starting an automobile
engine. The anode of this cell is metallic lead while the cathode is also made of
lead but it is covered with a layer of compressed, insoluble Pb02. Both the
cathode and the anode are immersed in an aqueous solution of sulphuric acid
which acts as an electrolyte. Under normal operating conditions, lead(ll) sulphate
will be formed at both electrodes.

(i) Write the ion-electron equations occurring at the cathode and the anode.
[2]

(ii) If the starter motor of the car draws a current of 50A from the cell for 5.0
seconds, calculate the mass of lead consumed.
[2]
(iii) Suggest how the battery can be recharged and state what changes occur
during this recharging process.
[2]
[Total: 10]

L
[".-

L
t
L1

u
l~
2

L
L

[
[

(a) (i) Define the term standard enthalpy change of neutralization.

[1]

(ii) The table below shows the standard enthalpy change of neutralization when
various acids are neutralized by sodium hydroxide

Acid
hydrochloric acid
nitric acid
ethanoic acid

Standard enthalpy change of neutralization


-57.3 kJ mor1
-57.3 kJ mor1
- 55.2 kJ mor1
-

----

----

-------

Explain the above data as fully as you can.

[
[

[2]
(b) (i) Both aqueous potassium carbonate and aqueous hydrogencarboilate react
with hydrochloric acid to give the same products.
In an experiment to determine the enthalpy change of reaction between
aqueous potassium carbonate and hydrochloric acid, 35 cm 3 of 1.0 mol dm--3
aqueous potassium carbonate was mixed with 45 cm 3 of a 2.0 mol dm-3
hydrochloric acid. The temperature of the mixture rose by 5.2 C. Calculate
the standard enthalpy change for the above reaction.
(Assume the heat capacity of all solutions to be 4.2 J K-1 g-1)
[2]

c
,,

(ii) The enthalpy change of reaction between aqueous potassium


hydrogencarbonate and hydrochloric acid is given as + 12.2 kJ mor1 .
Use this information and the result of (b)(i), find the enthalpy change for the
following reaction.

[
[

c
->

2HCo3-(aq)

-7

C032-(aq) + H20(I) + C02(g)

[2]
(c)

Use the following data as well as any other relevant data from the Data
. Booklet, draw an appropriate energy cycle and calculate the enthalpy change
of formation of gaseous carbonate ion, C032-(g).
Enthalpy change of atomization of potassium
Enthalpy change of formation of potassium carbonate
Lattice energy of potassium carbonate

[
[

- 1151 kJ mor1
- 2084 kJ mor1
[3]
[Total: 10]

L
L'
t

+ 89 kJ mor1

[
[
3

f_

Either

(a) The table below gives the acid dissociation constant, Ka. of 3 acids at 25C.
Name of acid
Ethanoic acid

Formula
CH3COOH

Chloroethanoic acid
Methanoic acid

CH2CICOOH
HCOOH

Kal mol dm-3


1.8xl0-5
1.3xio-3
1.8xl0-4

'~

~~

State and explain in terms of structure and bonding the obseNed trend in the
acid strength as shown.
[2]
(b) An analytical procedure for separating certain components of a dietary
supplement requires that they be dissolved in a buffered solution having a pH
of 3.85. It was found that methanoic acid is the best acid to use for the
preparation of this buffer. Explain why this is so and suggest how such a
buffer may be prepared.

[3]
{c) 25.0 cm 3 of 0.05 mol dm-3 of methanoic acid was titrated against 0.1 mol dm-3
of NaOH.

[
[

[
l.~

Calculate
(i) the initial pH of methanoic acid.
(ii) the pH after 10.0 cm 3 of NaOH had been added.
Hence sketch the pH cuNe during this titration, indicating the significant
points.
[5]
[Total: 10]

[
C'
L

L
L
f!
t_j

u
l~
4

L
L
l

c
r
[
3

Or
(a) The blood in our body uses carbonic acid and the hydrogencarbonate ion to
maintain a remarkably constant pH.
Write equations to explain how the constant pH is maintained.

[2]

r
,.,.

[
[

c
[

(b) 25.0 cm 3 of a 0.05 mol dm-3 aqueous solution of sodium carbonate was
titrated against 0.05 mol dm-3 HCI using screened methyl orange as indicator.
Given Ka 1 (H2C03) = 5 x 1o-7 mol dm-3
Ka 2 (H2C03) = 5 x 10-11 mol dm-3

(i) Write an expression for Ka 1 of carbonic acid, H2C03.

[1]

(ii) Explain why Ka 2 is much smaller than Ka 1

[1]

(iii) Calculate
(I) the initial pH of 25.0 cm 3 of aqueous sodium carbonate.
(II) the pH after 10.0 cm 3 of HCI had been added.
Hence sketch the pH curve during this titration, indicating the significant
points.

[6]
[Total: 10]

L
(
-,

[,,

L
L
[

[
[

L
['

c
c
Section 8
Answer all questions in this section.

(a) "Copper is a transition elemenr


Explain what is meant by the above statement, illustrating your answer with a
suitable example.
{1]

(b) Some physical properties of potassium and copper, which both contain one
Potassium , K
[Ar]4s 1
64
0.231

electron in the outermost shell of their atoms are shown below:

Electronic configuration
Melting point I C
Metallic radius/ nm

Copper, Cu
{Ar]3d 104S 1
1083
0.128

L
[

Explain the difference in

(i) the melting points, and


(ii) the metallic radii
of these two elements.
[3]
(c) Gemstones are valued for their ability to reflect light. They usually appear
coloured as a result of small traces of impurities. Turquoise is one such
gemstone highly priced for its sky-blue colour. Suggest how the copper present
in turquoise can give rise to this colour.
[3]

(d) When copper(ll} chloride is dissolved in. concentrated hydrochloric acid, the
complex ion [CuCI4f- is formed.
Using appropriate equations, explain the gradual change in colour from yellow to
green and then blue as this solution is diluted.
[3]
[Total: 10]

L
L
[

f
[
!'
lj

[I

u
6

L
[
\

L
[
[
[
[

[
[
[

[
~,

f
[

,_,

-,

Either
This question concerns the Group n elements and their compounds
(a) Group II elements are known to be reducing agents. Give an example of a
chemical reaction that illustrates the trend in the reducing ability of the
Group II elements.
[2]
(b) Magnesium reacts with ammonia to form a yellow binary salt, X, and hydrogen.
In another experiment, 0.198 g of compound X upon complete reaction with
water gives a white solid Y. Upon filtering to remove the white solid Y, the
resultant solution which contains ammonia requires 20.0 cm 3 of 0.1 mol dm-3
sulphuric acid for reaction.
Construct balanced equations for the two reactions.
[3]
(c) Beryllium carbonate must be kept in an atmosphere of carbon dioxide,
whereas no carbon dioxide is detected if barium carbonate is heated using a
bunsen burner. Explain these observations fully.
[2]
(d) The solubility products of some Group II compounds are shown in the
following table:
Solubility products of Group II compounds

Ba(OH)2

BaC03

2.60 X 10-9
10-9

Ca(QH)z

7.88

10-6

CaC03

4.95

Mg(OH)z

5.66x 10-12

MgC03

1.15 X 10-5

_J

(i) With reference to the data given above, suggest a reason why when
small quantities of carbon dioxide are to be detected, barium hydroxide
, solution is occasionally used instead of limewater.
(ii) State one disadvantage of using barium hydroxide.
[3]
[Total: 10]

[
L~
[

L
L
[_

L
['
[
[
5

Or

(a) Bromine is obtained from the sodium bromide dissolved in sea water in the
following manner:

Sea water is
concentrated by
evaporation

I --+ I

[
Step I
Pass chlorine in

Step V
Distil out the
bromine

I-. I

Step II
Blow air in

Step IV
Acidify the
solution

Step Ill
Channel into vats
containing sodium
carbonate solution

(i) Explain why air is introduced in step II.


(ii) Sodium bromate(V) is produced in Step Ill. Explain why such a reaction
occurs and write an ionic equation for the reaction.
(iii) Suggest a reason for adding acid in step IV.

[4]
(b) The halogens are oxidizing agents. Give 2 experiments by which the trend in
the oxidizing power of the halogens can be compared.
[2]
(c) Iodine and chlorine react together to give compound A, lxCiy. When a sample
of A was dissolved in an excess of aqueous potassium iodide, a brown
solution was obtained. The brown solution required 34.0 cm 3 of 0.50 mol dm-:-3
sodium thiosulphate, for reaction. When the experiment was repeated with
another sample of A of the same mass with excess potassium iodide, it was
found that a yellow precipitate was first obtained with aqueous silver nitrate.
When the yellow precipitate was filtered off and more aqueous silver nitrate
solution was added, a white precipitate of mass 1.83 g was obtained.
Construct a balanced equation for the reaction between compound A and
potassium iodide.

[4]

[
[
[

r'
['

L~

[
[

[Total:10]

L
8

L
L

u
r

L
[
[
Section C
Answer all questions in this section.

[
6

[
[
[
[

Coumarin is a naturally occurring organic compound with a grassy odour. A series of


reactions is shown below.
-

II

coumarin

CH=CH-C-CI

COH
.1

heat

HCI(g)

NaOH(aq)l Heal

0,0(1)

H2 S04 (aq)

KMn04(aq)
heat

[
(a) Name the type of reaction in I.

[1]
(b) Draw the structures of organic compounds P-T.

[5]

(c) Suggest separate simple chemical tests you could use to distinguish coumarin
from X and Y below:

-,

[-

[
[
[

X:

ro

Y:
0

[_

[4]
[Total: 10]

L
[

ro

For each test, give reagents and describe what you would see.

OH

l_

f'
.i

[
[
7

Benzocaine is well known for its local anaesthetic properties and is commonly used
in antiseptic creams and sunburn remedies.

~CH2CH3

[
[

NH2
Benzocaine

The following shows the synthetic route to benzocaine.

Step 1

N02

Compound X

C02CH2CH3

a series of
----J>-

reactions

QJ

r'

NH2

Benzocaine

(a) State the reagents and conditions for step 1, and describe the mechanism for this
conversion.
(4]
{b) Suggest how Compound X can be converted to benzocaine. Give reagents,

conditions and the structural formula of the compound obtained for each stage of
the conversion.

r
.)

rl
__ )

(3]
{c) Explain how a mixture of Compound X and benzocaine can be separated by
chemical means.

[3J
[Total: 10]

[J
[j
[I
[

10

L
L
l

[
[
8

When a mixture of alkane and chlorine gas is exposed to sunlight, a mixture of


chloroalkanes is formed. When one of the chloroalkanes, A, is boiled with aqueous
sodium hydroxide, an organic compound, B, is produced.

['

Other than the molecular ion peaks, the mass spectrum of A shows a set of peaks at
m/e values of 83, 85 and 87 with relative intensities in the ratio of 9 : 6 : 1
respectively and another peak at m/e value of 29.

The NMR spectrum of 8 consists of a triplet at <> 1.1, a quartet at <> 2.5 and a singlet
at <> 9.8. The integrated areas of these three peaks are in the ratio 3 : 2 : 1
respectively. None of these peaks is affected by the addition of D20. 8 also has no
reaction with aqueous alkaline iodine.

A reacts with hot ethanolic sodium hydroxide to produce a polar organic compound
C. When C reacts with HBr gas, an optically active compound D is produced.

Identify compounds A-D, and explain the above observations.


You should write equations for the reactions described wherever possible.

[
[
[
[

[
-,

Either

[Total: 10]
8

Or

Compound A has the molecular formula C 10H 13NO. On heating with aqueous
hydrochloric acid, it gives compound 8, C 6 H8 0 2 and compound C, C2HsNCI. The
NMR spectrum of B shows a 3-proton singlet at 2.5, a 4-proton multiplet at 8.0
and a 1-proton singlet at o 11.5.

Compound B reacts with bromine in the presence of FeBr3 to form the organic
product D, CsH102Br. Compound C reacts with aqueous sodium hydroxide to form
Compound E, which with excess CH 3CH 21gives compound F, CsH2oNI
Deduce the structures of compounds A;.;.. F. Explairl"the spectrum a-nd the chemistry
of the reactions with relevant equations wherever possible.
[Total: 10]

[~

L
[

l
l
[
L"

11

l
[-_

[
[
Section D
Answer all questions in this section.
9

A series' of experiments were performed on a food mixture M.


Experiment I
A small sample of M was heated with aqueous sodium hydroxide, followed by
treatment with Fehling's solution. A brick-red precipitate was obseNed.
Experiment II
A sample of M was subjected to heat in the presence of air. The black residue
obtained was divided into two portions. One portion was added to water and shaken
vigorously. A second portion was shaken with ethoxyethane. Chemical analysis of
the ethoxyethane extract showed carbonyl groups to be present

~~

[
[

Experiment III
Iodine value determinations were carried out using samples of M. The results
obtained were inconsistent and inconclusive.

[
--,

(a) Explain as far as you can in structural terms the obseNation in Experiment I.

[2]

[1]

(b) What was responsible for the black residue obtained in Experiment II?
(c) What food component gave rise to the observation of the ethoxythane extract in
Experiment II? Briefly describe how this food component changed chemically
during heating.

[3]
(d) No micronutrient analysis was carried out on either extract. Predict what type of
micronutrient could be present in each extract.
[2J
(e) Briefly explain the results of Experiment ill.

[2J
[Total:10]

-"

---,

_)

[J

[I

u
[

12

L
L
l

[
[
[

10 (a) Explain why potato crisps, although they are contained in sealed packages, may
become soft on exposure to light. How can the shelf-life of such food materials
be extended?
[3]

(b) The ingredient lists on food packaging often include such compounds as
sulphur dioxide (E220) and ethanoic acid (E260)

(i) State TWO functions of sulphur dioxide in food processing


(ii) What problem can arise if the concentration of ethanoic acid becomes too
high?
[3]

[
[

(c) Give TWO reasons for the careful control of the moisture content of certain
foods during processing.
[2]
(d) Lead and cadmium are contaminant metals. Suggest how they may enter the
food chain.
[2]
[Total: 10]

[
[

[
[

L
[

L
[

13

r:--'

,--,
I

,....._._..

'

I)

!:-----1

l.

'~~

s~ c. ~... l:h-""-cJ:-v:, ~s. ~'""" ~b t ~


O'-- ..___;,"-".
<A.eu.j
PbO.,_ r-"'f/'-....i-i>> CA...

"d

VVIAA

<.~~ ~ ~
p b 0;. (h-0--~ ~.......

'':"""

~"'--"

P60 -

(l>) (_;)

ct-...... a.\ho. c__h., ~! (b I )

-\-o so-.Q..t-

Ci _.

in t{cu>'--t~
.

ct ~v-..t ~ 1--~ v<:..R_. ,.,__

~~ v-> G\ ~ c.J--vo ,......_, ~~

L'"J ~ "'--- ~

'f!6 ~<i-

-t \--il.. 0

'

-==-ws

ftt

HtCv 3

flit /-~)10=1-

{11-\-::: ioS

Ct0

~ t?b
------::> P bO:>-

5"-t,.jer--- a.--rt:.~c--t;(;i'l -h-r iit

\:>11)

Cc\)

~ Go...k-k.-'---'-' -:.

Li-i z. Cos J

~-2. fl. .

t-<..-U-e..-v-.'L_.

~ t-l1 Co J

Li-ttJ [ i-{Cv~ -J

~-=l

J<n-"--'0

i:>~ .~h.~H H U

tJ.-v.....oo0,_ ~. fh --t ~-o'-~-1.- --"> Pb vo'< -t-?.x_.-:o-..i-1---.::>"""- " PbO2 -r4-~r+ '*'> ...L--+ k -">f'GR-0't t-2.i~ ' 0

\_'.->_

H Co r- f K i

\-\z.C<> 1 1 olt- -") H:Co

PbO.,_-o~ tA)"'--V~-t-~Hcz+-oi:..Q,_

--vt ~c.{),.__

V>

~ v~ ~ wo-.o-.JL,

(_;:)

(d

4-

[
[

'.~Or

(~)(i) {\AQ..Qk,~ ") (760 ~I.M.-~~ '"'"f~


~'-'k'/\ t; o v~ C...O<-"'-Q... S +-vo ~"} .,_._b. J-w -

SitwJ~ ~Jpy c-h~e 0 ~ 1\ eukJilJJOI)


sutfh"eJ as it-,e heJ-euolvPJ uJhe.., I ..,..o\e of
Jet~ eJ i 5 ~ fYt\fJ h~ the r.Xf\ of CVi' vtcid lVIcl C(
()(,(? J s~~Ja.J rt7YI o{;ho o<; Cl J M I .)q &k'.)

~fKZ~

;>

l)

~c

V1>(

CH Cf J

(\( i)

[
[

4-W) A -hwl<;;ihJv\l.eu\'~

a-f

stkMV'f

OVtiL

e.~. Cu ~Gf_

11"1--Jiil-! ~I'WJ<:;'

()VU,

tu~rUV11-1J

vvi+k

0...

puvfiC1\~

d-~ul:>~~~ (-

-t"iiLul
!

I(

U. 2t : (A,] 3d '1

~cii)- AI\ s{.{ 4 &~ lfttd-with N40H foprocL.ue\4-(b)Ld-ttw~ cldoo. tt1e_d ~<"- f llteclwV'\ ~ ~.
C\ fYIOI('o f wak- ti b~ 9 5"7. 5 k l 0 r ell-ei'q 0 i
- Sfw<1cyv 11'\.tft.l!:c '::>u: J _

-ncl ~ct !-\NO~~ Jhon~ ctc.u:JJ ]1J-N'~ jt~Ud-t s~ietdt.4 ~.:: f ..,~.ett~ ....c:. 4,.. shi&~l&J
d.\ C,<,o ci J pc,(
I
~ ~' r o.~J. d --(,n~S . d- e{t (tv. ... s ""-<.
~ C\h[D L H ;~ c{ we.4t ctad ~ porl ot ~ e j
pooY Sv~~
(,; ~ eMPv{ ~ To be .0C'J to he{p i
Cv. etpe"";~~c.l i1'9hY, ~<1''-<. n""ct.J..... a.r
.n ~~

...
l -- .Siu.Vly
t:tttvuc.t,\,._.. -r.- ~.-u~U....tz dx.d~.svos .
-the cW~ w "'/>I Je (~ cl.i< soU: a.f e
If(,) - rc.. <H.o'1 n 'ii'"-' "'k +" 4lu ;;..,_ """"'

[
[

[
[

f\

fv1t.WL ~

V-ttiv<,\01

f'.:J

l?v.te-. k2

-f-4. !-J<-vt./ .

)Ci)

- Colw,- 4w.

Al~fjfl::- 41~!LJr.-wl-l

q._(()

Aitt

e 11\1e"
Ca]

_;

f.. a-'1.)
~ p0lc-rf}~

~b)

PK"-

1'\e..-re,-

-to

[ 5utq =. [c. q J

~Ct)

~C\\ )

2-SL

"

k,_,vJ

tvtJ.t~I'J [(_t.,(H10}41q

1:,"'-f~lJ Cv('

~ph

e,fryif

<0 RP...odio11

of

reck<P-.9f

"to

,]

Cjj'y.,i,~ h'l.~ .f-Q

lb..

-f{....._

ole.r.Jell-fs "';-+J., H.:>.O t<>~e~ H.. o is


cler di{f<>-~rrl- co-tlddions ( .steoM ---7

WI

wofer )

-r f\93N2.

n..,.,H3 =-J.n 14 >-."""0


=- :~r~
~ u 1 = o-o"'t-"'t::ti
;- 4
1000 "
IL/1g3 JJ,. "" 0 I qo/'loo 'j :- 0 0 0 I q f,
A.

fl.rJH3 ~

71-

v<>I c~-,.~~~<H-l )

Gp 11

343 -+ ::lfJf/3 --7 3H"-

;,rn Mj3N,_

re"bd

't ) It-

.j::;Y>'-\5

b\ 1M- col I.>W'

{h_trJ(_

1-trO sl.;fK -~"" +.:J 1-ff f-.,,J ~i~ . MY~


f U.(\-4()ltt1n ti, ... J. Cc.....e&,-]1- Cf\<.." +~ ~- w~.v..-.

ool d

1 "J'!;

Ac{dt't~'--'-

..~~

Co'i,J "t "- ceq, J _.

6 -I:-\

colv-..-.

4-

Cfi~CICu\ll-t > Hwoi-{) (f\.JC0uti


rt

[Ct.CILty- ~IV<.~ VI\~ h +~ le\\Gv<~

) :: - <o\k.)t'Ylol-\

((()~}-

+._, d-S'plt f-/:.1..) <w.J.. c.f -c.\ -f"'""'~sift-, ,

4ld) [u..<.tt.o)+Y* + 4-<r ~ [ Cv.-014-Jl- -t t ~ ().

bcii ) 11\irj(f\ ::: i 14 s 1L J ~o 1-'

Ll

L
L
L
L

\ _If

or

etf' ,

_ o -004
- 0 -ool'l'

;:;_

Mg3 N2 -t .3 H,_o ----7 3 nsO + .;tNH 3


l4g 3 th -+ H:>.O ----7 ]Mj {oH\. -t- .2. Nfb

?='

-<
'../

'<..
z:

~'

c;"':>

=t

:c

"'0

<::;>

--<

r-

~.

e j:

1\

t-'

"Z
.:I:

}J

\$1

;:!:

('""

(\\

['

.::t::.
"'-+

I"
'2::

<>'

..:r:

("":>

("'>.
(

----------:----H------.--------------~----------

!l.-

I'

.~

..,
r

~
I

""--'-

-<

r-r;-

.:L:."t>

,....

vr

0>

~\

~
~

(9

..

I"

/
(")

1!

z
i'.._

(V;f:>

,..,

'\

(;

'J'"

'"I:>

'-'

'--'

'

,.-.
L ,

r-;

;------'
l

,--I

,---

/
{"-- .... _,'

[
[

[
[
[
[

[
[
[

CANDIDATE NAME _ _ _ _ _ _ _ _ __

Class _ _ __

SERANGOON JUNIOR COLLEGE


JC2 PRELIMINARY EXAMINATION 2006
CHEMISTRY
PAPER 1
THURSDAY

9251/1
9258/1
21 SEPTEMBER 2006

1 hour

Additional materials:
Multiple Choice Answer Sheet
Soft clean eraser
Soft pencil (type 28 is recommended)
Data Booklet

TIME : 2.00 - 3.00 pm


INSTRUCTIONS TO CANDIDATES
Do not open this booklet until you are told to do so.
On the separate optical mark sheet given, write your name, subject title, class in the spaces
provided and ensure that you shade correctly the item index number, starting with the number "2"
and the last 4 numbers of your FIN/NRIC number.
Eg. If your NRIC is S8706789C, shade 26789 for the item index number.
There are forty questions in this paper.
Answer all questions.

[
[
[

For each question, there are four possible answers labelled A, B, C and D.
Choose the one you consider to be correct and shade your choice in soft pencil on the separate
optical mark sheet.

INFORMATION FOR CANDIDATES


Each correct answer will score one mark. A mark will not be deducted for a wrong answer.

L
[

A Data Booklet is provided. You may use a calculator.


Any rough working should be done in this question booklet.

L
[

l
L-

[Turn over

..

~~--

SERANGOON JUNIOR COLLEGE


PRELIMINARY EXAM 2006- CHEMISTRY PAPER 1

Section A
For each question, there are four possible answers A, B, C, and D. Choose the one you
consider to be correct.

c
D

Which of the following quantities is equal to the Avogadro constant?


A
B

The number of atoms in 1 dm3 of carbon monoxide at 273 K and 1 atm.


The number of oxygen atoms in 100 g of allactite, Mn 7(As0 4 )z(OH) 8 , of molar
mass 798 g mor1 .
The number of ions in 165 g of K 3 Fe(CN)6.
The number of ions in 168 g of Reinecke's salt, NH4[Cr(NH 3 }2(SCN) 4 ].

A sample of the hydrocarbon C6 H 12 is completely burned in dry oxygen and the


product gases are collected as shown. [A,: H, 1 ; C, 12; 0, 16]
excess of 0 2

H2 0 + C02 + excess of 0 2

_...

___..__.,_

(to absorb C02)

(to absorb H20)

The increases in mass of the collecting vessels P and Q of the apparatus are Mp and
M0 respectively.
What is the ratio Mp I M0 ?

B 0.82

1.2

SRJC (2006) Chemistry Paper 1

+2

+3

[
[
[

[
l~

D 2.4

1.978 g of an oxide of an element X, represented by ~On (Mr: 197.8), is found to


react with 0.008 mol of acidified potassium manganate(VII) solution: In the reaction,
aqueous HX04z- ion is formefi. By considering the reduction of manganate(VII) ion to
manganese(II) ion, deduce the oxidation number of X in X 20n.
A +1

+----:'-- sodalime .

A 0.41

anhydrous calcium--+-~
chloride

L
I~

D +4

[Turn over

'~

L
r

A mass spectrum is shown below:


Relative
Intensity

10

A
B

!)

20

28 30

40

44

m/e

A
A
A
A

mixture
mixture
mixture
mixture

of CH 4 and N2
of CH 3CH3 and NzO.
of C02 and CH 3CHO.
of N02 and CH3CN.

Acetonitrile, C2 H3N, is used as a solvent


What is the number of cr and n bonds present in the molecule?

A
B

[
[

14 16

Which one of the following gives the complete mass spectrum illustrated?

()

1t

2
5
6
7

2
2
1
0

A/C/3 reacts with LiA/H 4 and (CH3)sN to give (CH3)sNAZH3.


Which statement about (CH 3 )sNAZH 3 is correct?

A
B
C
D

It contains hydrogen bonding.


It is dimeric.
The Al atom is electron deficient
The bonds around the Al atom are tetrahedrally arranged.

L
l
L
l~
t_ .

SRJC (2006) Chemistry Paper 1

[Turn over

L
. ~
7

In an experiment, 0.10 g of the volatile liquid Z formed 0.025 dm3 of vapour at 100 oc
and at atmospheric pressure. By assuming that 1 mol of vapour occupies 22.4 dm3
at s.t.p., what is the relative molecular mass of Z?

0.025

273 X 22_4

0.025

0.10 X 373

0.10 X 273

22.4

0.10 X 373 X 22.4

0.025x273

Using relevant information from the Data Booklet and the following data, which is the
correct energy cycle to represent the average bond energy of the Si-C! bond?

11H
!1H
A

at(si)

=+ 338 kJ mor

f(SiC/4)

=- 610 kJ mor1

Energy

. Si (g)+ 4C! (g)

f'.

+ 488 kJ

Si (g)+ 2C/2 (g)

I'

I'

4 X B.E (Si-C/)

+ 338 kJ

Si (s) + 2C/z (g)

-610 kJ

Energy

. Si (g) + 4C/ (g)

,,

+ 976 kJ

Si (g)+ 2Clz (g)

"I'

4 X B.E (Si-C/)

isi

[
(g)+ 2CI,(g)

.if'

+ 338 kJ

Si (s) + 2C/z (g)

B.E (Si-C!) ..

Si (s) + 2CI2 (g)

-610 kJ

-610 kJ
'II SiC/4 (g)

Energy

' Si (g) + 4C/ (g)

[
[

II SiC/4 (g)

II SiC/4 (g)

+ 338 kJ

4 X B.E (Si-C/)
Si (s) + 2Clz (g)

-610 kJ

+488 kJ

I'

Si (g) + 2C/z (g)

+ 338 kJ

Energy

[
[

. Si (g) + 4CI (g)


+ 244 kJ

373 X 22.4

0.010 x273

0.025x 373

'

' it SiC/4 (g)

L
[~

L
L
SRJC (2006) Chemistry Paper 1

(Turn over

[
~

In an experiment, 80 cm 3 of water at 25 C was bought to boiling point by


burning propane in excess of oxygen.
Given that the enthalpy change of
combustion of propane is -2220 kJ mor1 , calculate the volume of propane
needed if this process is only 90 % efficient.
Assume the heat capacity of water is 4.3 J g-1 K 1 and 1 mole of gas occupies 24 dm3
at ordinary temperature and pressure.

10

103 cm 3

251 cm 3

279 cm3

310 cm 3

The data below refer to the radii and charges of six ions:

v-

R+
0.05

Ions
Radius/nm

0.05

PT, QU and RV are ionic solids of the same lattice structure. Which of the following
gives the correct order of their lattice energies with the highest numerical value first?

[
[

A
B

QU

RV

PT
PT
QU

RV
RV

RV

PT

QU
PT
QU

[
11

[
[

L
[

A current of I ampere was passed fort seconds through a molten salt containing yn+
ions. The mass of Y deposited was mg. If M is the relative molecular mass of Y,
and e is the charge on an electron, then Avogadro constant is given by

12

fxtxM
mxnxe

fxtxm

Mxnxe

mxnxe
fxfxM

Mxnxe

Jxtxm

The following reaction is one of the steps involved in the manufacture of sulphuric
acid.

280 2 (g) + 02 (g)

~
~

flH

2S03 (g)

=-197 kJ mol-

The operating conditions are:


Temperature:
Pressure:
Catalyst:

450-550 C.
10 atm
V20s catalyst

'

Which factor influences the choice of these conditions?

A
B
C
D

The V20s catalyst decreases the equilibrium yield of S03 .


At lower temperature, the rate of formation of S0 3 decreases.
At lower temperature, the equilibrium yield of S03 decreases.
At higher pressure, the rate of formation of S03 decreases.

l
SRJC (2006) Chemistry Paper 1

[
L~

[Turn over

c
13

Cu3(P04)2 is a sparingly soluble sail


concentration of P043- at equilibrium?

A(~r5

(~)V5

If its solubility product is Q, what is the

108

(g~r5

D(80)1/
27

r
[
[

14

The graph below shows the variation in molar enthalpy change of vapourisation, AHv
for 8 consecutive elements in the Periodic Table, all with atomic number Z.:::;; 20.

~Hv/ kJ mor

[
[
[

[
Nuclear charge
What can be deduced from the above?
A

B
C
D

15

Element A forms a chloride which exist as dimer in the vapour phrase.


Element D is in the same group as nitrogen in the Periodic Table.
Element F exists as diatomic molecules.
Element G forms an oxide which is acidic in aqueous solution.

,The inorganic part of gallstones (formed in the gall bladder) is calcium ethanedioate
which is insoluble in water. The corresponding magnesium ethanedioate, however, is
soluble in water.
Which of the following best explains the difference in solubility between calcium
..ethanedioate and magnesium ethanedioate?

A
B

C
D

16

Magnesium is more electronegative than calcium.


Magnesium ethandioate has a lower solubility product than calcium

ethanedioate.
Magnesium ions has a higher enthalpy change of hydration than calcium ions.
Magnesium ethanedioate has a numerically lower lattice energy than calcium
ethanedioate.

Lime, CaO, is used to reduce the acidity of soil, and ammonium sulphate is a
nitrogenous fertiliser. Why must they not be used in a mixed form?

A
B

The soil will be too alkaline for plant growth.


CaS04, formed on mixing, causes hard water.
When dampened, ammonia is given off.
Sulphuric acid will be formed.

SRJC (2006} Chemistry Paper 1

L
[
[

r
L

L
I

L
L
[Turn over

L
l.

17

Aqueous sodium chloride (brine) is electrolysed by using inert electrodes in a cell


which is stirred so that the products of electrolysis are able to react. The cell is kept
cold. Which one of the following pairs of substances is among the final products?
A
B
C
0

18

hydrogen and sodium chlorate(!)


hydrogen and sodium chlorate(V)
hydrogen, chlorine and sodium hydroxide
hydrogen, sodium chlorate(!) and sodium chlorate(V)

An aqueous solution containing both sodium chloride and sodium bromide treated
with an excess of aqueous silver nitrate. The precipitate formed is filtered off and
washed with distilled water. The precipitate is then shaken with aqueous ammonia
and filtered off again.
Which ion is present in the final filtrate?

A chloride
19

[
[

c
0

',

20

-"

L
[

( '

At room temperature

After heating

srsr-and BroBrsr- and Bro-

BroBro3Bro-and Bro3Br- and Br03-

Chrominum(III) chloride combines with ammonia to form compounds in which the coordination number of chromium is 6. When 0.01 mole of one of the compounds is
added to aqueous silver nitrate, 2.87 g ofsilver chloride precipitate is obtained.
What is the formula of this compound?

A Cr(NH3)5C/3

21

c
[

0 silver

When bromine is passed into aqueous sodium hydroxide at room temperature and
the mixture is then heated, the products formed are

A
B

C sodium

B bromide

0 Cr(NH3)4C/z

Which transition metal in the following species has an unpaired electron in a d orbital?

A TiC/4
22

C Cr(NH3)4C/3

B Cr(NH3)sC/3

Mnoi-

[Fect4r

0 [Co(NH 3 ) 6] 3+

The medical treatment for a person suffering from carbon monoxide poisoning is to
administer pure oxygen. However, the same treatment is ineffective for a person
suffering from cyanide poisoning. What can be deduced from this?
A

c
0

The Kstab of the haemoglobin-cyanide complex is larger than the Kstab of the
haemoglobin-carbon monoxide complex.
Haemoglobin contains iron(II) ions which are hexa-coordinated.
The dative bond formed between haemoglobin and carbon monoxide is
weaker than that between haemoglobin and oxygen.
The dative bond formed between cyanide and haemoglobin cannot be
broken.

[
SRJC (2006) Chemistry Paper 1

[
t'

[Turn over

,f
23

Answer this question based on the following information:


[Fe(Hz0)6]3+ (aq) + e ~ [Fe(Hz0)6f+(aq)
[Fe(CN)6]3- (aq)
+ e ~ {Fe(CN)6] 4 -(aq)

E9 = +0.77 V
E9 = +0.36 V

Which statement is correct?


A
8

c
D

24

CW ligands stabilised Fe(II) with respect to Fe(III).


CW ligands stabilised Fe(III) with respect to Fe(II).
[Fe(CN) 6]3- is a stronger oxidising agent than [Fe(H 2 0)6] 3+.
H20 ligand binds more strongly to Fe(III) than CW ligand.

A new industrial preparation of ethyl ethanoate has been developed using cheap
sources of ethanol.

CH3CHzOH

Cu catalyst " CH3CHO

Cu catalyst

CH 3CH(OH)OCH2CH3

+ CH3CH 20H

- H2

Cu catalyst
-Hz

CH3COzCHzCH3

Which process is involved at some stage in this reaction sequence?


A
8
C
D

25

.How many isomers are possible for the compound of molecular formula C 3H5 Cl?
A

5
6

... 26

disproportionation
electrophilic addition
nucleophilic addition
reduction

[I

Which one of the following statements is true?


A
8

c
D

L~
[

Cracking is carried out to obtain a large proportion of volatile hydrocarbons.


Petrol mixtures with- high octane number are rich in aromatic compounds.
1,2-dibromoethane is added into petrol as anti-knocking agent.
Tetraethyllead (N) is added to reduce the amount of pollutant emitted.

L
[I

L
SRJC (2006) Chemistry Paper 1

[Turn over

L
f

r
[

27

One mole of compound M gives one mole of hydrogen gas with excess sodium metal
and one mole of NH 3 when heated with aqueous NaOH.
Which of the following could compound M be?

HO~CH(OH)CONHCH3

I
r-f(}~CHzCH
HOH2~

NHz

booH

Q-cH(OH)CH 2CONH 2

Q-cH(OH)CH(OH)CN

[
28

Which reaction could give an organic compound having deuterium incorporated into
the molecule?

c
['

Deuterium, D, is the ~H isotope of hydrogen.

CH 3CD2CH(Cl)CH(CH3)2

NaOD (ale.)
reflux

<'

H20, D+

CD3 CH 2COOCD3

reflux
DCN, eN

CD( CH 3)zCD 2CHO

10- 20C

.\

..J

29

Arrange the following in increasing pKb values.

Cl~

[
fj

CH(CH3)2CH20D + DBr

A
B

c
D

SRJC (2006) Chemistry Paper 1

6H2
R

NH3

S,Q,R,P
R,P,S,Q
P,S,R,Q
Q,S,P,R

l
L
[

NH,

[Turn over

L
30

Which one of the following reagents will distinguish U and V?


CH20H

C=O
OH

HO

A
B
C
D

aqueous bromine
Fehling's reagent
phosphorus pentachloride
alkaline aqueous iodine

Section B
For each of the questions in this section one or more of the three numbered statements 1 to
3 may be correct.
Decide whether each of the statements is or is not correct (you may find it helpful to put a
tick against the statements which you consider to be correct).

c
[

The responses A to D should be selected on the basis of


A
1, 2 and 3 are
correct

B
1 and 2 only are
correct

.2 and 3 only are


correct

D
1 only is correct

No other combination of statements is used as a correct response.

31

32

['

Which of these statements correctly describes an. electron shell with the principal
quantum number, n = 3.

1
2
3

Electrons occupy its orbitals starting with that of lower energy first.
It is made up of 9 subshells.
A total of 8 electrons can be accommodated in this shell.

Which of the following are correct statements about the H 2/02 fuel cell?

1
2
3

Fuel cell converts chemical energy of fuel into electrical energy by reduction
with oxygen from air.
Fuel cells work indefinitely as long as the reactants are supplied.
Fuel cells are pollution free since water is the only product formed.

SRJC (2006) Chemistry Paper 1

(Turn over

L
L

L
L
{

r
[

The responses A to D should be selected on the basis of

[
[

2 and 3 only are


correct

1 only is correct

No other combination of statements is used as a correct response.


33

34

For which of the following pairs does the first species have a smaller bond angle?
1

OC/2 , SnC/2

2
3

13-. N3-

IF 4-, BF4-

The graph shows the results of an investigation of the initial rate of hydrolysis of
maltose by the enzyme amylase. In the experiments, the initial concentration of
maltose was varied, but that of amylase was kept constant.
Initial
reaction
rate

[
['
[

[
...

B
1 and 2 only are
correct

A
1, 2 and 3 are
correct

What conclusions can be deduced from these results?

,.

-~

.1
2
3

L
[

[maltose]

35

. - When [maltose]- is high, the rate is independent of [amylase].


When [maltose] is high, the rate is independent of [maltose].
When [maltose] is low, the rate is first order with respect to [maltose].

Compound N

..

does not conduct electricity when in a liquid state,

when added to water produces a solution that readily conducts electricity.

What could N be?

1
2
3

MgC/z
SiC/4

PC/3

L
SRJC (2006) Chemistry Paper 1

10

[Turn over

l
[
[

The responses A to D should be selected on the basis of

B
1 and 2 only are
correct

A
1, 2 and 3 are
correct

D
1 only is correct

2 and 3 only are


correct

No other combination of statements is used as a correct response.


36

Which of the following statements describe the characteristics of nickel but not
strontium?

1
2
3

It is denser.
It forms coloured complexes.
It is a better thermal conductor.

[
r-

-.

37

The diagram represents a section of a catalytic converter on the exhaust system of a


car. Harmful gases are converted into carbon dioxide, nitrogen and water vapour.

C02
-H20

co

hydrocarbonsNOx
--

--

N2

platinum and rhodium catalyst

Which processes take place in its catalytic converter?

1
2
3

.38

Carbon monoxide and hydrocarbons react together.


Carbon monoxide and nitrogen oxide react together.
Platinum and rhodium catalyse redox reactions.

Which of the following methods can be-used to prepare 3-nitrobenzoic acid?

1
2
3

c
[

The nitration of benzoic acid.


The nitration of methylbenzene followed by oxidation.
The reduction of 3-aminobenzoic acid.

C.

L
l

L
[
SRJC (2006) Chemistry Paper 1

11

[Turn over

L
L
r

r
[

The responses A to D should be selected on the basis of


A
1, 2 and 3 are
correct

[
[

B
1 and 2 only are
correct

c
~

2 and 3 only are


correct

--

---

D
1 only is correct
---

~-

No other combination of statements is used as a correct response.


39

Halogen-containing organic compounds can be hydrolysed under suitable conditions


to produce the corresponding halide ions, x-.

A student investigated the amount of x- produced by hydrolysing CH 3 CH 2C/ and


another halogen-containing compound, W. In a given time the amount of x- formed
was greater with W than with CH 3CH 2C/.

c
[

number
of
moles
ofx-

[
r-,

CH3CH2C/

0
0

time

Which compound could be W?

c
[_,
~,,

2
3

40

C 6 H4 ClCH 2 Cl
CH 3 CH 2COC/
C/CH 2CHzCl

Morphinf} and codeine are both effective pain killers.

[
OH

HO

c
L
L
[

OH

H3CO

morphine

codeine

Which of the following statements are true?

2
3

Both decolourises bromine in tetrachloromethane.


More hydrogen chloride gas is evolved for morphine than codeine when both
compounds are reacted with PCl5.
One of them forms an orange precipitate with 2,4-clinitrophenylhydrazine.

SRJC (2006) Chemistry Paper 1

12

r--.

,._..,
'--------'

[
SRJC 2006 JC2 Prelim Exam

Chemistry Paper 1 Answers

[
[

[
[
[

.,
f
.__.,

[
[

['

[
'

,,

L
[

L
[,
f

2
3
'4
5
6
7
8
9
10

c
c
B
0
0
B
0
B

11
12
13
14
15
16
17
18
19
20

B
0
A

c
c
A
A

0
B

21
22
23
24
25
26
27
28
29
30

B
A

c
c
A

c
c
0
A

31
32
33
34
35
36
37
38
39
40

c
B

c .
c
A

c
0

c
0

[-,
1.

-)

Class _ _ __

CANDIDATE N A M E - - - - - - - - - -

SERANGOON JUNIOR COLLEGE


JC2 PRELIMINARY EXAMINATION 2006

9251/2
9258/2

CHEMISTRY
PAPER2

THURSDAY

21 SEPTEMBER 2006 .

1 hr 30 min

Additional materials:
Data Booklet

[
[

INSTRUCTIONS TO CANDIDATES
1

Write your name and class on this cover page.

Answer all questions.

Write your answers in the spaces provided on the question paper.

INFORMATION FOR CANDIDATES


The number of marks is given in brackets [ ] at the end of each question or part question.

r'

You are advised to show all working in calculations.

A Data Booklet is provided.

You may use a calculator.

FOR EXAMINER'S USE

. 1

/11

/8

/9

/71

/9

/9

17

TOTAL

[60]

L
L
L

[Turn over

l
-[

SERANGOON JUNIOR COLLEGE


PRELIMINARY EXAM 2006- CHEMISTRY PAPER 2

The reaction between iodine and propanone is catalysed by hydrogen ions.


CH3COCH3 + h

H+

CH3COCH2I + HI

The rate of reaction can be measured by recording the reduction of the


concentration of iodine by the decrease in the intensity of its colour as
measured in a colorimeter.
It is found that the reaction is first order with respect to CH 3COCH 3 . To find the
order of reaction with respect to H+ and 12 , two sets of separate experiments
are performed in which the initial concentration of H+ and Iz are varied in turn,
while keeping the concentration of CH 3COCH3 constant at 0.20 mol dm-3. The
results are obtained as shown below.
(I2]/mol dm-3

[1 2]/mol dm"3

0.004

0.004

0.003

0.003

0.002

0.001

(a)

[
[
'---+-+--1----t--1---1--- Time/min
4
5 6
3
2

What do you understand by the term first order reaction?

Use the graphs to deduce the order of reaction with respect to

(i)

[!

L
[1]

(b)

0.001

'---+--+---!--+-+--+---_.. Time/min
2 .3 4 5 6

[
[

0.002

[H+];, 0.80 mol dm3

H+

L:

u
SRJC {2006) Chemistry Paper 2

[Turn over

L
L

L
[

(ii)

[
[

[
[3]

(c)

Using your answers in part (b), write the rate equation for the reaction and
hence calculate a value for the rate constant, k, when the concentration of
H+ is at 0.40 mol dm-3. State the units of k.

[
[

.,

.,

[
[
[

L
[.

L
L
l
L
[
l"

[2]
(d)

Only one of the following outline reaction mechanisms is consistent with


the observed kinetics.

I2 + H+ -7 intermediate

[slow]

Intermediate + CH 3COCH 3 -7 products

[fast]

GH3COCH3 + H+ -7 intermediate

[slow] .

Intermediates+ I2 -7 products

[fast]

CH3COCH3 + H+ -7 intermediate

[fast]

Intermediates + h -7 products

[slow]

CH3COCH 3 + h -7 intermediate

[slow]

Intermediate+ H+ -7 products

[fast]

SRJC (2006) Chemistry Paper 2

[Turn over

l
-C
~

Decide which mechanism is consistent, explaining the reasons for your


choice.

Mechanism letter (A, B, C or D) ............................... .


Reasons:

[
[

[
[2]

(e)

Explain, with the aid of a suitable diagram, how a catalyst affects the rate
of a chemical reaction.

l-~

[
['

-
l

__ ,

[
[

[
[3]
[Total: 11]

l
L

L
SRJC (2006) Chemistry Paper 2

(Turn over

L
L
I

L
[

2(a) Ammonium nitrate, NH 4 N03 , is widely used as an explosive. When ignited, it


decomposes to give nitrogen, oxygen and steam.

(i)

Write a balanced equation with state symbols for the decomposition of


ammonium nitrate.

(ii)

Given that the enthalpy change of atomisation of ammonium nitrate is


+2928 kJ mor1 and using appropriate bond energy data from the Data
Booklet, calculate the enthalpy change of decomposition of ammonium
nitrate with the aid of a suitable energy cycle.

[
[

[
[

[
[
[

[
[
-,

-~

L
L

(iii) Suggest two reasons why the decomposition of ammonium nitrate is


explosive.

L
L
l

[5]

SRJC (2006) Chemistry Paper 2

[Turnover

l.
{b)

Ammonium nitrate, when mixed with aqueous ammonia, can be used to


prepare a buffer solution. 42.6 g of ammonium nitrate is dissolved in 800 cm 3 of
0.65 mol dm-3 of aqueous ammonia and the mixture is diluted to 1.00 dm 3 .
(i)

Use one of the following values to calculate the pH of the prepared buffer
solution.

NH/ (aq),
NH3 (aq),

=6.00 x 10Kb =1.67 x 10Ka

10

mol dm-3

[
[

[.
[

mol dm-3

[
[
[''
C"'

l_~
~-

L,
(ii)

Write equations to show how this buffer system regulates the pH level on
addition of H+ ions and OH- ions respectively.

ll
[
l1

u
I
[

[3]
[Total: 8]

_;

u
l_,

SRJC (2006) Chemistry Paper 2

[furn over

L
[

3(a) By using relevant information from the Data Booklet, sketch a suitable graph
using the axes below to illustrate and explain the thermal stability trend of
hydrogen halides.

[
[
[
[
[
[
[

-,
l

[3]

-'

(b)

C'

When aqueous chlorine was added to anunknown sample and the aqueous
mixture is shaken with tetrachloromethane, a violet organic layer was observed.
With relevant information from the Data Booklet, explain the above observation
as fully as possible, commenting on the relative oxidising powers of species
involved.

L
[
[
[3]

L
l.
L

SRJC (2006) Chemistry Paper 2

[Turn over

L
t:
{c)

The amount of carbon monoxide in a sample of polluted air can readily be


determined by passing it over solid iodine {V) oxide, 12 0 5 , to give carbon
dioxide and iodine.
The iodine produced is then removed and titrated with aqueous thiosulphate. A
1.0 dm 3 sample of air produced iodine that required 20.00 cm 3 of 0.15 mol dm-3
aqueous thiosulphate to discharge the iodine colour.
(i)

Write balanced equations for the two reactions mentioned above.

[-"

L
[

[
[
[

{ii)

Calculate the mass of carbon monoxide in this sample of polluted air.

r~

r
[

L
[
[

l[3]
[Total: 9)

SRJC (2006) Chemistry Paper 2

[Turn over

l
L
L
L
r

f
[
r

[
[

4(a} The colour of blood is due to oxygen-carrying organic molecules surrounding a


transition element ion. In human, this transition element is iron, and the blood
is red. In horseshoe crabs and sea squirts, the colours of their blood are not
red due to the presence of different transition elements. The spectrum provided
below shows the major absorption peaks for the blood of human and horseshoe
crabs.

II :

absorbance

Hoffi~hoe

cmbs

[
400

[
[

600

500

700

wavelength/nm

(i}

What do you understand by the term transition element?

(ii}

Suggest the colour for the blood of horseshoe crabs.

L
[
[
(iii} Given that the blood of sea squirts is green que to the presence of

[
[

vanadium metal, sketch and label on the axes above, the absorption peak
for the blood of sea squirts.

(iv} Explain why transition elements form coloured compounds.

l~

l
l
L
[
(-

[4]

SRJC (2006) Chemistry Paper 2

[Turn over

{b)

Another iron compound, iron (II) sulphate, forms a yellow solution when boiled
with an excess of sodium cyanide. Upon acidification of the yellow solution and
adding aqueous chlorine, a red solution is obtained. When evaporated, the
solution forms dark red crystals that contain 19.9% iron, 25.6% carbon, 29.9%
nitrogen and 24.6% sodium by mass.
(i)

Determine the formula of the dark red crystals and state the oxidation
number of iron in the crystals.

ll.

c
[

[
[
[
['
[
(ii)

With reference to the Data Booklet, suggest the identity of the yellow
solution.

{iii) Construct a balanced equation for the reaction between chlorine and the
yellow solution.

',

'

c
[

L
[
l~

L
[3]
[Total: 7]

l
l_

SRJC (2006) Chemistry Paper 2

[Turn over

l
I'

The apparatus shown can be used to prepare ethene from ethanol.


mineral wool
and ethanol

[
[

\I

strong
heat

water

[
-,

(a)

(i)

State the type of reaction that takes place on the hot pumice.

(ii)

Write an equation for this preparation.

--'

_,

-"

[
[

(iii) Given that this process is only 80% efficient, calculate the volume of
ethene collected at room temperature, when 0.40 g of ethanol is
used.

[
[
[
lr,
l_,

[2]

[
SRJC (2006) Chemistry Paper 2

L
l'

10

[Turn over

l
(b)

~
[

Dichloroethane can be synthesised from the chlorination of ethane or ethene.

"C=C
/

H
(i)

-----:"""~

'\.

H H
I

H-C-C-H

...:

IT

Cl C!

H H
I

[~

H-C-C-H

H H

State what type of reaction takes place in step I.

[
(ii)

Suggest suitable reagents and conditions for step II and describe the
reaction mechanism using relevant equations.

[
[
[
l

-~

~'

f~

[
[
(iii) Suggest one possible by-product in step II and how it may be separated
from dichloroethane in the industry.

L
L
[~

L
[4]

l
l_,
SRJC (2006) Chemistry Paper 2

11

[Turn over

L
~-

L
[

(c)

Ethene polymerises with 1,3-butadiene under suitable conditions.


(i)

Draw one repeat unit of the polymer formed.

(ii)

Write an equation for the reaction of 1,3-butadiene with cold dilute


acidified manganate (VII) ions. Describe what will be observed.

[
[
[

[
[

[
[

[2]
{d)

Hot, concentrated manganate (VII) ions break the double bond in alkenes.
When diethylstilbestrol (DES), an animal feed that causes several types of
cancer, is subjected to hot concentrated manganate (VII) ions, the only product
formed is:

HO~yHzCH 3
~C=O

Suggest the structure of DES.

[
[

[
[1]

[Total: 9]

[_~
SRJC (2006) Chemistry Paper 2

L
L

12

[Turn over

l
-ll
6

One substance which is responsible for the fragrance


2-phenylethanol. An isomer of 2-phenylethanol is 2-ethylphenol.

of roses

is

OH

g-CH2CH20H

2-phenylethanol

(a)

(i)

6-CH2CHa

2-ethylphenol

[
[

Name the type of isomerism exhibited by these two compounds.

[
(ii)

Another isomer of 2-phenylethanol, Y, is optically active. State the type of


isomerism Y displays and draw diagrams to represent the isomers.

[
[

[
[2]
(b)

By using simple chemical tests, describe how 2-phenylethanol, 2-ethylphenol


and Ycan be distinguished from each other.

c
[
[

[
[

[
[3]

l
l_

SRJC (2006) Chemistry Paper 2

13

[Turn over

L
r

L
l~

(c)

(i)

Explain why 2-phenylethanol is the least acidic, while benzoic acid is the
most acidic when compared with phenol.

(ii)

How would you expect the acidity of 2-fluorobenzoic acid to compare with
that of benzoic acid? Explain your answer briefly.

[
[

[
[
[

[
[-,
[

r
[
[

..
"

[
[4]

[Total: 9]

[-

L
l
SRJC (2006) Chemistry Paper 2
~

14

[Turn over

L
7

Three bromine-containing organic compounds are shown below.


CH 3
l
H3C-C-Br
I
CH 3

CH 3CH2CH 2CH2Br

1-bromobutane

(a)

-[
[
[

H
I
H3CH2C-C-CH3
I
Br

2-bromo-2-methylpropane

2-bromobutane

1-bromobutane reacts with aqueous sodium hydroxide to form butan-1-ol.

Give a balanced equation for this reaction and name the type of reaction.

[
(b)

[1]

1-bromobutane, 2-bromobutane and 2-bromo-2-methylpropane can also react


with an ethanolic {alcoholic) solution of sodium hydroxide.

Identify, by means of the structural formula, the organic product(s) formed from

1-bromobutane

III

2-bromobutane

L
[
[
II

2-bromo-2-methylpropane

[2]

SRJC (2006) ChemistryPaper 2

15

L
l
L

(Turn over

l_

(c)

(i)

Complete the reaction sequence giving the intermediate, the reagents and
the conditions for the synthesis of 2,2-dimethylpropanoic acid.

[
[
[
[

CH 3
Step I
I
H3C-C-Br - - - - +
I
,
CH 3

Step II

yH3
---~ H3C-C-COOH
I
CH 3

Step I: reagents ........................................................................ .


conditions .......................................................................

Step II: reagents .........................................................................


conditions .......................................................................

(ii)

State and explain how the rate of reaction in step I changes when 2bromo-2-methylpropane is replaced by 2-iodo-2-methylpropane.

[
[
[

[
[
[
[4]

l
L

[Total: 7]

L
l~

SRJC (2006) Chemistry Paper 2

16

__

r:

,--,.

~
l "
J

'

"SRJC 2006 JC2 Prelim Exam

Chemistry Paper 2 Answer Scheme

(a)

In a first order reaction, the rate of reaction is directly proportional to


concentration of one of the reactants; i.e. rate cc [A]'.

(b)

(i)

(e)

(0.00 3 - 0004 ) -1.667 x 1o-.mol dm'3 min' 1


6.0-0

fraction of molecules with ~nergy <: E,

[!2:J

fraction of molecules with energy<:: E

energy E

Rate 2 = - (0.00 2 - 0004 ) =3.333 x 1o- mol dm3 min'1


6.0-0

A catalyst works by providing an alternative reaction pathway of lower


activation energy Ea

. When [H] doubles while concentration of others are kept constant, the
rate also doubles, :. order of reaction wrt W = 1.

Number of reactant particles with at least the activation energy Ea Increases.

(il)

Number of effective collisions taking place In the reaction increases.

Rt _ (0.0036-0.004)_ 667 10 _s
ld -am-1
- . x
a e1 - a.O _0
mo m
tn
Rate2 = -

j::;:::;:;l

Since rate of reaction is proportional to the frequency of effective collisions,


rate of.reaction increases in the presence of catalyst.

(00016 - 0002 ) =6.67 x 1o-5 mol dm-3 min-'


6.0-0

2(a) (i)
When [I2rdoubles whiie concentration oi others are kept constant, the rate
remains constant. :. order of reaction wrt 12 = 0.
(c)

=k [H1[CH~COCH3l
1.667 X 1Q-4 =k (0.40) (0,20)
k =2.08 x 10'3 mol' 1 dm 3 min' 1

(d)

NH.N03 (s) -7 N2 (g) + Y.02 (g)+ 2H20 (g)

(ii)

Rate

By Hess's Law:
Slow step of B contains 1 mol of H+ and 1 mol of CH3COCH3 which Is
consistent with the rate equation given.

D.Hrxn = 2928- {994 + Y. (496) + 4(460)}

=-154 kJ mol'1
(iii)

SRJC Chemistry Paper 2

Large number of moles of gases evolved will cause an increase in


volume and hence build up of pressure;
Exothermic decomposition results in heat being given out.

SRJC Chemistry Paper 2

800
x 0.65 =0.520 mol dm3
1000
[NH4 ) =
42.6
3
14 + 4 +14 +3(16 )- 0.5325 moldm
[NH3] =

(b)

(b)

C/2 oxidised r to I2
C/2 (aq) + 2r (aq)

0 5325 )
pOH = -lg (1.67x1 0' 5) + lg (
(0.52)
pOH = 4.7876

NH3 (aq) + H (aq)

NH4 (aq) + OH' (aq)

2Cr (aq) + !2 (aq)

E9 (C/2/Cr) is more positive than E9 (!2/r)


Ch has stronger oxidising power than !2

pH= 14-4.7876 = 9.21


(II)

The violet organic layer contains 12

Iodine. formed is solubl~ i~ tetrachloromethane as the weak VDW forces of


attract1on between the 1odme molecules is similar in strength with the weak
VOW found in tertachloromethane.

NH/ (aq)

NH3 (aq) + H20 (I)

(c)

(i)

hOs (s) + 5CO (g)

3(a)

5COz (g) + 12 (s)

lz (s) + 2SzOt (aq) ~ 2r (aq) + S40l (aq)

Bond energy/
kJ mor 1

562~

(ii)

moles of CO in polluted air=

431

~xo
I5x~2 =7.5x !O''
1000
.

mass of CO in polluted air= 7.5 x 10'3 x (12 + 16) = 0.210 g

366

299

4(a)

l;rH:;:::F:-\----:-t:-+------- Hydrogen halides


absorbance

HCI

>

Order of thermal stability decreases from .HF to HI.

>

Down the group, covalent bond length of HX increases.

>
>

Covalent bond strength decreases.


Bond dissociation energy decreases.
wavelength/nm
(i)

Transition element is a d-block element that forms at least one compound


with a partially filled d-orbital.

(il)

Blue I Indigo I VIolet

(iii) Refer to spectrum


SRJC Chemistry Paper 2

r-l

SRJC Chemistry Paper 2

r--;

'

:::--1

'

,.._

1---"1
I
,

(b)
(iv) d-orbitals of transition elements are split into two groups in the presence
of ligands. The d-electron undergoes d-d transition and is promoted to a
higher d-orbital. The d-electron absorbs certain wavelength of light from
the visible spectrum and emits the remaining wavelength which appears
as the colour observed.

(i)
(ii)

Electrophilic addition
C/2 in CCI., ultraviolet light

uvlight
(b)

(I)

2CI

Na3[Fe(CN)s]
Oxidation state: +3

(ii)

Propagation:

Cl + CH3CH3 ... HC/ + CH3CH2

[Fe(CN)s]'"

CH3CH2

+ C/2 ... C/ + CH3CH2CI

(iii) 2[Fe(CN)aJ" (aq) + C/2 (aq) - 2[Fe(CN)sf (aq) + 2C/' (aq)

The apparatus shown can be used to prepare ethene from ethanol.


mineral wool
and ethanol

umlce

Termination:
Cl + C/ - C/2

(iii)

water

Chloroethane

By fractional distillation.
(a)

(c)

(i)

Elimination

(ii)

CH3CH20H (I) _ __.,

(i)

H H H H H H
I
I
I
I
I
I
-C-C-C-C=C-C1

(ii)
(iii)

No of moles of ethanol
Volume of ethene

= 0.4 = 0.0086957
46

=~
x 0.0086957 x 24 =0.167 dm
100

Decolourisation of purple KMno. solution.

SRJC Chemistry Paper 2

SRJC Chemistry Paper 2

H H H H
I I I I
H-c-c-c-c-H
I I I I
OHOHOHOH

(ii)
(d)

~COOH

<Q>-cooH

>

F
6(a) .(i)

The electron-withdrawing fluoro group increases the withdrawal of


electrons from the H atom which causes weakening of the 0-H bond and
promotes the loss of proton hence 2-fluorobenzoic acid is more acidic than
benzoic acid.

Structural isomerism

(II)

Optical isomerism

7(a)

Nucleophilic substitution

C""CH
H/ \
3
OH
(b)

(b)
III

1-bromobutane

Add FeC/3 solution to each of the 3 compounds separately.


Observations: Violet complex formed for compound 2-ethylphenol

CH3CH=CHCH3
and

Add iodine in aqueous NaOH to the remaining 2 compounds separately and


warm.

II

2-bromo-2-methylpropane

Observations: Decolourisatlon of brown iodine solution and pale yellow


precipitate formed for compound Y.

(c)

(i)
The electron withdrawing carbonyl qrouo in benzoic acid increases the
withdrawal of electrons away from the H atom in the hydroxyl group, this
causes weakening of the 0-H bond and promotes the loss of proton. The
negative charge in the carboxylate anion is delocalised over two oxygen
atoms thereby stabilising the conjugate base relative to the acid. Hence
benzoic acid is the most acidic.
The electron donating alkyl group intensify the negative charge on the
oxygen atom hence destabilising the anion relative to the acid. Thus 2phenylethanol Is the least acidic.

SRJC Chemistry Paper 2

(----....

SRJC Chemistry Paper 2

2-bromobutane

~;
J

I~

-=.
J

,-..,

r---"',.

(c)

(i)

. '

yH3
H3C-y-CN
CH3

Step I: alcoholic KCN, reflux


Step 11: dilute HC/, reflux
(II)

Covalent bond length: C-Br < CI


Covalent bond strength: C-Br > C-I
Bond energy: C-Br < CI
When 2-bromo-2-methylpropane is replaced with 2-iodo-2-methylpropane,

rate of reaction Increases.

SRJC Chemistry Paper 2

t1

,....-.
I
.

'

.I~'

r---;,
l

~.
-)

.<:)

L
[

CANDIDATE N A M E - - - - - - - - - - - -

Class - - - -

SERANGOON JUNIOR COLLEGE

JC2 PRELIMINARY EXAMINATION 2006

CHEMISTRY
PAPER3

9251/3
9258/3

[
WEDNESDAY

[
l...-"'

-,
f

2 hour45 min

Additional materials:
Answer Paper
Cover Sheet
Data Booklet

r
[

13 SEPTEMBER 2006

INSTRUCTIONS TO CANDIDATES
1

Write your name and class on the Cover Sheet provided.

Answer all questions in Sections A, 8 and C. The last question in each section is of the

-,-'

c
[

L
r.:

form either I or. In Section D, you are required to answer both questions on only one out of
the two option topics.
3

Write your answers on the separate answer paper provided.

Start each question on a fresh sheet of paper.

At the end of the examination:

Staple or fasten all your work securely together with the Cover Sheet on top.

Hand in the question paper separately.

INFORMATION FOR CANDIDATES


A Data Booklet is provided.
The number of marks is given in brackets {] at the end of each question or part question.
You are advised to show all workings in calculations.
You may use a calculator.
You should not attempt questions on option topics for which you have not prepared.
You are reminded of the need for good English and clear presentation in your answers.

L
[

[Turn over

\
.[
SERANGOON JUNIOR COLLEGE

PRELIMINARY EXAM 2006- CHEMISTRY PAPER 3

Section A

Answer all the questions in this section.


The equilibrium

[
N204 (g)
pale yellow

2 N02 (g)
deep red-brown

is reached fairly quickly in the gas phase.


(a)

When 3.50 g of N204 is placed in an evacuated 1.28 dm3 flask at 300 K, the
equilibrium pressure is 0.880 atm.
(i)

Calculate the number of moles of N204 at the start.

(ii) Calculate the number of moles of gas at equilibrium, assuming the gas
behaves ideally.

(iii) Hence, calculate the percentage of dissociation for N20 4 .


(iv) Write an expression for Kp of the reaction and calculate its value at
300 K.
[6]

(b)

(c)

L
r~

When N2 0 4 at 300 K is suddenly compressed in a gas syringe, the mixture


first darkens and then slowly becomes paler. Suggest why N2 0 4 behaves in
this way.

[':

(2]

NO (g) dimerises to N202 (g) in a way similar to the dimerisation of N02 (g) to
N204 (g). By drawing dot-and-cross diagrams, explain whether the
dimerisation of NO (g) is endothermic or exothermic.
[2]

[Total: 10]

l-J

Ll

u
u
[

l
SRJC (2006) Chemistry Paper 3

[Turn over

L
L
\

Acrylic acid, C3H402, has an acid dissociation constant of 5.6 x 10-5 mol dm3.

2
(a)

[
[

[
[

(i)

Calculate the pH of a 0.10 mol dm-3 solution of acrylic acid.

(ii) Using the value calculated in (a)(i), explain why acrylic acid is a weak
acid.

(iii) If the solution in (a)(i) was to be diluted tenfold, the pH is expected to rise
by

(b)

unit. However, it rises by less than one unit. Explain why this is so.
[3]

25.0 cm 3 of 0.10 mol dm-3 acrylic acid (in a conical flask) was titrated against
0.10 mol dm-3 of barium hydroxide. For this titration,
(i)

[~

on~

State the volume of Ba(OH)2 that must be added to produce a buffer


solution of the best buffer capacity.

(ii) Calculate the pH of the solution when 12.50 cm 3 of barium hydroxide has
been added. Explain, with the aid of an appropriate equation, why the pH
at this point is not neutral.

(iii) Calculate the pH of the solution when 20.00 cm 3 of barium hydroxide has
been added.
(5]

-,

,J

L
[
-

(c)

In another experiment, 25.0 cm 3 of a solution containing the diluted acrylic acid


and another monobasic aeid with pKa value of 8.9 was titrated against sodium
hydroxide. The pH change was monitored during the titration using a mixture of
two appropriate indicators, which in turn changed colours at two different
stages of the titration.

(i)

State the basis for the appropriate selection of the indicators in terms of
pH change during the titration.

(ii) Sketch the shape of the Pt-i curve for this titration till a large excess of the
alkali has been added, indicating the different stages of the titration at
which the indicators changed their colours.
[2]

[Total: 10]

----

{--

[
~L
..

r
L
L

SRJC (2006) Chemistry Paper 3

[Turn over

l
.[
3 either
(a)

Batteries for electric cars can be made with one electrode of zinc, and the
other of carbon. During discharge, the zinc dissolves as ions, which migrate
through the electrolyte to the carbon electrode, where they combine with
oxygen from the air and water to form zinc hydroxide.
(i)

By choosing two suitable electrode processes from the Data Booklet,


write the overall equation for the reaction that occurs during the
discharge.
Calculate the e.m.f. of one cell of the battery, assuming standard
conditions.

['

[
[

[
[I

(ii) Suggest a suitable substance to use as an electrolyte.

[2]

(b)

(i) Draw a labelled diagram to show how you could measure the standard
electrode potential of the Co2+(aq) I Co(s) system.

(ii) What effect would a decrease in the concentration of Co2 + (aq) have on

the E9cell of the system? Explain your answer.

[4]
(c)

Aqueous bromate (V) acts as a strong oxidising agent in acidic solution:


2Br0 3- (aq) + 12H+ (aq) + 10e ~ Br2 (aq) + 6Hz0 (I)

E0

=+ 1.48 V

Predict if a reaction will occur when bromate (V) in acidic solution is mixed with

(i)

hydrogen peroxide

(ii) exces;s tin metal


In each case, calculate E 9cen and predict what will be observed. Write an
equation for any reaction-that occurs.
[4]
[Total: 10]

[~

n
u
u

u
u
L
L
L

SRJC (2006) Chemistry Paper 3

[Turn over

L
[
\.

{..

3 or
(a)

Strontium metal can be obtained by the electrolysis of molten strontium


bromide.
Steel
Atmosphere of argon

Molten strontium

cr,

Molten strontium

bromide~

(i)

Heat
State the polarity of the graphite and steel electrodes.

(ii) Why is an atmosphere of argon used around the cathode?

(iii) Calculate the mass of strontium that can be obtained if a current of 2.0 A
is maintained for 45 minutes in this process.

[4]

(b)

The following aqueous solutions were connected in series as shown below:


.-------j

II If--------,

[
~::::

Cu (s)

[~
(i)

r
[

l
L

I
CuS0 4 (aq)

f'

Ag (s)

AgN03 (aq)

H2 S04 (aq)

Which of the above electrode are cathodes and which are anodes?

(ii) Write equations for the reactions occurring at electrodes A, D and E.


[3]

(c)

A household bleach contains sodium chlorate (1), NaC/0. When bleach


solution is acidified, chlorine is evolved by the following reaction:
C!O- (aq) + 2H+ (aq) +

cz- (aq) ~ C/2 (aq) + HzO (I)

The chlorine liberated will oxidise iodide to iodine and the amount of iodine
produced can be estimated using a standard thiosulphate solution.

A 25.0 cm 3 sample of household bleach is diluted to 250 cm 3. A 25.0 cm 3


portion of this diluted solution is acidified and then added to excess of
potassium iodide solution and titrated against 0.200 mol dm3 thiosulphate
solution. The volume required is 18.50 cm 3. What is the concentration of
sodium chlorate (I) in the household bleach?
[3]
[Total: 10]
SRJC (2006) Chemistry Paper 3

[Turn over

L
.
Section B

Answer all the questions in this section.

4 (a)

Transition metals are usually good homogenous catalyst. One example is


Co2+ (aq) catalysing the reaction between iodide and peroxodisulphate ions.
2r (aq) + S20l- (aq) 7 2SO/- (aq) + I2 (aq)

(i)

f'
[

What features of Co2+ enable it to act as a homogenous catalyst?

(ii) Suggest how Co2+ is able to participate in this reaction by using E8 values
from the Data Booklet and writing relevant equations.
[4]

[
r~

(b)

When 1.00 g of impure solid copper (I) oxide, Cu20, is warmed with dilute
sulphuric acid and a blue solution is obtained, together with 0.370 g of pink
solid.

(i)

Suggest the identity of the blue solution and the pink solid.

(ii) Write a balanced equation for the above reaction and hence calculate the
percentage purity of Cu 20 in the 1.00 g sample.

(iii) When excess ammonia is added to the blue solution obtained in (i), the
solution becomes deep blue. Upon adding edta4-, the deep blue solution
decolourises. Explain the above colour changes, writing equations where
appropriate.
[6]

L
[

[
[
[

Group II carbonates show a difference in stability when subjected to heat. A


solid mixture containing 2.00 g of magnesium carbonate and strontium
carbonate is heated slowly until no further change takes place.

The residue was then shaken with 50.00 cm 3 of water. 10.0 cm 3 of the solution
was pipette out and titrated with p.200 mol dm-3 dilute hydrochloric acid.
20.65 cm3 of the acid was required for complete reaction.

(i)

Identify the carbonate that has. a higher decomposition temperature and


explain w~y.

(ii) Write equations to show the reaction of each residue (if any) with water.
(iii) Calculate the percentage by mass of strontium carbonate in the original
mixture.

[5]
(b)

r-

[Total: 10]

5 either
(a)

l~/

Compare the reducing power of magnesium and barium, with reference to cold
water. Support your answer with relevant data from the Data Booklet.

[2]

L
[

L
L

--

SRJC (2006) Chemistry Paper 3

[Turn over

L
I

r
(c)

(i}

[3]

[Total: 10]
5

or

(a}

Explain the variation in melting points of the chlorides of sodium and silicon in
terms of structure and bonding.
[2]

(b)

When some solid sodium carbonate is added to an aqueous solution of


aluminium chloride, effervescence is observed. Identify the gas evolved and
explain this observation.
[2]

(c)

Outline the reactions, if any, of the oxides of sodium, aluminium and


phosphorus with water. Give the approximate pH of any solution formed and
write equations where appropriate.
[3]

(d)

Commonly used as a liquid source of chlorine, the chloride of an element X is


represented as X0 2 Cln with a relative molecular mass of 135. When 0.01 mol
of this chloride reacts with water, it forms a mixture of two acids. Addition of
excess silver nitrate solution to the mixture yields a white precipitate that when
filtered off, weighs 2.87 g. Subsequent addition of lead (II) nitrate solution
forms another white precipitate.

r-~

L
-,
f

--~

[
~,

-.-'

L
l

Describe the bonding in magnesium oxide and explain why it acts as an


insulator.

(ii) Suggest two reasons why magnesium oxide is preferred to pvc as an


insulator.

Electric cable used in fire alarm systems has copper wire surrounded by
magnesium oxide which acts as an insulator.

(i)

Calculate the value of n and hence find the relative atomic mass of X.

(ii) Deduce the .identity of X and the formula of the chloride.

(iii) Write an equation for the reaction of this chloride with water.

[3]
[Total: 10]

~~,
~~

L
f
L

L
SRJC (2006) Chemistry Paper 3

[
[

[Turn over

L
-C
[

Section C
Answer an- the questions in this section.

6 (a)

Jasmone is the active ingredient of jasmine extracted for use as perfume.


Suggest the structure of the products, and type of reaction undergone when
jasmone reacts with:
0

QCH2CH=CHCH2CH3
H2 I Ni

(ii)

2,4-dinitrophenylhydrazine

[
[

CH3

(i)

(iii) KMn04 I H+

[3]

[
(b)

Compound S can be produced from benzaldehyde by the following reaction


scheme:

~CHO

II

.. P

..

Benzaldehyde

?H
C-H ~III

booH

[
--

~oXa

~0~0

_,

CH3

s
(i)

Draw the structural formula of P.

(ii) Suggest reagents and conditions required for steps I, II and III.

(iii) Name and outline the mechanisrrlfor step I, indicating cieady why a trace
amount of sodium cyanide can act as a catalyst.

[5]
- (c) When benzaldehyde is treated with concentrated potassium hydroxide,
followed by acidification, two compounds Q (C1HaO) and R (C1H602) are
formed in equimolar amounts. q and R both evolve hydrogen gas when
heated with sodium metal. While R is soluble in aqueous NaOH, Q is not. R
can be obtained from Q by treatment with acidified potassium dichromate (VI).
Deduce the structure of Q and R and give your reasoning.
(You do not need to explain the reaction of benzaldehyde with concentrated
potassium hydroxide)

[2]

L
L
l_:

[Total: 10]

L_
SRJC (2006) Chemistry Paper 3

[Turn over

L
f

c
7

(a)

A section of the polypeptide is said to be built from the amino acid, threonine.

H H 0

II

II

--N--C--C-N-C-C-1
I
HO-C-H HO-C-H
I
I
CH3
CH3

(i)

Draw the structure of threonine when the polypeptide is hydrolysed.

(ii) Suggest the reagents and conditions that you could use to hydrolyse the

polypeptide.

(iii) Draw the full structural formula of the products obtained at room
conditions when threonine reacts with

I
II

.,

NaOH (aq)
HCI (aq)

.... ~-'

(iv) The molecules of threonine can also react to form another polymer
isomeric with the given polypeptide above. Draw the structure of the
polymer.
[3]

(b)

L
.,

Two other amino acids are aspartic acid and serine:


NH2-CH-C02H
I
CH 2
I
C02H

NH2-CH-C02H
I
CH 2
I
OH

aspartic acid

serine

-'

Draw the structural formula of a dipeptide formed from these two amino acids
and its ionic form that would exist at pH 12.

[
[

L
[

L
L

[21
(c)

Explain the differences in solubility and melting point of the following


substances as fully as you can.
Substance.

Melting point I C

I
!

C5HsNH2

insoluble

-6

C5HsC02H

sparingly soluble

121

C5HsCH2CH(N H2)C02H

soluble

> 200C
with decomposition

[51
[Total: 10]
SRJC (2006) Chemistry Paper 3

r_

Solubility in water

[Turn over

L
'c
8 either

When an unknown organic compound A, C1oH11N0 3 , is heated with aqueous


potassium hydroxide, two organic products B and C are obtained. The product B
forms a white precipitate with aqueous bromine and reacts with ethanoyl chloride to
form D.

r
[

Acidification of C produces a compound E. On warming 1 mol of E with aqueous


sodium carbonate,, 1 mol of colourless gas that forms white precipitate with aqueous
calcium hydroxide is evolved.

o 2.2, a 1-proton

D produces a NMR spectrum that consists of a 3-proton singlet at


singlet at o3.1 and a 5-proton multiplet at o 7.5.

(a)

(i)

Suggest the structure of D that gives rise to this spectrum and explain the
splitting pattern observed.

(ii) What would be the effect on the NMR spectrum on adding 0 20 to the
sample? Explain your answer.

[3]
(b)

Deduce the structures A, B, C and E and explain the reactions involved.

[
_,

-~

(5]

(c)

Suggest a starting organic compound required to synthesise B in two steps.


State the reagents and conditions required for each step and draw the
structure of the intermediate compound.
[2]
(Total: 10]

[
[
r<

[
[

l
['

.'
l
L
L
L
--~

SRJC (2006) Chemistry Paper 3

[Turn over

\1

L
8

or

An optically active organic compound F, C9H11 N0 2 , dissolves in both aqueous


sodium hydroxide and dilute hydrochloric acid. 1 mol of F forms a white precipitate
with 2 mol of aqueous bromine. F also reacts with ethanoyl chloride to form
compound G, C11H13N0 3 Compound H, C9H1 3NO, is obtained when F is subjected
to lithium aluminium hydride in dry ether.

F produces a NMR spectrum that consists of a 3-proton doublet at o 1.2, a 1-proton


quartet at o 3.0; a 2-proton singlet at o 5.9, a 4-proton pair of doublets at o 7.6 and a
1-proton singlet at o 11.0.
(a)

Use the NMR spectrum to deduce the structures F, G and H and explain
both the splitting pattern observed and reactions involved.

(ii) What would be the effect on the NMR spectrum on adding 0 2 0 to the
sample? Explain your answer.

[7]
..,

(i)

_J

-,

(b)

Compound I reacts with alkaline aqueous iodine to form a yellow precipitate.


Suggest how I can be synthesised from H in 3-steps. State the reagents and
conditions required for each step and draw the structure of the intermediate
compound.
[3]

-J

[Total: 10]

[
--~.

1
["'__
_,

--,

[--"

r
[,

l
L
L
l
[

SRJC (2006) Chemistry Paper 3

10

[Turn over

c
r-

Section D
Answer both questions on one option.

BIOCHEMISTRY OPTION
Answer both questions on the paper provided.
9 {a)

Label the structural components A-D of the diagram of the fluid-mosaic model
given below:
[1]

m-~--~-1~ 6~ ~ ~ ~ ~

.~ ~ nn
\,~ ~ w~ ~ ~ ~ ~ ~ ~ ~ ~ 81~0~
{b)

Draw the structural formula of a basic unit at A.

[
[1]

{c) . Explain why:


(i) water and water-soluble species cannot pass through A.

(d)

rL,

[1]

(ii) ions can pass through D against a concentration gradient

[1]

{iii) carbohydrate residues labelled E can strengthen the membrane

[1]

An enzyme, F, isolated from the surface of membrane, is found to show the

following kinetics:
At a given enzyme concentration the maximum rate of enzyme-catalysed
reaction is 80 ,umol dm-3 of substrate is converted per minute when the
substrate .concentration is 100 .umol d m -3 ;
The -reaction proceeds at half .of its maximum rate when the substrate
concentration is 12.5 ,umol dm3

(i)

Incorporate all this information on a labelled sketch of the rate against


substrate concentration curve, indicating the value and unit of Michaelis
constant, Km.

[2]
(ii) Explain how the presence of Ag+ ions affects the activity of the enzyme,
including the effects on Vmax and Km values. Add to your graph another
sketch of the enzyme kinetics in the presence of Ag +_
[3]
[Total: 10]
SRJC (2006) Chemistry Paper 3

11

[
[

~~n
~~~

......... _... .__,"

[Turn over

[
[~.

[
'

_,

c
[
['

L
[

L
L

L
r

[
[
[
[

c
c
[

1 0 Enzyme chymotrypsin is specific for splitting peptide linkages in which the carbonyl
group is next to an aromatic amino acid residue.
When chymotrypsin is added to a heptapeptide chain, the following fragments are
obtained:
ala-gly-glu
glu-ala-gly-phe

You may need to use the information provided:

C02H
I

CH 2
I
CH 2
I
H 2N-CH-C02H
glutamic acid (glu)

CHzOH

Hl

f''

[
[

cy

'l)-2

phenylalanine (phe)

adenine

r:~H

o-

It-t

O=P-o'

ribose

phosphate

OH

glucose

.,1:__N"

CH 2
I
H2N-CH-C02 H

Anticodon codes: GAG - glu GCC - ala

GGC - gly

1
o-

UUU - phe

UGU - cys

(a)

Deduce the primary structure of the heptapeptide.

(b)

Deduce the sequence of bases in the corresponding mRNA and DNA strands.
[2]

(c)

Explain the meaning of genetic code.

(d)

Show how two of primary structure of polypeptide chains cqn be bonded to


[1]
form secondary structure.

(e)

Draw the structures of


(i) a dipeptide formed from phenylalanine and glutamic acid

[1]

(ii) two repeat units in cellulose

[1]

(iii) a deoxyribonucleotide

[1]

[
r,
L

[2]

[2]

[Total: 10]

L
L
L
t

SRJC (2006) Chemistry Paper 3

12

[Turn over

l
[
CHEMICAL THERMODYNAMICS

Answer both questions on the paper provided.


11 (a)

The Ellingham diagram below shows the reduction of various oxides- by


carbon.

c
[

r
[
['
'

2500
Temperature/K

(i)

With reference to the diagram, identify the oxides that can be reduced by
carbon at a temperature below 1000 K.

(ii) What is the minimum temperature at which ZnO can be reduced to the
metal by carbon? Write the chemical equation for this reaction.

r--

(iii). Calculate the approximate A.Ge_when carbon reacts with GaO at 2500 K.

(iv) Why does !:J.G9 for reaction of C to form CO becomes more negative with

increasing temperature?
[5]

[
[

L
[
L"
SRJC (2006) Chemistry Paper 3

13

[Turn over

L
L
I

11 (b)

The half equations listed below show the reactions occurring in a


methane and oxygen fuel cell.
CH4 (g)+ 02 (g) --7 C02 (g)+ 4H+ (aq) + 4e

02 (g)+ 4H+ (aq) + 4e --7 2H 20 (I)

CH4(g}
02 (g)
H20 (I}
C02 (g)

c
{i)

['

S9 (JK-1 mol"1 )
186.26
205.14
69.91
i
117.60
'

Determine ~se of the overall reaction and comment on its value.

{ii) Calculate the values of ~Ge and E9cen of the reaction between methane
and oxygen.

-'

[
[
-.
f
[

~H 9 t (kJ mol"1 )
-74.81
0.00
-285.83
-413.80

{iii) When methane is replaced with ethene as the fuel, I:!.G 9 is found to be
-1252 kJ mor 1. Given that I:!.H 9 of combustion is -1411. 1 kJ mor1,
calculate the absolute entropy (S9) of ethene.
[5]
(Total: 10]

-~'

12 {a)

Explain why the molar heat capacity of a gas at constant volume is different
[2]
from that at constant pressure.

(b)

Explain how liquid oxygen can be obtained from air via adiabatic expansion.[2]

(c)

(i)

_,

State the First Law of Thermodynamics.

_.)

(ii)- Calculate the amount of work that must be done at room temperature and
pressure for the following reaction:

2CaH1a (I) + 250z (g) --7 16COz (g) + 18Hz0 (I)

L
['
r-

l~

L
L
L

~H

=-5471

kJ mor

(iii) Hence use First Law Thermodynamics to determine the change in internal
energy of this reaction.
[3] .
(d)

Explain the following phenomena:

(i)

When water freezes at any temperature below 0C, the final state has
less entropy than the initial state. This reaction is spontaneous despite a
decrease in entropy.

(ii) The enthalpy change of solution of ammonium nitrate is endothermic,


suggesting that ammonium nitrate should be an insoluble salt. However,
ammonium nitrate is soluble in water at room temperature and becomes
very soluble at 350K.
[3]
[Total: 10]

SRJC (2006) Chemistry Paper 3

14

,.----.
L__ _

'

~,
)

1....----,
'~

r---1
'
J

;-,

r--1
\

'

SRJC 2006 JC2 Prelim Exam


Chemistry Paper 3 Answer Scheme

(c)
(i) The pH transition range of each indicator must lie within the sharp pH change over the
respective equivalence points.

(ii)

(I) No. of mole of N20 4 present .,0,0380 (3s.f)

pH
14

(ii) n .,0,0456
(IIi) N20 (g) :;:::

2 N02 (g)

% dissociation "'20.0%
(iv)

Kp = (PNoY
PN,o,

(b)

Colour of mixture is proportional to (N02].


The compression results in a decrease in volume. Hence increasing [N02]. Therefore,
the mixture darkens.

= 0.146 atm

The mixture slowly becomes paler because by Le Chatelier's Principle, the equilibrium
position will shift left to decrease the total number of moles of gaseous particles. Hence
more N204 is produced.

1st end pt

znd

Volume of NaOH added I cm 3

end pt

. .._. ..__._.. . . ._..__.. . . . . .~. . .

~Jm,~

(c)
0

XX

"

N ; 0 ~

:0

X X

(a)

, o

(i) 2Zn (s)

E8cell =0.40- (0.76)

The reaction is exothermic.

(a)(i) pH

+ 02 (g) + H20 (I) 7 2Zn(OH)2 (s)

=+1.16 v

(ii) Aqueous KOH or NaOH

= 2.63

(li) For 0.10 mol dm'3 of strong acid,

(b)(i)

pH=Ig(0.10)
= 1.00
HoweVer, pH of solution is 2.63. The acid dissociates partially.

Co(s)

(iii)
C3H402 (aq) + H20 (I)

C3H30 (aq) + H30+ (aq)

With dilution, [H 20] increases.


By Le Chatelier's Principle, the above equilibrium position shifts right to reduce
[H20], thus new [Hj increases.
The degree of dissociation of the acid increases and the reduction of [W] is less
than tenfold.

(b)

(i) 6.25 cm

(ii) Let eqm [OH'] be x mol dm'3

Kw ==-x-2_
K.

(iii)

0.06667

x =3.4503 x 1o6 mol dm3

pOH = 5.462 Hence pH ..8.54


pH ""12.52

SRJC Chemistry Paper 3

C3H3o2 (aq) + H20 (I)


C3H402 (aq) + OH. (aq)
Reason:
C3H30 2- undergoes salt hydrolysis and releases OW
into the solution.
[OHl > [Hj
pH at end-point is > 7 (basic)

- - - - (ii)

Co 2(aq) + 2e

Co 2(aq)
3
I mol dm'

Co(s)

When [Co 2] decreases,


by Le-Chatelier's Principle, the above equilibrium position shift left to increase [Co 2]
Oxidation reaction of Co Is favoured
E6ox becomes less positive/decreases
=> E6ceu becomes more positive/increases

SRJC Chemistry Paper 3

(c)
(i)

E8cell = +0.80 V
Since Ee;;>O', reaction Is feasible
5H20 (aq) + 2Br03- (aq) + 2W (aq) 7 6H 20 (I) + Br2 (aq) + 50 2 (g)

(ii)~

S20s (aq) + 2Co 2 (aq) 7 2SO/' (aq) + 2Co 3 (aq)


= 2.01- (1.82) = +0.19 V
8
Since E c~n > reaction is ~

Observation: Colourless solution turns reddish brown

(ii)

E"cell

E8cell =+1.62 v
Since
E9cell

es;;::Q, reaction is ~

Step 2
2Co 3 (aq) + 21' (aq) 7 lz (aq) + 2Co2
E8 cell =1.83- (0.54} = +1.29 V
Since E8oeiJ2:....Q.. reaction is~

=+1.21 V

Since

Ee;;;>O, reaction Is ~

6W (aq) + Br03' (aq) + 3Sn (s) 7 Br'(aq) + 3H 20 (I) +3 Sn 2 (aq)

(b)
(i)

Observation: Grey metal dissolves I decreases in mass

(~)

(ii)

(i) Graphite: positive


Steel : negative
(Iii)

(Iii) mass of Sr = 2.45 g


(b)
(I) Cathode: B, 0, F

(ii)

(c)

Blue: Cu2 lag)


Pink: Cu Is)
2Cu20 (s) + 4W (aq) 7 2Cu 2 (aq) + 2Cu (s) + 2H 20 (I)
%purity = 83.3 %

(li) This is to prevent the oxidation of strontium by 0 2 from the air to form strontium oxideiSrO.

Anode: A.

o.

c. E

A: Cu (s) 7 Cu 2 (aq) + 2e
D: Ag (aq) + e 7 Ag (s)
E: 40H' (aq) 7 02 (g) + 2H20 (I) + 4e

[Cu(H20)e] 2+ (aq) + 4NH3 (aq)


[Cu(NHa)4(H 20)2] 2 (aq) + 4H 20 (I)
Initial complex present: [Cu(H20)sf
When NHa Is added, [Cu(NHa)4(H20l2f may be formed
=> [Cu(NH3)4(H20)2] 2 is much more stable than [Cu(H 20) 6]2
=> Stronger NH3 ligand replaces the weaker H20 ligand
=> Formation of [Cu(NH3)4(H 20)2f'is favoured
=> Blue solution tums deep I dark blue .
[Cu(NH3)4] 2(aq) + edta4'(aq)= J.Cu(edta)]2'(aq) + 4NH 3(aq)
Initial complex present: [Cu(NHJ)l
When NH3 Is added, [Cu(edta)f may be formed
=> [Cu(edta)f is much more stable than [Cu(NH 3)4]2
=> Stronger edta 4 ligand replaces the weaker NH3 ligands
=> Formation of [Cu(edta)t is favoured
=> Deep I dark blue solution turns colourless

Cto <=Ch =h ""2S20l

0.0~37

18 50
ns2o3 in 18.50 cm 3 solution.=
x 0.200 =
1000
n12l nc12 lnc1o. in 25.0 cm 3 of diluted solution = Y. x 0.0037 = 0.00185
Hence [CIO'] in original solution= 0.740 mol dm'3 (3 s.f)

SRJC Chemistry Paper 3

r-:

rl

r-->

,,___...,
(

'

~)

,-)

(~

~
I

(b)
(a)

(i)

AICI3 undergoes h!;t?lY,~!~to give an g~iCjj solutlon(pH "'3)


[AI (H20)s] 3 (aq) ':<::::::~: . [AI (H 20)s(OH)] 2 (aq) + W (aq)
The high charge density of hydrated Al 3 ion enables it to attract electrons away from one of
its surrounding water molecules, thereby polarising and weakening the 0-H bond which
results in the release of a proton I H.
W (aq) + col- (aq) ~ C02 (g) + H20 (I)

Ionic radius of cation:. s,-2 > Mg 2


Charge density of cation: s,-2 < Mg 2
Polarising power of cation: s,-2 < Mg2
Polarising effect on Col anion : sr < Mg2
SrC03 has higher decomposition temperature

(ii) SrO (s) + H20 (I)

(Iii)

(c)

Sr(OH)2 (aq)

nsr(OH)2 in 10 cm = 0.002065
nsr(OH)2 in 50 cm 3 = = 0.010325

Reacts vigorously to form basic solution

Na20

SrO (s) + H20 (I) ~ Sr(OH)z (aq)


SrC03 (s) ~ SrO (s) + C02

~2NaOH

pH=13114

(aq)

Does not react with water due to its very stable ionic lattice structure, which is
difficult to be broken down by water

AI203

SrC03 =SrO =Sr(OH)z


nsreo3 = 0.010325

pH= 7

mass of SrC03 = 0.010325 x (87.6 + 12 + 48)

P40e (s) + 6H20 (I)

=1.52397 g

1 52397
x1 00% = 76.2 %
2.00

% mass of SrC03 =

P.Os or
P401o,

(b) Ba reacts vigorously with cold water to from Ba(OH)2 while Mg does not read readily I
reacts slowly I no reaction with cold water. Instead, Mg reacts only with steam to form
Mg(OH)2.
The E9(Mg 2./Mg) is less negative than E8(Ba 2/Ba) .. Hence El~ is a better reducing agent
than Mg.

Reacts readily to form acidic solution


pH= 1/2

(i) MgO is a giant ionic compound. There are strong electrostatic forces of attraction between
oppositely charged ions.

Thus MgO cannot conduct electricity in solid state as there are no free mobile ions.
(il)

1. MgO has high melting point, thus it is able to withstand high temperature while pvc
will melt and decompose under high temperature to release toxic fumes of HCI.

(d)

No. of mol of cr = ~
108+35.5
!X02Cln = 2
:. n 2

= 0.0200 mol

Cd

:. Ar of X= 32.0 (1 dec. p)
(ii) X is sulphur ; S.Qz~

2. MgO is a good thermal conductor. Hence allowing heat to react the copper wire
while providing insulation.
(a)

NaCI
is a giant ionic compound with high melting point
Large amount of energy required to overcome strong electrostatic force of attractions
between oppositely charged ions
SiCI4
is a simple molecular compound with low melting point
Small amount of energy required to overcome weak van der Waals' forces of attraction
between molecules (or 'intermolecular')

~ 4H3P03 (aq)
or:
P401o (s) + 6H20 (I)

(i)

SRJC Chemistry Paper 3

Na20 (s) +H20 (I)

SRJC Chemistry Paper 3

6.

4H3P04 (aq)

(i)
H H 0
I
I
II
H-N-C-C-OH
I
HO-y-H
CH 3

Condensation

Reduction

(ii)

Dilute HC/, heaUreflux

(iii)

or Aqueous NaOH, heaUreflux

H-~-t-~-oNa'
I

H-0-C-H
I

H-C-H

(iii} Oxidation

OH

C:;'s""'/:oo

(iv)

I
I
II
o-y-y-c-+

CH3
(b)

CH 3 NH2

(b)

(i)

Dipeptide:

OH

\V)--9-H

0 H
II I
H2N-yH-C-N-yH-C02H

CN

yH2
C02H

(II)

(c)

I;

HCN, aqueous NaOH or NaCN, cold 10- 20C

II:

dilute HCI, heat

III:

CH3CHIOH\COOH and concentrated H,so reflux

Melting point of CeHsNH2 < CaHsC02H:

(Iii)

Mechanism: Nucleophilic addition


Step 1:

NaCN
Step2:

yH2
OH

Both are simple molecular compounds


Extent of strong Intermolecular hydrogen bonding :
CsHsNH2 < CsHsC02H
Energy required: CaHsNH2 < CsHsC02H

Melting point of CsHsCH2CH(NH2)C02H is highest:


It exists as a zwitterion and is thus an ionic compound
Melting involves overcoming strong electrostatic force of attraction between oppositely
charged ions I zwitterions
Very large amount of el'\ergy required

Na+ +eN
H

<Qj-?-o
CN

C6HsNH 2 is insoluble in water due to presence of hydrophobic benzene ring that outweighs the
ability of -NH 2 group to form hydrogen bonds with water
Step 3:

+CN"

C6HsC02H is sparingly soluble in water due to ability to form more extensive hydrogen bonds
with water despite the presence of hydrophobic benzene ring
CaHsCH2CH(NH2).C0:2H is soluble in water:
formation of ion-dipole interactions/bonds with water molecules results in the release of
energy which causes detachment of the ions from the crystal lattice structure for hydration

(c)

Q:

SRJC Chemistry Paper 3

r--1

SRJC Chemistry Paper 3

:--.
'
)

r---"1

'

~
I

'

(----,

,,----...,

'

(a)(i)

l---"'J

(a) (i)

Structure of D:

IR\_N-C-CH3
~-Ill
H 0
Structure ofF:

(a)(il) D has a labile proton:


Signai at Q..::..;W_ corresponding to labile proton will disappear on adding 0 2 0

(b)

H:

G:

(II)

F has two types of labile protons:


Signal at o= 5.9 and 11.0 corresponding to labile protons will disappear on adding 020
B(c)

(b)

Step 1

Step f

H
I
H-C

-15''

concentrated HN0 3, concentrated H,S04, reflux 55

Step 2:

Sn, concentrated HCt

oc

Stop Z

11

HzN~9

CHl

Step/:

SRJC Chemistry Paper 3

H
I
H-C-OH

~ 1
H 2 N~q-H

CH3

Intermediate
Step t:

concentrated H,S04 , 180 oc

Step2:

KMn04, dilute H,so4. reflux

SRJC Chemistry Paper 3

10


(a)

(b)

(b)

glu-alaglypheala-glyglu
Since the <Donly amino acid with aromatic ring is phenylalanine,
hydrolyses 1 amide bond at C=O end of phe.
mRNA: CUe eGG CeG AAA CGG CeG CUe
DNA : GAG Gee GGC TTI Gee GGC GAG

0
II
R,-C-0-CH,

(c) Genetic code is a <D code for the sequence of amino acids in a protein
It consists of a> triplet code or codon in which a sequence of 3 bases along the DNA
molecule codes for 1 amino acid in a protein

R 2 -~-0-yH

{d)

~ I
II

(c)

Due to the <Dhydrophobic nature of the non-polar tail facing the a> interior of membrane,
there are no favourable solute-solvent interactions with polar water molecules or
water soluble species for them to pass through the membrane
<DActive transport via NatK pump which alpumps K into the cell and Na out of cell.
This involves energy from hydrolysis of ATP & membrane proteins s.a. Na-K
ATPase

Vnwc 80pmol mln1

stabilised by hydrogen bonds

~N...c~

~c/N~

II

II

or

p chains]

[anti-parallel P chains]

(e)

(i)

(Iii) Prevent the rotation of the membrane from one side of membrane to the other.
Ra1e /pmoldm 4 mrnt

0
;

stabilised by hydrogen bOnds

[parallel

(I)

II

II

X Is contnbuted by an alcohol

(d)

~c/~

~c/N~

OH

(ii)

xo-~-o-cH,

(i)

(Ill)

yH2 ~

.-------=--

yH2 9

yH2

H2N-CH-C-~-CH-C02H

yH2

H2N-CH-C-~-CH-C02H

or

~ (lv) presence of Ag

...
..
.

Ym... 40
2

25

(ii)

,~:,roD.

~~~<~OJ

[Substrate]/ pmol dm.,

100

(ii) Ag ions act as <Dnon-competitive inhibitor by combining with some region in or near the
active site and hence alprevent the substrate from binding to the active site or deforms
the enzyme so that the ES complex cannot be formed
It decreases Vmax but Km remains constant curve

~.
Fig. 1st

Unit

2nd

Unit

(iii)

~N

c.~~~r~o

'---v---' '---v---'
adenine

11

SRJC Chemistry Paper 3

r
-1

chrymotrypsin a>only

12

SRJC Chemistry Paper 3

r--1
'

deoxyribose phosphate

.....--,
l
'
When the compressed gas expands, work Is done i kinetic energy is used to overcome
intermolecular forces of attraction.
:::::> Gas ccols I temperature decreases
Cooled gas is used to cool more incoming air, until temperature is below critical
temperature. Liquid air is obtained.
Oxygen can be obtained from liquid air via fractional distillation.

(i)
:. The oxides are:

FeO. ZnO, CuO and Ag,o

(ii) C Is) + ZnO Is\ 7 Zn Is\ +CO lgl

(c)(i) First law of Thermodynamics states that energy can neither be created nor destroyed
and the energy of the Universe remain ccnstant.
(iv)

c.n, change in no. of mol of gas = + %

~ !lSa Is positive

From !lGa = LlHa T!lSa,


As temperature is increased, magnitude ofT!lSa >magnitude of LlHa
:::::> !lGa will become more negative as temperature increases.
11(b)
(I)

CH4 (g) + 202 (g) 7 2H20 (I)


!lSa

+ C02 (g)

= 2Sa (H20) + sa (C02)- [Sa (CH.) + 2Sa (02)]


=- 339.12 JK'1mor1

t.n in no. of mol of gas =- 2. :::::> t.s is negative


(ii)
LlHa

=2LlHar(H20) + LlHar1 (C02)- [LlH 6r (CH.) + 2LlHar (02)]

=- 910.65 kJ mor

t.G = t.H - T6S = -8.0959 x 105 J mor1

For t.Ga = -nFE 6celt :::::>


:::::>

-8.0959 X 105 = 4(96500) Eacell


E6ce/1=+2.10V

(iii)

W = - p.W =- MRT = + 22.3 kJ

(iii) LlU

= LlH + W = LlH + (- p!lV) =- 5450 kJ mol'1

(4)(i) Water liberates heat to its surrounding when it freezes.


:::::> t.S of surrounding (L>Ssurr) Increases
:::::> t.S of water (L>Ssys) decreases
Using LlStot = I'ISsys + LlSsurr
Since LlSrut > LlSw => !lSI2l > 0
:. Freezing of water is still a feasible/spontaneous reaction.
(ii) When NH.N03 dissolves, solid NH.N03 dissociates into its aqueous ions
NH.N03 (s) 7 NH. (aq) + N03- (aq)
=>There is increase In entropy. Hence LlSsotn is positive
Using 6Gsoln LlHsoln- TLlSsolno
where both .dHsoln and .dSsoln are~ in this case:

At room temperature, since salt is soluble


:::::> LIGsoln < 0 (negative)
:::::>magnitude ofT LISsoln > magnitude of LIHsoln
When T increases, magnitude ofTLlSsoln magnitude of LlHsoln
:::::> LlGsoln becomes more negative
:::::> Solubility increases rapidly with temperature

-1252x10 3 = -1411.1 x10 3 -(298)!1Sa


=>
LlSa = 533.89 J mol' 1 K' 1
LIS 6 = 2 Sa(C02) + 2 Sa (H20)- [S 6 (C2H4) + 3
:::::> Sa (C2H 4 ) 293.492
+ 293 J moi'1 K'1

(ii)

s (02)]

Cp > Cv
At constant pressure, energy absorbed during heating is partly used to expand the gas I
to do work (gas has to do work against the surrounding)
Hence, more energy is required to raise the temperature of the gas by 1K.
At constant volume, no work is done, all energy absorbed is used to raise the
temperature by 1 K.
(b)

Water & C0 2 is first removed from air.


Air is cealed, then ccmpressed.
Compressed air Is allowed to undergo adiabatic expansion
=:>There must be no transfer of heat between system & surrounding
SRJC ChemistryPaper 3

13

SRJC Chemistry Paper 3

14

r-:

11
l

'

r----''

L
[
[
[

YISHUN JUNIOR COLLEGE


Preliminary Examination 2006

925.1' 9258/01

CHEMISTRY.
PAPER 1 Multiple Choice'

25 AUGUST 2006
FRI 1430h - 1530.h
.1 hour

Additional material:
Multiple Choice answer sheet
. Data Booklet

!UN JUNIOR COLLEGE YISHUN JUNIOR COLLEGE YISHUN JUNIOR COLLEGE YISHUN JUNIOR COLLEGE YISHUN JUNIOR COLL
!UN JUNIOR COLLEGE Y/SHUN JUNIOR COLLEGE
.
.
YISHUN JUNIOR COLLEGE.YISHUN JUNIOR COLL
VN JUNIOR COLLEGE YISHUN JUNIOR COLLEGE
~-~ ., YISHUN JUNIOR COLLEGE YISHUN JUNIOR COLL .
!UN JUNIOR COLLEGE YISHUN JUNIOR COLLEGE
J ;;"
YISHUN JUNIOR COLLEGE YISHUN JUNIOR COLL
ff.JN JUNIOR COLLEGE YISHUN JUNIOR COLLEGE~,.
a J Y/SHUN JUNIOR COLLEGE YISHUN JUNIOR COLL
!UN JUNIOR COLLEGE YISHUN JUNIOR COLLEGE
' . . . .
. .YISHUN JUNIOR COLLEGE YISHUN JUNIOR COLL
TUN JUNIOR COLLEGE YISHUN JUNIOR COLLEGE
X"'4i
Y/SHUN JUNIOR COLLEGE YISHUN JUNIOR COLL
J .. ~~ . YISHUN JUNIOR COLLEGE YISHUN JUNIOR COLL
!UN JUNIOR COLLEGE YISHUN JUNIOR COLLEGE
TUN JUNIOR COLLEGE YISHUN JUNIOR COLLEGE~- ,.

,;:_Yf.SHUN JUNIOR COLLEGE YISHUN JUNIOR COLL


!UN JUNIOR COLLEGE YISHUN JUNIOR COLLEGE

. ll'ciSHUN JUNIOR COLLEGE YISHUN JUNIOR COLL.


!UN JUNIOR COLLEGE YISHUN JUNIOR COLLEGE
.
~ YISHUN JUNIOR COLLEGE YISHW JUNIOR COLL
TUN JUNIOR COLLEGE Y/SHUN JUNIOR COLLEGE
. ....,
YISHUN JUNIOR COLLEGE YISHUN JUNIOR COLL.

"f

~~~ ~. ~~

i..ai .

T(JN JUNTOR

COLLEGE Y!SHUN JUNIOR COLLEGE YTSHUN JUNIOR COLLEGE Y!SHUN .JUNIOR COLLEGE Y!..'>Hl!N .Jl!NIOR COLI.

[
READ THESE INSTRUCTIONS FIRST

Write in soft pencil.


Do not use staples, paper clips, highlighters, glue or correction fluid.
Write your name, class and index number on the answer sheet in the spaces
provided.

There are forty questions onthis paper. Answer all. questions. For each question
there are four possible answers, A, 8, C and D.
Choose the one you consider correct and record your choice in soft pencil on the
separate answer' sheet.

Each correct answer will score one mark. A mark will not be deducted for a wrong

answer. Any rough working should be done jn this booklet.

L
l
L
(

I~

This question paper consists of 13 printed pages.

9251, 9258/P1/YJC2006

L
ll

Section A

For each question there are four possible answers, A, B, C, and D. Choose the one you
consider to be correct

1.

The main. ingredi~nt. in chocolate bars is sugars, typically about 47% of milk
chocolate bar's. If these sugars are represented by sucrose,. C12H22011 (Mr = 342),
how many sugar molecules are there in 1000kg of chocolate bars?
9.67 X 1025 ..
2..82 X 1026

B
D

[
[
[

8.27 X 1026 .
3.57 x.10 27

[
2

Consider the following half equations

Fe2+ -7 Fe3+ + e
2
.
C204 - -7 2 COz + 2e
In an experiment, 20 cm 3 of an acidified solution of 0.02 moldm-3 FeC20 4 is
oxidised by 15 cm3 of 0.02 moldm-3 KMn04. What is the final oxidation state of
manganate(VII)?
A

+2

+3

+4

4 dm 3 of ethene were burned in 16 dm3of oxygen and the resulti-ng mixture cooled

to room temperature. The residual gas would coritaln

equal volume of carbon di.oxide and oxygen.

twice the volume of carbon dioxide as oxygen.

0
4

+1

... twice the volume ofoxygen as

~rbon

dioxide.

only carbon dioxide.

Samples of Fz-gas and Xe gas are mixed in a container of fixed volume. The initial
partial pressure of the Fz gas is 8.0. atmospheres and that of the Xe gas is
1.7 atmospheres. When all of the Xe gas reacted, forming a solid compound, the
pressure of the unreacted F2 gas was 4.6 atmospheres. The temperature remained
.
constant. What is the formula of the compound?
A.

XeF.

XeF2

C XeF3

XeF4

L
[

[
9251/P1NJC2006

L
L
r

~
~

The diagram shows the mass spectrum of an organiG compound, X:


abundance

42

ll

29

15
74

57

. 10

[
[
6

L
[

II

.,l

20

30

40

. 50

CH3CH2COOH

CH3CH2CH20CH3 .

CH3CH2COCH3

CH3CH2CH(OH)CH3

Triton has a low atmospheric pressure.

Solid methane has a metallic structure.

Methane molecules contain strong C-H bonds.

The intermolecular forces between methane molecules are weak.

When 25 cm 3 of 1 moldm-3 sodium hydroxide. is neutralised with an equal volume of


1.0 moldm-3 hydrochloric acid, the temperature of the mixture rose by 6.8 C. What
will be the temperature change if 50 cm 3 of 0.5 moldm-3 sodium hydroxide is
neutralised with an equal volume of 0.5 moldm-3 hydrochloric acid? (Assume heat
. losses to be negligible in each case.)

1.7C

3.4 C

6.8 C

f~

I.

70
80
relative mass (m/e)

The voyager probe has shown that the surface of Triton, a moon of the planet
Neptune, contains solid methane which flows rapidly.

L
L

I
60

Which of the following statements explains the mobility of solid methane?

ro

I.

What ls X most likely to be?

9251/P1/YJC2006 .

13.6 C

l_

L
[

H2P04- + HBO/- ~ HPO/- + H2B03-

The equilibrium constant for the reaction represented by the equation above is
greater than 1.0. Which of the following gives the correct relative strengths of the

acids and base in the reaction?


eases

Acids

H2P04- > H2B03-.

and

. HBol- > HPo/-

H2B03- > H2P04:

and

HBol-> HPO/-

H2P04" > H2B03-

and

HPO/-> HBol-

1-:12803- >

and

HPO/- > HBol-

H2P04-

[/
[

P.E.

[I'

X+Y

ill

reaction co-ordinate
The en~rgy diagram for the reaction X + Y -7 Z is shown. The addition of a catalyst
to this reaction woulq cause a change in which of the indicated energy differences?
A

10

I only

II only

I and II only

II and

m only

Which value would be required to estimate the lattice energy for compound UH?
A

The electron affinity of hydrogen:

The first ionisation energy of hydrogem.

The lithium-hydrogen bond energy.

The standard enthalpy change of formation

9251/P1/YJC2006

r-,

[
[
[

of UH.

L
l
L

L
{

11

[_

11

The standard electrode potentials of Cu 2+(aq) I Cu(s) and Zn 2+(aq) I Zn(s) are
+ 0.34V and- 0.76V respectively.
Which of the following conclusions can be clrawn from these data?

[
[

Cu is an oxidising agent.

Cu displaces Zn from a solution ,containing Zn ions.

Zn ions can act as a reducing agent.

Zn has a greater tend(;!ncy than Cu to form positively charged ions ..


.

12

[
[

13

14

AI

Na

Si

cr, c1o-

Clo-. cJoz-

cl-, c1o3-

r
[

Atomic number
A

Boiling point of CCI4, SiCI4, SnCI4 and PbCI 4

First ionisation energy of Mg, Ca, Sr and Ba

Oxidising power of Fz, Cl2, Brz and

Atomic radius of Na, Mg. AI and Si

1z

L
L

cr. c1o4-

Which property when plotted against increasing atomic number gives the shape of
the following graph?

The anions containing chlorine which are formed when chlorine is passed into hot,
concentrated aqueous potassium hydroxide are
A

The following elements are reacted with oxygen and the products dissolved in water.
The element which produces the solution with the greatest pH is

9251/P1/YJC2006

L
~

15

Which of the following compounds includes a transition metal in oxidation state zero?

[Ni(C0)4]

[Go(NH:~)a]Ch ..

[Fe(HzO )a]S04

D.

[Fe(H?Oh(OHh]

[
~

[
16

AICh reacts with -UAIH4 and (CH3)3N to give (CH3)3NAIH3.

Which ofthe_following will be true of (CH3)3NAIH3?

17

The A/ atom is tetrahedrally co-ordinated ..

The N atom is electron deficient:

It contains hydrogen bonding.

It is dimeric.

[
[

Which of the following statements about the elements Ca, Sr, and Ba is correct?
A

Their oxides are amphoteric.

Aqueous solutions of the chloride have pH> 7.

Their nitrates decompose to produce oxygen only.

The elements react with cold water liberating hydrogen gas.

[
[

[
18

Although the second ionisation energy ofMg is about double that of its first ionisation
energy, the Mg+(aq) ion does not exist in aqueous solution because
A

Mg2 + (g) has a higher energy content than. that of Mg+(g).

The following reaction readily occurs

2 Mg+ (aq) 7_ Mg (s) + Mg2+ (aq)

19

The hydration of the doubly charged cation gives sufficient stability to


overcome the second ionisation energy.

Hydrogen bonding formation in aqueous solution stabilises the doubly


charged cation.

An element has the following successive ionisation energies (in kJmor1):

u
u
[j

762 (1st). 1540, 3300, 4390, 89so, 11900


Which group of periodic table does the element belong to? A

Group I

Group II

Group Ill

9251/P1/YJC2006

Group IV

L
L

[
[
[

20

Which of the following hydrides most readily decomposes into its element on contact
with a hot glass rod?

Sodium hydride, NaH

. Hydrogeri iodide, HI

. Ammonia, NH3

21

Water, H2 0

A dibromoalkane, CH3CH2CHCHCH2CH3, was treated with excess alcoholic

I I

.Br Br

potassium hydroxide. Which of the following is expected to be the main product?

[
[
[

22

CH3CH2CH20 -K+

CH3CH=CH-'-CH=CHCH3

CH3CH:lCH=CHCH2CH3

CH3CH2CH(OH)CH(OH)CH2CH3

Which of the following reagents and conditions would be used to convert CH 3COOH
into CICH2COOH?
Chlorine in bright sunlight at 100C

f'
L

Boiling with hydrochloric aCid at 100C

c
l

. Phosphorus pentachloride at room temperature

23

Chlorine in the dark with anhydrous aluminium chloride

The antibacterial drug tetracydine has the structure


N(CH3)2
OH

CONH2

OH

How many pairs of stereoisomers are there?

L
L
L
l

3.

9251/P1/YJC2006

L
f'
8

24

Chloroethane CH2 =CHCI is a monomer of polyvinyl chloride, PVC. What are the
bond angles along the polymeric chain in PVC?

25

They are all109.5.

They are all120.

T~ey are an 1809 .

Half are 109.5 and half are 120.

Which of the following is most basic?

&CI

!Q(Gfi,CH,NH,

CH3

. 26

EH

. The following compound is heated with phosphorus_tribromide, PBr

HO

COOH

Which of the following is the main organic product obtained? .


A

CH 2 Br

HO

.c

ECOOH

.Ecos,

Br

HOx)(COBr

ECOOH

sr
9251/P1NJC2006

L
L
L
[

L
L
l
L

r
[

27

A compound Z was boiled with aqueous NaOH and the resulting mixture was cooled
and acidified with dilute sulphuric. acid. The final products included a compound
C 3 H6 0 2 and an alcohol which give a. positive
triiodomethane
test.
.
.
Which of the following formula could represent Z?

[
[

CH3CHzCOOCH3

. CH3CHzOCOCH3

(CH 3)2CHOCOCH_3

D . CH3CHzCOOCHzCH3

28

Which one-of the following compounds is most likely to react with ethane-1,2-diol to
give a polymer of high molecular mass?
.A

HOOCJ\2CHzOH

HO~OH

HOOC~COOH

ct-Q-cooH

29

An optically active compound, CsH 120 is oxidised by an excess of acidified


potassium dichromate(VI) to an optically inactive product, CsH1 0 0. Which of the
following is a possible structure for compound CsH1zO?

A
.B

CH3CHzCH(OH)CH2CH3 .
CH3CHzCH2CH(OH)CH3

CH3CHzC{OH)(CH3)2

CHz(OH)C(CH3)3

[
30

[
[

Some aminoethanoic acid is dissolved in a buffer solution of pH = 9.0. Which of the


following gives the structures of the two main forms of aminoethanoic acid at
this pH?

+NH3CH2C0 and NHzCHzCOz-

+NH3CH2C02_H and +NH3CH2C02-

+NH3CH2C02H and NH2CHzCOz+


.
.
NH3CHzC02H and NHzCHzCOOH

L
L
t'

9251/P1N JC2006

L
[
[

10

Section 8
For each of the questions in this section, one or more of the three numbered statements
. 1 to 3 may be correct.

Decide whether each of the statements is or is not correct (you may find it helpful to put a
tick against the stater:nents that you consider to be correct).

Th~ responses A to D should be selected on the basis of


A

1, 2 and 3

1 and 2 only

2 and 3 only.

1 only

are correct

are correct

are correct

is correct
;

31

Which of the following molecul~s are planar?

32

N2H4

BF3. NH3

sr-

Ca2 +

Sc3 +

,.

Particles having the electronic configuration 1s2 2s2 2p6 3s2 3p6 include

r
[

CeHsCN

[
[

r
"'-,J

. 33

4 HCI (g)

02 (g) ~ 2 Ciz {g) + 2 .H20 (g)

Equal number of moles of HCI and 02 in closed system are allowed to reach
equilibrium as represented by the equation above. Which of the following must be true
at equilibrium?
1

[HCI] must be tess than [Ciz]

[02] must be greater than [HCI]

34

. [CI2] must be equal to [H 20]

Which of the following would constitute a buffer solution?

100 cm 3 of 1 mol om-3 ofCH3COOH and 50 cm 3 of 1 mol dm-3 NaOH


50 cm 3 of 1 mol dm3 of HCI and 100 cm 3 of 1 ~of dni-3 NH (aq)

50 cm of 1 mol dm- HCI and 50.cm 3 of 1 mol drh-3 of NH3 (aq)

L
L

9251/P1/YJC2006

L
Ll

L
L
L
\

L
["'
'"

L
[

11

The responses A to D should be selected on the basis of

. _1, 2 and 3

1 and 2

are correct

35

~mly

are correct

2 and. 3 only .

1 only

are CO(rect

is correct

. The rate equation for the reaCtion P + Q ~ R + Y , A H < 0, is


Rate= k

Which of the following graphs are correct?


1

Rate

[
[Q]

2
[Q]

c
c

time

3
Energy

l
[

\/

L
L
L
['

progress ofreaction

36

Which of the following concerning the element astatine (atomic no. = 85) are
consistent with its position in group VII?
1

Silver astatide is insoluble in aqueous ammonia.

The element is a solid at room temperature and pressure.

Hydrogen astatide is less stable to" heat than hydrogen iodide.

9251/P11YJC2006

\.
f~

12

The responses A to D should be selected on the basis of


A

. 1, 2 and 3

1 and 2 only

2 and 3 only

are correct

are correct

are correct

. 1 only
is corre.cf.
-

37

38

39

. In which of the. following pairs of species will the d-block elements' have the same
. electronic ~onfiguration?

V02:r and .Tiel 3

. Mn04 - and Cr201 2-

Cr and Co(H20)a 3+

In which of the following pairs can the members be distinguished by their different
solubilities in water?
1

Ethanol and ethanoic acid

Phenyl benzoate and sodium benzoate

Propanone and methylbenzene

-_

-~

Cinnamaldehyde is a fragrant compound of cinnamon.

o-~=CHCHO
Which of the following reagents, when added to cinnamaldehyde, would show a
change of colour?

Dilute acidified potassium manganate (VII)

Bromine

Fehling's reagent

c
L
[

9251/P1/YJC2006

L
l.
l
L
L

c
[

13

The responses A to

[
[

40

b should be selected on the basis of

1, 2 and 3

1 and2 only

2 and 3only

1 only

are correct

are correct

are correct

is correct

I
I

The following scheme involving cold aqueous sulphanilic acid,


H0 3 S---o-NH2

~-

and

pota~sium nitrite is a possible test for traces of gunshot.

-,

_,

c
[

H03 S - o - N H2

11

~I

Ho-s-Q-+
II
N=N
0
-

II
K+

02S - o - N = N
u

c
[

HO-

-j

so -~

Which of the following statements concerning the scheme is true?


1

Compound T is formed as a result of sulphanilic acid acting as a base.

The potassium sulphonate groups make compound U soluble.

Step II involves an electrophilic substitution reaction.

L
[

END OF PAP-ER

L
L
[

L
t.

9251/P1/YJC2006

.----:J

[
[

Name: ____________

YtsHuN JUNIOR coLLEGE


Preliminary Examination 2006

9251' 9258/2

CHEMISTRY
Paper 2

23 AUGUST 2006

L
'

WED 0800h - 0930h


1 hour 30 min
Candidates .answer on the Question Paper.
Additional materials:

Data Booklet

'

c
[..

[
(

CTG: - - -

ruN JUNIOR COUEGE YISHUN JUNIOR COLLEGE YISHUN JUNIOR COLLEGE YISHUN JUNIOR COLLEGE YISHUN JUNIOR COLL
ruN JUNIOR COLLEGE YISHUN JUNIOR COLLEGE\' .:/'_,. : ...... ...... .-: '<\.'YISHUN JUNIOR COLLEGE YISHUN JUNIOR COLL
rUN JUNIOR COLLEGE YISHUN JUNIOR COLLEQE ,:;:(: ~-~~~ ,:.i}:YISHUN JUNIOR COLLEGE YISHUN JUNIOR COLL
rUN JUNIOR COLLEGE YISHUN JUNIOR COLLEGE .:.::
::)'[SHUN JUNIOR COLLEGE YISHUN JUNIOR COLL
rUN JUNIOR COLLEGE YISHUN JUNIOR COLLEGE(''' .
.'-YISHUN JUNIOR COLLEGE YISHUN JUNIOR COLL
fUN JUNIOR COLLEGE YISHUN JUNIOR COLLEGE;.. .... . .
., :' '' . . YISHUN JUNIOR COLLEGE YISHUN JUNIOR COLL
iUN JUNIOR COLLEGE YISHUN JUNIOR COLLEGE
'riSHUN JUNIOR COLLEGE YISHUN JUNIOR COLL
rUN JUNIOR COLLEGE YISHUN JUNIOR COLLEGE ' ...,.. . ::;~
.YISHUN JUNIOR COLLEGE YJSHUN JUNIOR COLL
rUN JUNIOR COLLEGE YISHUN JUNIOR COLLEGE :_a,
, l) :: :': :\r:. _...YISHuN JUNIOR COLLEGE YISHUN JUNIOR COLL
rUN JUNIOR COLLEGE YISHUN JUNIOR COLLEGE:.~ .
. .
~JiiYISHUN JUNIOR COLLEGE YISHUN JUNIOR COLL
rUN JUNIOR COLLEGE YISHUN JUNIOR COLLEGE:.:-}~-- . . -;.L~:: YISHUN JUNIOR COLLEGE YJSHUN JUNIOR COLL
ruN JUNIOR COLLEGE YISHUN JUNIOR COLLEGE
:: . . Y~
YISHUN JUNIOR COLLEGE YISHUN JUNIOR COLL
TUN JUNTOR COLLEGE Y!SHCJN J(JN/OR COLLEGE YJSHCJN J(JN/OR COLLEGE YISHCJN J[JNJOR COLLEGE YTSHrJN.![JN/OR COLL

.;;~:\ a
.r.,.
.. .. . \..:
::~f .,.~~ . ~, ~~,
..::. ..

,/

READ THESE INSTRUCTIONS FIRST


Write your name and CTG in the sp<'!ces at the top of this page. .
Write in dark blue or black penin the spaces provided on the Question Paper.
Do not use staples;paper clips, highlighters, glue or correction fluid.
Answer all questions.
The number of marks is given in brackets ( ] at the end of each question or part question.
You are advised to show all working in calculations/
For Examiners'
:You may use a calculator.

'

2
3

..

TOTAL

L
L
L
t.'

-----

This question paper consists of 12 printed pages.


9251,9258/ P2JYJC 2006

Use

\
J

2
1.

['

The yellow gas, chlorine dioxide, CI02 has been used as a flourimproving agent in bread-making. It is a powerful oxidizing agent in
acid solution:

czo2(~q) + 4H+(aq).+ se ~ cr(aq) + 2H20(J), E9 =+1.50V

{a) . Draw. th~...dot-<:rois diagram for GZ02. Using the coneept of


valence-stiell..;electron-pair repulsion theory, predict and explain the
. shape of CI02.

[2l

[
[

c
(b)

Predict the reaction of chlorine dioxide, in acidic solution, with


H202. Write an equation for the reactions that occurs and calculate
the E9cell value.
[2]

[
{c)

Consider the following reaction involving chlorine and silver (i)


chlorate (v):

L
L

2AgCI03(s) + Ch(g) ~ 2AgCI(s) + 2Cl02(9) + 02(g), AH=O kJmor1

0)

Draw an energy cycle to calculate the standard enthalpy change of


formation of CI02(g) using the following data:
Standard ~flange of formation of AgC/03(s) = -25 kJmor1
Standard change of formation of AgC/(s) = -127 kJmor1

9251,9258/ P2/YJC 2006

[2]

L
L
L
L
L
\

[
[

(ii)

Comment on the value obtained in c(i).

[1]

C/0 2 oxidises KBr c;!s well as KI to Br2 and I 2 respectively.


The lattice energies of potassium halides are given below:

Lattice Energy/ kJmor1

Compoun~

. -670
-629

KBr

KI

(iii)

Suggest an explanation for the trend shown in lattice energy.

(iv)

Using the full information:

[2]

-,

._,;

[
[

c
[

K+(g) + aq
K+(aq). ~H
B((g) + aq ~ B((aq) ~H

=-322 kJmol1
=-335 kJmor1

Calculate the enthalpy change for the process:

KBr(s) + aq ~ ~(aq) + B({aq)

c
[

L
L
L
[

9251, 9258/ P2 I YJC 2006

[2]

r
[

4
(d)

In an experiment to determine the equilibrium constant, Kp. of the


reaction in (c), chlorine gas at 100 kPa with an excess of silver(!)
chlorate(V), AgCZ03(s) was allowed to reach equilibrium in a vessel
at constant volume at 90

oc:

l,

The partial pressure variation of CZOz(g) was graphically presented


~~-

Pressure/

kPa

100

80

.:
10

(i)

Time/min

Determine the value of the equilibrium constant, Kp, including its


units.

[
[2]

[
[
(ii)

How would you expect the equilibrium composition to change when


the temperature is increased?

[
[1]

c
[~

(iii)" At the tenth minute, the volume of the vessel is suddenly halved.
.Sketch the variation of pressure of CIOz(g)with time.

9251, 9258/ P2 I YJC 2006

[2]

L
l
L
l
\

r
[

5
(e)

Explain why boiling point of C/Oz is higher than boiling point of 0 2

[2]

[
[
[

[total: 18 marks]

c
[

2(a) Under suitable conditions iodine and chlorine reacts to give


interhalogen compound, !Clx. When a pure sample of ICZx was
dissolved in an excess . of. aq KI, the _iodine liberated requires
17.1 cm 3 of 0.1 moldm-3 NazS2 0 3 solution for the titration whereas
the ch.loride ion liberated requires 12.8 cm 3 of 0.1 moldm-3 silver .
nitrate for complete precipitation.
Determine the value of x. Hence, write a balanced equation for the
[4]
reaction between !Clx and r.

c
[

L
[
[

L
l
[

L
L

9251,9258/ P2/ YJC 2006 .

c
[

6
(b)

Under suitable conditions, chlorine and fluorine gas react to give an

interhalogen compound, CJF.

Sketch the vaiiation of pV against p at 300k for two moles of (i) an .


ideal gas and (ii) ClF ga~f.

Label your graphs clearly~ (p = pre~sure, V

=volume)

(2]

r
[

c
['
f~'

(c)

2 moles of C!F(g) is subjected to higher temperature of 600 K


Using the same plot in sketch (b), show the change of variation of
pV against p at 600 K for 2 moles of C/F(g) .. Explain your sketch.
3]

[
['

[
[Total: 9 marks]

9251, 9258/ P2/ YJC 2006

l
l
l
L
L

l
[
[
[

7
3.

(a)

Consider the two metals, A and B, whose electron configurations


are as follows:
A: 1s2 252 2p6 3s2 3p6 4s2 .
B: 1s2 2s2 2p6 3s2 3p6 3d 5 4s 1
Explain which of the two metals are ahle to form ions of stable
variable oxidation states.

[2] .

[
[

c
r

(b)

List the possible oxidation states for metal A and metal B.

[2]

(c)

One of the metals form stable complex ions.

(i)

Write down the formula of the complex ion that would be formed
between the M3 + of this metal and excess OH- ions.

(1]

(ii)

Whatshape would the coinplex ion take?

(1]

(iii)

Explain why this complex is coloured.

[2}

-~

.J

_,

[
[
[

[
[

L
l
L
L
l

[Total: 8 marks]

9251, 9258/ P2 I Y JC 2Q06

c
[

4.

This question concerns the preparation of an organic compound


QQ using the apparatus illustrated below.
.
;

._..

.:

. . .

..;;:til1!"""" . :.,.,. /'.''""" ::.":./


:-

--~..

-: . ~ _::

...--.

-:
. ..

:: _-:. .=:

.:txf'n; .:

,~

..

.:

.'
._.:.:

[
........

...
:

c
L

. . -c
A mixture of concentrated H2S04 and CH3CH(CHs_H3}CH(OH)CH 3
heated in the flask to about 170
A compound PP, which is the
major product of the reaction, was formed and PP reacted with
bromine in the second test tube to form QQ.

oc.

PP~

(a)

Give the structural formula of the compound

(b)

Give the structural formula of compound QQ and give its IUPAC


name.
[2]

(11

c
L
L
[

L
[
9251, 9258/ P2 I YJC 2006

u
[1
\

'

c
[

9
(c)

Suggest a reason why an excess of concentrated H2S04 was used


for the reaction taking place in the flask.
[1]

[
[

(d) -.~The reaction mixture in the flask developed a light brown colour._
which becomes black on heating to about 170 ac. Suggest a
po_ssible identity for the black colouration: .
. [1}

(e)

State the type of organic reaction mec(lanisin involved in the


formation of QQ from PP and write down clearly the steps involved
in its formation.
[2]

(f)

Compound QQ formed is usually contaminated with excess Br2.


Suggest a suitable way to remove excess Br2.
[1]

c
[

[
[
l

~-,
--'

[
[

L
L
L.

9251, 9258/ P2 I YJC 2006

I
J

10

{g)

suggest a simple chemical test which can be carried out to show


that pure QQ is a bromine containing compound.
[2]

[
[
[Total: 10 marks]

5.

to

Novocaine is a local anaesthetic which is used for mirior surgery:


areas such as the nose, throat or eyes. It is applied to the surface

before surgery.

[
[

Novocaine has the following structure:

~C2H5
N~C-O-CH 2CH 2 ~
~~
c~s

H,

(a)

IT

Name two functional groups in this substance.

[
[1]

[
f'

{b)

Novocaine reacts with hot dilute HCI.

(i)

Name the type of reaction which has taken place.

L
[1]

[
[

. {ii)

Draw the structure(s) of product(s) formed in this reaction.

[2]

L
[J

u
u
9251,9258/ P2/ YJC 2006

L
\

['
[

11
(iii)

Give the obsetvation when aqueous Brz is added to novocaine.


Draw the structure of the product formed.

[2]

[
[

[
[

(c)

Novocaine has a Kb of 4.1 x 10 -7 moldm-3.

(i)

Account for this basic character in terms of its structure.

[1]

[
[
[

Another isomer of novocaine has the structure shown:


H

L
[

/C2H5

HO ~c--N-CH,CH,,

(ii)

II

C2H5.

Compare arid explain the basicity of this isomer with novocaine.

L
L
L
l~

9251, 9258/ P2/ YJC 2006 .

[2]

t
~-

12
(d)

The synthesis of novocaine is given by the following path.

-~I~.n-~
-~

-~N~

~N~

No'\tlcaine

. m

CH 3 CI
AICI3

' \ f!,N--cooH /
(i)

Give the reagents and conditions for steps (I). tq (IV)

[4]

c
r~

(ii)

Give the structure of A.

[1]

r.
[
[
[

(iii} Give another likely product of A.

[1]

L
L
[

[fotal: 15 Marks]

9251, 9258/ P2 I YJC 2006

L
[
\

[
[
YISHUN

JUNIOR

COLLEGE

PRELIMINARY EXAMINATION 2006

9251,9258/3

CHEMISTRY
PAPER 3.

18 AUGUST 2006

Friday

0800h-1045h

Additional materials:
Data Booklet
Pap_~r

[
['
-,

_,

[
[

r,

[
[

TIME

2 hours 4'5 minutes

INSTRUCTIONS TO CANDIDATES
Do not open question paper until you are told to do so.
Write your name and .CTG on the on all the work that you hand in:
Write iri dark blue or black pen on both sides of the paper.
You may use a pencil for any diagrams, graphs or rough working.
Do not use staples, paper clips, highlighters, glue or correction fluid.
Answer all questions in Sections A, B and C. The last question in each .section is of the form
. either/or. Answer both questions for the section on chemical thermodynamics.
At the end of the examination, fasten all your work securely.
The number of marks.is given in brackets []at the end of each question or part question.
A Data Booklet is provided.
You are reminded of the need for good English and clear presentation in your answers.

L
L
[

This question .paper consists


cif 13 printed pages
9251 ,9258/P3NJC2006

~:

,.--.,
)

L
2

Section

Answer all the questions from this section.

[
1

{a)

(i)

Determine the nufTiber, charge and location cit the sub-atomic particles in
an atom of gallium, 69 Ga and give the electronic configuration of Ga ion.
[3]

. (ii) .

[
[

{b)

The solubility product ofgalliurn chloride has a value of 1.60 x 105 mol4 dm-12 at
298 K. Give an expression for the solubility product. of gallium chloride and
calculate the solubility of gallium chloride in a solution of gallium chloride in Which
the total concentration of gallium ions is 0.1 mol dm~3 at 298 K.

.
.
[3]

(c)

Gallium is manufactured by the electrolysis of molten gallium chloride. A current


of 10 A flows for 30 min 10 s. Calculate the mass of gallium produced.
[2]

Naturally OcCUrring gaiJium, Ga,: is a mixture of tWo iso"topes, gallium-69


and gallium~71. Use this infoimation together with the relative atomic
.mass of gallium from - the Data Booklet, calculate the percentage
abundance of each isotope.
[21

[Total: 10]

[
[
[
[
[

L
L
[

9251,9258/P3/YJC2006 .

t
[
[-

(a)

The bromination of propanone is acid-catalysect


H+
.
- CH3COCH3 + Brz ~ CHaCOCHzBr + H+ + Br-

The rate of disappearance of bromine was measured for several different


concentrations ofJiropanone, bromine and.W at a certain temperature:

Experiment
No

1
2
3'
4
5

[CH3COCH3] I
moldm-3
0.30
0,30
.. 0.30
0.40
0.40

- [Br2] I .
mol dm-3

[H+) I
moldm-3

0.050
0.100

0.050
0.050
0.100
0.200
0.050

0~050

0.050
0.050

Rate of disappearance
- of Br2/
mol dm-3 s-1
5.7 x-1o-5
5.7x10-5
1.2 X 10-4
3.1 X 10-4

___I.6 x_1o-~

(ii)

Determine the order of reaction with respect to each of the reactants and

hence the rate equation. Show your working clearly.


[3]

By means of an energy profile diagram, explain how a catalyst affects the rate of
a chemical reaction.
[2]

(c) _ The following reaction A~ B is found to be first order with respect to A If the
initial concentration of A is 1.6 mol dm-3 and the half-life for the reaction is 5 mins,
sketch a graph-of ~oncentration of A against time for the first 20 minutes of-the
. reaction.

[2]
(d)

Determine the rate constant, k, stating its units.

[1]

(b)

[
[

..

(i)

8.H

=-90 kJ JnOI-

CD+ C~E

L
l:

slow
fast

. Given _that the activation energy of the reaction is +35 kJ mol-1


energy profil~ diagram for the above reaction.

_,

The reaction occurs via the following mechanism:


C+D~CD

.,
l

C reacts with D. according to


. the equation:
2C+D-')>E

Sketch. the

. [2]
[Total: 10]

L
[

9251 ,9258/P3/YJC2006

L
[

L
4

[
3

either
(a) Human plasma is buffered mainly by dissolved C02 which hasreacted to form
carbonic acid.

. H2C03 (aq) .= H+ (aq) + HC03- (aq)

[
[

(i)

Explain how carbonic acid can buffer human plasma ..

(ii)

Write an expression for the acid dissociation constant,


.

c
['

_,

Usually the pH of human plasma is 7.4. Calculate the ratio of the [HC0 31
to [HzC03] in the plasma.
[6]

(b) Aspirin, a monobasic acid is a drug which lowers the body temperature rapidly
and effectively in a feverish patient as well. as a painkiller. It was found that 22.5
cm3 of 0.100 mol dm-3 of NaOH completely neutralised 25.0 cm 3 of aspirin.

(i)

Calculate the concentration of aspirin.

(ii)

Explain, with an aid of an equation, why the pH of the salt solution formed
at neutralisation is not equal to 7.

(iii)

Suggest a suitable indicator for this titration, giving a reason for your
choice.

[4]
[Total: 10]

[
[

['

_(iv)

When the concentrations of carbonic acid and hydrogencarbonate ions are


equal, the concentration of H+ ions is 7.90 x 10-7 mol dm-'3. Calculate the
value. of Ka. '
.

l~

. (iii)

[
[

Ka. of carbonic acid.

9251,92S8/P3/YJC2006

l
~

or
(a)

Ammonia is a weak base, pKb

=4.76.

Calculate the pH .of


0.20 . mol dm-3 of ammonia solution at 298 K.
assumptions were made in your calculation?
.

What

[2]
(b)

Ammonium ~hloride; N.H4Cl is added to 1 dm3 of the above


8.9 is noted.

(i)_

soi~tio~ and a

pH of

Calculate the mass of NH 4Ci added.


[2]

(ii)

(c)

cm 3.. of

[
[

50
0.2 mol dm- of hydrochloric acid is added to the reaCtant mixture
in (b).- Calculate the change in pH and comment onthis change.
[3]

Sketch a graph of pH against volume of HCi to illustrate what happens during the
titration of 50 cm 3 of 0.2 mol dm-3 ammonia with HCZ of the same concentration.
You are required to show calculated values in your sketch.
[3]
[Total: 10]

[
[
[
[

['
[

[
[~

l
9251,9258/P3NJC2006 .

L
\-

[
6

Section B

Answer all the questions from this section.

(a)

[
[

State and explain how each of the following properties varies across the. third
period of the Peri~dic. Table.from Na to C/:

(i)

th~. first

(ii)

tlie boiling point of chlorides

(iii).

the acid/base behaviour of the oxides

(iv)

the electrical conductivity of the elements

ionisation of the elements

[8]
.

(b)

-.

....

-:..;..._

"Beryllium differs from the other group li metals bufshows a strong resemblance
to aluminium instead." .

Comment on this statement by referring to the bonding of the oxides and


chlorides of beryllium ..
[2]

[Total: 10]

[
[
[

L
[

L
L
l
l"
L,

9251 ,9258/P3/YJC2006

l
[
~

either
(a) Describe the physical states and colours of Ch, Br2 and
and explain the observed trend in their volatilities.

h at room temperature

. [2]
(b)

The oxidis.ing power of C/2 , Br2 and hare different. By quoting relevant E0 values
from the Data: Booklet,. describe and expl~in:

(i)

tt'le reactions of the elements with H2 and the thermal st_abilities of tf:le
resulting hydrides

(ii) . The reactions of the elements with redudng agents such as iron(ll) ions.
.
.
~
(c) . A haiogen was passed through 250 cm 3 of a 0.05. mol dm-3 aqueou.s Na2 S2 0 3

until the reaction was completed. Excess halogen was .then removed by some
suitable means. 25.0 cm 3 samples of the resulting-solution were found

[
[
[
[
[

to require 12. 50 cm 3 of 1.00 mol dm-3 KOH for neutralisation


II

to require 10.00 cm 3 of 1.00 mol dm-3 AgN0 3 for reaction

Ill

to give 0.583 g of white precipitate, BaS04 when treated with aqueous


Ba(N03)2.
.

Construct a balanced equation for the reaction.


[3]
[Total: 10]

[j

lJ
[j
t~

l
L
9251,9258/P3/YJC2006

l~

[
[

or
(a)

Describe and explain what will happen in each of the following experiments,
. giving oalanced equations when necessary.
.

(i)

(ii)

[
[

A large amount of cold water is added to: calcium followed by. the addition
of universal indicator.
!;qual mass ofMg(N03)2 and Sr(N03)2 are heated over the bunsen flame,._
. [5] .

. (b). Compare two physical properties of calcium that are markedly different from that
ofiron.

. [2]

(c)

Agricultural-lime is manufactured from limestone, CaC03 by first heating the rock


to a high temperature in a lime kiln. The product is then allowed to cool and a
calculated amount of water is added. A highly exothermic reaction takes place
and2.00 g slaked lime is produced.

. (i)

Write balanced equations for these two reactions.

(ii)

Calculate the mass of limestone needed to produce 2.00g of slaked lime.

(iii)

Why is lime used in agriculture?

[3]

[Total: 10]

L
[

[
[

[
[

l
~~
(

. 9251,9258/?3/YJC2006

l
[
[

Section C

Answer all the questions from this section.


6

(a)

An organic compound, A has the following composition by rt:~ass:

C.: 40.0 %

.H: 6.65 %

0: 53.3 %

Determine_ the molecular formula of A given the molecular mass is 90.

[2]
(.b)

Compound A shows the following properties or reactions _

It is optically active ..

II

It gives a brisk effervescence with aqueous Na2C03.

Ill

U gives a yellow precipitate when warmed with alkaline aqueous 12 .

(i)

State what can be deduced from each ofthe above statements.

(ii)

Suggest an identity for compoun~ A and draw its displayed formula.

(iii)

The product, B formed when A reacts with acidified K2 Cr20 7 has been
_found to react with HCN. Describe the mechanism of 8 with HCN by
means of equations.
[6]

..

. (c)

When A is warmed with concentrated sulphuric acid, a compound of molecular


formula C6 H8 0 4 can be isolated. Suggest a displayed formula for C6 H80 4 ahd
state the functional group present in the molecule.

[2]
[Total: 10]

[
[
[
[~

lJ

u
ll
l
9251,9258/P3/YJC2006

L
l~

L
f~
10

l.

7 A herbicide, mecoprop has the following structure

OHCMOC(CH,),COOH

Cl

(a)

CH=CHCH 3

(i)

What type of isomerism is displayed by mecoprop? Draw the displayed


formulae of all the isomers.
[2]

(ii)

Draw the structure of the product formed when mecoprop reacts with

PC/s
II

2,4-dinitrophenylhydrazine

[2]

(b)

[I;

[
[
[

Suggest a simple one-step chemical test by which the isomers in the following
pairs can be distinguished from each other. State the reagents and conditions for
each test and describe how each of the isomers in the pair behaves.
(i)

CH3CH2CH2CHO and CH3COCH2CH3

(ii)

CH3CH2CH2CONH2 and CH3CH2CONHCH3

(iii)

J:oH

LcH,

~a~dg
[6]

[Total: 10]

L
[

l
L
l
l
L

9251 ,9258/P3/YJC2006

L
[
11

either

Given isomers A and B of formula C4H1 1N. Both A and B are primary amines.
The NMR spectrum of A shows 2 singlets. However, the NMR spectrum of B shows a
6-proton doublet, a 2-proton doublet as well as a 1-proton multiplet.
(a)

Identify the structures for isomers A and B.

[2]

(b)

[,

Compare the difference in boiling points of A and B.


[2]

(c)

(d)

Give the tertiary structure of isomer, C of formula C4H 11 N and describe its NMR
spectrum.
[4]
What is the difference in the NMR spectra obtained for A and C before and after
adding D20?
[2]

[
r,
L

[Total: 10]

8 or
X contains C,H and 0 atoms only and it has a relative molecular mass of 120.
(a)

X is unreactive towards acidified KMn04 but gives a yellow precipitate with an


alkaline solution of lz. Its NMR spectra shows a 3-proton singlet and a 5-proton
multiplet.
(i)

Deduce a structure for X.

(ii)

Give an equation for the reaction of X with alkaline lz.


[3]

(b)

Y, an ; isomer of X can be oxidised by acidified KMn0 4 but has no reaction with


alkaline lz. However, Y reacts readily with Fehling's solution.
(i)
- (ii)

(iii)

[
IL

Deduce a structure for Y.

Describe the NMRspectrum of Y.


Give an equation for the reaction of Y with Fehling's solution.

[5]
(c)

[
[

Another isomer, Z can react with Tollen's reagent but not with Fehling's solution.
It can be readily oxidised by acidified KMn0 4. Suggest a possible structure for z.
[2]
[Total: 10]

9251 ,9258/P3/YJC2006

l
(

L
L
L

[
[

12

CHEMICAL THERMODYNAMICS

r
[

Answer both questions on the paper provided.

Ellingham diagrams, showing the variation of standard free energy change, L\G 9 , with
temperature, have proven to be useful in deciding the best conditions for the extraction
of metals from their ores.

[
[
[:

[
[

(a)
(i)

Determine the AG 8 value for the following reaction at 1300 K :


ZnO (s) + C (s) ------.. Zn (s) + CO (g)

[2]
(ii)

Predict , with reasoning, whether carbon or hydrogen gas is the better


reductant for the extraction of zinc.from its-oxide at 1600 K.
[2]

1'

(iii)

Comment on the change in entropy for the reaction 2C (s) + 0 2 (g) ------..
2CO (g). Explain.
[2]

(iv)

Explain why the gradient of the graph for zinc changes at 700 K ?

L
L
L
l
L

[1]
(b)

With reference to the Data Booklet and given that


E9 ( AgBr (s) I Ag(s), 8( (aq))
+ 0.071 V,
calculate the value of the solubility product of silver bromide at 298 K.

[3]
[Total: 10]
9251 ,9258/P3/YJC2006

r
[

13

10

(a)

Nitrosyl chloride, NOCI, can decompose into nitric oxide and chlorine:

2 NOCI (g)

~===

2 NO (g)

The forward reaction is endothermic, with L1H 6


(i)

Cl2 (g)

= 77.16 kJ mor1 of Cl 2 .

Calpulate the standard internal energy change for the above reaction.
[1]

(ii}

(b)

It may be assumed that both L1H 9 and L1S9 are temperature invariant. The
standard molar entropies for NOCI, NO and Cb are 261.6, 210.7 and
223.1 Jmor1K-1 respectively.
Calculate the numerical value of the equilibrium constant for the reaction
at 650 K.
[3]

Suggest explanation for the following:

C'

(i)

The molar heat capacity of a gas at constant volume, Cv, is less than that
at constant pressure, Cp.
[2]

r
L

(ii)

Propanone obeys Trouton's rule but propan-1-ol does not.

[2]
(iii)

The enthalpy change of solution of ammonium nitrate is endothermic.


In spite of this unfavourable enthalpy change, ammonium nitrate is
soluble in water at room temperature, and it becomes very soluble at
350 K.
[2]

[Total: 10]

L
[

L
[

L
9251 ,9258/P3/YJC2006

L
I

[
[
Yishun Junior College
Chemistry Preliminary Examination 2006
Paper 1 (MCQ)

Answers

c
[

c
[

'

[
[
f~

[
[

[
[

L
[

L
L
t-

QnNo.
1
2
3
4
5
6
7
8
9
10

Answer Qn No.
c
11
c
12
B
13
D
14
D
15
D
16
c
17
A
18
c
19
20
A

Answer Qn No. Answer


D
21
8
22
8
A
c
23
D
.A
A
24
A
25
c
A
26
8
D
27
D
c
28
D
D
29
8
8
30
A

QnNo.
31
32
33
34
35
36
37
38
39
40

Answer
D

c
c

8
8
A
8

r--:-

/']

.,...__..,
'

Answer Scheme for YJC 2006 Chemistry Prelim Paper 2

Answer Scheme for YJC 2006 Chemistry Prelim Paper 2

1.

(ii)

The yellow gas, chlorine dioxide, G/02 has been used as a flourimproving agent In bread-making, It is a powerful oxidizing agent in
acid solution:
CI02(aq) + 4H+(aq) + 5e "" Cf(aq) + 2H20(I), E0

(a)

Comment on the value obtained In c(l).

[1]

The enthalpy change of formation of CI02 is endothermic shows


that C/02 is energetically unstable.

= +1.50V

Draw the dot-cross di~1gram for C/0 2 Using the concept of


valence-shell-electron-p!lir repulsion theory, predict and explain the
shape of C/02.
[2]

C/0 2 oxidlses KBr as well as K! to Br2 and h respectively.


The lattice energies of potassium halides are given below:

XXX
00

xx Clxx

oo 0

Qoo
00

00

(iii)

2 bonding'electron pairs, 2 non bonding pairs::;. bent shape


(b)

Suggest an explanation for the trend shown in lattice energy.

[2]

Ionic radius of r > Br"

Predict the reaction of chlorine dioxide, in acidic solution, with


H2 0 2. Write an equation for the reactions that occurs and calculate
the E0 ceuvalue.

E0 ceu

Lattice Energy/ kJmor1


-670
-629

Compound
KBr
KI

Since lattice energy


[2]

cc

I (g+

g')/ (r' + ()

Lattice energy of Kl < KBr.

=1.50-0.68 =O.B<~V

E0ceu positive, reaction is energetically feasible. C/0 2 oxidises H20 2


to 02.
(iv)
(c)

Consider the following reaction involving chlorine and silver (I)


chlorate (v):
2AgCI03(s) + C/2(g) ""' 2AgCI(s) + 2CI02(g) + 02(g) t.H = 0 kJmol"

(i)

K(g) + aq -+ K+(aq) t.H


Br"(g) + aq -+ B((aq) t.H

= -322 kJmor 1
= -335

kJmoi" 1

Calculate the enthalpy change for the process:

Draw an energy cycle to calculate the standard enthalpy change of


formation of C/0 2(g) using the following data:
Standard change of formation of AgC/0 3(s) = -25 kJmoi" 1
Standard change of formation of AgC/(s) = -127 kJmol" 1

Using the full information:

KBr(s) + aq -+ K+(aq) + Br"(aq)

[2]

[2]

KBr(s) + aq -+ K+(aq) + B({aq)

-6~

;:22

K(g) + B({g)

ts

/-355

t.H = 322 -335 + 670 = +13 kJmor 1


2(-25) + 0 = 2(-127) + 2t.Hr
b.Hr = +102 kJmor 1

L......'

L__:

c.._;

!__J

-.L_l

r...__.

(____,;

L_..)

-,
~

,/..

\.-....)

L_

1..-_-j

;___j

:.,__)

.....,._._.,'I

.___j)

..____\

~--

Answer Scheme for YJC 2006 Chemistry Prelim Paper 2

Answer Scheme for YJC 2006 Chemistry Prelim Paper 2

(d)

(e)

In an experiment to determine the equilibrium constant, Kp, of the


reaction in (c), chlorine gas at 100 kPa with an excess of silver(!)
chlorate(V), AgC/03(s) was allowed to reach equilibrium in a vessel
at constant volume at 90 c.

Explain why boiling point of C/0 2 is higher than boiling point of 0 2.


IMF of C/0 2 is permanent

dipole-perm~tnent

[2]

dipole interaction

and IMF of 02 is Vander Waal's forces.


The partial pressure variation of CI02(g) was graphically presented
below:

Van der Waal's forces weaker than permanent dipole-permanent

Pressure/

dip_ole interaction.

kPa 100

60

[Total: 18 marks]

60

2(a)

40
20

Time/min

10

(i)

Determine the value of the equilibrium constant, Kp. including its


units.

I
E

Determine the value of x. Hence, writEl a balanced equation for the


reaction between ICI, and r.

[4]

[2]
IC/x + f -t !2 + xCT

2AgC/03(s) + Ch(g) .,. 2AgCI(s) + 2C/02(g) + 02(g)


.
0
100
0
40
20
80

Under suitable conditions Iodine and chlorine reacts to give


interhalogen compound, ICI,. When a pure sample of !Cix was
dissolved In an excess of aq K!, the iodine liberated requires
17.1 cm 3 of 0.1 moldm"3 Na 2S 203 solution for the titration whereas
the chloride ion liberated requires 12.8 cm 3 of 0.1 moldm" 3 silver
nitrate for complete precipitation.

!2 + 2S203 2

-+ s.oe2 + 2r

:1.

KP =

40 20
=400kPa 2
x

1 17.1x0.1)=8.55x10"
n,, =2(
1000

4 mol

*~

nc, =
(ii)

How would you expect the equilibrium composition change when


the temperature is increased?

12.8x0.1 = 1.28x10"3 mol


1000

[1]
I2:

cr

2:3

x=3

Since C.H = 0, Increase in temJ?. does not affect Kc.

IC/3 + 3!' -t 212 + 3Cf


Eguilibrium constant remains unchan~ed.

(Iii)

At the tenth minute, the volume of the vessel Is suddenly halved.


Sketch the variation of CI0 2(g)

[2]

,.----,

,........-,
1

,..--,

I'

Answers
K _ [NH:J[OW]
[NHal

Qn No.
3 or
(a)

Marks

Answers

Qn No.

Marks

(c)
pH

.Jr-i.-74-x-1o---x-0-.2 =[OW)
[OW]= 0.00186moldm- 3
pOH =-Jg0.00186 =2.72
pH =14 -2.72 =11.3
< 7 ......... , __ ,. ""'""""""""""""

Assumption: negligible ionisation OR 0.2- x"' 0.2


(b) (i)

Buffer forms.
[NW]

pOH = pK. + l g -4[NH 3 ]

25

[NW]
4
14-8.9 =4.76+1g-[0.2]

50

Volume of HCI

0.34 =I [NHJ]
g [0.2]

[NH;] = 0.2x2.188
= 0.438mo/dm- 3
mass of NH4Cl = 0.438 x 53.5 = 23.4 g
(ii)

W + NHa-+ NH4+
0 2 50
= 0.00952moldm-3
x
1050
0 2 0 00952
new[NH 3 ] = -
= 0.181mo/dm-3
1.05
0 38 0 00952
new[NH]=
0.426moldm-3
.4 + 4
1.05
26
pOH = 4.76 +lg 0.4 = 5.13
0.181
pH= 14-5.13 8.87
t::.pH = 8.9-8.87 0.03

f-1]=

=
=

pH changes is negligible as mixture of NH4CI and NH 3 form a


buffer solution which resists pH changes when small amounts of
Wand OH' are added.

&____;

L-l

l_....,

Qn No.
4 (a) (I)

(ii)

(iii)

(iv)

(b)

I
...___

(.____,

L_,)

\---.J

Answers
First Ionisation increases generally across the period as valence
electrons added to the same quantum shells experience constant
screening. However, nuclear charge increases across the period
and atomic radius decreases.
NaCI, MgCI2
'--y---..J
strong ionic bonds
-? high boiling
point
NazO, MgO
'--y---..J
Ionic oxides
-?basic

\__) t___j

Marks

"---'

Qn No.
5 either
(a)

Na, Mg, AI
'----r--'
Metals with sea of mobile
electrons
-? conductor of electricity

(b) (i)

eiOz, P401o, S03, ClzO;

Ionic with covalent


character
-? amphoteric

-,

'----'

-.---,

Answers
Colour
yellowish green
brown
black

Halogens
Cb'
Brz

:--- J

___;

-.:.J

Marks
state
gas
liquid
solid

Volatility of halogens decreases down the group as molecular


size increases, strength of van der Waals' forces increase.
Hence, attraction between molecules Increases, leading to an
increase In boiling points of the halogens and a decrease In
volatility.

weak van der Waals' forces


between discrete molecules
-+low boiling point
AI203

--~

AICb, SICI4, PCis, S2CI2

'-y-'

'.-_.)

v
covalent oxides
-?acidic

Clz + 2e'"' 2 Cl'

E9 =+1.36 V

Brz + 2e' .. 2 Br'

E9 = +1.07 V

lz + 2e' .. I

E9=+0.54 V

E9 becomes less positive and so the oxidising strength


decreases. Hence the reaction with H2 needs different
conditions.

Si, P4, Sa, Cl2


'--y---.1
Covalent compounds with no
free mobile electrons or ions
-? non-conductor

If X2 = Cl2 sunlight required


= Brz heat at 1200 C
= lz heat::+ catalyst

I AI"

and Be2' have high charge densities and so are polarising


Ions. Hence, both A120 3 and BeO are predominantly ionic with
covalency as 0 2' polarised.

H-X bond length increases as the atomic radius of X Increases


down the group. Bond energy decreases due to less effective
overlap, leading to a decrease in thermal stability of HX down the
group.

Cl, having a larger atomic size than 0, is polarisable. Al 3+ and


Be 2, with high charge densities, polarise Cl' extensively to form
covalent bonds.

(ii)

Fe 3 +a"" Fe 2 E9 = +0.77 V

E9(lz/l') < 0.77V, hence lz is not able to oxidise Fe 2 to,Fe 3.


E9(Ciz/CI') & E9(Br2/Br') > 0.77 V, hence both halogens are able
to oxidise Fe 2 to Fe 3.
(c) (I)

(II)

(Ill)

OH' + H+ -? HzO
No. of mol. of H+ = 0.0125 mol
No. of mol. of X2 = 0.00125 mol
Ag + x-? AgX
No of mol. of x = 0.01 mol
2
0.583
No. of mol. of S04 ' =
= 0.0025mo/
137+32+64

H+:
x:
soi
x2:
0.00125; 0.00125;
0.01 : 0.0025
. 8:
1:
10:
2
4 Xz + NazS203 + 5 HzO -? 8 HX + NazS04 + HzS04

...

.
8

ri
L

Qn No.
5 or
(a) (i)

Answers.
Ca, being reducing, reacts vigorously with cold water to liberate
Q.(gl. An alkaline solution of Ca(OHh forms and hence the
universal indicator turns blue.

(ii)

Mg(NOs)2 decomposes more readily than Sr(N0 3)2. Hence


amount of products formed by Mg(N0 3)2 is greater than that of
Sr(NOs)2.

Marks

(ii)

(iii)

-1-:

(b) (i)

6.65

53.3

16

2:

= 90

I -A has a chlral carbon atom.


II -A is a carboxylic acid.
111- presence of CH 3 - C - or

I
0

H H
I I
I I

(ii)

H-c-c-c
..

CaCOs ~ CaO + C0 2
CaO + H20 ~ Ca(OH)2

Mass of CaCOs

40
:
12
1:

A: C3Hs03

1. Ca has lower melting point than Fe.


Ca has lesser number of valence electrt>ns per atom and hence
weaker metallic bonds.
'
2. Ca is less dense than Fe.
Atoms of Ca .are less closely packed due to weaker metallic
bonds and smaller A,.
3. Electrical conductivity if Ca is lesser than that of Fe.
Ca atoms have lesser valence electrons than Fe.

2
No of mol. of Ca(OH)2 =
40+2(17)

H:

n::o3

sr

(c) (i)

C:

(CH20)n

Charge density of Mg 2 is greater than that of


and hence it is
more polarising. This causes Mg(N0a)2 to be less stable than
Sr(NOs)2.
(b)

Marks

Answers

Qn No.
6 (a)

(Iii)

0.02703mo/

OH

I
I

CH - C s
OH

,;-0
'OH

Nucleophilic addition
CN", a nucleophile reacts with B to form a base, oxide.
COOH
yOOH

~CN"
-CH 3
6'61 S+
1

= no. of mol. Of CaC03


= 0.02703 x 100 = 2.70 g

v'

CaO is basic and so, can neutralise acidic soil.

v'

CN" +

3
co-C-CH
I
CN

yOOH

H 0-C-CH + -

6N

.,-CN

Oxide accept H+ to from HCN to form an addition product.


(c)

Ester

H3 c-t-c=o
I

0
I

0
I
O=C-C-CH
I
3
H

10

.;___

(- -1.

L-

c-

c::=,;

Ci

,~----

(__)

I
o~"c)O(o -c-co
H
I
CHa

H/

CH 3/H

trans

c=c--.......cH

/
H

o~

"c)O(o I

OR
Add PCis.
WhKe fumo.of

c=c--.......H

o,NXN-N=~ ~~/cH,
N02

Marks

60H

I
o- C-C02H

/C

Cl

HCI-

&
0

CHa

OR
Add aq. Br2.
Doool'"""''" of red-brown "'' '"' whllo ppt-

CH3

I
H

____;;

HCI- 60H

No white fum" of

CH 3/CH3

Cl

'...._)

&CH,

CHa

.._____)

cis

I
0~"c)O(o -C-COCI
I
H/

&
0

CH3

'-....H

H
(b) (I)

Add neutral FeCI3.

N"lolot ""''""'''"-

H
(ii)

..
'
.._

l..-.-

c-c--.......H

----

_:_H3/CH3

Cl

[.__)

Vlolot ""''""'"'"-

CH3
I
-c-C0 H

H/

'

(iii)

Cl

J..--...1

Answers
Qn No. I
Add aqueous NaOH and heat.
(ii)
NH3(g)- CH3CH2CH2CONH2
No NH3(g) - CHaCH2CONHCHa

Marks

Qn No. I
Answers
7 (a) (i) I Cis-trans/geometric isomerism

\...__,..J

No """""""'" aod white ppt

60H

I Add Tollen's reagent to both compounds and warm.


Ag mirror CH3CH 2CH2CHO
No Ag mirror- CH3COCH2CHa
OR
Add 12 and NaOH and heat.
No yellow ppt CH3CH2CH2CHO
Yellow ppt- CH3COCH2CH3
OR
Add KMn04, dil H2S04 and heat
Decolourise purple KMn04 CH3CH2CH2CHO
No decolourlsatlon - CHaCOCH2CHa
11

12

.____)

r----'
l

'

Qn No.
8 either A:

Answers

B:

(a)

Marks
CH 3 H

I I
H-C-C-NH
I I

Qn No.
8 or
(a) (I)

NMR of X shows a 3-proton singlet that Indicates a -CH 3 group


with no adjacent proton.
5 proton multiplet indicates the protons on the benzene ring.

X. Hs c'-...... ,!'0

CH 3 H

I I
H C-N-C-CH
I

H
(ii)

3 signals
2 equivalent -CHs groups -7 singlet
CH2 group most deshlelded proton, shows splitting
pattern of a quartet.

CHs group most shielded proton, shows splitting pattern


of a triplet.
(d)

II

A has a lower boiling point than B.


A has smaller surface area and l)ence, less attraction and
weaker van der Waals' forces.

(c)

0
X: absence of 1, 2 alcohol and aldehyde.

) CH3 H
(b)

Marks

Answers

X has CH 3

Q-c OCH + 3
3

l2

+ 4 OH' -7

Q-c -o + 3 H20
II

0
+ 3 I'+ CHb

(b)(i)

A :. signal due to -NH 2 group disappears.


Th1s Is due to the labile proton In -NH 2
This Is not seen In C.

(ii)

(Ill)

01-c=o
~ ~
The NMR spectrum of Y shows a 5-proton multiplet due to the
protons on the benzene ring. The -CH2 group shows a doublet
while the 1 proton In -CHO group shows a triplet.

o - - ? - r = O + 2 Cu + + 5 OH'
H

~ o-?-r=O
H

(c)

13

14

___;

"----

L-...

Qn No.
9(a)(l)

l__J

L.:.J

L.J

L-J

Answers
From the Elllngham diagram, at 1300K,
2 C (s) + 02 (g) 2 Zn (s) + 02 (g) -

2 CO (g)
2 ZnO (g)

'

(_]

Marks

----,,

'

----,

'-----'

...__.,
~---~.

Qn No.
1O(a) (I) I t.H

Zn (s) +CO (g) at 1300K,

AU
(il)

t.G 9 = Y.(-690)- Y. ( 220) = 235 kJmor 1


t.G 9 .. Y. (270)- Y. ( 220) = 25 kJmol' 1

(b) (I)

AgBr (s)
t.G 9

(s) + Br"(aq)
Ag (aq) + e

~ Ag (aq) + Br" (aq)

ee

=+0.071 V

= t.U

+ (+1)(8.314)(298)x10'3

= 74.7 kJ

= ill

I w = . pt.V

(II)

I Propan1ol

(Iii)

I t.G = t.H Tt.S

= -0.80 v

E9 =0.729V

= nFE = (1) ( 96500 )( 0.729) = 70348.5 J mol' 1

t.G 9 RT In Kap
70348,5 = ( 8.31 ) ( 298 ) K1p
Ksp

Marks

Extra energy needed to do work against external pressure


during a constant pressure change, since volume changes.

Melting of Zn occurs.
e~Ag
~

'--'

-1.685 x 1000 = (8.314)(650) In KesoK

t.S" Is positive.
The Increase In number of moles of gaseous particles result In an
Increase In disorder.

AgBr (s) +
Ag (s)

-----J

It.S 6--;-[2-(210.7) + 223.1]. (2(261.6))=-121.3 J mor1l<-:r

KesoK

(b)

"'---]

t.GasoK '" RT In KaeoK

for reduction of ZnO with C Is more exergonic than the


reduction of ZnO With H2.

(lv)

t.GasoK= t.H Tt.S = 77.16 (650)(121.3/1000)


= 1.685 kJmol1

t.G 9
(Iii)

___J_

= t.U + t.n RT

Carbon.
Carbon:
:
H2

Answers

t.G 9 = 540 kJmor 1


t.G 9 = 410 kJmol" 1

t.G 9 = Y. (540) ..., Y. ( -41 O) = 65 kJmor 1


(II)

= t.U + pt.V

77.16
Hence for the reaction
ZnO (s) + C (s) -

has Intermolecular hydrogen bonding, resulting


In greater orderliness.
When propan-1-ol changes from liquid to gaseous state, It
experiences a larger Increase In entropy.
Hence, standard entropy of vaporization Is greater than
88 Jmoi'1K'1
"'

t.Hootn Is endothermic but t.S Is positive making t.G


negative. .
6S is positive because as solid dissolve to form aqueous
Ions, there Is an Increase In disorderliness.
As temperature Increase, Tt.S term becomes more negative

= 4.60 X 10 13 mol 2 dm-4

15

16

r_

TEMASEK JUNIOR COLLEGE

[
[

11~.'7,~
..
-<.."
~

C'o
(L.eaE fO~

[
[
[
[

"''i-

9251/01

CHEMISTRY
PAPER 1
Thursday

21 SEPTEMBER 2006

Additional materials:

Data booklet
Multiple choice answer sheet

INSTRUCTIONS TO CANDIDATES

r~

Do not open this booklet until you are told to do so.

L
[

1 hour

Write your name, centre/ index number and CG on the answer sheet in the spaces
provided.
There are forty questions on this paper. Attempt all questions.
For each question, there are four possible answers labelled A. 8, C, and D. Choose
the one you consider correct and record your choice in soft pencil on the separate
answer sheet.

Read the instructions on the answer sheet very carefully.

Each correct answer will score one mark. A mark will not be deducted for a wrong
answer.
Any rough working should be done in this booklet.
You may use a calculator.

L
L
[
l_

This question paper consists of 15 printed pages


(Turn over

l
[

Section A
For each question there are four possible answers, A, 8, C and D. Choose the one you
consider to be correct.

1 mole of potassium chlorate(V), KCI03 is thermally decomposed to give potassium


chloride and pxygen. Excess sodium is burnt in the oxygen produced. How many
moles of sodium oxide is produced?

A
B

C
D

[
[

1.0
1.5
2.5
3.0

Which of the following is incorrect for 75 As3 ?


A
B

It is isoelectronic with Kr.


It is highly polarizing.
It is bigger than an arsenic atom.
It can form a compound M:As2 with a metal M.

L
[

Chlorine consists of two isotopes, 35CI and 37CI in the abundance ratio 3:1. In the
mass spectrum of trichlorosilane, 1H 28SiCh. what would be the abundance ratios
corresponding to the m/e values 134,136, 138 and 140?

A
B

c
0

9:3:3:1
27:9:3:1
27:18:9:1
27:27:9:1

The density of-tee is 1 ;00 g cm3 . What-isthe volume of steam produced when 1 cm 3
of ice is heated to 596 Kat a pressure of one atmosphere?
3
(1 mole of a gas occupies 24.0 dm at 298 K and one atmosphere.]

A
B

c
0

,0267 dm 3
1.33 dm 3
3
2.67 dm
48.0 dm 3

L
u
[

u
l
[Turn over

L
\

L
[

- 5

SbF3

SnCiz

[
[
[

Identify the shapes of the following species: SnCb. SbF3 , BH 4-_-

planar
planar
pyramidal
pyramidal

BH4Square planar
Tetrahedral
Tetrahedral
Square planar

Which solid exhibits more than one kind of bonding?


A
B
C

Trigonal
Trigonal
Trigonal
Trigonal

Bent
Linear
Bent
Linear

Ice
Bronze
Silicon
Zinc

[
7

If the rate of decay of a radioactive isotope decreases from 1200 counts per minute
to 75 counts per minute in 48 hours, what is its half life?
A
B
C
D

6 hours

8 hours
12 hours

16 hours

[
8

[
l

Which one of the following statements is most likely to be correct concerning the
reaction shown below:
A(g) + 2B(g) ~ C(g)

-~

A
B

The yield of C increases at lower pressure.


The yield of C decreases when the volume of the reaction vessel is halved.
The equilibrium concentration of C is given by the expression:
2
[C] Kc [A][B]
The rate of the forward reaction is given by the expression:
Rate k [A][B] where k is the rate constant.

l
L
[
l_

t'

(Turn over

r
9

The table below give tlie enthalpy change of combustion of ethyne (C2H;i). hydrogen
(H 2 ) and ethane {C 2Hs)
1---

Substance
Ethyne
Hydrogen
Ethane

AHc/ kJmor 1
-1300
-285
-1560

[
I

What is the enthalpy change for the following reaction?

C2H2(g) + 2H2(g) -7 C2Hs(g)


A
B

10

+310 kJ
+155 kJ
-155 kJ
-310 kJ

n
u

The pKb value for aqueous ammonia at 25c is 4.8.

[]

What is the correct pKa value for the ammonium ion at this temperature?

A
B

2.2
-4.8

9.2
11.8

11.

[1
[J

The standard electrode potential E0 for each of the three electrode reactions in
aqueous solutions, is given below.
Electrode reaction

E0N

2C02 + 2e ~ C204 zFe3+ + e ~ Fe 2 +


Mn04 - + 8H+ + 5e ~

-0.49
+0.77
+1.52

Mn 2 + + 4H 2 0

1/5

2/5

3/5

c
L:

How many moles of Mn04 - will react with 1 mole of iron(ll) ethanedioate in acid
solution?
A

5/3

L
[
[
L

L
4

[Turn over

[
L

c
[

f2

Two cells are connected in series as shown in the diagram where M, N, 0 and Pare
electrodes. Which of the following correctly shows the products formed at each
electrode?

....-------1~

lead

:._.;;.;;r pI atin um

1 moldm-3 ------'1-dil HN03

[
A
B

----'1----

M
02
Oz
Pb;::+
Pb 2 +

N
H2
Pb
Hz
Hz

02
Clz
Clz
02

Fe 2+
Hz

1 moldm-3
FeCb (aq)

Hz

Fe2+

[
[

13

B
C
D

L
[

,,
l
L
L
l
~

Two moles of each of the :following compound is strongly heated using a Bunsen
flame and any gas produced is collected at room temperature and pressure. Which
compound is likely to give 48 dm3 of gas/es?

14

MgO
MgC03
Mg(N03)z
Mg(OH)z

What changes occur in the magnitudes of the following as Group II is descended?


(i)
(ii)
(iii)

the standard electrode potential of M2+(aq)/ M(s) electrode, E6 .


the lattice energy of the oxide, L'l.H1~u.
the standard enthalpy change of hydration of Mz+(g), L'l.H:yd
E

A
B

0 2
M +!M

illaf?C1JQ.S

decreases
decreases
increases
increases

L1H1att of MO(s)

L1Hh:d of ~+(g)

decreases
decreases
increases
increases

decreases
increases
decreases
increases

[Turn over

c
15

An aqueous solution containing both potassium chloride and potassium iodide is


treated with an excess of aqueous Silver nitrate. The precipitate formed is filtered off
and washed with distilled water. The precipitate is then shaken with aqueous
ammonia and filtered off again. Which ion is present in the final filtrate?
K+ (aq)

r (aq)

c1 (aq)

No3 (aq)

[
16

The table shows the possible oxidation states of four d-block elements in the Periodic
Table. (The elements are represented by letters which are not their symbols).
Element

Possible oxidation numbers

3
3
3

5
5
5

2
2

6
6

Which of the following ions is likely to exist?

wo3-

vo4-

XO/

zo43.

[
[

17

Which graph correctly describes a trend found in Group VII?


A
X-X
bond
length

X-X
bond
energy

~
I
Cl2

Br2

12

strength of
intermolecular
forces

boiling
point
of X2

Br2

l2

~
Cl2

~
Cl2

Br2

12

L~-

I ~
Cl2

Br2

[
l2

u
6

[Turn over

[j

L
\

c
[

Ions of the two common isotopes of nickel are shown below:

18

58 N. 2+
28 I

60 N. 2+
28 I

Which one of the following statements is true?

A
B

The electronic configuration of both Ni2 + ions is 1s2 2s 22p63s 2 3p6 3d 64s2 .
The 60 Ni2 + ion will have more protons in its nucleus than the 56 Nf+ ion.
In the same magnetic field strength, the 60 Ni 2+ ion will be deflected more than
the 56Ni2+ ion.
Both Ni2 + ions have the same number of electrons but different number of
neutrons.

[
19

When aqueous ammonia is added to CuS0 4 solution, a pale blue precipitate is


formed which dissolves to form a deep blue solution in excess aqueous ammonia.

Which process does not occur in this sequence?

A
B
C

20

formation of dative bond


formation of complex
precipitation of copper(ll) hydroxide
reduction of Cu 2 + ions

What is formed when this compound reacts with hydrogen cyanide?

CH 2 0H

'

[
CH 2 0H

L
[

IT

CH 2 CN

CN

~A~

OH

hl

CH 2CN

CN

OH
CN

9H 20H

CN

D
CN

OH

I!J

L
[

L
l

[Turn over

c
21

Acetaminophen is a drug used in headache remedies. It has the structure:

HO-o7 ~

N-C-CH 3

Which of the following reagents reacts with Acetaminophen?

22

23

cold hydrochloric acid

sodium carbonate

LiAIH4 in dry ether

2 ,4-dinitrophenylhydrazine

Which of the following reagents distinguishes propan-2-ol from 2-methylpropan-2-of?


sodium metal

alkaline aqueous iodine

phosphorus pentachloride

neutral iron(lll) chloride

10 cm of a hydrocarbon was mixed with I 00 cm of oxygen gas which is in excess. The


mixture was exploded and after cooling to room temperature, the residual gases
3
occupied a volume of 70 cm . Upon addition of aqueous NaOH, the volume decreased
3
to 20 cm . Assuming all volumes are measured at room temperature and pressure,
what is the possible formula of the unknown hydrocarbon?
CzH4

[
[

CzHa

CsH1o

CsH12

.,
l

'

[
24

E, F and G are three organic compounds. E undergoes oxidation with hot acidified
potassium manganate(VII) to give F and G. F gives a orange precipitate with 2,4dinitrophenylhydrazine but gives negative results with Tollen's reagent G reacts with
sodium carbonate to give strong effervescence of carbon dioxide.

Which of the following compounds could be E. F and G?

E
CH 3CH(OH)CH=CHCH 3

CH3COC02H

CH 3CHO

CH 3CH(OH)C(CH 3)=CHCH3

CH3COCOCH3

CH 3COzH

CH 3CH(OH)C(CH 3)=CHCH 3

CH3CHzCOCHO

CH3COzH

CH 3COC(CH 3)=C(CH3)z

CH3COCOCH3

CH 3CHzC0 2 H

L
r1
LJ

u
[Turn over

L
L
L
\

L
[

25

What are the organic compounds in the final reaction mixture?

[
[

A mixture of one mole of CH 2CICOCI and one mole of C6H 5 CI is heated under reflux
with dilute aqueous NaOH.

26

CH 2 CIC02 and C6HsOH

CH 20HCOCI and C6HsCI

CHzOHCOz. and C6Hso

CH20HCQz. and CaHsCI

Which of the following statements is true about ethene?

A
B

The nuclear magnetic resonance spectroscopy signal of the protons in ethene


would be a singlet.
It reacts with aqueous bromine to give 1,2-dibromoethane as a major product.

It reacts with cold dilute alkaline manganate(VII) to give an optically active product.

It can be obtained by heating excess ethanol with concentrated sulphuric acid at


140C.

[
-,

27

_,

Which of the chloroalkanes, with the chemical formula C5 H1 1CI, gives the largest
number of organic products when treated with ethanolic potassium hydroxide under
reflux?
CH3
CH3
A

CH3-CH2-CH-CH2CI

CH3-C-CH2CI

CH3
Cl

Cl

CH3-CH2-CH2-CH-CH3

CH3-CH2-C-CH3

CH 3

[
[

L
l
[

L
[

28

A halogenoalkane, S, can give an amine wllen heated in a sealed tube with excess
ammonia. A nitrile, T, can give an amine as well by reacting it with LiAIH4 in dry ether.
Which of the following pairs of S and T give the same amine?

s
A

CH 3CH 2CI

T
CH 3CHzCN

C6HsCI

CsHsCN

CH 3CH(CH 2 CI)CH 3

CH 3 CH(CN)CH 3

CH 3CH 2CHCICH3

CH 3 CH(CN)CH 3

[Turn over

c
29

Which of the following shows the correct order of increasing basic strength?
Weakest

c
D

Strongest

OCONH, ONH,
ONH,
OCONH, ONH, ONH,
ONH, NH, ONH, . OCONH,
OCONH, ONH, ONH,
NH3

NH3

NH3

[
[

L
[

[
30

Ethynylestradiol is a synthetic estrogen used in birth-control pills and has the structure
shown:
HO

c=cH

0
-.....::

HO

r
[

[]

How many chiral centers are present in Ethynylestradiol?


A

u
[i

L
10

[Turn over

L
L
r

L
[
[
[

Section B
For each of the questions in this section, one or more of the three numbered statements 1 to

3 may be correct.
Decide whether each of the statements is or is not correct. (you may find it helpful to put a
tick against the statements which you consider to be correct).
The responses A to 0 should be selected on the basis of

[
[

1, 2 and 3
are correct

1 and 2
only are correct

2and3
only are correct

1 only is correct

No other combination of statements is used as a correct response.

31

Which of the following statement(s) is/are correct?

-,

The nucleon number of an element is the number of neutrons in one atom of


the element.
The atomic number of an element is the number of protons in one atom of the
element.
The magnitude of the charge on an electron is the same as that on a proton.

_J

32

Reasons why a real gas deviates from ideal gas behaviour include(s) that the
molecules in a real gas

2
3

have different speeds.


have definite sizes.
experience intermolecular forces.

r~

L,

['
-,
r

33

Some data on two acid-base indicators are shown below.

bromocresol gr~er:t__
phenol red

--~-

-"

L
[

Approximate pH range

Indicator

3.8-5.5
6.8-8.5

Colour change
Acid
Alkali
__yellow ______Q~
yellow
red

Which one of the following conclusion(s) can be drawn about a solution in which
bromocresol green is blue and phenol red is yellow?

1
2
3

It is weakly acidic.
It can be a solution of sodium ethanoate.
It can be aqueous ammonia.

11

[Turn over

L.
rt.
l

1, 2 and 3
are correct

1 and 2
only are correct

2 and 3
only are correct

1 only is correct

34

For which of the following reactions does the value of L\H 9 represent both a standard
enthalpy change of combustion and a standard enthalpy change of formation?
1
2
3

35

pH of oxides

[
-,
r-[

o~~--~~--~---

Mg

Ca

Sr

Ba

decomposition temperature of nitrates

Temp
1

X
'

Mg

X
'

Ca

[
'

Sr

Ba

first ionisation energy

r:

1st I.E

ol

pH

C(s) + 02(g) ~ COz(g)


H2 (g) + 1/202 (g) ~ HzO (g)
N 2 (g) + 20 2 (g) ~ 2N0z (g)

Which diagrams correctly show the trend ina property-of the Group II compounds
from Mg to Ba?
1

X
I
Mg

I
Ca

Sr

[1

Ba

12

[Turn over

[
[

1,2and3
are correct

1 and 2
only are correct

2 and 3
only are correct

1 only is correct

Why is warming concentrated sulphuric acid with solid sodium bromide unsuitable
for the preparation of hydrogen bromide?

36

1
2
3

37

Hydrogen bromide is not displaced by sulphuric acid.


Hydrogen bromide is oxidized to bromine.
The product is contaminated by _sulphur compounds.

Terfenadine (sold under the trade name Seldane) is a drug that alleviates seasickness
and asthma.

~
CH
o-r--cN-CHzCHzCHz-yH-Q-?~CH3

[
[

OH

OH

CH3

Terfenadine

What deduction(s) about Terfenadine can be made from this structure?

One mole of Terfenadine will liberate two moles of HCI(g) on reaction with two
moles of PCI 5 .

Terfenadine reacts with concentrated sulphuric acid at 180 C to form a product that
decolourises Br2 (aq) at room conditions.

Terfenadine. reacts with dilute aqueous HCI to form. a salt.

[
'[

l
L
L
[_

13

[Turn over

L'

38

1,2and3
are correct

1 and 2
only are correct

2 and 3
only are correct

D
1 only is correct

A polymer has the repeat unit:

--CH-CH2-CH2-CH=CH-CH2-

[
[

Which statement(s) about this polymer is/are correct?


1

It can be hydrolysed by dilute aqueous sulphuric acid under reflux.

It is an addition polymer.

6
CH

The monomers are

=CH2

and CH,=CHCH=CH,

[
39

Which of the following molecules show(s) optical activity?

Q
I

G===C

H2N

)<I

H3C\ _ /

/c

c===c

H H

"-

'

NH2

CH3

H2C

3 ' CH 2 =CHCHBrCH3

[
14

[Turn over

L
l

r
[
[

40

1, 2 and 3
are correct

1 and 2
only are correct

c
2 and 3
only are correct

1 only is correct

Fenoprofen is an anti-arthritic agent.

OOOCHC~H

CH3

Which of the following could be part of a sequence for synthesising Fenoprofen?

[n the sequence below, R"

0 OJ
0

[
[

,,

NaCN (ethanolic)

RCHBrCH 3

R-CHCONHCH 3

heat

intermediate

H+ (a )

Fenoprofen

heat

-J

intermediate

room temp

reflux

R-C-CH 3 HCN in NaOH .

II

1n

terme d'1ate

10-20C

L
L
L

L
f'

Cr 2o?-; W (aq)

CH3

[
[

LiAIH 4 in dry ether

15

H+ (aq)
heat

Fenoprofen

Fenoprofen

[
J
L

Name

Centre/Index No:- - - - - -

------------------------

C.G. _ _ _ __

TEMASEK JUNIOR COLLEGE

o~~
"'
1>
. ' ' ' .. ' cS
<'o . . . -""
l(f:GE FOl'- ~

[
[

CHEMISTRY

9251/02

PAPER2

t8 1h SEPTEMBER 2006

Monday
Additional material:

1 hour 30 minutes

Data booklet

INSTRUCTIONS TO CANDIDATES

Write your name, centre/index number and


CG in the spaces provided on this page.

Answer all questions.

uestion
1

No

Marks

[
[

L
[

L
[

3
Write your answers in the spaces provided on the
question paper.

5
INFORMATION FOR CANDIDATES
The number of marks is given in brackets [ ] at the end
of each question or part question.

You are advised to. show all working in calculations.


You are reminded ofthe need for good English and clear presentation in your answers.

This question paper consists of -14 printed pages.


[Turn over

L
[
[

- Answer ALL Questions

1.

One reaction which occurs in air polluted with nitrogen oxides is showri below:
2NO(g) + 02(g)

2NOz (g)

3 experiments were carried out at constant temperature to find the relationship


between the initial total pressure of NO and 0 2 , and the initial rate of formation of
N02.
Experiment
Initial total ~ressure of NO and 02/atm
Initial pressure of NO/atm
Initial rate of formation of N02/ atms
(a)(i)

I
1.00
0.40
1.08

II
1.60
0.40
2.16

Ill
3.20
0.80
17.28

Explain what is meant by the term order of reaction.

[
[

(ii)

Use the above data to determine the order of reaction with respect to each of the
reactants. Show how you arrive at your answers.

[
[
[
[
[

"(iii)

Use these--answers to write a rate equation for.the:reaction.

[
l_
[Turn over

L
L_

l'

l-[

(iv)

Determine a value of the rate constant k, stating the units.

[
,r.~
L!

[6]
(b)

The first step in a possible mechanism for the above reaction is:

fast
2NO(g)

NzOz (g) ............................Step I

(i)

Draw dot and cross diagrams for the two molecules NO and Nz02

(ii)

From the diagram of NO you have drawn, suggest why Step I is a likely step in the
reaction.

c
[

[
r~

.L-.~

[
['
[Turn over

l
l

L
[
\

I.

(iii) Sketch on the same axes below, the distribution of the molecular speeds of

1: nitrogen molecules at a given temperature


II : same number of nitrogen dioxide molecules at the same temperature.

Label your sketches clearly.

Number
of molecules

r
[_
[
[

Molecular speed
[5]
[Total: 11]

2. In a gaseous reaction at constant pressure of 1 atm. XY3 is synthesized from X 2 andY2


x2 {g) + 3Y2 (g)

r
L

r~

2XY3 (g)

A mixture of 4.0 moles of X 2 and 12.0 moles of Y 2 was placed in a vessel of volume 20 dm 3
at 450K. When the system had reached equilibrium, it was found that 4.0 moles of XY 3 was
present.

(a) Calculate the partial pressures of each species at equilibrium.

r-

L~

L
L
[2]
[Turn over

uI '

u
L
\

(b)

Hence calculate the value of Kp of this reaction.

[
[1]

(c)

c-

When the volume of the vessel was increased and the system allowed to come to a
new equilibrium at the same temperature, 9.0 moles of Y2 were found to be present.
Calculate the new equilibrium pressure of the system.

L
[
[,
[2]

[
(d)

What effect will the use of a catalyst have on the Kp value? Explain your answer.

[
[
[2]

[Total: 7]

(Turn over

l~-f:
--- ------

- -

----------~----

3.

Reactions

--"'
+2e

6H+fkJmor

Ba2(aa) +2e

+286

"' so/(ag2___j__~_7__1
..,.. BaS04 ls

(a)

You are provided with the following thermochemical data:

-1465

Calculate the standard enthalpy change for the reaction:


Ba2(aq) + So/(aq)

---r"'

BaS04 (s)

r
[

r
'

[2]

(b)

Given the standar<;l enthalpy change of atomization of barium is +176 kJmor 1 and the
1
first and second ionization energies of barium are +502 and 966 kJmol" respectively,
calculate the hydration energy of the barium ion.

[
["

[
[

L
.ufl

L;
[3]
[Turn over

L
L

l.

(c)

Explain how the magnitude of hydration energy of magnesium ion differs from that of
the barium ion?

[
[

[1]

[Total:6]

4 (a)

When paper is manufactured, it needs to be 'sized' to fill in small cavities in the


surface to prevent ink from spreading. In the past, many books were 'sized' by
adding aluminium sulphate. However, if the paper becomes damp, the aluminium
sulphate is hydrolysed, causing the paper to become acidic. Eventually, the acid
causes the paper to crumble.
One method of reducing the damage is to use diethylzinc, Zn(C 2 H5 )z, a volatile liquid.
This reacts with moisture to produce zinc oxide, which neutralizes the acid in the
paper.

(i)

Give the ionic equation for the neutralization of acid by zinc oxide.

(ii)

Explain with the aid of equation(s ), why moisture causes paper 'sized' with aluminium
sulphate to become acidic.

(iii)

Suggest Why the vapour of diethylzinc is more effective for neutralizing the acidity of
paper than using zinc oxide directly.

[
[
[

'

[
~

1..

[
[
[

L
L
[

l
L-

[4]
[Turn over

L
(b)

Platinum is a transition metal which forms many complexes with ligands such as NH3
and cr. One example is cisplatin.

Cisplatin is an anti-cancer drug which has the molecular formula of Pt(NH 3)zCI 2 . It is
a deep yellow solid with the following structure:

H3N

"'

H3N /

o_-)0
o-c,,

Pt/

""Cl

H3N
H3N

/p'

(i)

[
r~

,,

"' / -)<>
0

dicarboxylate ligand

[~

Cl

Cisplatin can degrade into other toxic products. In constrast, carboplatin, which is
similar to cisplatin except for the presence of a bidentate dicarboxylate ligand,
degrades less readily.
The structures of the dicarboxylate ligand and the carboplatin molecule are shown
below:

r
r

0-C

carboplatin

What do you understand by the term "bidentate" ligand?

r-

[
(ii)

. Explain why the presence of the bidentate ligand slows down the degradation of
carboplatin as compared to cisplatin.

L
L
[

r
'--

[
[Turn over

IL
8

L
[
1-

c
[

(iii)

Another platinum complex, Q has the molecular formula Pt(NH3 )4CI4. On treating 1
mole of this complex with an excess of aqueous silver nitrate, 2 moles of silver
chloride was produced as precipitate.
Suggest a displayed structure for complex Q. State the presence or absence of
dipole moment in the displayed structure of complex Q.

[
[

['
[

(4]

'

~/

1.

(c)

'

~!

The stability constant, Kstab. of the nickel complex [Ni(en)3] 2+ (aq) has the value of log
Kstab
18.0 (en
ethane-1 ,2-diammine). When this complex is treated with
butanedione dioxime, the Ni(ll) ion forms a red complex with this ligand in the mole
ratio 1:2.

Butanedione dioxime has the structure:

-'

-.
f

H 3 c""-

I __,

c-c

/c H 3

I I

N N
HO/
"oH

r.

(i)

Predict the log

Kstab

value of this red complex.

'

. (ii)

What is the coordination number of this red complex?

L.

L
L

(iii}

State the shape of this complex.

[3]

[
[Turn over

l
L
L

r
(d)

An aqueous solution of chlorine can be used as a disinfectant, for example in


swimming pools. The disinfecting action is due to the presence of chloric(l) acid,
HCIO, formed by the reaction of chlorine with water:
Cl 2 (aq)

H2 0 (I)

HCIO (aq)

H (aq)

Cl" (aq)

Chloric(l) acid ionizes as a weak acid:


HCIO (aq)

(i)

--------

H+ (aq)

c
L

Cto (aq)

In many swimming pools, chemicals other than chlorine are used to form the
chloric(l) acid. This is partly because the use of chlorine gas causes much more
corrosion of metal parts in the pool than does chloric(l) acid.

Compounds used to chlorinate pool water in this way include calcium chlorate(!) and
chlorine dioxide, CI02 .

[,

Why should chlorine in pool water cause much more corrosion of metal parts than
chloric(l) acid?

L
:r~
/

r
(ii)

Suggest one other reason why the use of chlorine itself is undesirable.

['
(iii)

Chlorine dioxide undergoes a disproportionation reaction in water to give a mixture of


chloric(l) acid and chloric(V) acid. Suggest a balanced equation for this reaction.

L
[
[5]
[Total:16]

J
<.

>

l
(

[Turn over

10

L
L
f1

[
[

Name:

5.

CG: - - - - - - -

o,b

A reaction scheme is shown below.

v - C H 2CH 2CH3

[
Excess cone

L
[

..,OzNb

Stage I

180oC

H,so,
OzN

OCH CH CH
2

D
Stage IV

Stage Ill

II

CH3-C-Nh

CHb

0 2N

(a)

Give the structural formulae of organic compounds B, C and D.

Q-cH,CH,CH,

or:~H2 CH,

L
L
[
[

L
i'
L~

NaOH(aq)
heat

~~

A
H

lz/

OCH,~:CH,

HBr(g)
room temp

[
'[-,

~Stagell

D
[3]
[Turn over

II

r
[

(b) What reagents and conditions are used in stages I, II, Ill and IV?
--

Stage

Reagents

Conditions

I
II

Ill

IV

[4]
(c)

Suggest a simple chemical test to distinguish between compounds A and E.

[
L~

c
[2]

(d)

Give a brief outline of the mechanism involved in stage I.

[
['
r,
L

r.

L
[.
[
[3]
[Total:12]

[Turn over

12

L
l
l
.[
f

L.

c
[

[
[

6(a)

An organic compound, X, has the followfng composition by mass:


C: 58.8%

H: 9.80%

0: 31.4%

(i)

Find the empirical formula of X.

(ii)

Compound X is an unsaturated diol. When heated with acidified potassium


manganate(VII), 1 mole of compound X gives 1 mole of CH 3COC0 2H and 2 moles of
carbon dioxide.

[
[

Deduce two possible structures of X. In each case, label any chiral carbon atoms
present.

t~
:[

c
[

(iii)

X rotates plane polarized light. Using one of the structures from part {ii), draw diagrams
to illustrate how X gives rise to optical isomerism.

r
L

c
L.
[

L
L
L

[Turn over

13

r
(iv)

State the number of stereoisomers compound X exhibits.

c
[5]
(b)

Serine is one of the 20 amino acids that can be found in the human body. It has the
structure:
H 0

II

L
[

H2N-C-C-OH

CHz

OH

(i)

.Serine exists as zwitterion in aqueous solution. Draw the structural formula of the
zwitterion formed, and write equations to show how it can react as a buffer.

c
[

r
~._,

f'

[
(ii)

Explain why serine has relatively high melting point.

[,

c
'0

L
[3]
[Totai:S]

END OF PAPER

14

L
fL
{

r
,.

TEMASEK JUNIOR COLLEGE

c
01>~~~

c
[

[
[

f'

"o'-t~E"GE. f0"- ~;:;

CHEMISTRY
PAPER 3
Wednesday

9251/03
13th SEPTEMBER 2006

2 hours 45 minutes

Additional materials: Answer paper


Data booklet
Graph paper

INSTRUCTIONS TO CANDIDATES
Write your name, centre/index number & CG in the spaces on the cover page provided
and on all the work you hand in.
Write in dark blue or black pen on both sides of the paper.
You may use a pencil for any diagrams, graphs or rough working.
Do not use staples, paper clips, highlighters, glue or correction fluid.
Answer all questions in Sections A, B and C. The last question in each Section is of the
form either/or. In Section D, you are required to answer both questions on only one out
of the four option topics.[Only two options are offered here]
Begin each answer on a fresh page. At the end of the examination, fasten all your work
securely together.

[~
',"7

INFORMATION FOR CANDIDATES


The number of marks is given in brackets [ ] at the end of each question or part question.
You may use a calculator.
A data booklet is provided.
You are reminded of the need for good English and clear presentation in your answers.
You should not attempt questions on option topics for which you have not prepared.

c-----~~~~~~---

L
l

This question paper consists of 14 printed pages.

[Turn over

L
[

lrv
Section A

Answer all questions in this section

(a}

Predict the shapes of carbon tetrachloride molecule, CCI 4 and


hydroxonium ion, H3 0 .... Account for any differences in the bond angles of
these species.
[3]

(b)

The boiling points of 2-nitrophenol and 4-nitrophenol are to be determined


in an experiment. Which of the two compounds will show a higher boiling
point? Explain your answer.
[3]

(c)

(i)

In a movable piston chamber, CO and 0 2were present in the


molar ratio 2:1. They then react according to the following
equation:
2 CO (g) + 02 (g) -7 2 C02 (g)
After complete combustion at atmospheric pressure, the
volume of the vessel decreased by 20 cm3 and the temperature
increased from 25 c to 50 c. Calculate the volume of the vessel
before the start of the reaction.
[3]

(ii)

In a separate vessel, a fixed mass of-an ideal gas, X, is heated at


a constant pressure. Sketch a graph of V against T, where V is
T
the volume of gas X in cm 3 and Tis the temperature in Kelvin.
(Numerical values are not required in your answer.)
[1 J
[Total: 10]

c
[

L
[

L
f'

c
L
[,

l
L
2

[Turn over

>

'

L
11:

(a)

[
[

The hydrogen phthalate ion, HC8 H5o4- is a weak monobasic acid.


0.525g of potassium hydrogen phthalate is dissolved in water and made
up to 250 cm 3 in a standard flask. The pH of the resulting solution is 4.24, -

(i)

Calculate the acid dissociation constant Ka for the hydrogen


phthalate ion.
[1]

(ii)

20.0 cm 3 of the standard solution prepared is titrated against an


excess of 0.0102 mol dm-3 aqueous NaOH using phenolphthalein
indicator. Sketch the pH curve for this titration.

{3]

[
[

(b)

Slurries of barium sulphate are swallowed by patients before X-ray


examination of the digestive tract since the slurry is opaque to X-rays but
cell tissue is not. Barium sulphate is very slightly soluble in water and the
Ba2 + ions are poisonous.

(i)

Explain why potassium sulphate is often added to slurries of


barium sulphate.
(2]

(ii)

The solubility product for barium sulphate in water is


1.1 x 1 10 mof dm-6 at 25c. Calculate the solubility of barium
sulphate at this temperature.
[2]

r~

(iii)

Will barium sulphate precipitate when 10 cm 3 of 0.02 mol dm-3


barium chloride is added to 30 cm 3 of 0.001 mol dm-3 potassium
sulphate solution at 25.C? Show how you arrive at your
conclusion.
[2]

[~

o-

[Total: 10]

[
[
[

L
['
,_

L
L
L
l

(Tum over

t-

r3

either

-(a)

(i)

Draw a fully labelled set-up of the cell under standard conditions.

(ii)

Calculate the emf of the cell and write a balanced equation for the
reaction occurring in the cell.

(iii)

(b)

A cell is set up using the systems Cu 2+(aq)/Cu(s) and H20(1) I


acidified H 20 2 (aq).

Explain what is observed and any effect on the emf of this cell if
NaOH (aq) is added to the Cu 2+(aq)/Cu(s) half cell.
[6]

In the electrolysis of aqueous sodium sulphate using graphite electrodes,


it was found that the volumes of gas formed at the anode and cathode are
25 cm 3 and 40cm 3 respectively. The anode gas was found to be a mixture
of oxygen, carbon monoxide and carbon dioxide. When the anode gas
was collected and shaken with aqueous sodium hydroxide, its volume
3
was reduced to 15 cm

[
[

r
[
[

[
{-

F~

L
- aqueous sodium sulphate

[
['

graphite
~

lRt_

I .. \

Inert e(()dl'l)do

cathode

anode

(i)

Why are oxides of carbon produced at the anode?

(ii)

:Calculate .the volumes of carbon dioxide;::carbon monoxide


and oxygen in the anode gas.

r..
[4]

[Total: 10]

L
4

[Turn over

L
L

L
\

r
(

or

3
(a)

A cell set up between acid-ified Cr20/.(aq)/Cr3+(aq) system and an


unknown metal electrode M2+(aq)/M(s) has the following cell notation:

M(s) IM 2+(aq) II Cr20/'(aq),Cr3+(aq) I Pt(s)


The standard cell potential was found to be +1.77V.

(i)

Use the data booklet to calculate the standard electrode potential


of the M2+(aq)/M(s) system and identify M.

(ii)

Explain the effect, if any, on the emf of the cell on adding dilute
sulphuric acid to the Cr20/'(aq)/Cr3+(aq) half ceiL

(iii)

Describe what you might observe when the identified metal rod, M
is dipped into a solution of acidified KzCrzOy.

,,

[4]
(b)

An element Y has two isotopes of masses, 69 and 70, in the ratio 1:2.
When a current of 2.5 A was passed through for 50 minutes, 1.81 g of the
metal was deposited. Using the results of the experiment, calculate the
value of x in the electrolyte, YCix.
[3]

(c)

Unlike iron, aluminium is resistant to atmospheric oxidation because of


the oxide layer on the surface. Draw a fully labelled diagram and with the
aid of equation(s), explain how this oxide layer can be thickened by
electrolysis.
[3]

{~

[
[,

[Total: 10]

r
,,,

L
L
L
[

l,

(Turn over

t-

Section B

Answer all questions in this section.


4

(a)

Describe and explain the variation down the group of the ease of thermal
decomposition of the carbonates of Group II elements magnesium to
barium, writing equations where appropriate to support your answer.

[4}
{b)

{i)

Comment briefly on the fact that ions of transition metals are


usually coloured.
[2]

(ii)

Explain the following observations, quoting relevant


values
from the Data Booklet and writing balanced equations {including
state symbols) where possible.

When zinc metal is added to a green solution of V3 +(aq) under


acidic medium, the solution turns violet. The excess zinc is filtered
off and the violet solution is allowed to stand for several hours.
[4]
The solution slowly turns back to green.
[Total: 10]

[
[

[
[

either
{a)

The melting points of three common chlorides are given in the


following table.
Compound
Sodium chloride
Silicon tetrachloride
Phosphorous
-pentachloride

Formula

Melting Point/ C

NaCI
SiCI4
PC Is

801
-70
162

r,
L

Briefly relate these melting points to the structure of, and bonding in, each
of these chlorides.
[3]
{b)

r~

Write a general equation for the decomposition of halogen hydrides,


H-X.
... State andexplainthe trend ofthe thermal stabilities of HF, HCI; HBr
and HI, using appropriate data from the Data Booklet.
[4]

l
L
r
L

L
[Turn over

C
l
\

r
l

(c)

In a laborato1y experiment, a chemist discovered that a Group II element,


Z, burnt in nitrogen to for_m an ionic nitride Z3 N2 . When 0.20 g of this
nitride was reacted withwater, ammonia was liberated as shown in the
following equation.
Z3 Nz + 6H 2 0

3 Z(OH)z + 2 NH 3

The ammonia obtained was passed into 70.0 cm 3 of 0.100 moldm'3 dilute
HCI. The unreacted HCI is requires 42.90 cm 3 of 0.100 moldm-3 dilute
NaOH for complete neutralisation. Identify Z.

[3]

[
[

[Total: 10]

c
r
_j

or

(a)

MgO, Alz03 and P40 1oare common oxides found in our daily lives.
Describe their solubilities in, and their reactions with water, if any, relating
any differences to their structure and bonding. Write equations where
[7]
appropriate and suggest the pH of any aqueous solution formed.

(b)

In order to speed up the rate of the reaction between iodide and


peroxodisulphate (VI) ions as represented by the following equation:
21"(aq) + SzOt(aq)

--4

2Sol-(aq) + lz(aq)

a small amount of Co2+(aq) will be added to the reaction sample.


Explain the role of cobalt(ll) ions in this reaction giving equations where
necessary.
[3]
(Total: 10]

_,

c
[

'~

L
L

L
l

(Turn over

l. -

Section C

Answer all the questions in this section.

(a)

Compound C has the structure shown below:

[
[

Compound C

Suggest structural formulae for the product formed when C reacts with

(i)

2, 4- dinitrophenylhydrazine

(ii)

LiAIH 4 with ether as solvent

r-,

(iii)

hot, aqueous NaOH

(iv)

hydrogen bromide

{5]

c
/

(b)

Propene and 2-methylpropenoic acid combine to form an addition


polymer.
(i)

(ii)

(iii)

Assuming that the monomers join together in a 1:1 ratio, suggest


a repeat unit for the addition polymer.
Explain why an increase in the amount of 2-methylpropenoic acid:
propene ratio used in the polymerization will increase the strength
of the polymer. Draw a suitable diagram to illustrate your answer.
When excess sodium hydroxide is .added to the above polymer,
another polymer is formed. Compare the strengths of the two
polymers, giving a suitable explanation for the change in strength.
[5]
[Total: 10]

L'
[

c
[

L
r

L
8

[Turn over

l
l
L
r

L
r

t.

(a)

You are given the following scheme of reactions.

~'/Mn04- H~-CH 20H

PC1s1

c
l~

c
[

H~-CONHCH2CH~OH.
H

(i)

Identify the lettered compounds E and F.

{ii)

Give a synthetic route, involving not more than three steps, from D to G.
In your answer, state the reagents and conditions and intermediates
involved in the synthetic route.

(iii)

Name the functional group(s) present in compound H.

(iv)

Give the structural formula of another possible organic product that may
be formed during the formation of H from F and G.

'

'

[7]

.~

[
['
'

(b)

Predict whether the following compounds are more or less acidic than
phenol.
OH

H3C

OH

CH3

02

_,N02

c
[

CH3

N02

[3]

[ Total:10]

[
[
9

L
L
{

[Turn over

l
8

either
Compound J is neutral and has the molecular formula C 10 H 13 N02 . It was used
principally as an analgesic and can also be used to treat rheumatoid arthritis. The
compound is not commonly used as a drug now as it was found to be
carcinogenic.
The NMR spectrum of J is shown below. The singlet at
addition of 0 20.

o7.9 disappears on

('
[[

[
3
4

,--A---,

['
______.)I...._.) ' - - - - - - 1

~--r--,---r--.---.--.--.---,_--r--T--.---T--.--_,--.---.--r___,--.---r--.-~~

11

10

pprn

When J is boiled under reflux with aqueous sodium hydroxide, compound K,


CaH 11 NO, and a sodium salt, C 2H302Na, are formed. K turns red litmus blue and
reacts with bromine water at room temperature to give a white precipitate.
The sodium salt (C 2H30 2Na) can be acidified using hydrochloric acid. The
resultant acidified product can be reduced by lithium tetrahydridoaluminate(lll) to
form compound M. M reacts with hot aqueous alkaline iodine to give a yellow
precipitate.

0
y

reflux at temp

~ 60C

..

Sn I cone HCI

L
reflux

. OCH 2CH 3
Account for the splitting pattern in the NMR spectrum of J. Identify the
compounds J to M and explain the above observations.
[ Total:10]

10

f'

L
r

L~

K can be prepared in the reaction scheme shown below.


cone HN03 + cone H2S04

(Turn over

L
l'
f'
l

L
L
L
t

c
[

c
[

c
c

or
Compound P has the molecular formula C 10H 140. It is isomeric with the
compounds, thymol and carvacrol. In ancient Egypt, thymol and carvacrol were
used for the preservation of mummies because of their bactericidal and fungicidal
characteristics.

The NMR spectrum of P consists of a doublet at 8 1.2, a singlet at 2.3, a


multiplet at 8 3.1, a singlet at 8 4.8 and a multiplet at 8 6.9. The integrated areas
of these peaks are in the ratio 6:3:1:1:3 respectively. On addition of D20to the
sample, the peak at 8 4.8 disappears.
Compound P reacts with chlorine gas in the presence of UV light to form
compound Q, C10H130CI.

When Q is refluxed in alcoholic potassium hydroxide, the following compound is


formed:
OH

[
[
[

CH3
H
H3 C

t'l

On addition of bromine water at room temperature, P forms a white precipitate R.


When treated with acidified potassium manganate (VII) under reflux conditions, P
forms compound S, C8 H6 0s. 1 mole of S reacts with 1. mole of sodium
carbonate.
Account for the NMR spectrum of P. Identify the compounds P to S and 'explain
the above observations.
[ Total:10]

L
[

L
[

L
L
l
l

II

[Turn over

l'

rSection D

r~

Answer both questions on one option

BIOCHEMISTRY
Answer both questions on the paper provided.

(a}

Give a brief account of the interactions present in the tertiary structure of


a protein.
When a protein is denatured, the tertiary structure is disrupted. Explain in
chemical terms how R group interactions are broken in each of the
following examples. In your answer, identify the type of R group
interaction which is broken for each. example. Give specific examples.

(i)

In the presence of metal ions eg Ag+ and Hg 2+

(ii) .

At pH 2

[
[
[

[6)

(b)

Integral proteins are important components in cell membranes. What is


the function of integral proteins in membranes? Explain why bilayers
cannot perform this function.

[4]
[ Totai:10J

10

(a)

(i)

Draw the structural formula of the triglyceryl ester formed from the
fatty acid palmitoleic acid, CH3(CH2hCH=CH(CH 2)?C0 2 H. Draw
the ester groups as displayed formulae.

(ii)

When broken down in the body, palmitoleic acid (Mr = 282)


releases 39 kJ g- 1 The simple sugar glucose releases 16 kJ g- 1 .
Using the formula of the fatty acid and the formula of glucose,
.-.explain quantitatively why thereJs a difference between these two
energy values.

(iii)

Use the structures of palmitoleic acid and glucose to explain two


differences in their physical properties.

[7]
(b)

The primary structure of large proteins can be determined by breaking up


the protein using a process called recombinant DNA technology .
. Explain how a knowledge of the genetic code of the DNA enables the
primary structure of the polypeptide to be deduced.
[3]
[ Total:10)

12

[Turn over

[
[
[

[
[
[
.

'

L
L
L
L
l

c
[

FOOD CHEMISTRY

Answer both questions on the paper provided.

11

(a)

[
[

Name and draw the structure of one essential amino acid obtained from
[1]
protein in the human diet.
(b)

[
[

[
[

Of the amino acids obtained from proteins in the human diet, only eight
are essential to adult.

The folding of the a-helix structure in the proteins such as enzymes is


caused by interactions of the R groups.
Give simple examples of four different group interactions, illustrating
each one with a suitable formula for the R group concerned.
[4]

(c)

Processing techniques may cause denaturation of food proteins. State


and explain one way, other than pH and temperature, by which proteins
can be denatured.
[1]

(d)

An extract V from a food material was divided into three equal portions
and subjected to the following investigations.
(i)

A sample of V was warmed gently with ethoxyethane and the


mixture was then filtered. The resulting filtrate was then
evaporated to dryness. No residue was observed after
evaporation.
Which food component(s) was/were absent from the extract V?

(ii)

A second sample of V was treated with a strongly alkaline solution


of copper (II) sulphate and produced a purplish-violet solution.
Give the formula of the functional group which this test indicated.

(iii)

A third sample of V was boiled with a dilute acid solution and the
mixture filtered. Explain, using a generalized equation, how this
condition affects the food component shown to be present in (ii).

(iv)

One other major food component, which gives a blue colouration


with iodine solution, was present.

L
L

Indicate, using a generalized equation, what would happen to this


food component when the extract was heated in acid solution.
[4]
[Total: 10]

L
[

13

[Turn over

l
C'
12

(a)

(i)

The browniog of potato crisps, the crust of baked bread and the
edge of roast meats are examples of the Maillard reaction. This is
the reaction between a reducing sugar and an amine group, e.g.
from a protein.
Comment on the Maillard reaction and suggest what reaction
occurs between the two functional group involved.

[
[

(ii)

Explain how potato crisps, although they are contained in sealed


packages, may become soft and tallowy on exposure to light.

(iii)

By considering two named food components present, explain what

happens when meat is cooked.

(iv)

Why must the moisture content of certain foods be carefully


controlled during processing?

[7]
(b) --

A student is investigating the causes and preventions of browning


process occurring in some fruits and vegetables e.g. banana, apple and
potato. Three slices of apple are cut for comparative experiments on
browning.

[
[

The three slices I, II, and Ill are as follows:


I
II
Ill

untreated apple;
covered with lemon juice;
covered with sugar syrup.

Predict the order in which the apple slices become brown - placing first
the one which becomes brown most rapidly. Explain your answer.
[3]

[
[

[Total: 10]

L
l
l
14

L
l

r----".

!'

Qn No.
2(a)(i)

YISHUN JUNIOR COLLEGE


200fPRELIMINARY EXAM PAPER 3 (FREE RESPONSE QUESTIONS)
MARK SCHEME
Qn No.
1(a)(i)

Answers
Location
.Charge
No.
nucleus
0
38
Neutron
nucleus
31
+
Proton
shells
31
Electron
Electronic configuration of Ga : 1s 2s" 2p" 3s" 3p" 3d1

Marks

Answers

Rate

=[Br2]' [CH3COCH3]Y [H ]z

0.05)' = 5.7x1 o:
( 0.1
5.7x1o

Marks

x=O

=5.7x1 o
( ~)y
0.4
7.6x1 0"

5
5

y =1
(ii)

A = 69 X+ 71(1 00 - X)
r

100

0.2 )'
( 0.03

69x + 71 oo -71x = 69700

3.1x1 0-4

=7.6x1 0"5
z =1

-2x=-130
x=65
:.

(b)

69

Ksp

Ga- 65%

71

. . _. . . -.. . . . .I.

Ga-35%

= [Ga"' )[CI"]"

(b)
energy

Let the solubility of GaCh be


(X+ 0.1)(3d "'1.6X1 o-s
27xa =

x mol dm' ,

,,....,..................~. .....
:

1.6X10"5
reactants

0.1
x = 0.0180mo/dm 3

~\ I~::~~~~~-~. . . . . .

'.

E, for

Ea for

uncatalysed rxn

. L.. . '........
'

pro uc s

(c)

progress of rxn

Ga 3 + 3e -t Ga

Catalyst provides an alternative route with a lower activation


energy. More reactant molecules have energy greater than
activation energy, leading to an Increase in effective collisions.
Hence rate of reaction increases.

3(96500) = 69.7
10X1810 mass
mass= 4.36g

(c)

[A]

Rate= k [A]

::~

g~;~;-=~=. . . 10

15

20

time/min

-..I

c__

L__,

t__._;

L...:-

L-

-~

Answers

Qn No.
(d)

r
'
........__.,

L.__;

..

1-;-~

L-J

QnNo.
3 either
(a) (i)

Marks
..

Energy I kJ mol"'

C+D

L-J

L.._t

'---.-!

:__j

.___j

._j

Marks

Answers
When small amounts of H is added, it is removed by the large
reservoir of HC03. and hence the concentration of H+ remains
approximately constant and so pH is approximately constant.
H+ + HCOi ~ H2C03
When small amounts of OH" added Is removed by the large
reservoir of H2C0 3 pH is approximately constant.
OH" + H2COa ~ HCOa + H20

.eo

...X.kJ.mol~..........-

(ii)

K = [W][HCO;J

[H2 C03 ]

(iii)

Ka

Reaction progress

=7.90 x 10

mol dm"3
~

(iv)

pH= pK. + lg [HCO;]


[H2C03 )
7.4 = -lg(7.9x1o- 7 ) + lg

[z~o;J
[

03]

lg [HC03] =7.4-6.1
[H2C03 )
=1.3
[HCO;] = 20
1
[Hp03)
(b) (i)

HX + NaOH ~ NaX + H20


Concentration of HX

(ii)

= ~ =0.0900 mol dm"3


25

x + H20 ,.. HX + OK'


pH> 7
x is a stronger base than H20 and it accepts a proton from H20.
H20 polarises and forms OH", (OR
hydrolysis to 'produce OH")

(Iii)

'

undergoes anionic/salt

Phenolphthalein.
pH at the end-point for neutralisation is greater than 7 and
phenolphthalein changes colour at pH which coincides with the
pH at the end-point.

L,.__

Answer Scheme for YJC 2006 Chernlstry Prelim Paper 2

Answer Scheme for YJC 2006 Chemistry Prelim Paper 2

(b)

(b)

Under suitable conditionn, chlorine and fluorine gas react to give an


interhalogen compound, C/F.

List the possible oxidation state of for metal A and metal B.

[2]

A: +2

Sketch the variation of pV against p at 300k for two moles of (i) an

ideal gas and (ii) C/F.gas..

8:+2,+3 +6
Label your graphs clearly. (p = pressure, V =volume)

[2]

pV

(c)

One of the metals form stable complex ions.

(i)

Write down the formula of the complex ion that would be formed
between the M 3 of this metal and exce.ss oH ions.

[1]

[M(OH)sf"
(ii)

(c)

2 moles of C/F(g) is subjuCted

to

Octahedral

higher temperature of 600 K.

Using the same plot in Siketch (b), show the change of variation of
pV against p at 600 K for 2 moles of C/F(g). Explain your sketch.
[3]

At higher temperature, average K.E. of C/F molecules

[1]

What shape would the complex ion take?

(iii)

Explain why this complex is coloured.

[2]

d orbitals are split into 2 different energy levels in presence of

t, move

of ligands. Electrons from lower energy gets promoted to higher


further away from each other, I.M.F. becomes less significant.
energy level absorbs energy in visible region. The energy not
absorbed is transmitted giving rise to the colour.
[Total: 9 marks]

3.

Consider the two metals, A and B, whose electron configurations


are as follows:
2

[Total: 8 marks]

A: 1s 2s 2p 3s 3p 4s
B: 1 s2 2s 2 2p 6 3s 2 3p 6 3d' 4s 1
(a)

Explain which of the two metals are able to form ions of stable
variable oxidation states.

[2]

The close energy of 3d and 4s orbitals electrons in B allows B to


form stable ions of variable oxidation no. by losing different no. of
electrons with small amount of energy.

'-.,.__:

..__,

L-.J

,,

!~

-(

!_::..j

'

1.__]

;_)

[____,;

I
'
._.....:

l-''

.___]

-.-...-1

...__)

'--.J

,r
Answer Scheme for YJC 2006 Chemistry Prelim Paper 2

Answer Scheme for YJC 2006 Chemistry Prelim Pnper 2

4.

(c)

This question concerns the preparation of an organic compound


QQ using the apparatus illustrated below.

Suggest a reason why an excess of C()ncentrated H2S04 was used


for the reaction taking place In the flasl<.

[1]

To grevent formation of ether.


DmpptJ<g finnel'and \tr)r)pdr

(d)

'rcc~ih:g

The reaction mixture in the flask developed a light brown colour


which becomes black on heating to about 170
Suggest a
[1]
possible identity for the black colouratlon.

oc.

11\i Mure

carbon
(e)

State the type of organic reaction mechanism involved in the


formation of QQ from PP and write down clearly the steps Involved
in Its formation.
[2]

r--sr"

electrophilic addition

~
H 3 C-y\:J_CH~ _t:

A mixture of concentrated H2S04 and CH3CH(CHzCH3)CH(OH)CH 3


heated in the flask to about 170 C. A compound PP, which is the
major product of the reaction, was formed and PP reacted with
bromine in the second test tube to form QQ.

CH CH

(a)

Give the structural formula of the compound PP.

[1]

Br

Br

Br
I
- - H3C-y-y-CH 3
C2H5 H

H3 C-C=C-CH
I
3
Br

CH 2 CH 3

Br

H3 C-y-y-CH 3
(b)

C2H5 H

Give the structural formula of compound QQ and give its IUPAC


name.
[2]

~r

H C-C-C-CH
3

(f)

~r
I

Compound QQ formed is usually contaminated with excess Br2.


Suggest a suitable way to remove excess Br2.
[1]

C2 H5 H

Car!Y out fractional distillation.

2,3-dibromo-3-methylpentane

r-:

,--..

~I

L,

Answer Scheme for YJC 2006 Chemistry Prelim Paper 2

Answer Scheme for YJC 2006 Chemistry Prelim Paper 2

(g)

(iii)

Suggest a simple chemical test which can be carried out to show


that pure QQ is a bromine containing compound.
[2]

,-.........,

i.

'

Give the obseNation when aqueous Br2 Is added to novocaine.


Draw the structure of the product formed.

[2]

Decolourisation of aq Br2, white ppt is formed.

Addition of aq NaOH, boil followed by cooling. Acidify with dil.


HN0 3 then add AgN03. A cream ppt of AgBr formed shows the
presence of Br In QQ.
[Total: 10 marks]

5.

Novocaine is a local anaesthetic which is used for minor surgery to


areas such as the nose, throat or eyes. It is applied to the surface
before surgery.

(d)

Novocaine has a Kb of 4.1 x 10 "7 moldm"3.

Novocaine has the followil1g structure:

(i)

Account for this basic character in terms of its structure.

Lone pair of electrons at N atom accepts protons,

(a)

[1)

Name two functional groups in this substance.

[1]

H'.

Another isomer of novocaine has the structure shown:

ester and amine


(b)

Novocaine reacts with hot dilute HCI.

(i)

Name the type of reaCtion which has taken place.

[1]

hydrolysis

(li)

Draw the structure(s) of product(s) formed in this reaction.

[2]

(ii)

Compare and explain the basicity of this isomer with novocaine.

[2]

The isomer is less basic.

HO-CH2-CH2N-(CH 2CH 3) 2

Less no. of basic amine group.

10

._,
(

'--'

__

{.

LJ

!___L

J~

'~-

\....--i

Answer Scheme for YJC 2006 Chemistry Prelim Paper 2

(c)

'--J

i.\.-J

i,

...............

'

L--.1

.__.

c~

,------~

'--'

------l
----"

:......:.:J

__;

A11swer Scheme for YJC 2006 Chemistry Prelim i>aper 2

The synthesis of novocaine is given by the following path.

@__!___

@ - N 02

@ - N H2
CH 3CI

Novb::alne

~ H N-@-cooH
2

(I)

Give the reagents and conditions for steps (I) to (N)

AICI3

[4]

I - cone. HN03, cone. H2S04, 60 c


II- Sn, cone. HCI, reflux
III - dil. H2S04, KMn04, heat

IV- HOCH2CH2N(C2Hsh,conc. H2S04, heat


(II)

Give the structure of A.

[1]

Q
CH 3
(Ill)

Give another likely product of A.

[1]

bCH,
[Total: 15 Marks]

11

12

-J

L.

c ~~>
[
[

1.

JVlNIO~

letl1PrS-K

Enter your NAME ( as in NRIC ).


Enter the SUBJECT TITLE.

3.

Enter the TEST NAME.

CD L-1..-l?lA- G
U1 C""YYJ \ ST~ 'T

:J. oo b 1Sc

RUB OUT ERRORS THOROUGHLY

I~ll
yg ~ENCIL ONLY
ENTRIES ON THIS SHEET

Pre/wYl

PvqJ.ev

-,_
r

5.

6.

Enter the CLASS.

1
Enter your CLASS NUMBER or j

I
N

INDEX NUMBER.

E
X

Now SHADE the correspondingt


lozenge in the grid for
EACH DIGIT or LETTER

[L

c:
c:

[--.
-~.

[:

L
L:

[.

0
c::i.

II

M
B

=
0
=
0
=

E
R

I I

r,

1
2
3
4
5
6
7.

8
9

10
11
12
13
14
15
16
17

18
19
20

56

==-====
0
1
2
3
4
5
.6
=-=====
o 1 2
3
4
5
6
-======

SHADE APPROPRIATE BOXES

_j_

7.

012

~--,,;

/S:;::?'

IL__
WRITE

[[-

.Ml~l~TRY OF EDUCATION

2.

4.

=
1
=
1
=
1
=
1
=
B
=

2.

=
2
=
2
=
2
=
2
=
c
=

3.

=
. 4.
=
4
=

=
3
=
4 :
3
= C::::J
D
E
= =

:>.

=
7
=

.8 .

= =
5
6
8
9
= =
= =g
5 6 7 . ' 8
C::::JC::::J C::::l: =
=
6
5
7
8
9
= .6
= = = =
.5
B
7
9
c::i.
=: =. = =
F
H
I
G
= = = =

'[~.=.

3
q
3

ir

q.

INSTRUCTIONS FOR RECORDING ANSWERS


Suggested answers to each question are given in the question paper. Choose an answer and shade the corresponding
lozenge. If there are only four suggested answers, A, B, C, D, ignore Eon this sheet. Don't worry if the question paper
has less than the 60 questions allowed for below.

c
b
= =
B
A
c
=
=
A
ir
c
o .= b
A
B
c
A

i:

=D

=
E
=
.E

iiil!!l

..c:::J

=
=
A
.~~ =D
0
A
B
c D
= = =
A
B
iJ
.C
= =
=
A
B
c D
= =
=
A
B
c D
0
= = -'
A
B
c D
= =
=
A
B
c D
= =
=
A
B
c D
= = =
A'
a c D
=
= =
A
B
c D
= = =
A
B
c D
= 1111!1 = =
A
B
c D
= = =
'A
B
c D
= = t:::J
A
B
c D
= = =
B
c o
A
= o =
A
B
c D
= = =

- -

-.fl!!iil

lllili

..

24
....

=
E
=
E
=
E
=
E
=
E
=
E
=
E
=
E
=
E
=
E
=
E
=
E
=
E
= .
=

23

o
E

22

=E

21

25
26
27
28
29
30
31
32
33
34
35
36
37
38
39
40

=
.A
=

c D E
= =
B
c D E
illll!!l
= = =
l:l
c D E
d
... d
=
B
c D E
= = =
8 . c
D
E
:c::l =-: =
B
c D
E
= = = =
.B
c D .. E
= =
=
B
c D E
= . Ill!!!!!
= =
c D E.
B
B

- A
A

=
A
=

.i\

=
A
=
A
=
.A
=
A
=
A
=
A
=

lilli'l

=B

=
B
=
B
=

..,

=
c
=

=
D
=
c D
!1111!1. .=
c 0
=
c D
=
c D
=
c n
= b
c D
=
c D
= =
c D
=
c D
=

--

-- -A

=
A
=
A
=

B.
B

=
B
=
B
=
B
=
B

c
=

=
E
=
E
=
E
=

42
43
44
45
46
47
48
49
50
51

52

53

=
E
=
E
=
E
=
E
=
E
=
E
=

54

=E

c
B
= =
B
A
c
=11i' .. =
=
. :c
'B
b
= ' t:::J
B
c
A
= = =
c
B
A
't::::J. t:::::J
=
c
A
B
=
=
=
A
B c
A

41

55
56
57
58
59
60

..

.b

i::::J .

[::j

= = .=c
B
-.:A
= = [::j
c
A
B
= = =
c
A
B
= = .=
A
B
c
= = =
A
B
c
= = =
A
B
c
= = =
c
A
B
= = t::J
A
l:l
c
= = =
B
c
A
= .-= =
A
B
c
= = =
c
A
B
= = =
A
B
c
= = =

=
D
=

=
E
=

b.
D

D..

E
E

= =
E
D
=. =
E
D
= =
E
.D
= =
0
E
= =
D
E
= =
0
E
= =
D
E
.= =
0
E
= =
D
E
d
=
D
E
= =
D
E
= =
D
E
= =
D
E
= =
D
E
= =
D
E
= =
E
D
= =

I'

I
I
I
I

SG101 SCANNING SYSTEMS PTE LTD

l'

T
[

06TJC Prelim P2 Answers


Answer ALL Questions

1. One reaction which occurs in air polluted with nitrogen oxides is shown below:
2NO(g) + 02(g)

2N02 (g)

3 experiments were carried out at constant temperature to find the relationship between
the initial total pressure of NO and 0 2, and the initial rate of formation of N02.
Experiment
Initial total pressure of NO and Ojatm
Initial pressure of NO/atm
cJ~itial rate of formati()!!_oJN_Qi_ a@s~ _ _

I
1.00
0.40
1.08

L_

--~--------

II

Ill

1.60
0.40
_2.16

3.20
0.80
17.28

!
!
I

It is the sum of the power to which the concentration of the reactants are
raised in the rate equation.

(ii) Use the above data to determine the order of reaction with respect to each of the
reactants. Show how you arrive at your answers.

Compare expt I and II,


Po2 doubles, rate increases by two times
Rate a Po2

Compare expt II and Ill,

Rm =

17.28
2.16

=-

'

k (0.80)" (2.40)
k (0.40)" (1.20)

n=2
Rate a

. (a}(i) Explain what is meant by the term order of reaction.

(PNo )2

.,

(iii) Use these answers to write a rate equation for the reaction.

Rate= k Po2 PNo2

Or

L
L

Rate= k [0 2][N0] 2

l
1

l
L

L
{-

(iv) Determine a value of the rate constant k, stating the units.

[
[
[

Use expt I, rate


1.08

=k
k

[
-.

~"~

(PNo )

0.60 X (0.4)2
11.25 atm2 s1
X

[6]
(b) The first step in a possible mechanism for the above reaction is:
fast

2NO(g)

==k Po

(i)

N202

(g) ................... Step I

Draw cross and dot diagrams for the two molecules NO and N20 2

~0 x.

XX

..N=io~
xxx

..

N.Nxo:
xxxX

_o

[
[

(ii) From the diagram of NO you have drawn, suggest why Step I is a likely step in the
reaction.

The molecule NO has a single unpaired electron implying that it is a radical


and is very reactive so step I is a fast step since radicals combine very
quickly.

[
[
[

L
[

l
L
L
l

r
[

(iii) Sketch on the same axes below, the distribution of the molecular speeds of

1: nitrogen molecules at a given temperature


II: same number of nitrogen dioxide molecules at the same temperature.

Label your sketches clearly.

[
[

Number
of molecules

[
r

[
Molecular speed

[5]
[Total:11]

2. In a gaseous reaction at constant pressure of 1 atm, XY3 is synthesized from X2 and

Y2.

x2 (g)

+ . 3Y2 (g)

2XY3 (g)

A mixture of 4.0 moles of X 2 and 12.0 moles of Y2 was placed in a vessel of volume 20
3
dm at 450K. When the system had reached equilibrium, it was found that 4.0 moles of
XY3 was present.

(a)

Calculate the partial pressures of each species at equilibrium.

x2 (g)

Initial moles
Change
Moles at equilibrium:

+ 3Y2 (g)
12
4
-6
-2

2XY3 (g)

0
+4
4

Total no. of moles of gas at equilibrium =12

= 1/6 atm
= 1/2 atm
Pxv3 = 1/3 atm

Px2

PY2

I'Li

[2]

[
[

Hence calculate the value of Kp of this reaction.

(b)

Kp

(1/3} 2
1/6

= 5.33

(1/2) 3

atm2

[1]
(c)

When the volume of the vessel was increased and the system allowed to come
to a new equilibrium at the same temperature, 9.0 moles ofY2 were found to be
present. Calculate the new equilibrium pressure of the system.

Xz (g) + 3Y2 (g)

-,
r
-j

Kp

= (2/14 x Prf

+3
9

4
-2
2

=5.33

(3/14 X Pr)(9/14 X Pr) 3

solving, Pr

Let Pr be the new equilibrium pressure

2
+1
3

Total no. of moles of gas at equilibrium =14

Initial moles
Change
Moles at equilibrium:

2XY3 (g)

= 0.259 atm
[2]

What effect will the use of a catalyst have on the Kp value? Explain your answer.

(d)

No effect on the value of Kp


A catalyst only speeds up both the forward and backward reaction to the
same extent. It has no effect on the composition of equilibrium mixture.

[2]

rL~
[

l
L
L

c
f'

- You are provided with the following thermochemical data:

AWl kJmor1

Reactions

Ba(s) + aq

Ba2+(aq) +2e
~ so/-(aq)

S(s) +20z(g)_ +aq+2e

-1465

Calculate the standard enthalpy change for the reaction:

(a)

Ba 2+ (aq) + SO/- (aq)

Ba2+(aq) + so/-(aq}
\

+286~

-907

., BaS04 (s)

Ba{sl + S(_s) + 202(g)

l.

+286

--~ BaS04 (s)

aH, 9

\-907

BaS04 (s)

.,.

-1465

Ba (s) + S (s) +20z(g)

By Hess' Law, AHr = -286 +907 -1465


= -844 kJmor 1 of Ba 2+(aq)

[2]
Given the enthalpy change of atomization of barium is +176 kJnior1 and the first
and second ionization energies of barium are +502 and 966 kJmor1 respectively,
calculate the hydration energy of the barium ion.

(b)

[
[

+286
Ba(s} + aq

Ba 2+{aq) +2e

[
+176

Ba(g)

\ .1.Hhyct

+502...

+966
Ba+ (g) +e

Ba 2+(g) +2e

[,

cycle
By Hess' Law, .1.Hhyd = -966-502-176 +286
= -1358 kJmor 1

u
[3]

u
5

L
. l1

r~
[

(c)

L
[

[
[

Explain how the magnitude of hydration energy of the magnesium ion differs from
that of the barium ion?

~Hhyd a

q+

--------r+
a 1/r+
since cationic radius of Mg 2+is smaller than that of Ba2+, ~Hhyd for Mg 2+ will be
more exothermic.
[1]
[Total: 6]

4(a)(i) ZnO (s) + H+ (aq) ~ H2 0 (I) + Zn 2+ (aq)


{ii)

[state symbols are optional]

Al 3+ has high charge density/ polarizing power. When Al 3+ interacts with the
water molecules, electron density is drawn away from the oxygen and the 0-H
bond becomes more polarized, eventually giving H+ in solution (acidic).
[AI(HzO)s] 3+ (aq) + H20 (I) ~ H30+ (aq) + [AI(H 20)s0Hf+ (aq)

{iii)

(b)(i)

It is a volatile liquid which makes it easy to spread evenly on affected parts of


the paper, it vapourises easily, removing all moisture easily from the paper,
hence preventing further hydrolysis of aluminium sulphate.
[4]
A bidentate ligand can form 2 dative bonds to the Pt central atom.

(ii)

A more stable complex is formed.

(iii)

CL

H3~ ~-!-;/tH.J

H3N

l
L
L
l

OR

v+~l'IHJ
CL.

NH;

zero dipole moment

}I3

rJ~------1-~----CL
1
I
I
I

I
I
I

H&Mv-r=~-~CL
NH3
net dipole moment
to the left
[4]

(c)(i)

> 18

(ii)

(iii)

Tetrahedral/ square planar (both acceptable)

[3]

t
-r.
[
(d)(i)

The aqueous HCI formed from chlorine in pool water, is a strong acid, which
dissociates completely to give a high [H+] unlike chloric (I) acid, which is a weak
acid, only dissociates partially to give a low [H+].
The high [W] will cause much more corrosion of the metal parts.

r-_

(ii)

Chlorine is a potent irritant in humans to the eyes, skin, upper respiratory tract
and lungs.

(iii)

CI0 2 (aq) + 3H+ (aq) + 3e -+ HCIO (aq) + H 20 (I)


CI02 (aq) + H20 (I) -+ HCIOa (aq) + W (aq) + e

4CI02 (aq) + 2H20 (I) -+ HCIO (aq) + 3HC10a (aq)

[
[5]
[Total: 16]

L
r-,

r
[

[
[

[
[
[

L
7

L
L

r
[

5. A reaction scheme is shown below.

o,b

[
[

o,h

Stage I

,. Q-cH,cH,cH,
.

Excess cone HzS04


180oC
0 2N

lz/

-\

.,

[..

l
o,h

NH,
Q-!HCH CH,

II I
CH,-C-NR
Q-cH,CH,CH,

Stage IV

Stage Ill

NaOH(aq)
heat

.~

A
H,NR
Q-cH,CH;,CH,

(}cH,b:cH,

HBr(g)
room temp

l-.'

~II

CHI3

(a)

Give the structural formulae of organic compounds B, C and D.

[
02

02N

L
L
L
L
L

(}cH,L-Na'

0-r=CHCH

'0-

8
,
bHCH 2CH3

D
[3]

L
[
(b)

What reagents and conditions are used in stages I, II, Ill and IV?
Stage

Reagents

Conditions

CH 3CH 2 CH 2 CI, anhydrous AICb ,

room temperature

II

Sn/ cone HCI

Reflux

Ill

CH 3COCI

Room temperature

IV

Cone NH3

Heat with excess NH3 in sealed


tube or alcohol solvent

[
----------

--~

[4]

bromine water at room temperature


JCompound A: white ppt and removal of reddish brown colour
Compound E: no visible observation

OR

NaOCI solution at room temperature


Compound A: violet complex observed
Compound E: no visible observation

t-'
[2]

(d) Discuss the mechanism involved in stage I.


Electrophilic substitution reaction
1 mark for steps 1 and 3
Step 1: Generating the electrophile, CH3CH2CH 2+.

AICb

AICI4-

CH3CHzCH2CI

[
[

+ . C!-hCJ:izCH2+

Step 2: Attack on benzene ring by CH 3CH 2CH 2+ followed by proton expulsion.


H
vi\:CH 2 CH 2 CH3

J)+CH 3 CH2 ~H2 ~ o2N~

o 2N--((

Step 3: Regenerating the catalyst.


H+

AICI4-

'
/

CH 2 CH2CH 3

fast., o , N - o +H+

AICb

[
[
['

S(Jggest a simple chemical test to distinguish between compounds A and E.

(c)

\.

[
[

HCI

[3]
[fetal: 12]

lJ
9

u
L!

[
[
[
[

G(a)

An organic compound, X, has the following composition by mass:


C: 58.8%

[
[
r~

0: 31.4%

(i) Find the empirical formula of X.

58.8

Ratio

4.90
I Ar
I smallest no. 2.5

[
[

H: 9.80%

ratio

9.80
9.80
5
10

31.4
1.96
1
2

Empirical formula of X is C5H1002

(ii) Compound X is an unsaturated diol. When heated with acidified potassium


manganate(VII), 1 mole of compound X gives 1 mole of CH3 COC02H and 2 moles of
carbon dioxide.
Deduce two possible structures of X. In each case, label any chiral carbon atoms
present.
H OH
OH
I I
I
H-C-C-C=C-C-H

I *I

H H

H CH20H

OH

I I
I
H-c-c=c-c-H
I
I I
H

-,

[--

(iii) X rotates plane polarized light. Using one of the structures from part (ii), draw diagrams
to illustrate how X gives rise to optical isomerism.

OH

[
[

H'''l'C"-.CH=CHCH20H
H3 C

OH

/c .. ,,,,,H
HOH2CHC=CH

~CH 3

L
[
[

L
L
L

10

l
[
(iv)

State the number of stereoisomers compound X exhibits.

l_

[5]
Serine is one of the 20 amino acids that cali be found in the human body. It has
the structure:
H 0

(b)

I II

H2N-C-C-OH

[
[

CH2

OH

Serine exists as zwitterion in aqueous solution. Draw the structural formula of the
zwitterion formed, and write equations to show how it can react as a buffer.

(i)

[
[

H 0

I II

H3W-c-c-o-

CH2

OH

H 0

I II

H3W-c-c-o- + H

H 0
+

CH2

OH

OH

H 0

H 0

I II

H3 N+-c-c-o- + OH-

(ii)

II

H2N-,C.,-C-o- + H 2 0

c~

c~

OH

OH

r-

CH 2

I II

H3N+-c-c-0H

Explain why serine has relatively high melting point.

A large amount of energy is required for melting as melting involves the breaking of
strong electrostatic forces of attraction or ionic bonds between the zwitterions.

u
u

[3]
[Total: 8]

11

[_'

L
\

L_

[
[

06TJC Prelim P3 Answers


Section A

1 (a)

C9l--:JIO'l5o

[
/

[
[

1\ ... -

<1Dq.5\l

~.'V\~M

c~

~ ---~9.
-\

)(.., ............. ('ll


/
~,

-~

- - -\ / "/
H......
......... H"

Tetrahedral (4 bond pairs)


Bond angle is 109.5.
Extent of bp-bp repulsion is the same.

Trigonal pyramidal (3 bp + 1 lp)


Bond angle is <109.5".
lp-bp repulsion is bigger
than bp-bp repulsion.

(b)

Boiling point of 4-nitrophenol is higher than that of 2-nitrophenol.


Being isomers (same Mr) the extent of van der Waals' attracti()n between
molecules would be very similar. However, due to close proximity of -OH andN02 groups in 2-nitrophenol, an intramolecular H-bond can be formed in each of
these molecules . This reduces the extent of H-bonding with other molecules of
the same isomer. Since boiling involves breaking intermolecular H-bonds
between molecules, more energy is involved in boiling 4-nitrophenol because
there is more extensive intermolecular H-bonding.

(c)

(i)

[
[
-,

-'

PN1
PNz

= n1RT1 -------- (1)

=nzRTz -------- (2)

Let V cm 3 be the volume of the vessel before the start of the


reaction. Hence, V 1 = V and V 2 = V- 20

Given thatreaction took place in a movable piston chamber,


P1 P2 and CO and 0 2 were fed in molar ratio of 2:1 and

[
[

complete reaction had taken place, n2

= ~ n1.

T 1 = 25C = 298 K
T 2 = 50C = 323 K
Combining equations 1 and 2,

V
V -20

n
2
-n
3 I

298
323

- = -1X -

V= 72.1 cm 3

(N. B. Correct substitution of values for any 2 out of 3 variables gives only 1 mark.)

L
[

L
L
L_

(ii)

T /cm3K 1

[
[
T!K
2 (a) (i)

(ii)

0.525
1000
3
39.1+6+96+64 x 250 = 0.0102 mol dm
-+ [W] = 104 24 mol dm3 = ~Ka x c
Given pH =4.24
-+ Ka =(10 4 24lt 0.0102 = 3.25 x 1Cl7 mol dm3

[HCaHs04-]

Initial pH =pH of standard solution used


- pH the solution tends to after titration

=4.24

=pH of aqueous NaOH


=14- (-lg 0.0102) = 12.01

(pH 12 is acceptable)
- correct shape of curve
r--:--d-~----thlfk:- 1
_
1 - tn rca es a mar
~
NaOH HCaHs04
'-----'
- Volume of NaOH required at equivalence point =20.00 cm3
- pH at equivalence point > 7
- At maximum buffering capacity, NaOH added = 10.00 cm 3
- pH =pKa (Acidic buffer is present) =- lg (3.25 X 1o-7 )= 6.49

n
u
[]

f}
[

N.B. Correct sketch (with well-labelled axes) gives 1 mark.

pH

12.01 1--

Equivalence point, pH > 7

Maximum buffering capacity


4.24

(b) (i)

Reason:

I 0.00 20.00

BaS04 (s) ~

Volume of NaOH added I cm3

Ba 2+ (aq) + SO/- (aq) ----- (1)

Fucassium sulphate dissolves in water (in digestive tract) to give So/ ions.
Concentration of S0 42- ions increa_ses and according to Le Chatelier's Principle.
(LCP), position of equilibrium in equation (1) shifts towards the left.
The concentration of Ba 2+ ions is reduced to a safe level when sparingly soluble
BaS04 is formed.

'--

L
L

I
_,

L_

(ii)

~
[

(iii)

Lets mol dm3 be the solubility of BaS04 in water at 25'C.


Ksp
= [Ba 2 +][S04 2 -]
10
1.1x10- =(s)(s)
-+ s
=
1.05x 105 moldm-3
After mixing the solutions,
= 5.00 x 10-3 mol dm-3
[Ba 2 +] = (10 x 0.02) /40
[SO/-]= {30 X 0.001) /40 = 7.50 X 10-4 mol dm-3
Ionic product = [Ba 2+](so/-] = 5.00 x 10-3 x 7.50 x 10-4
=3.75 x 10-6 mof dm-6
Since ionic product > K.Q, the precipitation of BaS04 will occur.

3
Either
(a) (i)
Each correct half cell: 1m
Every error minus 1m
except for temp

potentiometer

[
Cu electrode

salt bridge

[
3

1 moldm HzOz (aq)

[
[

1 mo ld m-3

1 moldm3 H+ (aq)

c u2+ (aq)

Temp: 298K
Pressure : 1 atm (optional)

(ii) E9 cen= E9 red- E9 oxid = + 1. 77 -. 0.34 = + 1.43 V (sign and units)


Overall Equation: H20 2(aq) + 2H+(aq) + Cu(s) ~ 2H 2 0 (I) + Cu 2 + (aq)

[
[
r-,

L
[
L-

L
["

Platinum electrode

(iii) A blue ppt of Cu(OH)z is observed. Emf will increase.


. Cu 2 +will react with OH- to form Cu(OH)z so [Cu 2+] decreases. By Le Chatelier's
Principle, position of equilibrium Cu 2 +(aq) + e ~ Cu (s) will shift to the left to
counteract decrease in [Cu 2+] so Eoxict becomes more negative so emf
becomes more positive.

(b)(i)

Oxygen formed at the anode reacts with graphite electrode to form


carbon dioxide and carbon monoxide.

(ii) volume of C0 2 = 25 -15 = 1Ocm 3


Let th.e volume of CO in the anode gas be x cm 3 .
Volume of 0 2 in anode gas = 15 - x
C (s) + 0 2 {g) --4 COz(g);
2C(s) + Oz(g) --4 2CO(g)
Initial volume of 0 2 before reacting with graphite= 1/2 x 40 = 20 cm 3
Vol of 0 2 reacted with C to form C0 2 = 10 cm 3

Vol

~f 0 2 reacted with C to form CO = ~


2

[
cm

- + 10 + 15 -X =20
2
X

25--

=20

2
x = 10 (= vol of CO)
vol of 0 2

=15 -10 =5 cm3

3 Or
(a)(i) Metal M is the anode.

E9 cen= E 9 red- E 9 oxid =+ 1.33- E 9 0Xid =+ 1.77V


E9 oxid =-0.44V
(for Fe2+(aq} + e F
Fe(s})

D
-J

M is Fe.

(ii) [H+] will increase. By le Chatelier's Principle, position of equilibrium


Crzol(aq) + 14H+(aq) + 6e- F
2Cr+(aq) + 7H 20(1) will shift right to
counteract decrease in [Hj so Ered becomes more positive and
So Ecen will be more positive.

(iii) Fe will be oxidized to Fe2+ (green) and Cr2 ol will be reduced to Cr3+ (green).
There is a colour change of solution from orange to green
(optional: Hydrogen gas evolved and Fe will become smaller.)

(b)

relative atomic mass ofY = (1/3 x 69) + (2/3 x 70)


2.5 X 50 X 60
7500 C
Q It
no of mole of electron 7500 + 96500 0.07772

= =

=
=
=

I 81
no of mol of Y deposited = -- =0.0260

[
[

=69.7

69.7

yx+ + xe ~ y
No mol of electron needed to deposit 1 mole Y

=x =0.07772

..

r-

r-'-

.....__

Aluminium anode

......_

Anode: 2H2 0 (I)


4AI (s) + 30 2 (g)

~ 0
~

'

1m: correct choice of


electrode and AI as anode
1m: correct electrolyte

.-'-

=3

(c)

Graphit e
cathode

..;-0.0260

Dilute H2S04 or Na2S0 4(aq)

L
L
L

(g) + 4H+ (aq) + 4e-

2Aiz03 (s)

l
l

l:
L
[

Section B
4 (a)
Thermal stability of Group II carbonates increases down the group.
(Ease of thermal decomposition decreases down the group)

Down the group, the size of the cation increases and thus the charge density of
the cations decreases.

The polarizing power of the cation decreases. When the anions are less
polarized, the electron clouds of the anion is not as distorted, hence they are not
so easily decomposed to the oxide ion and carbon dioxide.

MC03(s) ~ MO(s) + COz(g)

(b) (i )
For transition metals, in the presence of ligands the five 3d orbitals are no longer
degenerate and they split into two energy levels.
When d-d transition of electrons takes place, radiation in the visible region of the
electromagnetic spectrum corresponding to ilE is absorbed. The light energy not
absorbed will be seen as the colour of the complex.

[
[

(ii)

E9 (V3+/ V 2+)
= -0.26 V
E9 (0 2/ H2 0) =+1.23 V
Ecen 9 +1.23- (-0.26) +1.49 V > 0
Thus, V 2+ is oxidised by oxygen in the air to V 3+ (green)
4V2+ (aq) + 0 2 (g) + 4H+(aq) -+ 4V3 +(aq) + 2H 20 (I)
(-1 overall for no state symbols given)

[
[

E9 (V3+I V 2+} = -0.26 V


E9 (Zn 2+/ Zn) =-0.76V
Ecen9 -0.26- (-0.76) +0.50 V > 0
Thus, Zn is oxidised to Zn 2+ and V3+ is reduced to V 2+ (violet).
Zn(s) + 2V3+ (aq) -+ Zn 2+(aq) + 2V2+ (aq)

Either (a)

NaCI exists as giant ionic lattice with strong ionic bonds between the ions. Large
amount of energy of energy is required to overcome the strong ionic bonds and
hence, a high melting point.

SiCI4 and PC15 exist as simple molecular structures. Melting involves overcoming
the weak Van der Waals forces of attraction between them. Little energy is
required to overcome the intermolecular forces of attraction and thus, they have
lower melting points as compared to NaCI.

PCI5 has a larger molecular size compared to SiCI4 and thus the intermolecular
forces in PCI5 are more extensive compared to that of SiCI 4 and thus PCI 5 has a
higher melting point..

[
[

L
l
L
l

J__ --- ---

n
[

(b).

Compound

HF

HCI

HBr

HI

Bond Energy
/kJmor1

+562

+431

+366

+299

[I

2HX --+ H2 + X2
Ease of decomposition of halogen hali(;fe increases down the group
(or thermal stability decreases down the group)
Down the group, as the atomic radii of halogens increase, the electrostatic
attraction of nucleus for the shared pair of electron decreases resulting in a
decrease in H-X bond energy.

OR

(c)

Down Group Vlrthe size of the halogen increases. Thus the bond length in the
hydrogen halides increases, bond strength decreases and H-X is more easily
broken.
No. of moles of ammonia

= no. of moles of HCI reacted


= (70 -42.90) X 10-3x 0.100 = 2.71

n
n

u
D

10-3

Z3N2= 2NH3
No. of moles of Z3N2 = 312 (2. 71 x 1o-3) = 1.36 x 1o-3

MrofZ3N2= 0.20/1.36 x 10-3


3(Ar Z) + 2 (14) = 0.20/1.36 x 10-3
Ar Z = 39.7 Hence, Z is Calcium.

[
[

5 or {a)

(b)

MgO is an ionic oxide and has a giant ionic lattice consisting of ions held by
strong ionic bonds. It dissolves sparingly in water due to its relatively high lattice
energy.
0 2- (aq) reacts with water to give a strongly alkaline solution (pH- 9).
[Acceptable range for pH = 8 - 10] 0 2- + H20 ---+ 20HAb03 is an ionic oxideWith very high lattice energy.
It is insoluble in water and does not react with water.(pH- 7)
P40 10 is an acidic oxide with simple molecular structure and dissolves readily in
water to produce an acidic solution.{pH -.2). {Acceptable ,range for pH = 2 - 4]
The reaction with water is due to the-energetically accessible 3d orbitals for
dative bonding with water.
P401o (s) + 6H20(I) ~ 4H3P04 (aq)
I mark for all pH values given correctly.
Step 1
2Co2+ + S20a2- ---+ 2SO/- + 2Co3+
Ece119 = + 0.19V
3
2
Step 2
2Co + + 2r ---+ 2Co + + b
Ecen9 = + 1.28V
2
Co + acts as a homogeneous catalyst, increasing the rate of reaction by providing
a pathway of lower activation energy.
Reaction between oppositely charged ions is highly favoured due to electrostatic
attraction unlike the re'action between r and S2ol- which is slow due to repulsion
between the negatively charged species.

L
[
r-

t
l
l
t:

r
[

Section

(a)

0~

(i)

tNH

(iii)

~-'P

HO~

NOz

HOCH 2~~
(iv)

-Na+

0
(b)

H
CH3

(i)

I
I

-CH-CH2-CH2-C-

CH3
(ii)

or any other possible addition


polymer of the 2 monomers

C02H

An increase in the proposed ratio gives rise to an increase


in the number of hydrogen bonds per unit length, thus
increasing the strength of attraction between polymer chains.
CH3

-CH-CH 2 -CH 2 - C -

CH3

o+r,l0-C=O

- - - hydrogen bonding

; 0-

(show partial charges)


HO--c=b

o-

-cH-CH2-cH2-c-

CH3

HO

(ii)

,....

(iii)

CH3

Indicate that the addition of sodium hydroxide forms the polymer


CH

shown below:
3

(-rH-CH,-CH,-t- )
CH 3

L
L
l
L

o=C-0-Na+ n

Or Mention that ionic bondi.ng now exist between the polymer


chains as NaOH will react with carboxyl group to form sodium salt
Since ionic bonding is stronger than hydrogen bonding, the
new polymer is stronger.

\_j

n
(a) (i)

_7

H~C02H

H~COCI

(ii)
D

PCis

Ho--@-cH2CI

KCN
alcohol
reflux

-0
~

H'"'u

UHCH2CN

~
ether

LiAI

room temp

[1]

(iv)

[1]

H,~CH,

9<
CH3

[
[

o,~No,
<

9
N02

[
L

L
[

either
J has a high C:H ratio, J contains a benzene ring.
K turns red litmus blue and reacts with bromine water at room temperature to give a
white precipitate (electrophilic substitution). K is a phenylamine.
J is neutral and undergoes alkaline hydrolysis to give K and sodium salt. J is an
amide.
M reacts with aqueous alkaline iodine to give a yellow precipitate. M contains
OH group or M is ethanol.

CH-C3

-CH 3 groups are electron-releasing. It reduces the delocalization of negative


charge of 0 into the benzene ring, thus decreasing the stability of phenoxide ion.
Hence 2,4,6- trimethylphenol is Jess acidic than phenol.
-N02 groups are electron-withdrawing. U increases the delocalization of
negative charge of 0 into the benzene ring, thus increasing the stability of
phenoxide ion. Hence 2,4,6 -trinitrophenol is more acidic than phenol.
[3]
8

[3]

(iii) amide and phenol

H~E-o-Q-cHzCHzNH2

fl

Ho--{5\-cH2CH 2NH;)

[1 mark for each step, total3 marks]

(b) order of acidity:

\:
fl

[2]

,.,I

.l
l
l
l
f

{.

r
r~

NMR spectrum
0

J:

(e)

(b)

H.......__N_.,....., '-...CH3

(d)H

H (d)

(d)H

H (d)

[
[
[
[~

OCH 2 CH3
(c)

(a)

Triplet at 8 1.4 due to -CH3 (Ha) protons adjacent to -CH 2 protons


Singlet at 8 2.1 due to -CH 3 protons (Hb) next to C=O group (no adjacent proton)
Quartet at 8 4.0 due to -CHz (He) protons adjacent to -CH3 (Ha) protons and 0 atom
Multiplet at 8 7.1 due to 4 phenyl (or benzene) protons
Singlet at 8 7.9 due to -NH proton next to benzene ring and C=O group
The peak disappears on addition of 0 20 => presence of labile proton
( 1 mark for each correct structure)

J:

K:

II

H'-...N/ "---cH3

L
r,

OCH2CH3

OCH 2CH3

L:

M:

CH3CHzOH

[
[
[

OCH 2 CH3
Note:
For .compounds J, K, and L, ortho-disubstituted or meta-disubstituted compounds
are also accepted.
[11 points, maximum 10 marks]

L.

L
L
l

L
-~,
8_

or

P has a high C:H ratio, P contains a benzene ring.

Q is formed from the reaction of P with Cb(g), UV light (free radical substitution). Q is
a halogenoalkane.
On addition of bromine water at room temperature, P undergoes electrophilic
substitution to form a white precipiate R. P is a phenol.
1 mole of S reacts with 1 mole of sodium carbonate. S contains 2 -C0 2H groups.

r-
~

Credit is also given for discussing the reactions that have taken place when Q is
refluxed in alcoholic potassium hydroxide or when P is treated with acidified
potassium manganate (VII).

NMR spectrum

(d)

P:

OH

(e)H

H (e)

(e)H

CHjb)

/CH,
H3C

(c)

(a)

CH3
(a)

Doublet at o 1.2 due to -CH 3 (Ha) protons adjacent to -CH protons


Singlet at o2.3 due to -CH3 protons (Hb) next to benzene ring (no adjacent protons)
Multiplet at o 3.1 due to -CH (He) protons next to benzene ring and two groups of -CH 3
protons
_
Multiplet at o 6.9 due to 3 phenyl (or benzene) protons
Singlet at o4.8 due to -OH proton on benzene ring or phenolic proton
The peak disappears on addition of 0 20 => presence of labile proton
(1 mark-for each correct structure)
OH
OH
Q:
or
P:
OH

/CH,
H3C

H3C

CH3

R:

/CH,

S:

OH
Br

CH2CI

OH

CH3
H3 C/

C0 2 H
'CH3

C02 H

10

[
[.

/CI

H3C./ "---cH3

[11 points, maximum 10 marks]

Br

CH

CH3

CH3

CH3

r~

r
L

L
L
L
L
1-

[
[
[
[

[
'

Section D
BIOCHEMISTRY

9 (a) Hydrogen bonding between polar R groups containing -OH or -NH.


Ionic bonding (weak) between oppositely charged groups eg C0 2- and NH 3+
Induced dipole-induced dipole forces of attraction between non-polar R groups
(eg -CH 3 )
Covalent bonding between -SH groups to form S-S.
[1m for any two points or 1m for diagram below, not 2m as type of group must be
identified]

c
[

[
[
[

o Figure 2.14 A summary of the interactions


responsible for maintaining the tertiary structnre
of a polypeptide chain.

(i)

-ve charged R groups form salts or complex ions with the metal ions
eg -cooAg+
eFor disulphide linkages, Ag+ will break the S-S bond
Eg -CHrS-Ag

OR

(ii)

(b) Integral proteins allow water-soluble materials (ions or sugars) to enter the cell
as their channels contain polar groups which form hydrophilic interactions with water"
soluble materials.
Bilayers have hydrophobic groups on interior of the bilayer which prevent the
passage of water and polar solutes.
Some solutes have to be transported against a cone gradient, hence require an
enzyme to catalyse the reaction.
Require 'pump' I carrier pr'otein I involvement of ATP (eg Na/K pump)

addition of H+ breaks ionic bonds by neutralizing basic/alkaline groups)


Eg -coo + H+ -7 -COOH
.

l
[

L
f_.

11

L
[
[

10(a) (i)

H
0
I
II
H-C-O-C-(CH2)?CH=CH(CH2)?CH3

II
H-C-O-C-(CH2)7CH=CH(CH2)?CH3

[,

II
H-C-O-C-(CH2)?CH=CH(CH2)?CH3
I

,(ii)
1 gram of fatty acid releases 39 kJ of energy
No. of moles offatty acid in 1 g: 1/ (18x12+36+2x16) = 1/282
-<. 1 I 284 mol of fatty acid releases 39 kJ of energy
AH (fatty acid)= -39 x282
-11000 kJ mol"1
1 gram of glucose releases 16 kJ of energy
No. of moles of glucose in 1 g: 1/ (6x12+12+6x16) = 1/180
1 I 180 mol of fatty acid releases 16 kJ of energy
{ AH (glucose) = -16 x 180
-2880 kJ mol"1
In order to have a valid comparison between the fatty acid and glucose, we need to
compare the same number of C atoms, ie compare 3 glucose with one fatty acid. This
means that AH (3 glucose) = -2880 x 3 = -8640 kJ mol-1
Glucose has a higher oxygen : carbon ratio compared to the fatty acid. Glucose is more
oxidized than the fatty acid and therefore releases less energy per carbon atom than the
fatty acid ..
[one bonus]
(iii)
CH20H
I

*C-o

II
HO-::-C-:-.(CH2)?CH=GH(CH2)?CH3

7/~

*y
OH
H0\1

\ ;H

C\*
OH
*c-c*
I
I
H

If

structure and
labeling of chiral
centres

OH

Fatty acids {palmitoleicacid) are insoluble in water (hydrophobic) as it contains longchain hydrocarbon groups which are unable to form strong interactions with water,
while glucose is soluble in water (hydrophilic) as it contains many polar -OH groups
that can hydrogen bond with water.
Glucose shows optical activity as it rotates plane polarized light. It contains chiral
centres which are not symmetricaL Fatty acids (~almitoleic acid) are optically inactive as
all of its carbons are symmetrical.
Glucose forms a crystalline solid with a higher melting point than the triglyceride as the
numerous -OH groups on glucose are able to form strong hydrogen bonds. Since

12

[
[

l
t
.

-~

Ao

c
rL

[
[

l-

L
L
L

L
r-

t.

L
[
[
[

r
[
[
[
r"

melting involves breaking these sfrong hydrogen bonds, glucose has a high melting
point.
The fatty acid, on the other hand, has weaker van der Waals forces of attractions which
are easily overcome. Hydrogen bonding takes place only to a small extent with the
carboxyl group.
[any 4 points, one bonus]
(b) Each amino acid residue is coded by a small number of consecutive nucleotides.
More than one nucleotide is required as there are only four bases in DNA but 20
amino acids present in proteins. If groups of three bases in DNA code for one amino
acid, 43 64 different pairs can be generated. Hence, at least three bases must code
for one amino acid.

Each group of three bases is called a codon and specific codons code for
specific amino acids. There are also specific codes for the initiation and termination
of tha coding process. This system of three bases coding for particular amino acids is
called the triplet codon.
The sequence of amino acids in the synthesized protein depends on the sequence
of bases in the mRNA molecule that codes for the protein. In turn, it is the sequence of
bases in the corresponding gene of the DNA molecule that determines the sequence
of bases in the mRNA. The sequence of bases in mRNA is complementary to that in
DNA. Knowing the sequence of DNA and which triplet codes for which amino acid will
enable the primary structure (ie amino acid sequence) of the polypeptide to be known.

-~

.,
l
'

['
[
[

L
[

L
l
{

13

r
c

FOOD CHEMISTRY

11

(i)

Name & structure

[1 mark each]

I
I

R-C -co2H

NH2
Name

1. valine
2.1eucine
3. isoleucine
4. threonine
5.1ysine
6. methionine

R
-CH(CH3)2
CH2CH(CH3)2
-

CH(CH3)(CH2CH3)
CH(OH)CH3
(CH2)4NH2
(CH2)2SCH3

7. phenylalanine

c5
8. tryptophan

(b)

{c)

(d)

The four different interactions are


(i)
Hydrophobic (or van der Waals) e.g: alkyl or aryl group
(ii)
Hydrogen bonding, e.g. -CH20H, -C0 2H, and -NH 2 groups
(iii)
Ionic, e.g. -C02-. -NH3+ and> NH/ groups
(iv)
Disulphide, e.g. -SH or -S-S- groups
Addition of salt e.g. NaCI.
ions from salt will interact with charges on amino acid and affect their ionic
bonds, van der Waals' attraction and dipole-dipole attraction in protein.
- Denaturation occurs when the strand break apart.
Or mechanical agitation
destroy hydrogen bonds and lead to denaturation of protein.
(i)

Fats/lipids/oils and fat soluble vitamins

(ii)
0

II

r
L
[

u
u
u
u

-c-N-

L
14

l.
[
[

(iii}

Hydrolysis of protein

rf

r r

R1

-N-c-c-N-c-c-

1
H

._ H2NCC02H

I
H

+ H2NCC02H

I .

(iv}
Starch

[
[

R2

12

(a} (i}

Maillard reaction is a non-enzymic browning reaction in food. It is


the combination of the reducing sugar with an amino compound.
-NH2
+
-CHO
(from amino acid protein)
(from a reducing sugar)

Number of rearrangement

[
[

(ii)

The crisps become rancid by oxidation to aldehydes and ketones, a


process which catalysed, or initiated, by light.

(iii)

Heat brings about changes in proteins and fats.


Cooking of meat affects its texture (i.e: firming up the meat) and
.removes its raw appearance (i.e: colour change from red to brown) These
~changes are broughtabout by the decomposition of myoglobin,
coagulation I denaturation of protein of meat fibres & .gelatinisation of
collagen.
Heat also melts and oxidizes the fat in meat.

(iv)

The moisture content of certain foods must be carefully controlled during


processing in order to
reduce hydrolytic rancidity from occurring;
inhibit the growth of micro-organism
prevent destruction of colloidal system;
retain the texture.

Brown m1anoidins

c
L
L
[

L
l
t.

15

c
(b)

Apple slices I, Ill, II


The untreated apple, slice I will brown at the normal rate as enzymes
come into contact with oxygen when the apple is cut and polyphenolases
catalyse the oxidation of polyphenol compounds.
Slice Ill will brown slower than I as sugar solution will partially exclude
oxygen, causing oxidation to slow down.
Slice II will brown the slowest as the lemon juice contains citric acid and
ascorbic acid which lower the pH, causing the enzymes to fall out their
working range. Moreover, ascorbic acid act as an antioxidant, reacting with
oxygen before it can take part in other oxidation reactions.

c
[

c
[
[

c
[

r
[

[
16

l
L
l
{

L
[

Name

--------------------------

Class:

Reg Number: _____

Meridian Junior College


JC2 Preliminary Examination 2006
Chemistry 9251

[
[
[

20 September 2006

1 hr

Paper1
Additional Material
Data Booklet
OMR Answer Sheet
INSTRUCTION TO CANDIDATES

L
[
[
C'

Write your name, class and register number in the spaces provided at the top of
this page.
There are forty questions in this section. Answer all questions. For each
question, there are four possible answers labelled A, 8, C and D. Choose the
one you consider correct and record your choice in pencil on the OMR Answer
Sheet.
You are advised to fill in the OMR Answer Sheet as you go along; no additional
time will be given-for tMe transfer of answers once the examination has ended.
Use of OMR Answer Sheet

L
L

L
[

l
1
L

Ensure you have written your name, class register number and class on the
OMR Answer Sheet.
Use a 28 pencil to shade your answers on the OMR sheet; erase any
mistakes cleanly. Multiple shaded answers to a question will not be accepted.
For shading of class register number on the OMR sheet, please follow the
given examples:
If your register number is 1, then shade 01 in the index number column.
If your register number is 21, then shade 21 in the index number column.
This question paper consists of Pages 1 to 17.

r
r~
Section A
For each question, there are four possible answers labelled A, B, C and D.
Choose the one you consider to be correct.

Which one of the following statements concerning N02+ and NO is


incorrect?

A
B
C
0
2

There are more protons than electrons in N0 2+.


Both have the same number of nucleons.
Both have the same relative formula mass.
Both have the same shape.

One method to inflate air bags in cars is to use nitrogen produced


chemically from the decomposition of sodium azide, NaN3.
2NaN3(s)

2Na(s) + 3N2(g)

[
[
r-

['

What is the volume in dm3 of nitrogen generated at 21C and 101 kPa
pressure upon the decomposition of 60.0 g of NaN3.

A
B

D
3

0.022
0.034
22
34

25.0 cm3 of 0.05 mol dm-3 KC/0 4 (aq) required 50.0 cm 3 of 0.20 mol dm-3
TiC/3(aq) to reach end-point. GivenJhat Ti(III) ion .is oxidised to. Ti(IV) Jn
this reaction, which one of the following formulae correctly represents the
ion?.
reduction product of the

czo4-

a
c

oczczo2cr

C/03-

L
[

L
[
[

l
l

l~

L
L
{

[
[

Which of the following pairs of substances have different types of bonding


and structure?

A/F3

[
[
[
[

AsH3

D
5

A/C/3

Diamond
- MgC/2

SiC
Bai2
SiC/3H

The figure below shows the trend of 3rd ionisation energy of successive
elements from Period 2 and Period 3 of the Periodic Table.

3rd IE I kJ mor1

9000
8000

7000
6000

5000

4000
3000 -

u
T

2000
1000
0

[~

Atomic Number

Which Group does element T belong to?

Group II

Group III

Group IV

Group V

L
[

L
L
l

r
L
6

Flask X contains 1 dm 3 of helium at 2 kPa pressure and flask Y contains 2


dm3 of neon at 1 kPa pressure.

If the flasks are connected at constant temperature, what is the final


pressure?

1.33 kPa

1.67 kPa

1.5 kPa
2.0 kPa

The graphs below show the variation of the percentage of gaseous


products present at equlibrium, with temperature and pressure.

[
['

%products
at
equilibrium

...... ___ _
~-~

-~-

- ... __

(T +a) C

------TC
Pressure
Which one of the following systems could the graphs represent?

A
8
C

2N2(g) + 02(g)

CO:i(g) + C(s) F

N2(g) + 3H2(g)

2NH3(g)

302(g) + 4NH3(g)

2N2(g) + 6H20(g)
2N20 (g)

2CO (g)

n
[j

L'lH

=-92 kJ mor1
L'lH =-1248 kJ mor 1

L'lH = +82 kJ mor 1


L'lH = +173 kJ mor1

u
r11

L1

L
l
4

L
L

L
[

L
r~

Solutions P, Q, RandS contain a strong acid, a weak acid, a salt of weak


acid and a strong base, but not necessarily in the same order. The
concentration and pH of each solution are shown below.

Concentration in mol dm-"'

pH

1.0
1.0
0.010
0.001

2.4
9.4
12.0
3.0

Q
R

Which one of the following statements is incorrect?

[
[

Solution

A
B
C
D

-~

P contains a weak acid while S contains a strong acid.


Mixing 10 cm 3 of P and 500 cm 3 of R produces a buffer solution.
Mixing 10 cm 3 of Rand 500 cm 3 of S produces a buffer solution.
Mixing 100 cm 3 of P and 100 cm 3 of Q produces a buffer solution.

The solubility product of copper( II) iodate, Cu(I0 3) 2 is 4 x 1o-9 mol 3 dm-9 at

25C.

[
[',
[

What is the corresponding solubility of copper(II) iodate?

c
o

1.849 x 10-2 g dm-3


4.135x10- 1 gdm-3
5.210x10- 1 gdm-3
6.564 x 1o- 1 g am-3

r~

[,

L
L
L
L
L
L

l-_._

c
[
10

The decomposition of dinitrogen pentoxide, N205 , was found to be first


order with respect to N20s.

[
[

Which one of the following graphs confirms the result?

[
Rate of deromposition

Rate of decomposition
.lt.

ofNzOj

ofNzOs

.lt.

t~

[N 20:~]

[N20s]

r,

Rate of decomposition

Rate of decomposition
.lt.

ofN20~

ofNzO~

[I

"

[j
.....
--..-

----

fN~05}

11

fN20sJ

[1

Given the fellowing information:


1

r,1,

L\Hc (graphite) = -394 kJ mor

L1

L\Ht (water) -286 kJ mor


1
L\Ht (methanol)= -239 kJ mor

'1,

Which one of the following is the correct AHc (methanol) in kJ mor ?

A
8

c
D

-441
-727
-919
-1205

'

L
6

L
f

r
[
12

A
B
C
D

Which treatment is frequently used to protect aluminium articles from


subsequent corrosion?

13

making
dipping
dipping
coating

the
the
the
the

aluminium
aluminium
aluminium
aluminium

the anode during electrolysis


in hot aqueous sodium hydroxide
in molten cryolite
with a less reactive metal

The graph shows how the property of the elements Na to CI varies with
proton number.

property

[
[

[
[
[

~.

--

11

12

13

14

15

What is the property?


A

electro negativity
ionic radius
first ionisation energy
meting point

~~

L
-.,

[
--~

l
L
[

L_,

16

17

proton number

l
l.
[
14

P, Q and R are hydroxy compounds of the elements X, Y and Z


respectively. X, Y and Z are in the same period of the Periodic Table.

P gives an aqueous solution of pH< 7.

Q reacts with both strong acids and alkalis.

R gives an aqueous solution which is strongly alkaline.


Which one of the following statements is true?
A
8
C
0

15

The atomic radius decreases from X to Y to Z.


The electronegativity decreases from X to Y to Z.
The first ionsation energy increases from X to Y to Z.
The atomic number increases from X to Y to Z.

Down Group II, what changes occur in the magnitudes of lattice energy
and enthalpy change of hydration of sulphates?
lattice energy

A
8

c
0

16

decrease
increase
decrease
increase

decrease
decrease
increase
increase

[
[
[

Which of the following is not true of the Group II elements?


A
8
C
0

17

enthalpy change of hydration

They show only one oxidation state in their compounds.


All of them have negative values for their standard electrode
potentials.
Their sulphates(VI) are all insoluble in water.
They are manufactured by electrolysis of their molten compounds.

Brine is electrolysed by using inert electrodes in a cell which is stirred so


that the products of electrolysis are able to react. The cell is kept cold.
Which one of the following pairs. of substances is among .the. final
products?
A
8
C
0

hydrogen and chlorine only


sodium hydroxide and chlorine only
hydrogen and sodium chlorate(!)
hydrogen and sodium chlorate(V}

[
[
[

L
[

l
L
L
l

L
[

18

[
[

19

Which one of the following does not act as a ligand in the formation of
complexes?
A

CN-

NH/

CO

Platinum(IV) chloride combines with ammonia to form a compound which


contains a cation and only two chloride ions.

What is the formula of this compound?

A
8

-,

'

-~

20

Pt(NH3)3C/4
Pt(NH3)4C/4
Pt(NH3)6C/2
Pt(NH3)6C/4

The drug cortisone has the formula as shown below:

[
[

[
[
f

-~

-~

What is the total number of optical isomers present in the cortisone


molecule?
A
8

16
64

256

L
[

l
l
[

L
l_
[
21

Under controlled conditions, CH 3CH 2CH 2CH(CH 2CH 3)z can be cracked to
produce X and CH 3CH 2CH3.

Which one of the following could be X?

CH3CH2CH2CH=CH2
CHaCH2CH=CHCH3
CH3CH=CH(CH2CH3}
CH2=C(CHa)(CH2CH3)

B
C

D
22

22.4 dm 3
24.0dm 3
44.8 dm 3
67.2 dm3

l
[

How many total alkenes (including stereoisomers) are possible for the
product of the following reaction?
CH 3

Heat

H3CCHCH CH2CH2CH3
I
Br

A
B

2
3

24

In the hydrogenation of CH2=CHCH2CN using platinum as a catalyst, the


volume of hydrogen (measured at s.t.p.) that reacts with 1 mol of the
compound could be

A
B
C
23

KOH with alcohol


solvent

L
[

The compound X was heated with alcoholic potassium hydroxide.

H3C

CH

HO

Br

Br

l~'

Which of the following would be the major product?

(yCH
H 3 C~
HO

-CH

H3C

::::::.-._

Br

Br
10

PCH3
WCH3
l
D

H3 C

HO

H3C
HO

Br

OH

OH

r
L

L
L

L
~
[-"
25

The molecule Z below is the substance chiefly responsible for the smell of
ripe raspberries.

[
OH

Z gives an orange precipitate with 2,4-dinitrophenylhydrazine, no


precipitate with Fehling's solution and no reaction with phosphorus
pentachloride. It contains a chiral carbon atom.

The C4H 7 0 group in the molecule Z could be

L
...

---

O=C-CH(CH3)CH3

26

I
D

I -

CH 3-CHCOCH 3

CH 3-C(CH 3)CHO

In the presence of a dilute alkali, some aldehydes and ketones undergo


the "aldo reaction" where they dimerise to form a hydroxycarbonyl
compound (an aldo). For example, ethanal dimerises in this way to form
3-hydroxybutanal.

CH 2 =C-CH(OH)CH3

2 CH3CHO

OH- (aq)

ethan aI

CH 3CHCH 2 CHO

OH

3-hydroxybutanal
Which of the following earbonyl compounds will undergo the aldo reaction
to produce the aldol shown below?

CH 3

CH 3CCH 2C===O

--c

OH

CH3

CH3COCH3
(CH3)2CHCHO
CH3CH2CHO
CH3CH2COCH3

L
L
l

11

[
[
[
27

X, Y and Z are three organic compounds. X gives a yellow precipitate with


alkaline iodine. It undergoes oxidation with acidified potassium
manganate(VII) to give Y and Z. Y gives a yellow precipitate with 2,4dinitrophenylhydrazine but gives negative results with Tollen's reagent. Z
reacts with sodium carbonate to give strong effervescence of carbon
dioxide.
Which one of the following organic compounds could be X, Y and Z?

A
8

28

CH3CH(OH)CH=CHCHa
CH3CH(OH)C(CHa)=CHCHa
CH3CH(OH)C(CH3)=CHCH3
CH3COC(CHa)=C(CHa)2

CHaCOCHO
CHaCH2COCHO
CH3COCOCH3
CH3COCOCH3

CHaC02H
CHaC02H
CHaC02H
CHaCOCH3

In which of the following sequences does the value of pKa decrease


continuously?
A
8
C
D

29

CHaC02H
CC/3C02H
C2HsOH
C6HsOH

> CC/aC02H
> CHaC02H
> C6HsOH
> C2HsOH

> C2HsOH
> C2HsOH
> CH3C02H
> CHaC02H

>
>
>
>

C6HsOH
C2HsOH
CC/3C02H
CC/3C02H

The base strengths of ammonia and ethylamine are different because of


the electron-releasing properties of the ethyl group.
Which one of the following pairs correctly shows the effect of the ethyl
group?
effect on strength of C2HsNH2
as a base relative to NH3

A
8

effect on strength of C2HsNH3+


as an acid relative to NH/

. increase
decrease
decrease
increase

increase
increase
decrease
decrease

[
[
[

[
[
[

[
[
[

[
[
I.
L

12

l
L

L
,~

[
[

30

The amino acids glutamine and glutamic acid can react with each other to
form amide linkages.

040H

HO~OH
NH 2

glutamine

glutamic acid

What is the maximum number of different compounds, each containing


one amide linkage that can be formed from one molecule of glutamine and
one molecule of glutamic acid?
A

[
[

[
['..
[
[

[
[~

[
[

l
L
l'

NH2

13

l
l
[-

f'

Section 8
For each of the next 10 questions, one or more of the three numbered
statements 1 to 3 may be correct.
Decide whether each of the statements is or is not correct (you may find it helpful
to put a tick against the statements which you consider to be correct}.

[
[

The responses A to D should be selected on the basis of

A
1, 2 and 3 are
correct

8
1 ahd 2 only are
correct

2 and 3 only are


correct

The energy profile for a reaction, P

R, is shown below.

1 only is correct '

No other combination of statements is used as a correct response.

31

I
I

[
[

Energy

[
~~

L
p

-----i_:l~----~

[
[

Progress of reaction
From the diagram, it can be deduced that

2
3

the mechanism for the reaction is


P~Q
slow
Q~R
fast
the addition of a catalyst changes the value of E2
increasing the temperature decreases the value of E1

14

l'
l

L
[
\1,

l.
[
[

1, 2 and 3 are
correct

1 and 2 only are


correct

2 and 3 only are


correct

1 only is correct

L _ _ ----

32

2
3
33

Consider the following changes:

The second ionisation energy of M could be calculated from the energy


values associated with

r,
[

There are significant attractive forces between the gas molecules.


The volume of gas molecules is significant.
The gas undergoes dissociation.

ilH1: M(s) --4 M(g)


ilH2: M(s) ~ M2+(g) + 2e
ilH 3: M(g) --4 M+(g) + e
L1H4: M+(g) --4 M2+(g) + e
ilHs: M(g) ~ M2+(g) + 2e

When a sample of gas is compressed at constant temperature from 15 to


60 atm, its volume changes from 76.0 cm 3 to 20.5 cm 3 .
Which statements are possible explanations for this behaviour?

[
[

-----

2
3
34

ilH1 + ilHs
ilHs- ilH3
ilH2- L1H1- ilH3

One type of rechargeable battery makes use of the nickel-cadmium cell, in


which nickel and cadmium electrodes, coated with their respective
hydroxides, are immersed in potassium hydroxide solution. In normal use,
the cadmium electrode is the one that releases electrons to the external
circuit.
The relevant electrode reactions are:

L
[

l
l~

L
[

Cd(OH)2 (s) + 2e

~ Cd (s) + 20H- (aq)

NiO(OH) (s) + H20 (I) + e ~ Ni(OH)2 (s) + OH- (aq)


Which of the following statement(s) is/are likely to be true?

1
2
3

The mass of the cadmium electrode decreases as cadmium is used


up.
Cadmium is the negative electrode.
The concentration of the potassium hydroxide solution decreases.

15

L
f_

[
A

1, 2 and 3 are
correct

35

B
1 and 2 only are
correct

2 and 3 only are


correct

1 only is correct

The Group II metals have higher melting points than the Group I metals.
Which of the following factors could contribute towards the higher melting
points?
1
2

36

There are smaller interatomic distances in metallic lattices of Group


II metals.
Two valence electrons are available from each Group II metal atom
for bonding the atom in the lattice structure.
Group II metals have higher first ionisation energies.

The conversion of cral-(aq) into Cr20/-(aq} is represented by the


following equation:
Cro/-(aq) + 2H+(aq)

Cr20/-(aq) + H20(I)

L
[
[
',

[-
[

Which statements are true of this reaction?


1
2
3

37

Which of the following about methylbenzene and its derivatives are true?
1
2
3

38

Crol-(aq) acts as a base.


Addition of OH-(aq) to Cr2o/-(aq) causes a colour change.
The conversion Cro/-(aq) into Cr2o/-(aq) involves a change in
oxidation state.

Methybenzene will decolourise warm potassium manganate(VII)


solution.
There are four aromatic isomers of molecular formula C7 H7 Cl.
The molecule of methylbenzene is planar.

Which compounds can be prepared from C5HsCHBrCH3 by the action of


sodium hydroxide under different conditions?
1
2
3

C5HsC02-Na+
C6HsCH(OH)CH 3
C5HsCH=CH2

[
[
[

['

L
l
16

l
[
[
[

A
1, 2 and 3 are
correct

[
39

B
1 and 2 only are
correct

2 and 3 only are


correct

1 only is correct

I
I

Terfenadine alleviates seasickness and asthma in the same way as the


older drugs, but it does not cause drowsiness as a side effect.

c~
D-r~N-CH2CH.CH2?Ho-f-CH3

OH

OH

OH

Terfenadine

-,

What deductions about Terfenadine can be made from this structure?

"0

-,
r

3
40

One mole of Terfenadine will give out one mole of hydrogen on


reaction with two moles of sodium.
It can be oxidized to form a product which does not react with hot
Tollen's reagent.
It is soluble in aqueous sodium hydroxide.

Anthraquinoid dyes are of enormous technological importance, and much


work has been done in devising syntheses of large ring systems
embodying the quinone structure. The structure of lndanthrone, an
anthraquinoid dye is given below:
0

[
[

lndanthrone

L
l
l
L
L
[

Which of the following statements is true of lndanthrone?


1
2
3

It is able to form intramolecular hydrogen bonds.


It is capable of exhibiting stereoisomerism.
It gives a brick red precipitate when warmed with Fehling's solution.
THE END

17

-,

r-',

c----1

r--1
'
.I

-;-1

r---1
l

-)

L
[
[
[

Name------------

Class: _ __

Reg Number: _ _

Meridian Junior College


JC2 Preliminary Examination 2006
Chemistry 9251

b
[

[
[
[

[
[

1 hr 30 min

15 September 2006
Paper2
Additional Material
Data Booklet

INSTRUCTION TO CANDIDATESWrite your name, class and register number in the spaces provided at the top of this
page.
Answer all questions in this paper.

Write your answers in the spaces provided on the question paper.

INFORMATION .FOR CANDIDATES

[
[

The number of marks is given in brackets [


question.

] at the end of each question or part

You are reminded the need for good English and clear presentation in your answers.
For Examiner's Use

.5

l
l'
l'
[

This question paper consists of pages 1 to 18.

Total

Grade

L
[
[
1

The kinetics of the reaction of peroxodisulphate ions and iodide ions

S20/- (aq) + 2r (aq} ~ 2SO/- (aq) + I2 (aq)


can be investigated experimentally by varying the concentrations of the two
reactants involved and determining the time taken for the brown iodine colour to
first appear. Water is added to each reaction mixture to ensure that the total
volume is kept constant such that for each of the experiments, the concentration
of each of the reactant is directly proportional to its volume used.

(a)

[
[

. The following results were obtained in such an investigation:


Expt

Vol of S208 2 "/cm 3

Vol of r /cm 3

Vol of water /cm 3

Time/s

10
20
10

20
20
30

20
10
10

55.5

2
3

[
[

27.9
37.0

Define the term rate of a chemical reaction.

[
[1]
(b)

Deduce the order of reaction with respect to S2 0 82- and


reasoning.

r.

Explain your

[
[
[
[
[
[3]

L
l
L
r

r
[

(c)

Write the rate equation


peroxodisulphate(VI) ions .

for

the

reaction

between

iodide

ions

and

~
bJ

r
[

[1)
(d)

The reaction between the peroxodisulphate ions and iodide ions can be
increased by the addition of small amounts of homogeneous catalyst, Fe3 +.
(i)

Explain, using the concept of activation energy, how the addition of a small
amount of catalyst can cause a large increase in the rate of reaction.

[
[

[
[

(ii) Propose a mechanism for the catalysis of the above reaction using Fe3 +.

[
[
[

l
L
L
l~

[4]
[Total: 9]

l-

I.
[
2(a)

The compound furan, (CH)4 0, is unsaturated and has a structure in which the
four carbon atoms and one oxygen atom are connected together in a ring. It is a
liquid at room temperature with a boiling point of 31C. It is used to make
solvents and nylon. The displayed formula of furan is as follows:

[
I

1::~

H-C-C-H

II

II

H-C

C-H

\I
0
(i)

By constructing an appropriate energy cycle, calculate the enthalpy of the


following reaction of furan at 298K :
(CH) 4 0 (I)

4C (g) + 4H (g) + 0 (g)

[
[
[

Use the following given enthalpy values:

L1Ht[(CH)40] = -62 kJ mor1


1
ilHat (C)
+717 kJ mor
1
ilHat (H) = +218 kJ mor
ilHat (0)
+250 kJ mor- 1

r
[

L
l
L
4

-L

u
I

I,.
1

L.
~

[
(ii)

It is also possible to calculate the value of the enthalpy change for the
reaction above by using appropriate bond energy values from the Data
Booklet. The value calculated using this method differs from that calculated
by using the Hess Law approach in part (ai).
By considering the bonding- in furan and the definition of bond energy,
suggest one reason for the difference in enthalpy values.

~~

[
(iii) Liquid furan is transported by road tankers. Briefly explain why an explosion
may happen during transportation if precautions are not taken.

[
[
[:
__ .o

[
[

[5]

(b)

lron(II) oxide is a black-coloured oxide used for glazing.


(i) . What do you understand by the term lattice energy of an ionic compound?

(ii)

Construct and label carefully a Born-Haber cycle for the formation of iron(II)
oxide from its elements.
Use the cycle to calculate the lattice energy of iron(II) oxide.

C_

[.

l
L

L
t

Incorporate the relevant data from the table below as well as relevant data
from the Data Booklet.
Enthalpy term
Standard enthalpy change of formation of FeO
Standard enthalpy chang_e of atomisation of Fe
First electron_ affinity of 0
Second electron affinity of 0

AH 1 kJ mor1
-278
+416
-141
+790

[:
[

f
[

t-

L
[
[

r
\{_,

[,

(
[
--'

(iii) Explain qualitatively howyou would expect the -numerical magnitude of the
lattice energy, and hence the relative stability, of iron(II) oxide to compare
with that of iron(II} sulphide, FeS.

L
[
,-

[7]
[Total: 12]

l
L
u
r

'L

L
~

[
3

[
[

Industrialisation and urbanisation has significantly increased the demand for


electricity. A large part of this demand is met through the combustion of oil and
coal. Coal-fired power stations emit flue gases, which contain the oxides of
nitrogen, carbon and sulphur. In order to reduce sulphur dioxide emission, such a
power station is fitted with a flue gas desulphurisation (FGD) plant. Three stages
are described below.

[,
oalcl<nn
carbonate

,..

,
coal

[.

..

'-v---'
Stage I

S01 scrubber

Stage II

Stage Ill

Stage II

Lime (calcium oxide) reacts with sulphur dioxide to produce calcium


sulphite (CaS03).

Stage III

Any remaining sulphur dioxide is removed through a scrubber. This


involves passing the gas through an aqueous lime (calcium oxide)
suspension to form calcium sulphite.

(a) (i) Suggest balanced chemical equations for the reactions in Stage I and II.

(ii)

+ so} calciu~
sulphite
. L____j

Powdered limestone (calcium carbonate) is injected into the


furnace where it thermally decomposes to give solid lime (calcium
oxide) and carbon dioxide.

[I

Limo

Stage I

'L

...~ '\

\At ..

air

Flue\

Coal-fired furnace with FGD plant

The average sulphur content in coal is typically 1.5% by mass. Calculate the
:mass of limestone that is needed to remove the sulphur dioxide emitted
when 1 million tonnes of coal is used. [1 tonne= 1000 kg]

[
~

[4]

"'

L
L
[

l-_,

r_

r
(b)

The graph below represents the change in mass that occurs when 100 g of
limestone, is heated at a temperature, T.

Sketch on the same axis, the graph that would be obtained by heating 100 g of
magnesium car~onate, MgC03 , at T. Explain your answer.

[
r~
L

Mass I g

100
50

~--------------------------------

~--------------------+- Time

rL

c
c
[\
[3]
(c)

Based on your knowledge of periodiciey, .suggest why both of the metal oxides
produced would effectively remove sulphur dioxide.

n
u

[1]
[Total: 8]

L1

L
L
L

t.

l.
[
[

Copper is a transition element. Copper was named after the island of Cyprus
which was once rich in copper ores.

(a)

Write the electronic configuration of copper atom and Cu2+ ion.


Cu2+ ion:

copper atom:

[1]

When concentrated hydrochloric acid and aqueous ammonia are added in


excess separately to copper(II) sulphate solution, [CuCI 4] 2- complex and
[Cu(NH3)4(H 20)2]2+ complex are formed respectively. Both the complexes are
coloured.

(i)

(b)

[Cu(NH3)4(H20)2] 2+

[CuCI4f:

[
[

State the colour of both complexes.

(ii)

Explain why transition element complexes are coloured.

[
[

c
I'
[
[4]

L
l
l"

r
[
(c)

The numerical value for the stability constant for [CuCI4] 2 - complex is 5.6, and
that for (Cu(NH 3)4(H 20)2]2+ complex is 13.1.
Using the concept of stability constant, explain what will happen if ammonia
added in excess to [CuCI4 solution.

is.

[
[

[
[

r. . .,
..

~'

[
[3]
(d)

When concentrated hydrochloric acid is added in excess to copper(I) sulphate


.. solution, [CuCI4]3- .complexis.formed.
Suggest the colour of [CuCI 4]3- and explain your answer.

u
L
'

r~

tl

u
[2]
[Total: 10]

10

L
L
L
rL

r,

'

5(a)

The following represents part of a synthetic pathway to obtain the amino acid,
phenylalanine, and one of its other derivatives, D.

Br

CH2 C-H

CH 2 CHCN

f~

Step2

Step 1

[
8

Sfup3

NH2

NH2

CH2CHC-OH

CH2CHCN

II

NHCH2 CH 3
CH2 CHCN

Step5

Step4
~

phenylalanine

,[

(i)

Suggest suitable reagents and conditions for the following conversions:

5Mp1 : _____________________________________

Step 2 : - - - - - - - - - - - - - - - - - - -

5Mp3: _ _ _ _ _ _ _ _ _ _ _ _ _ _ _ _ _ _ ____

[
L
_,

L
L
[

1l

c:

r
[
(ii)

[,
[,

State and describe the mechanism for the conversion in Step 1.

[
[
,~

L
L
[
(iii)

It was observed that a solution containing compound A did not display any
optical activity. Explain this observation.

[
[

r
L
L
[7]

12

L
L
L
r

L
[

r
[

(b)

State and explain the relative basicity of compounds C and D.

[
1~

[
[

c
[
[~

['

L
[

[3]
(c)

Amino acids can be separated by the well-known technique known as


electrophoresis. This technique involves placing a solution of different amino
acids at the centre of a strip of gel moistened with an aqueous solution of a
known pH. An electric potential is then applied through electrodes which are
connected to the ends of the gel strip.
The following information about phenylalanine and 2 other amino acids are given
as follows:
Amino Acid

Structure

Phenylalanine

H
HN-C-COOH
CH 2(C6H5 )
H
H2 N-C-COOH

Glutamic Acid

Lysine

H
H2N-C-COOH

---

I- ------

CH2CH2CH2CH2NH2

L
'--~

5.50

3.22

CH 2CH 2 COOH

L
r
L

lsoelectric Point

[
:('

".

l3

9.74

[:
[_

An electrophoresis experiment is run on a solution containing the 3 amino acids


mentioned above.

-ve

t--

r-

r-

+ve

'-

'-

[
[

Gel strip at pH 5.50


Draw the structural formulae of the species found at positions X, Y and Z at the
end of each experiment.
Compound X:

L
[

r,
-~

Compound Y:

[
[

'f\':' ~--

CompoundZ:

[3]
[Total: 13]

14

c
[

L
L
~-

c
c
c

Many of the foam materials and packing used in everyday life are made up of
polymers. Some of these common polymers are polystyrene and polyurethanes,
and these can be found in the form of common household names such as
Styrofoam and Lycra respectively.
The monomer that can be used to make polystyrene is styrene.

~>

[
,_,
f

'~

styrene

(a)

(i)

Draw a section of the polystyrene chain, showing at least two repeat units.

(ii)

What type of polymerisation occurs when polystyrene is formed?

[
[

r
[

c
[

L
[2]

L
L

15

r
[
On its own, the polystyrene formed is too flexible to be of use since it lacks
strength and rigidity. As such, they are often strengthened and stiffened by copolymerisation with other monomers such as 1,4-divinylbenzene.

[
1,4-divinylbenzene

~
(i)

[
[

0
(b)

Draw a section of the co-polymer formed if each repeat unit consists of one
styrene and one 1,4-divinylbenzene molecule.

[
[
[
['
[

(ii)

Suggest how the addition of the new monomer, 1,4-divinylbenzene helps to


form a polymer with better strength and rigidity.

L
C'

[
[2]

16

L
L

L
[

r
c

On the other hand, polyurethanes are the most versatile family of polymers. One
possible polyurethane is formed from the polymerisation of 2 monomers :

O=C=N

N=C=O

and

HO-CH2CH2CH2CH 2-0H

The repeat unit of this polyurethane has the structure below :

II

II

-O-C-N
H

[
(c)

N--c--0-CH2CH2CH2CH2H

(i)

Deduce the type of polymerisation in the formation of the polyurethane


above.

{ii)

The diol used as the monomer can be replaced by a diamine such as ethyl1,2-diamine, H2NCH2CH2NH2.

[
,[

L
L

Draw the repeat unit of the polymer formed when ethyl-1 ,2-diamine was
used.

L
L

L
L

17

r:~

r
(iii) With the aid of a balanced equation (involving one repeat unit of the
polyurethane), suggest why polyurethanes are not used for packaging of
fruit juices.

[
[
[

[
[
[4]
The End

[Total: 8]

c
L
[

c
r~

u
n
Ll

u
L
18

L
L
J'

L
r_

Name------------

[
[

13 September 2006

Paper3

[
[
[
,,

-"

L
[_

Reg Number: _ _

Meridian Junior College


JC2 Preliminary Examination 2006
Chemistry 9251

Class: _ __

2 hr 45 min

Additional Materials
Data Booklet
Writing paper
INSTRUCTION TO CANDIDATES
Write your name, class and register number in the spaces provided at the top of this
page.
Answer three questions in Section A (Q3 is an Either I Or foim)
two questions in Section B (Q5 is an Either I Or form)
three questions in Section C (Q8 is an Either I Or form)
two questions in Section D
Begin each question on a fresh page of writing paper.
Fasten the writing papers behind the given Cover Page for Section A & B and C & D
respectively.
Hand in Section A & B and C & D separately.
You are advised to spend about 15 min per question.
INFORMATION FOR CANDIDATES

L,
l

L
L
[

The number of marks is given in brackets [


question.

] at the end of each question or part

You are reminded the need for good English and clear presentation in your answers.
This question paper consists of Page 1 to 16.

l-,
'.I

r
[

Section A
Attempt all questions in this section.

Begin the question on a fresh page of writing paper.

Cyanamide, NH2CN, is used for organic synthesis in a stable commercial form.

(a)

Cyanamide is formed from the acidification of calcium cyanamide, CaNCN.

(b)

[~

(i)

State the type of structure for cyanamide and calcium cyanamide.

(ii)

Draw dot-and-cross diagrams (involving only outer shells) to illustrate the


electron arrangement in cyanamide and calcium cyanamide. Predict, with
reasoning, the shape of cyanamide.

(iii)

Suggest, with brief reasoning, the solubility of cyanamide and calcium


cyanamide in water.
[6]

[
rL
t~

Cyanamide is capable of forming an addition product with boron trifluoride, BF3.


(i)

State and describe the chemical bond that is formed during the reaction.

(ii)

Draw a diagram to illustrate the likely shape of the addition product.

C'

[
[2]

(c)

Cyanamide may undergo dimerisation in the gaseous phase as given below.


2NH 2 CN(g) --- H2 NCNHCN(g)

II

NH
(i)

Calculate the enthalpy change of the reaction with the use of appropriate
bond energy data from the Data Booklet.

(ii)

State whether the reaction is expected to be spontaneous.

[2]

[
[

[Total: 10]

L
L
L
L

c
[

2(a)

A cell is constructed with zinc and carbon electrodes, each weighing 40 g, partly
immersed in aqueous KOH. Oxygen is supplied in the half-cell containing the
carbon electrode by bubbling air into it.

[
[

aq.KOH

aq. KOH

When connected in a circuit, some zinc passes into the solution as zincate(II)
ions, Znol (aq).

'.

__,

(i)

Write the equation for the reaction occurring in each half-cell, and hence
write a balanced equation for the overall reaction which takes place.

(ii)

The standard cell potential was found to be +1.62 V.


Use the Data Booklet to calculate the standard electrode potential of the
Znol-(aq) I Zn(s) system.

[
(iii)

.Calculate the quantity of electricity passing .through the cell and the length
of time the cell can last before replacement becomes necessary.

L/

L
[
[

L
l

L
fL

The cell delivers a current of 13.7 rnA. The zinc electrode is replaced once
. 60% of it is used up .

[7]
_(b)

The E9 values concerning chloric(I) acid, HOC/, and the anion chlorate(I), C/0-,
are given as follows:
9
E cro 1cr = + 0.89 V
9
E HOCIJ c12 = + 1.64 V

(i)

Suggest which of the two species, HOC/ or C/0-, is a stronger oxidising


agent.

(ii)

Predict, with the use of relevant E9 data, what happens when chloric(I)
acid is mixed with tin(II) ions under acidic conditions. Write a balanced
equation for any reaction that occurs.
[3]

[Total: 10]

L
f
'

either

(a)

The primary fermentation of grape juice produces wine which typically contains
tartaric acid (represented by RC0 2H) and ethanol. On keeping for some time, the
wine develops a fruity taste as the following equilibrium is established between
tartaric acid and ethanol.

RC02H (I) + C2H5 0H (I) ~ RC02C2Hs (I) + H20 (I)


In a typical reaction, 5 moles of tartaric acid react with 2 moles of ethanol in a 5
dm3 vessel at 300 K. Calculate the percentage dissociation of tartaric acid, given
that the equilibrium constant at 300 K is 2.0.

[3]
(b)

Grape juices contained a cynadin compound, CyH+ that accounts for its purplishred colour. The addition of small quantities of sulphur dioxide reacts with CyH+ to
form a colourless solution.
CyH+ (aq) + S02 (g)+ H20 (I) ~ CyS03H2 (aq) + H+ (aq)

----- (1)

L
[

(i)

Write an expression for the equilibrium constant, Kc for reaction (1 ).

(ii)

In an experiment, when sufficient sulphur dioxide was added to a sample of


2
grape juice to give an equilibrium [S02] of 2.5 x 1o- mol dm-3, the intensity
of the purplish-red colour decreased to one-fifth of its original value.
Given that the magnitude of the equilibrium constant for reaction (1) is 5.0 at
500 K, calculate the pH of the solution at equilibrium.

(iii) Predict whether the decolourisation _of the preserved grape juice will be
-more or less severe' at pH ''13 compared to pH 10. Explain your answer.

[7]
(Total: 10]

[
[

L
[

L
l
l
L

L.
[

f
[

Methanoic acid, HCOOH, occurs naturally in ants and used to be prepared by


distilling them. This gave rise to the earlier name for methanoic acid- formic
acid - after the Latin word formica for ants.

[
[
[

or

(a)

An aqueous solution of methanoic acid of concentration 1.00 mol dm-3 has a pH


of 1.9. Calculate the acid dissociation constant, Ka. for methanoic acid.

[2]
(b)

10.0 cm 3 of 1.00 mol dm-3 of aqueous methanoic acid is titrated against with
20.00 cm 3 aqueous sodium hydroxide of the same concentration.
(i)

Sketch the expected acid-base titration graph. Include in your sketch:

1
2
3

the initial pH
the end-point
the region of buffering capacity
the pKa of methanoic acid

(ii)

State whether the solution at the equivalence point of the titration is acidic,
basic or neutral. Explain your answer.

(iii)

In a similar titration using 1.00 mol dm-3 solutions of hydrochloric acid and
sodium hydroxide, the temperature rise was higher than that of when
methanoic acid was used.

[
[

Explain the above observation.

[6]
(c)

When an ant bites, it injects 6.0 x 1o-3 cm 3 solution containing 50 % volume of


methanoic acid. However the ant does not inject all of its methanoic acid but
keeps a little in reserve. Assuming a typical ant injects 80 % of its methanoic
. acid, calculate the number of typical .ants needed to produce 1.0 dm3 of pure
methanoic acid.

[2]

[,

[Total: 10]

r
['

L
[

l
r

L
..

,'

Section 8
Attempt both questions in this section.

Begin the question on a fresh page of writing paper.

4(a)

compound
sodium oxide
aluminium oxide
suphur dioxide

(b)

The melting points of three oxides are given below.


formula
Na20
AI203

so2

--~----------~--~

mJ!.: I 'C

--------

1275
2072
-73

(i)

Relate these melting points to their structures and bonding.

(ii)

Outline the acid-base nature of the oxides, giving equations where


appropriate.

(iii)

Describe their solubilities in, and reactions with, water.


Give the
approximate pH of any solution formed and write equations where
appropriate.
[7]

An ammonium iron(lll) salt has the formula (NH4)aFe(S04)b.nH20.


following information to calculate the values of a, band n.

Use the

A 1.00 g sample of the salt was dissolved in 100 cm 3 of water and the solution
was divided into two equal portions. To one portion, an excess of aqueous
potassium hydroxide was added and the mixture subsequently boiled. The gas
evolved neutralised 10.40 cm 3 of 0.100 mol dm-3 hydrochloric acid. To the other
portion, an excess of zinc was added which reduced Fe(III) to Fe(II). The re"-'OXidation of Fe(U) to ..Fe(IIl} in the.resulting filtered solution,requireEh20.80 cm 3 of
0.0100 mol dm-3 acidifted potassium manganate(VII).
(3]
(Total:10]

L
L

c
[
L.

c
['

[IJ

L
l'J

u
'

I.

r
[

either

(a)

When sodium chloride is treated with excess concentrated sulphuric acid, white
fumes of X are evolved. When sodium iodide is treated in the same way, a
coloured vapour Y is produced.

Identify X and Y and explain the difference in behaviour of both salts towards
concentrated sulphuric acid. Include any relevant equations.

[4]
(b)

Aqueous sodium chlorate(!), NaC/0, is sold under the trade name Parazone as a
common bleaching agent. The concentration of sodium chlorate( I) in Parazone .
can be determined by reacting it with excess acidified potassium iodide to form
chloride ions, C/- and iodine as the products. The iodine liberated is then titrated
with sodium thiosulphate solution, Na2S 20 3(aq), using starch indicator.

It was found that 1.0 cm 3 of a sample of Parazone liberated enough iodine that
required 60 cm 3 of 0.10 mol dm-3 of sodium thiosulphate for complete reaction.

(i)

Write the equation for the reaction between chlorate(I) ions and iodide
ions in acidic medium.

(ii)

Determine the concentration of sodium chlorate( I) in Parazone.

L
[
[

[3]
(c)

(i)

Under certain conditions, the chlorate(I) ion undergoes disproportionation.


Write a balanced equation for this reaction, specifying the condition.

(ii)

The iodate(I} ion, IO-, also undergoes a similar disproportionation reaction.


Predict,. with. reasoning,. the -ease -of disproportionation of iodate(!} ion

relative to the chlorate(I) ion.

[3]

[Total: 10]

[
[

L
L
L
L
[-

r
[

or

(a)

Describe and explain the relative thermal stabilities of the hydrogen halides.

[2]
(b)

Astatine, At, is the last member of the Group VII elements.


Predict and briefly explain what happens when aqueous sodium astatide, NaAt,
is added to aqueous silver nitrate followed by aqueous ammonia.
[2]

(c)

25.0 cm 3 samples of the resulting solution are found to


require 12.50 cm 3 of 1.00 mol dm-3 potassium hydroxide for neutralisation

r
II require 10.00 cm 3 of 1.00 mol dm-3 silver nitrate for reaction

aqueous barium nitrate.


Using the above information, deduce the balanced chemical equation for the
reaction between the halogen and sodium thiosulphate.
[3]
Another halogen __ upon reaction with aqueous sodium thiosulphate gives a
different sulphur containing product. This reaction is -commonly encountered in
experiments.

-(i) '

Write

[
[
[

Ill give 0.583 g of barium sulphate precipitate when treated with an excess of

(d)

A halogen is passed through 250 cm of a 0.050 mol dm- aqueous sodium


thiosulphate, Na2 S203, until the reaction is complete. The excess halogen is
then removed by some suitable means.

L
[
[

a balanced chemical equation for this reaction.

(ii)

What explanation can you offer for the difference in reaction between each
of the two halogens in (c) and (d) with aqueous sodium thiosulphate?

(iii)

Suggest another chemical reaction to support the explanation provided in


(d)(ii).
[3]

(Total: 10]

l
L

L
l

l
[''
~
Section C

Attempt all questions in this section.

Begin the question on a fresh page of writing paper.

Salbutamol, commonly known as Ventolin, is a drug that is used to treat acute


asthma and to relieve symptoms associated with other conditions pertaining to
reversible airway obstructions.

In a laboratory, salbutamol can be made from the starting compound, 2hydroxybenzoic acid via the following route:

HOOC)Ql

Step 1

HO

HOH~)Ql

Step2

H,CCOOH,C)Ql

HO

H3CCOO

2-hydroxybenzoic acid

[
CH3 COCI, AICI3l
heat

r
[

H;3CCOO

.. 3 steps
l
OH

CH
"-cH 2 NHC(CH3h

HO

salbutamol

L
L
c

'-..._CH3

Step3

H3CCOO

HOH2C

II

H3CCOOH2C
'-..._CH 2CI

II

H3CCOOH 2C

L~

[
[
(a)

Provide suitable reagents and conditions for Step 1 and Step 2.

[2]
(b)

(i)

(ii)

Suggest a possible 3-step synthesis for the conversion of compound J to


salbutamol. Your answer should include all essential reagents and
conditions, as well as the structures of any intermediates that are formed.
[3]

A laboratory technician suggested the use of chlorine gas under ultraviolet


light to convert compound H to J in Step 3.
Suggest why his choice of condition may not be appropriate for the
conversion stated.
[1]

(c)

A organic chemist realised that the labels to the bottles containing the starting
reagent, 2-hydroxybenzoic acid and another of its isomer, 4-hydroxybenzoic acid
had fallen out.

4-hydroxybenzoic acid

2-hydroxybenzoic acid

[
[

[
[
['

HO

HO

COOH

HOOC

Through determination of the melting points of the 2 compounds, she found that
one of them had a . melting point of 215C, whilst the other had a melting point of
159C.
Explain as clearly as possible how this rnelti11g point information may. be useful in
distinguishing the 2 compounds.
[2]
(d) Outline the mechanism for the mono-nitration of 4-hydroxybenzoic acid.

[2]
[Total: 10]

r
[

['

l
L,

l
10

L
L

L
r

and it can exists as a variety of stereoisomers.

(a)

(b)

Draw the structures of the geometric isomers of H.


[2]

Draw the structures of the product(s) that are formed when compound H reacts
with the following reagents under suitable conditions:

(i)
(ii)
(iii)
(iv)

[
[

CH=CH-CHCI-COOH

HO

C'
[

A compound H has the following structure:

aqueous bromine;
dilute nitric acid, followed by tin in concentrated HC!;
cold alkaline potassium manganate(VII);
hot aqueous potassium hydroxide.

[4]
(c)

Suggest suitable chemical tests to distinguish the pairs of compounds given


below. In each case, state clearly the reagents and conditions used as well as

any relevant observations.


(i)
CH 2 CH 20H

.,

HOH 2C

CH 2 Br

-'

[
[

[2]

(ii)
H3 C

CH2CHa

CH3

[
[2]

[
[Total: 10]

[
l_~

l
l. -

11

t'
r1
.

''

t-

either

Compounds A and B, which are isomers with a molecular formula of C6 H120 4 ,


react readily with potassium carbonate under room conditions to give a
colourless gas. Reaction of A and 8 with hot acidified potassium manganate
(VII) yields products C and 0 respectively.

C is optically inactive and gives an orange precipitate with 2,4dinitrophenylhydrazine. With alkaline aqueous iodine followed by acidification, C
gives a compound E, C5 H6 0 5 which is a symmetrical molecule.
0, another optically inactive molecule, has a molecular formula of C6H100s, and
gives the following spectrum in the region of o 1.0 to o4.0 when analysed
through NMR spectroscopy.

[
[

c
[
[
[~
4

0 also reacts with excess phosphorus pentachloride to form F with .the evolution
of vigorous white fumes. The mass spectrum of F shows 4 molecular ion peaks
at m/e 216, 218, 220 and 222.
F.reacts.rapidlywith ethane-1..;2-diol.to give a compound G thatexistsasa high
melting point solid with good tensile strength.
Deduce the structures of compounds A toG, giving your reasoning based on the
information provided.
[10]
[Total: 10]

12

c
[
I

l
l
L
l

L
r~

[
[

or
A hydrocarbon compound U was isolated and identified as a source of pollutant
in water. Upon analysis through mass spectrometry, the compound gave the
following spectrum :

100

80
>-.

-+J

'(i)

c
c

<I)

-+J

60

<I)

.~

-+J

0
q)

40

0:':

r~

20

-'

[
'

'

...

0 I 11111111111 '" 1II 1111111 ullllli 111111 II Ill 1ul'llitlllllli 1111 illlllllu 1111111111111111 Htlllllli 1111 11l iltlllt11 11111111111
75
100
25
50
125

m/e

r
[
--,

[---

- Upon irradiation of U withUV light in the presence ofbromine-gas, a product Vis


isolated. The mass spectrum of V showed 3 molecular ion peaks at m/e values
of 276, 278 and 280 in a ratio of 1:2:1.
Upon heating with aqueous potassium hydroxide, V yields a compound W, which
is sparingly soluble in water.
When W reacts with hot- acidified potassium dichromate, a compound X was
formed, which gives an orange precipitate with 2,4-dinitrophenylhydrazine. X
produces a silver mirror upon reaction with Tollen's reagent, but does not give
any precipitate when heated with Fehling's solution. X also gives a yellow
precipitate with a characteristic smell when it was heated with alkaline aqueous
iodine.
Conversely, when W reacts with hot acidified potassium manganate(Vll), a
compound Y is produced. Y readily dissolves in sodium hydroxide, and it is
isomeric with one of the monomers of Terylene.
-

[
[

13

L
L
[
Strong heating of Y under certain conditions causes each molecule of Y to lose
one molecule of water, and Z is produced. The NMR spectrum of Z before and
after the addition of D20 does not show any significant changes.
Deduce the structures of compounds U to Z, giving your reasoning based on the
information provided.
[10]

rrotal: 10]

[
[_

\
[
[
r

L~

L
L
[

L
[
14

L
L

L
r--

[
Section D

Attempt both questions in this section.

Begin the question on a fresh page of writing paper.


9

[
[

(a)

.. ,.:Membranes have a fluid mosaic nature and their chief components are
phospholipids and proteins. Phospholipids form bilayers. Molecules and ions
need to pass through the membrane in and out of each cell. This transport can
. be active or passive.
Give a structural formula of a typical phospholipid.

[1]

[
[--~

(b)

Explain, using your answer in (a), how bilayers are constructed. State the nature
of the intermolecular forces at each end of each phospholipid in a bilayer.
[3]

(c)

Describe, with the aid of a labelled diagram, the fluid mosaic model of a
membrane.
[3]

(d)

Explain what is meant by active transport. Describe briefly one common example
of active transport.

[3]

[Total: 10]

[ __,

[
[

L
L
[
[

15

L
[
[
10(a) The enzyme succinate dehydrogenase catalyses the following dehydrogenation
reaction of succinate :
succinate + oxygen

fumarate + water

The rate of reaction can be followed by determining the fumarate concentration


by the use of a coloured reagent and a colorimeter. The initial rate of the reaction
was measured at different concentrations of succinate.
In experiment Q, the reagent succinate in the presence of the enzyme was
subjected to reaction with oxygen. The experiment was repeated with the
addition of another anion, malonate and termed Experiment R.
The rate of reaction for both experiments Q and R against [succinate] was plotted
as shown below:

L
[
[
[
[
[,

Experiment Q
--17

-=t/1-

'7c

.E

(')

'E
"0

0
E

::1.

......

cr:
Q)

lU

JJ~

[
I I
0

10

12

rsuccinatel/ mmol dm-

(b)

14

16

I
18

I I l=r=j
20

(i)

Explain the shape of the graph in experiment Q.

(ii)

Determine the values of the Michaelis constant,


Q and R, stating the appropriate units.

(iii)

State the role of the malonate added in experiment R. Justify your answer.
[7]

Km.

for both experiments

Explain what is meant by the tertiary structure of a protein.

[3]
[Total: 10]
THE END

[~

L
l
L

16

L
r

L
,~

[
[

2006 MJC Chemistry Prelim Exam Paper 1 Answers

[
,

~J

[
[
[

L
[
t

[
f~

L.

l'

11

12

13

14

15

21

22

23

24

25

33

31

32

16

17

c
18

26

27

28

34

35

36

37

38

10

19

20

29

30

39

40

I~

~--
l

L'.
..

[._.

[
[

I
1

...

2006 MJC Chemistry Prelim Exam Pape~

2 Suggested Answ~r~-]

The kinetics of the reaction of peroxodisulphate ions and iodide ions

Szoi- (aq) + 2r (aq) ~ 2Sol- (aq) + h (aq)

r=

can be investigated experimentally by varying the concentrations of the two


reactants involved and determining the time-taken for the brown iodine colour to
first appear. Water is added to each reaction mixture''to ensure that the total
volume is kept constant.such that-for each ofthe ex.periments, the concentration
-of each of the reactant is dire~tly proportional to,-.tsvolunie used.

[
[

The following results were obtained in such an investigation:

Expt

Vol of S20{-/cm,

Vol ofr /em,

Vol of water /cm 3

Time/s

10
20
10

20
20
30

20
10
10

55.5

2
3

L_

[
(a)

27.9
37.0 ...

Define the term rate of a chemical reaction.


Rate of reaction is the change in concentration of reactants or products per unit
time.

[
[

(b)

Deduce the order of .reaction with . respect to


reasoning.

.
8 2 0 82- and

[1]
Explain your

Order of reaction is 1 with respect to 8 2 0 82

Order of reaction is 1 with respect to I.

. [3].

[
[

(c)

Write the rate equation


peroxodisulphate(VI) ions .

for

the

reaction

between

iodide

ions

and

.Rate = k [SzOll[Il

[1]

L.

l.
[
l'

-l

l.
L
(d)

The reaction between the peroxodisulphate ions and iodide ions can be
increased by the addition of small amounts of homogeneous catalyst, Fe 3 +.
(i)

(ii)

Explain, us.ing the concept of activation energy~ how the addition of a small
amount of catalyst can cause a large increase in the rate of reaction.

[
[''

When catalyst is added,


Ea is lowered via alternative reaction pathway
Number of particles with at least Ea increases
Number of effective collisions increases
Frequency of effective collisions increases
Rate of reaction increases

Propose a mechanism for the catalysis of the above reaction using Fe3+.

2Fe3+ +2r - 7 2Fe2+ +h


Fe2+ + S20a2--7 2Fe3+ +

2Soi[4]

The compound.furan, (CH)~O., is unsaturated and has a structure in which the


four carbon .atoms arid. one oxygen -atom are connected .together in a ririg. It is a
liquid at room temperature with a boiling point of 31C. It is used to makE)
solvents and nylon. The. displayed formula of furan is as follows:

2(a)

[
[

H-C-C-H

II

II

H-C

C-H

\I
0

(i)

By constructing an appropriate energy cycle, calculate the enthalpy of the


following reaction offuran at.298K':

(CH)40 (I) ._ 4C (g) + 4H (g) + 0 (g)

:Use the following given enthalpy values:


dHt[(CH)40] = -62 kJ mor1
. dHat (C)
+ 717 kJ mor1
dHat (H)
+218 kJ mor1
1
dHat(O)
+250 kJ mor

=
=
=

By Hess' Law,
ziHrxn
+4052 kJ mor1

(i)
.

::

=-

,,

'2.

,,

..

'.'"..<.
...

"".<:
-"..

'
;

:;.

;.

...

... ,: ..

... ~

~--.

L
L
L

.:,

,
L

....-._._._

.,

'

' ; ..

(ii) ..

It is also possible to calculate the valu~ of the enthalpy change for the reaction
above by using appropriate bond energy values from the Data Booklet. The value
calculated using this method differs from that catculated by using the Hess Law
approach in part (ai).
By considering the bondirig in furan and the definition of bond energy,
suggest one reason for the difference in-enthalpy values.

In a(ii),
Furan is a liquid at room temperature and bond energy is applicable for
gaseous molecules or delocajisation effed in furan is not considered.

[
[

(iii) Liquid furan is transported by road tankers. Briefly explain why an explosion
may happen during transportation if precautions are not taken.

[
[

Liquid can be easily vapourised to the gaseous state. This causes an


increase in pressure within the drum. This may lead to an explosion.
[5]
lron(II) oxide is a black-coloured oxide used for glazing.

(b)

(i)

What do you understand by the term lattice energy of an ionic compound?


Lattice energy of an ionic compound is the energy released when 1 mole of
the ionic compound is formed from its constituent gaseous ions under
standard conditions.

(ii) . . Construct and label ,carefully a Born-Haber cycle for the formation of iron(II)
oxide from its elements.

Use the cycle to calculate the lattice energy of iron (II) oxide.

Incorporate the relevant data from the table below as well as relevant data
from the Data Booklet.

-Enthalpy term
Standard enthalpy change of formation of FeOStandard enthalpy change of atomisation of' Fe
First electron affinity of 0
Second electron affinity of

L.

o-

By Hess Law
1
L\H 81att(Fe0) = -3913 kJ mor

L
L
[
[

3
.

L\H I kJ mor1
-278
+416
-141
+790

r'
r'.

'

(iii) Explain qualitatively how you would expect the numerical magnitude of the
lattice energy, and hence the relative stability, of iron(II) mdde compared
with that of iron(ll} sulphide, FeS.

I.6.H 91att

I oc

+r_
o Ionic radius of 0 2- < S2o IAHelatt Iof FeO > FeS
o Lattice energy of FeO is more exothermic than FeS
o FeO is more stable than FeS

I''"'SOJ

.coal

'----....f

80 2 scrubber

Stage Ill

.Powdered limestone (calcium carbonate) is .injected into the


furnace where it- thermally decomposes to give solid lime (calcium
oxide) and carbon dioxide.

Stage II .

, Lime (calcium oxide) reacts with sulphur diox.ide. to ,prodl!ce calcium

sulphite (CaS0 3).

Stage III

Any remaining sulphur dioxide is removed through a scrubber. This


involves passing the gas through an aqueous lime (calcium oxide)
suspension to form calcium sulphite.

Suggest balanc~d chemical equations for the reactions in Stage I and IL

l
l

.:;

~
-:,.

'

.... -

'---v-'

, Stage II

:;'.'
..

r-~

- sulphite

Stage 1: CaCOg~ CaO + C02 .


Stage 11:. CaO + SOi ~ CaS.Oa .
""

Stage I

(i)

:-

L
r

Flue~

'

calclu!n
.

- - St<!ge I

(a)

gases

''-y--'

;,::..:, .

Coal-fired furnace with FGD olant


Water and Lime

carbonate
celcimn

. '~

[7]
[Total: 12]
Coal-fired power stations emit flue_gases, which contain the oxides of nitrogen,
carbon and sulphur. In order to reduce sulphur dioxide. emission, such a power
station is fitted with a flue gas desulphurisation. (FGD) plant Three stages are
described below.

air

q+qr+

..

.....

.. ->A.

..

..

>

:, l

l
[.
[

...

'
..

_;:::

-..

:.~ ..

..

The average sulphur content in cocil is typic~lly 1.5% bymass.C~Ic~l~te th~ .


mass of limestone that is needed to remove the sulphur dioxide emitted
when 1 million tonnes of coal is used. [1 tonne 1000 kg]

(ii}

Mass of limestone= 4.67 x 108 x 100.1 = 4.67x 1010 g

[4]

(b)

The graph below represents the change in mass that occurs when 100 g of
limestone, is heated at a temperature, T.

Sketch on the same axis, the graph that would be obtained by heating 100 g of
magnesium carbonate, MgC03, at T. Explain your answer.

Mass I g

100

50
Mass I g

L-----~----..

100.

r:

50

CaC03
r-,-------------------------- MgC03
L __ _ _ _ _ _ _ _ _ _ _ _ _ _ _ _ _~

100
Mass of MgO = - - x 40.3
84.3

[
(b)

Time

=47.8 g

[3]
Based on your knowledge of periodicity, suggest why both of the metal oxides
produced would effectively remove sulphur dioxide.
MgO and CaO are basic oxides that would react with acidic gas S02

[1][Total: 8)

L
L
l~

Time

s
..
..

[
[
4

Copper is a transition element. Copper. was named after the island of Cyprus
which was once rich in copper ores.

(a)

Write the electronic configuration of a copper atom and a Cu2+ ion.

Cu 2+ ion: [Ar] 3d9

copper atom: [Ar]3d 104s 1

[1]
(b)

When concentrated .hydrochloric acid and aqueous ammonia are added in


excess. separately to copper(II) sulphate solution, [CuC14t complex and
[Cu(NH3)4(H20)2]2+ complex are formed respectively. Both the complexes are
coloured.
State the colour of both complexes.

(i)

[Cu(NH 3)4(H20)2]2+

[CuCI4f : yellow

deep blue

Explain why transition element complexes are coloured.

(ii)

In the presence of ligands, the set of degenerate 3d orbitals of transition


metal is split into 2 groups with different energy. This effect is known as d
. orbital splitting.

~
~E

~-

[
[
[

[
[

c:
[

d-orbitals
free.ion

octahedral complex

energy level by absorbing a certain wavelength of lig~t_ from the visible


region of the electromagnetic spectrum

d electrons from the lower energy level can be promoted to the higher

This effect is known as d-d transition.

. The complex thus emits the remaining wavelength which appears. as the
colour of the complex observed.

[
[

.; .....

:'."

.-.

.,.

. ;6

l
L
l
r,

!__

,.,.-

..

_;:

.. . ..
-~-

.-.

~.:

l.

..

(c) ... The nume~ical value for the stability corist~nt for
that for [Cu(NH 3)4(H20)2]2+ complex is 13.1.

[Cu~14f co~plex

is 5;6, ancl

Using the concept of stability constant, explain what will happen if ammonia is
added in excess to [CuCI 4f solution.

Yellow solution will change to deep/dark blue solution

[3]

(d)

When concentrated hydrochloric acid is added in excess to copper(I) sulphate


solution, [CuCI 4]3- complex is formed.
Suggest the colour of [CuCLi]3- and explain your answer.

Colourless
Copper (I) has fully-filled d-orbital or copper (I): [Ar] 3d 10 and d-d transition

cannot occur

[2]

rrotal: 101

[
[
_::

~-

.
,',

..

..

[
[

L
L
L
L
r

-:~~

.. ____

'

....

,;.

' / . .

.. .
.. _,.

- ~ ~- > ;
~--.

[:

r
r
[

5(a) _ The following represents part of a synthetic pathway to obtain the amino acid,
phenylalanine, and one of its otherderiv(itives, D.
c

[I

II

Br

r:

CH2 C-.- H

CH2 CHCN

Step2

Step 1

[
8

I'

Skp 3

NH2

NHz

CH2 CHC-OH

II

0.

CH2 CHCN

Step4

[I

NHCH2 CH3
CH2CHCN

StepS

[
c

phenylalanine

L
[I

(i)

Suggest suitable reagents and conditions for the following conversions:

Step 1 :-HCN with traceamts of base, 10-20C

1_J

~tep

2 : PBrs. at r.tp _

. Step,3: NH3 _(g), heat in a sealed tube.

:.

~:.'

'.

~-

':: ..

:..

L
L
l
L
r

'L
["

.
.

.: : ... ~

-.

.......
..

'

.. ..

[
[

. (ii)

State and describe the mechanism for the conversion in Step 1.


Mechanism : Nucleophilic addition

Step 1 :The nucleophile eN- attacks the electron deficient C of the carbonyl
group.
[

ll

0-

_yH2C"" H

CN

slow-

{\'

L
[
Step 2 : Protonation of the tetrahedral intermediate to form the cyanohydrin.

0~
I .

?H2C""C~

('

OH

. CH2C"' H

H . '6N

CN
+

[
[

:(iii)

-CN

It was observed that a soJution containing compound A did not display any
optical activity. Explain this observation.
After Step .1, the,solution containing compound A exists as a racemic( or
racemiate) mixture. Thus there is no net rotation of plane polarized light.

L
L
l
L
l~

[7]

".

. 9
.

'
.

.-:---

---~

- ./:

.,

-.-

:'

.
.
. ::

.':.

r
r
(b)

l.

State and explain the relative basicity of compounds C and D.

Compound Dis more basic than Compound C.


(c)

Amino acids can be separated by the well-known technique known as


electrophoresis. This -technique involves placing a solution of different amino
acids at the centre of _a -strip of gel moistened with an aqueous solution of a
known pH. An. electric potential is then applied through electrodes which are
connected to the ends of the gel strip.

IIt
C.

The following information about phenylalanine and 2 other amino acids are given
as follows:
Amino Acid
Phenylalanine

Structure

lsoelectric Point

H
HN-C-COOH
2

5.50

CH2(C 6 H5)
H
H N-C-COOH
2

Glutamic Acid

[
,

[
3.22

9.74

CH2CH2COOH
H
H2N-C-COOH

Lysine

CH2CH2CH2CH2NH2

An electrophoresis experiment is run on a solution containing the 3 amino acids


mentioned above.

L-

~-~

. . . II

-ve

+ve ,

L
L
L

Gel strip at pH 5.50


Draw the structural. formulae of the species found at positions X,
end of .each experiment.

":.

: -

.-::.

<

':,_

Y an'd Z at the

-:

,-

. ,..... .

JLJ

: -~':

:;_

'.

'10 ...
.. -..

-:...

..

L
I

L
'

t
-.:

. ~-

'

,_.,

..

~ : .. ,

.<.:..

Corn pound :X :
.

H
I

HsN-. C-COOH
.. ,

r~

- .

Conlpound

Y: .

. + .,

CH 2CH 2CH 2CH 2N H3

HsN-C-COO~ ''-~' ~- ;:

. ..

CH 2(C 6 H5 )

Compound Z:
.H.

.. I.

.. .

H~-C-C00-

1 .. .

Ct-J2CH 2COO-

'[3]
[foUII: 1.3].

[
f~
[

..

':'

.~-

..

[
[
[

l
l
l.

l.

. .: -....>.
-:":.-:

,.:

..,:_

11
n:.''

..

,-:-:

. : .... ' ...

L
[

[
6

Many of the foam materials and packing. used in everyday life are made up of
polymers. Some Of these common polymers are polystyrene and polyurethanes,
and these cari be found in the form of common household names such as
Styrofoam and Lycra respectively.
The monomer that can be used to make polystyrene is styrene.

[~

c
r
r

.styrene

~
(a)

(i)

Draw a section of the polystyrene chain, showing at least two repeat units.

C'

r
[

\. __ ,

(ii)

What type of polymerisation occurs when polystyrene is formed?


.

Addition Polymerisation
.:.-.._

:.

.:

r,1 "

'

~: .. '

L
L

.....

~ .

L
.

._: _.-:.::_:

..

__

,_.

. :..~ ". .

..

. .
'

L
I.

l
.,_;.

' . . . '

.~

. .:.:. .

..' '
.. ,.

....>

_:

,:.

'.::

..

On :its. own, the polystyrene formed is .too flexible to be of use since. it lacks
strength and rigidity. As such, they are often strengthened and stiffened by copolymerisation with other monomers such as 1,4-'divinylbenzene.

"'

. 1,4.:cfivinylbenzene

~--:~.

~.

/L

(b)

(i)

Draw a section of the co-polymer formed if each repeat unit consists of one

styrene and one 1,4-divinylbenzene molecule,

[
[

r
[,

[
[
[

(ii) .Su~gest ~ow theaddition of the new m~nomer, 1 ,4~di~inylbenz~n~


form a .Polymer with better strength and rigidity.

.hel~s to.

Each repeat unit of the polymer still contains a double bond which can be
used for another addition polymerisation process with a neighbouring chain.
Crosslinking causes the chains to be held strongly to one another, giving.
strength and rigidity.'

[2]

L
['

l.'

'13

t;

n
n
On the other hand, polyurethanes are the most versatile .family of polymers. One
possible polyurethane is formed from the polymerisation of2 monomers :

c
r~
'\.-

o=c=N

.N=c=o

and

HO--CH2 CH2 CH 2 CH 2 -0H

f~

The repeat unit ofthis polyurethane has the structure below :

II

II

-0-C-N
H

(c)

'

N--c--0--CH2CH2CH2CH2-

Deduce the type of polymerisation in the formation of the polyurethane


above.
Addition Polymerisation

(i)

(ii)

The diol used as the monomer can be replaced by a diamine such as ethyl1,2-diamine, H2NCH2CH2NH2.
ethyl~1 ,2-diamine

Draw the repeat unit .of the .polymer formed when

used

II

--11
-N-C-N

I ' .
H ,

With the aid of a balanced equation (involving one repeat unit of the
polyurethane), suggest why polyurethanes are not useci forpackaging of
fruit juices.

(iii)

In fruit juices, there contain acids which may hydrolyse the polymer.
As such, the packaging may disintegrate.

Write balanced equation ._


-~.

..

:._;:

..

:...::. . .

. ':

r
L

N--C--N-CH2CH2.H .
.

- "0

was

[
[
l.
--_-_ :. : L

::

:.

-14

..

"-

.. .....~ ..:
...

''.

. .. .

:~:

:':.

f
l

L
f". .

2006 MJC Prelim Paper 3 Suggested Answers

1{a)

(i) NH 2 CN has simple molecular structure but CaNCN has giant ionic structure

{ii)
XX

H ~ N ~ c:tNx

ex

f"

{/
2

Ca +

j,x

L~N ~ C;

-'<

]2-

Nx:

Shape of NH2 CN is trigonal pyramidal.

(iii)
NHzCN can form hydrogen bonds with H2 0 hence is solublein H20
CaNCN can form ion-dipole interactions with H2 0 hence is soluble in H2 0

(b)
(i)
Dative bond is formed when lone pair of N is donated to vacant orbital of
B.

[
(ii)

NC
F
H "N~B/ F
H/

.or

/F

"-p

""-N-CN~B - F

"-p

(c).

[.

(i)
LlHnm

=- 25 kJ mor1

(ii) .
Reaction is exothermic and is-expected to be spontaneous.

L
f~
k

L
L
f '
I,

,.

.. 1
-.";
p:

.-

;:.:

. :-..'

. .,
-;

L
. r~

2(a)

(i)

(ii)

Anode:
Zn (s) + 40H- (aq)
Cathode:
02 (g) + 2H20 (!) + 4e
Overall Equation:
2Zn (s) + 40H- (aq) + 02 (g)

E Znoi-l

Zn

+ 2H20 (l) + 2e

40H- (aq)
2Znoi- (aq) + 2H20 (l}

=-1.22 v

Q = 7. 08 x 10 4 C and t

(iii)

Znol- (aq)

[
=5.17 x1 0

(b)
(i)

HOC!

(ii)

E8cen = +1.49 V > 0, reaction feasible

2HOCZ (aq) + Sn2+ (aq)+ 2H+ (aq) ~Ch (g)+ Sn4 + (aq)+ 2H 20 (l}

Greenish-:-yellow Cl2 gas will be formed.

C'

3 either
(a)

% dissociation = 32.3 %

[
. 3(b)

.
[H+][CyS03 H 2 ]
(i) Kc = [CyH+][S0 ]

(ii) pH

=1.51

[
(iii)

When pH is increased,
[H+] is decreased.
By Le Chatelier's principle, the equilibrium position in (1) would shift right
The new equilibrium mixture would contain lower concentration of purplish red
. . CyH+. Hence, the decolourisa,tion would be more severe at pH 13.
-::. :

.
,

, ...

....

.2"

.;

--

.. .

_-.

L
L
L
L

> \.

C.
[

3 or

- (b)(i)

Ka = = 1.58 x 10-4 mol dm-3

(a)

pH
r-

[
f~
\l'

1.9

Buffer .

-,r
_,

[~

Volume ofNaOH
added/cm3

10

--(ii)
At equivalent point, salt solution is basic.
Hcoo- undergoes salt hydrolysis and releases oH-.

(iii)

(c)

Heat is released during neutralisation reaction.


HCI is fully ionized but HCOOH is partially ionized, so some energy is used for
dissociatron of the HCOOH molecules.
No. of ants required

= 4.20 x 105

L
[

-_.-, , ->.

L~

' .:.,

.
.

.....

,:

: ..... .

-~?

':.
..
:.

L
L--

. ::-_: ... .

,.

r
[
4(a}

(i)
Na 20 and Ab0 3 have giant ionic structures.
Hence Na20 and. Al 20 3 have high melting point, large amount of energy are
required to overcome .the strong electrostatic forces of attraction between
oppositely charged ions present in the crystal lattice.
Melting point of Ab03 > Na 20 since charge density of A1 3+ > Na +

[
[

802 has simple molecular structure.


80 2 have low melting points as small amount of energies are required to
overcome the weak van der Waals' forces of attraction between molecules during
melting.

(a)

(ii)
Na 2 0 is ionic and basic oxide.
Na20(s) + 2 HCI(aq)--+ 2 NaCI(aq) + H20(I)

['

Ab0 3 is an amphoteric oxide.


Ab03(s) + 6 HCI(aq) --+ 2 AICh(aq) + 3 H20(!)

AI203(s) + 3 H20{I) + 2 NaOH(aq) --+ 2 NaAI(OH)4 (aq)

L
L

S02 is covalent and acidic oxide.


S02(g) + 2 NaOH(aq) ~ Na2S03(aq) + H20(I)
(iii)
Na20 dissolves in water to give an alkaline solution (pH : : : 13)
Na20(~) + H20(I) --+ 2Na0H(aq)

Ab03 does not dissolve in water.

802 dissolves in water to give an acidic solution (pH : : : 2)


S02 (g) + H20 --+ H2S03 (aq)

(b)

~b:::

2 and n =12

,:._

;'

.>

-:-:-:. .'.

. :,
. ' .:

. _: . . . ~ . .

-..

...

': ~

.::-....

. . ~: .

....

::.:::_.

::,.--
~

.:::.'
.... ~.

.'

: .: '

..

::.

.- .

~-

: .:

..

.. ~-

;:.

..._ ........

...~

,.

. - ..

.
'.

' . ~~~

:-.

.
,

.. -.:
...

....

_,
;

..

..

,.

..

'.!4
_,_

. ....

~--

t~

.:..'.

r
[

..

5 either
(a)
2NaCI + H 2S04 ~ Na2S04 + 2HCI
X is HC/Iiberated.

2Nal + HzS04 ~ Na2S04+ 2HI


8 HI +- H2S04 ~ 4 lz + H2S + 4 H20.
Y is !z gas.

r is a stronger reducing agent than cr,

(b)

(i)
Overall equation: CIO- +

(ii) .[CIO-] = 3.00 mol dm-3

[
(c)

(i)

L~.

31o- ~

Bond length of 1-0 > Cl-'0


Bond strength of 1-0 < Cl-0
Bond dissociation energy of 1-0 < Cl-0
1-0 bond is more easily broken
Hence 10- can undergo disproportionation more readily,.

5 or
(a)

Down the group

atomic radius of X increases

bond length of H-X increases

.~ bond strength of H-X decreases


bond dissociation energy of H-X decreases
H-X bond is more easily broken

Hence thermal stability of the hydrogen halides decreases.

c
[

L
l

L
L
['

zr + 103-

(ii)

zcr

3Cio- ~
+ Clo3condition: warm I heat

[~

I'

zr + 2H+ ~ Cr + H20 + b

(b)
A dark coloured (yellow I brown) AgAt ppt is seen.
The ppt remains insoluble in aq NH3
Reason:
(1)
AgAt(s) ~ Ag+ (aq) + Ar (aq)
The Ksp value of AgAt is very low and there is insufficient Ag+ to react with NH 3 to
form complex ion
Thus the equilibrium position (1) does not shift right, hence AgAt remains
insoluble in aq NH3.

t
[
rS20l- (aq) + 4X2 (aq) + SH20 (I) -+ 1OH+ (aq) +

(c)

(di)

Y2 (aq) + 2s203 2- (aq)

ax- (aq)

+ 2SO/- (aq)

-+ S40s 2-(aq) + 2Y-(aq)

(ii)
X2 is a stronger oxidising agentthan Y2 .

X2 can displace Y from its salt solution.

(iii)

(a)

['

' Step 1: LiAIH4 , in dry ether, r.t.p

rL

Step 2: CH3COCI ,r.t.p

(b)(i)
0

[
[

H 3 CCOOH 2

CH 2 CI

Dilllte-HCI, heat

CHzCI

(CH3)3 CNHz
heat in sealed tube

l"

Followed by
diL NaOH, r.t.p.
OH

......---NHC(CH 3 )a
CH 2

HOH2 C

LiA~

t'

-~-o

I
H~CH'c<NHC(CH,),

HOHzC

in dry ether, rtp

L
L

(b)

(ii)
There is a possibility of multiple substitutions of the Hs. on any alkyl group found
in the compound. OR .. :.. : -. . :._.. -.. .
. _. .. ..... --.
.... ..
..
.,
.
,.
~.

:- _,.

....

.:.-

;' -::~:

..

.'

"

.
,.
..

'

:__

-: ~:

.:_- .'

..._.
..'"-

.:6

[
. ,_ ,.~.
--~

. .-

f1

r _- ,

[
. (c)

..

4-hydroxybenzoic acid generally has a higher melting point (of 215C) since there
is the presence of more extensive intermolecular hydrogen bonding than in 2hydroxybenzoic acid.

(d)

Type of reaction:

Electrophilic Substitution

Step 1

HN03

N02+

2 H2S04

2HS04-

H30+

Step 2

[-;
--

i J J.H
NOz
. ( + \ ...-OH

slow

+ NOz+
HOO

HOO

\J

Step 3

.h
H-N02

fast

OH

--=~0

+ HS04-

(~

HOO

rNOz
. OH

H2S04

[
[

,,'

C'

....
..::''

'

~;

..

;. :-~-- ,

L
l

-'

-,

:~~~.:~ -'-<:-~.

'

::

;-

-,

.:-
..

.;"
.,._

'

-:~_;-

::.:.

..

..... ..

~:.

:. .-

I ,

L---

l
[

,'

:'.:

.'.

-.._.

:. :i:.' :

' 7
-,:: ..

'.,

L
[
'

7 (a)

..

IH

.HO-g;c=c"'CHCI-COOH

.CHCI-COOH

\=I

"'H

HO

cis isomer
trans .isomer

(b)
aqueous bromine;

(i)

[
[

HO

GH-CH-CHCI-COOH

Br

Br

Br

L
HO

CH-CH-CHCI-COOH

<I l
Br

OH

Br

. (b) (ii):

[
HO

GH-:-GH-. CHGI-.GOOH

,.
-~

. _..

\:.. :

..

~ .< .
~-

. '" ., ....

<H2N

.. :_;

"

_,.

...
_;:

LJ
...

,_:

lJ

::.).

<"

lj

. a.
,;

L
1:

l.

r
[
(b)(iii)

CH-CH-CHCI-COO

I OHI

OH

f
L

(b)(iv)

CH=CH-CH-COO

OH

(~)(i)

["

Add aq NaOH to both compounds separately and heat. Subsequently, add


acidified AgN0 3 to both mixtures, at r.t.p.

The test-tube containing the first compound on the left will not give any
precipitate whereas the test-tube containing the 2nd compound will produce a
cream-coloured precipitate of AgBr

l
[

(c)(ii)

Add acidified KMn04 to both compounds separately, and heat.


NaOH (aq}, followed by b (aq), and heat.

Add excess

The test-tube containing the 1st compound initially will give a yellow precipitate of
cHh whereas the test-tube containing the 2nd compound will not give any
precipitate.

(~
_,

l:
L

- .
....,
< ..

. __ ,

<-:

.. -

[
..:::.:.

..

l:

__

l
(
-~

[
8 either

E is therefore likely to be
0

II

H2

H2

HO-C-C-C-C -C-OH

II

),

II

HO-C-CH-CH-CH-CH-CH
2
I _ 2 I . _3
.
.
OH
OH

II

HO-C-CH-C-CH-C-CH
2
II
. 2 II
3

0 islikely.to be
0

Hb

CH3a Hb

II .1 I ._
I I.

Ho-. C - C - C - C - C - O H
:I . . .
Hb

OH

Hb .

[
8 is likely to be
0

CH3

I I I I H2
Ho-c-c-c-_c-. c -..
.
I I I
H

OH
r~

.H

OH

Fis likely to be
0

CH3

II I . I 1 II
I l l
H Cl H

Cl-c-c-c-c-c-.cJ

and G is likely to be

OHCH
II I I I II
.

f
,

HO.

_---r-r-,C--a.--C~2CH2-0 __. ..

C-x--.
-

H.

Cl

H,_,

. ,

.::

;<

..'

....
,

--

. . . . .

. 10
'

..

'

'.,_.;c;.

_<.n

L
L
L
L
[

l
\

c
r~

:-.

. . .

Or
.Z is therefore likely to be
0

-~o-_)

_,

~c
-~

_,

. . .

Y is thus likely to be
0

. II

rQ(c::.

-,.

_,

.. :

. .. 0cII

-, .

..

.J

andX is likely to be

[I

. II

. rOY
. . . .. "":".

~c/
. :. 11 .

0.

Wis thus likely to be

L
l
l
L
l
L

'

J:G::~

rL
L

[
r

..

CH

.I .

OH

. stichthatV and U may be


.

CH2Br

/~

CCHI

0 ..

and

Br

: .; ~... ..

. ( .CH3

CH,CH,

...

. ;.:

..
.,.;

.. _

..... 11
.'

l
-c
[

9(a)
Non-polar I
tails
I

- - H- - - -

11
:
H-C-0-C-R I
1

: I
I
:
I

0- - - I
0

II

----- ___ -- _ ~l:i-::;-:..G..-70=-:.C.-::-B:..I


0

: R"--0--P--0-,C-H
II

Polar bead _II

1I
I _____________ ,.I

o-

(b)
2 hydrophobic
tails

(c)

~wwwwwwwwwwww~

Ihydrophilic

~~~~~~~~~~~~~V head

-~

}b;Wya

;E

rn~mn un~nnnB lnnn


W
UNU
"'- U
_U!UUUUUU~UU
.. ,.

phospholipids

bilayer

.....

....
.:._ .. .

nnn
uuu

......

/A

Integral protem

Peripheral
protein

channel protein
(Integral)

Cell membranes consist mainly of a lipid bilayer and proteins.


The proteins present in the _membrane . are either embedded in the bilayer or
completely spanning the.mernbrane ..

. ,.

.. ._._ .
-::.

:,'::

-:-.

r
L
['
L.
_,

.,.

:,

..

...

...

.
~

[
~

.:

.., .

?-

. :-,~~-.

..
., .. .

''

. .. :. . ..

- ....
:~>

. '12
.

...

.-..

':

: -"'

:'

l
L

. ~:-~

:._.,
. . _

,#

-..

L
(

L
r--

.'

.c

(d)

Active transport involves the passage of a substance through the membrane

against a concentration gradient.


Energy required for this process is derived from the hydrolysis of ATP molecules.

Active transport requires the participation of one or more membrane proteins and
sometimes even other molecules such as hormones to stimulate one of the steps.

- Na+ !!<."':Pump- An Example of Active Transport


The Na+ I K+ pump is a system responsible for the movement of Na+ and K+
across cell membranes.

The system maintains a high K+ concentration and a low Na+ concentrationin the
interior of the cell by moving K+ into the cell and Na+ out of the cell.

Both exchanges of Na+ and ~ are against a concentration gradient.


The protein in the lipid bilayer involved in this system is Na+-K+-ATPase , which

spans the entire membrane.


10(ai) 1; at low [substrate]

['

Rate of reaction increases linearly as a function of [substrate].

Rate is first order wrt [substrate}.

II: when [substrate] increases

Rate of reaction increases to a lesser extent, not proportional to [substrate].

Rate is mixed order.

Ill: further increase in [substrate]

Rate of.reaction is independent of[substrate].

Rate is zero order wrt [substrate}.

['
(ii)

t~

Km for Expt Q =
Km for Expt R =

1.6 mmol dm-3


7.5 mmol dm-3

[
[

L
L

. 13
...

(iii}

When malonate anion is added,


Vmax remains approximately the same

Km increases

=> Features of a competitive inhibitor


=> Malonate anion behaves as a competitive inhibitor.
(b)

Tertiary structure of protein:

Refers to the overall three-dimensional shape of a formed when a polypeptide


chain folds or coils extensively to form a precise; complex, rigid, 3dimensional structure.

The -folding and coiling frequently gives rise to a globular protein.

The different parts of the chain are stabilized by hydrogen bonds and other
weak interactions between the R groups of the amino acid residues:

',.

_:-:

ll
_j

..

..

~-.

:,.

'

.. .
~

''14:
,

. :

..

L
[
[
CATHOLIC JUNIOR COLLEGE
PRELIMINARY EXAMINATION 2006

[
'[
[
[
[

c
r

catholic junior college catholic junior college catholic junior college catholic junior college catholic junior
college catholic junior college catholic junior college catholic junior college catholic junior college catholic
junior college catholic junior college catholic junior college catholic junior college catholic junior college
catholic junior college catholic junior college catholic junior college catholic junior college catholic junior
college catholic junior college catholic junior college catholic junior college catholic junior college catholic
junior college catholic junior college catholic junior college catholic junior college catholic junior college cath

CHEMISTRY

9251/1

PAPERl
13 SEP 2006

lHOUR

INSTRUCTIONS TO CANDIDATES

~,

-.'

Do not open this booklet until you are told to do so.

There are forty questions in this paper. Answer all questions. For each question,
there are four possible answers, A, B, -C and D. Choose the one you consider
correct and record your choice in soft pencil on the separate answer sheet.

Each correct answer will score one mark. A mark will not be deducted for a
- wrong answer.

,<'

r
f'

[
[

L
[

A Data Booklet is provided.

r
(
For each question there are four possible answers; A, B, C and D. Choose the one you
consider to be correct.
1

Solutions of 45 cm3 of 0.8 mol dm-3 of potassimn iodide and 30 cm3 of 0.5 mol dm-3
oflead(II) nitrate were mixed together according to the equation below:

2 KI(aq) + Pb(N03)2(aq)

E4F

EF4

EF

EF6

Which of the following atoms or ions has 3 unpaired electrons?

S2 -

Zn2 +

~--

Consider the molecule:


0

II
H2C==CH-- C-CH2--c==N
I

5-

What is the hybridization of each of the carbon atoms, C-1 to C-5?

sp3

_C-5
sp
sp3

sp3

sp3

. sp

sp3

sp3

sp3

C-1.,

C-2.
sp2

C-3
sp2

C-4
sp3

sp2

sp2

sp2

. sp2
sp2

sp2
sp2

5.

[
[

An element E has the electronic configuration (Kr]4d 5s 5p2.


What is the likely formula for the fluoride ofE?

A
4

0.036 mol dm -J
0.080 mol dm-3
10

0.006 mol dm'3


0.023 mol dm-3

[
[

Pbl2(s) + 2 KN0 3(aq)

What will be the approximate concentration of the iodide ions in the solution after the
reaction has completed?
A
B

sp~

Tetracyanoethylene, C6N4, has the skeleton shown below:

N"

/N

L
L
L
L

c"
/c
/c-c"
/c
c"'N

How many sigma and pi bonds are there in one molecule of tetracyanoethylene?
A
B

5 sigma and 9 pi
6 sigma and 8 pi

Prelim Exam 2006 Chem P 1

c
D

9 sigma and 7 pi
9 sigma and 9 pi

[_

L
r~

The standard enthalpy change of combustion of ethyne, C2H 2(g) is -1301 kJ mor 1 and
the standard enthalpy change of the reaction below is -623 kJ mor 1

3 C2H2(g)

organa-nickel catalyst (60-70 C)

c6H 6(1)

What is the standard enthalpy change of combustion of benzene in kJ mor 1?

f~
[

-678

-2034

-3280

-3903

The yellow gas chlorine dioxide, Cl02 , has been used for many years as a flourimproving agent in bread-making. It can be made in the laboratory by the following
reaction:
2 AgCl03(s) + Cb(g)
2 AgCl(s) + 2 Cl0 2(g) + 02(g);
~H=O

Which of the following statements is correct about the above reaction?

A
B

The activation energies for both the forward and reverse reactions are zero.
The equilibrium constant is independent of temperature.
The concentration of the reactants are equal to the concentration of the
products.
Its rate constants do not vary with temperature.

r~

Consider the equilibrium system N20 4

2N0 2 .

~,

It was fotmd that N 2 0 4 is 30% dissociated at 303K and 1 atm. What is the Kp for-the
above reaction?

[
L
"

0.129

0.428

0.396

0.857

-'

['
[

A typical aspirin tablet contains 324 mg of aspirin (acetylsalicylic acid C 9H 80 4 , Mr =


180), a monobasic. acid of Ka = 3.00 x 10'4 .mol dm-3. When 2 aspirin tablets were
dissolved in a glass containing 300 cm3 of water, what is the pH of the solution?

10

1.22

2.72

2.87

5.44

The pKb value for aqueous ammonia at 25 oc is 4.80. What is the pH of a solution
containing 100 cm3 of 0.440 mol dm -3 aqueous ammonia and 100 cm3 of 0.240 mol
dm- 3 ofhydrochloric acid?
A

4.72

4.88

9.12

9.28

[_

L
l
[

Prelim Exam 2006 Chem P 1

r
c
11

When ammonium dichromate(VI), (NH4 )2Cr20 7 is heated, an exothermic reaction is


initiated which results in the evolution of much gas, and leaves as its only solid
residue a grey-green solid powder ofCr20 3 .
By using oxidation numbers, which of the following can be the possible identity of
the gas?
A

12

N2

N20

NO

N02

[
[

h + 2s2o/ -+ 2r + s4oi-

.,

The anion gets converted to Chat the end of the reaction. What is the value of n?

13

Chlorine is a Group VII element and has the ability to exist in more than one
oxidation state.
NaC1011 (where n represents the number of oxygen atoms) is a strong oxidizing agent.
When 2 moles of the salt was reacted with excess potassium iodide, iodine was
liberated. It was found that the iodine liberated required 10 moles of the thiosulphate
solution for complete reaction, according to the equation

[_,

The use of the Data Booklet is relevant to this question.


A strip of copper is placed in a 1 mol dm-3 solution of copper nitrate and a strip of
silver is placed in a 1 mol dm-3 solution of silver nitrate. The two metal strips are
connected to a voltmeter by wires and a salt bridge connects the solutions.

L
[

When the voltmeter is removed and the two electrodes are connected by a wire, which
of the following does not take place?

B
C
D

Electrons flow in the external circuit -from the copper electrode to the silver
electrode.
The silver electrode increases in mass as the cell operates.
There is a net general movement of silver ions through the salt bridge to the
copper half-,cell.
Some positive copper ions pass through the salt bridge from the copper halfcell to the silver half-cell.

l
[

Prelim Exam 2006 Chem P 1

L
L

L
[

14

In the conversion of compound X into compound Z, it


proceeded according to the rate equation:

was found that the reaction

Rate =k [X]

The intermediate Y can be isolated.


Which reaction profile fits these data?

r-

~IJ\

[
[
[

progress of reaction

progress of reaction

..

..

~~~

[
[
[

.::

~
progress of reaction

progress of reaction

15

-- The graph below shows the variation in the boiling points for eight consecutive
" elements in the Periodic Table; all with atomic number between 10 and 20.

:a

Pi.

.tiil,.

~'

L
[

L
[

Atomic number

Which of the following can be deduced from the above?


A
B
C
D

Element D forms an acidic oxide only.


Element G does not conduct electricity.
Element C can exist in two allotropic forms.
Element A and beryllium are in the same group.

Prelim Exam 2006 Clrem PI

L
[

.0

c
16

Aluminium ch101ide is used as catalyst in certain reactions, in which it forms


carbocations (carbonium ions) vvith chloroalkanes.
RCl + AlCb

--+

R+ + AlCl4-

Which property makes it possible for this reaction to occur?


A
B
C
D

[
[

AlC1 3 is a covalent molecule.


AlCh exists as the dimer AhC1 6 in the vapour.
The aluminum atom in A1Cl 3 has an incomplete octet of electrons.
The chlorine atom in RCl has a vacant p orbital.

L
[

17

A yellow precipitate of cadmium(II) sulphide is formed when H2S is passed into an


aqueous solution of cadmium(II) ions, Cd2+. This precipitate is also obtained in the
presence of dilute hydrochloric acid but not in the presence of concentrated
hydrochloric acid nor in an excess of potassium chloride.

Which explanation accounts for all these observations?


A
B

C
D

18

The presence of a high concentration of W(aq) suppresses the ionisation of


HzS(aq).
The concentration ofS 2 -(aq) is reduced by the formation ofSCl/-(aq).
CdS(s) is soluble in concentrated HCl(aq).
Cd2\aq) ions react with Cr(aq) to form the complex ion CdCl/-(aq).

r
[

L,

The hexa-aquairon(III) ion hydrolyses as follows:

[Fe(Hz0)6] 3+(aq) + HzO(l);.: [Fe(HzO)sOHf+(aq) + HJO\aq)


Which of the following deductions made from the above equilibrium is false?
A
B
C

The corresponding iron(II) ion, [Fe(H20) 6] 2+, is less likely to undergo


hydrolysis.
The hexa-aquairon(III) solution can react with aqueous NaOH.
The .iron tmdergoes.a change in oxidation:state.
This hydrolysis is favoured by high pH values.

L
[
[

19

Two moles of a calcium compmmd is strongly heated with a Bunsen flame and the
gas produced is collected at room temperature and pressure. From which calcium
compound is 120 dm3 of gas likely to be collected? (One mole of any gas occupies 24
dm3 at room temperature and pressure.)
A

CaCb

Ca(N03)2

CaC03

L
l

Ca(OH)2

[
Prelim Exam 2006 Cltem P 1

L
[

20

[
[

lz

A
21

22

HzO

!3-

S0z

In whi~h of the following pairs of compounds is the boiling point of the second
compound higher than that of the first compound?
A
B
C
D

[
[

Which of the following product is not formed when excess iodide ions react with
concentrated sulphuric acid?

CH3CHzOH and CH30CH3


cis-HOzCCH=CHCOzH and trans-HOzCCH=CHCOzH
cis-CH3CH=CHCH3 and trans-CH3CH=CHCH3
HzNCHzCOzH and CH3CH2CONH2

The structure of methoprene is shown below:


OCH3

CH3

CH3

CH3

II

H2
H3c/C"-...,. /C"-...,. /CH
/C~ /C~ /C"'--. /CH"-.
H3C
C
C
"-...C
C
~C
0
CH3
H2
H2
H2

methoprene

How many stereoisomers does methoprene have?


21

22

23

24

[
23

The diagram shows the mass spectrum of an organic compound, X:


Rei. Abundance
10

43
90II-

7 IfI

60lr-

5 II-

31

4 IF-

L
L
l
L
L
['

3 II-

2 II-

74

II)E._

llllilllllllllillll,,,,,l

10

20

30

40
m/e

50

60

70

80

What is X most likely to be?


A
B

(CH3)3CC02H
CH3CH20CH2CH3

Prelim Exmn 2006 Chem PI

c
D

(CH3)2CHCHzOH
CH3C(OH)=CH(OH)
7

c
[
24

The elimination reaction of some halogenoalkanes is known to undergo a mechanism


known as E2. The mechanism is shown below:
R R
s:-~H"'-.~ :'/

_.c-c

R_,/

'\:x

[
[

' C=C/R + HB +X:R,(


"-.R

(Note: B:" is a base, eg. Olf, CN, NH 3;R can be eitherH or alkyl group)

Which of the following is not likely to undergo the mechanism described above?

25

1-bromo-2,3-dimethylbutane

2~bromo-2,3-dimethylbutane

C
D

1-bromo-2,2-dimethylbutane
2-bromo-3,3-dimethylbutane

[
f

The structures of two alarm pheromones for ants are given.

CH3CH2CH2CH=CHCHO and CH3CH2CH(CH3)COCH2CH3


Which characteristic applies to both compounds?
A
B
C
D

Both can be obtained by the oxidation of alcohols.


Both decolourise aqueous bromine.
Both decolourise dilute alkaline potassium manganate(VII).
Both showsoptical isomerism.

r~
~~---

[
26

Butanedione, CH3COCOCH3 (a yellow liquid), is responsible for the cheese-like


smell in cheese and from unwashed feet.
Which set gives the results expected from the reaction of butanedione with
2,4-dinitrophenylhydrazine reagent and with Fehling's reagent?

A
B

c
D

. 2, 4-dinitrophenylhydrazine
positive
positive
negative
negative

Prelim Exam 2006 Chem P 1

.Fehling's
positive
negative
positive
negative

L
L
L
L
L
{

l
r~.

27

The mechanism of base hydrolysis of an ester is shown below:

+ oH- base hydrolysis

II
c

R/

R/ "OR'

step

I
.-C

p;; "oH

II

step2

ROH

"'-o-

11
o-

II
c

+ Ro-

~ "oH

RO

Which of the following describes the types of reaction occurring in step 1 and 2?

['

A
B

-,
r
-~

28

step 1
addition
addition
substitution
substitution

step 2
elimination
oxidation
elimination
condensation

Which sequence shows the nitrogen compounds of decreasing pKb?

-o-CH NH o,N-g-NH,

H,C~NH,

o,~NH, H3C~NH2

o,~NH,

(Q)-NH,

[
(

~,-

o-NH,

\Q)-NH, o-CH NH
2

-~

[
[

<Qr-NH HC-g-NH, Y-CH NHz


2

02N~NH2

o-CH,NH,

~NH,

~'

l
L
Prelim Exam 2006 Chern PI

l_

c
29

Part of the structure of a polymer is as shown below.


0

II I
I
-C-0-(CH) -O-C-N-(CH) - N Il
24

26

Which of the following statements about the polymer is false?

30

It is a condensation polymer.

B
C
D

It is both a polyester and a polyamide.


On acid hydrolysis, one of the products formed is C02
On alkaline hydrolysis, a salt, o(CH2)4o and free amine, H 2N(CH2) 6NH2 are
formed.

Which one of the following compounds will give rise to a 1H n.m.r spectrum
consisting of two singlets, one doublet and one quartet?
A

CH3CHzCH2NHz

CH3CH(CH3)0H

CH3CH(OH)2
H2NCH(CH3)C02H

c
[

L
[
[

[
[

L
[
[_

l
-L
Prelim Exam 2006 Chem P 1

10

r:
[

For each of the following questions, one or more of the three numbered statements 1 to 3 may
be correct. Decide whether each of the statements is or is not correct. The responses A to D
should be selected on the basis of:

L
[

1, 2 and 3 are
correct

1 and 2 only
are correct

2 and 3 only
are correct

1 only is
correct

No other combination of statements is used as a correct response.

31

The combustion of ethanol is described below

C2HsOH(l) + 302(g)

-t

2C02(g) + JH20(l)

L1H =- 1.37

103 kJ

Which of the following statements is/are true about the reaction?

1
2
3

L
[

32

-,

A car running on lead-free petrol is fitted with a catalytic converter to its exhaust
system. An investigation shows that its exhaust gases contain small amounts of
nitrogen oxides. Which modifications would lower the concentration of nitrogen
oxides in the exhaust gases?

1
2
3

[
33

The enthalpy change would be different if steam is produced.


This is not a redox reaction.
The products of the reactions occupy a larger volume than the reactants.

An increase in the surface area of the catalyst in the converter.


An increase in the rate of flow of the exhaust gases through the converter.
A much higher temperature of combustion in the engine.

Consider the following reaction mechanism:


P+Q

~R

~s

fast
slow

Which of the following relationships are consistent with the above mechanism?

-~

[
[

l
L
[

["

rate oc [P][Q]

Kc =

rate= d[S] = k2 [R]


dt
.

~
[P][Q]

Prelim Exam 1006 Chem P 1

(Kc: equilibrium constant)

11

[
t_
34

Which pairs of reagents when reacted together would produce a change in the
oxidation numbers of the metal atoms?
aqueous potassium manganate(VII) and acidified aqueous iron(II) sulphate
aqueous anunonia and an aqueous suspension of silver chloride.
aqueous ammonia and aqueous copper(II) sulphate.

l_

Which of the following property increases from fluorine to bromine in Group VII?

l.

1
2
3

35

1
2
3

36

The electron affinity of the element.


The acid strength of the hydrohalic acids.
The reducing power of the halide ion.

- Sodium lauryl sulphate, used as detergents, are converted industrially from long-chain
alkanes.

r~

II
II

CH3(CH2) 10CH2 0-S-ONa

[
[

sodium lauryl sulphate

Which of the following deductions can be made from the structure of this substance?
1
2
3

. Part of the structure is polar and is water-attracting.


The alkyl chain is soluble in oil droplets.
All the C-C-C bond angles are tetrahedral.

[
37

Which of the following explain why 2-methylpropan-2-ol, (CH3) 3COH, is more


difficult to oxidise than propan-2-ol,- (CH3) 2 CHOH?
1
2
3

2-methylpropan-2-ol does not contain the CH-OH grouping.


The 3 methyl groups on 2-methylpropan-2-ol, being much larger than
hydrogen atoms, pose .steric hindrance to .the attacking oxidising agent
2-methylpropan-2-ol has a higher boiling point.

L
L
l
Prelim Exam 2006 C/um P 1

12

L
L
r

L.

L
[

38

Below are the structures of compound X and Y:


0

r:

[
y

Which sets of reagents and conditions can be used to .distinguish between compounds
X andY?

1
2
3

39

Ethanoic acid does not react with ethylamine to form CH3CONHCH2CH3. Neither
does ethanoic acid react well with phenol to form an ester. Which of the following
statements are valid reasons to explain these observations?

2
3

[
40

acidified KMn0 4, reflux


aqueous alkaline iodine, heat
2, 4-dinitrophenylhydrazine

Ethanoic acid reacts with ethylamine in an acid-base neutralisation reaction


instead of a condensation reaction.
Phenol is not strong enough a nucleophile to attack the carboxylate carbon.
The C-0 bond in phenol is not easily broken.

Below is compound Z, a modified compound of the antipsychotic agent flupenthixol:

C02H

IN~OH

NJ .

r~
''-<'

L
[

OH

Which of the following statement is true?


1
The molecule has 4 stereoisomers.
2
It can react with 2 mols of dilute hydrochloric acid.
3
It reacts with 2 mols of aqueous bromine.

[
Prelim Exam 2006 Chem P 1

['

13

,---.,
l

-'

~----,

~
'

'

L
~
[

CATHOLIC JUNIOR COLLEGE


PRELIMINARY EXAMINATION 2006

[
[

catholic junior college catholic junior college catholic junior college catholic junior college catholic junior
college catholic junior college catholic junior college catholic junior college catholic junior college catholic
junior college catholic junior college catholic junior college catholic junior college catholic junior college
catholic junior college catholic junior college catholic junior college catholic junior college catholic junior
college catholic junior college catholic junior college catholic junior college catholic junior college catholic
junior college catholic junior college catholic junior college catholic junior college catholic junior college cath

NAME: .......................................... TUTORIAL GROUP: ............ .

CHEMISTRY

PAPER2

9251/2

13 SEP 2006

1 HOUR 30 MINUTES

INSTRUCTIONS TO CANDIDATES

Answer all questions.


Write your answers in the spaces provided on the question paper.

The number of marks is given in [ ] at the end of each question or part question.

A "Data .Booklet is provided.

("

You are reminded of the need for good English and clear presentation in your
answers.

--..._1

L
[
[~

L
[

Total

L
"

l(a)

Chromium oxide, Cr02 is an unusual oxide of chromium. It is normally found on the


magnetic layer of cassette tapes. This compound is both highly conductive and
ferromagnetic, which provides a good audio response.
When the oxide from a particular length of tape is dissolved in dilute sulphuric acid,
disproportionation occurs, forming Cr3+ and Cr2ol-. The resulting solution required
30.0 cm3 of 0.10 ~ol dm-3 Fe2+(aq) to completely reduce Cr20/- to c?+.
(i)

Suggest an equation tor the disproportionation of Cr02 in acid solution.

['

[
[

[
[
[

(ii)

Write an equation for the reaction of Cr20/- with Fe2+ in acid solution.

[
(iii)

Calculate the mass of Cr02 in the length of cassette tape.

[
[

[
[
(iv)

If 12.5% of the original length of tape was used for the experiment, calculate
the. original length of the tape. (Width -of the tape-= 0.5 em; thickness = 0:1
mm; coating density of Cr02 = 224 g em-\

[
[

[
[7]

L
[

Prelim Exam 2006 Chem P 2

L
l
l'

l
['
[

(b)

Phosphorus acid, H3P03, is a dibasic acid which can be obtained by the hydrolysis of
phosphorus(III) oxide with ice-cold water. In the molecule, one of the hydrogen atoms
is bonded directly to phosphorus and it is a tetrahedral structure around the
phosphorus.
(i)

Draw a dot-cross diagram for the structure of phosphorous acid, H3P03.

(ii)

What does your structure suggest about the relative strengths of the P-0 and
P=O bonds?

[
[
[
[
...........................................................................................................

[
(iii)

On the other hand, phosphoric acid, H3P0 4 is a tribasic acid.


Draw the dot-cross diagram for phosphoric acid.

[
(3]

L
[
[
[

[Total: 10]

2 (a)

The decomposition of hydrogen peroxide is a first order reaction.


2H202(aq)

2H20(l) + 02(g)

The rate constant k for the reaction is 1.06 x 10-3 min- 1.

(i)

Write the rate equation for the decomposition of H202.

(ii)

How long will it take for 15 % of a sample of H202 to decompose?

L
L
L
f_-

Prelim Exam 2006 Chem P 2

[
(iii)

The decomposition of hydrogen peroxide can be catalysed by bromide ions.


The rate equation is: rate = k(H20 2] [Brl Give a 2-step mechanism involving
Bro intermediate that is consistent with this rate equation.

[
[

[
[5]

(b) Iodide ions are oxidized in acid solution by hydrogen peroxide.

H20z(aq) + 2W(aq) + 2r(aq) --) h(aq) + 2H20(1)


(i)

State two conditions that determine whether a reaction would occur when
molecules collide.

[
(ii)

Assuming the rate of the above reaction is zero order with respect to [H+],
determin~ the order of reaction with respect to [H20 2] and [r] based on the
experimental results shown below. Hence, determine the rate constant for the
reaction. (Assume order of reactions with respect to H20 2 and r are positive
integers.)
Expt
number

[H202]/mol dm-3

[r]/mol dm-3

Initial rate/
mmol dm-3 s 1

0.01

0.02

0.08

0.02

0.03

0.24

0.03

0.04

0.48

[
[

[
[
[

jl mmol = 10"3 mol]

L
[
r

.................... ...............................................................................

[5]
[Total: 10]

Prelim Exam 2006 Chern P 2

l
l
L
i

3 (a)

A saturated solution of cadmium hydroxide, Cd(OH)i, has a pH of9.56 at 25 C.


(i)

Write the

Ksp expression

of Cd(OH)2 .

....................................................................................................
(ii)

\. J

Determine the Ksp value for Cd(OH)2 .

[
[

L
[

[5]
(b)

When hydrogen chloride gas is bubbled into a saturated Cd(OH)2 solution, the
solubility of Cd(OH)2 is observed to increase. Explain this observation with reference
to a relevant equation.

[
[2]

[
(c)

Assume that hydrogen chloride gas dissolves completely into the Cd(OH)2 solution.
Calculate the volume of hydrogen chloride gas that should be bubbled into a 2 dm3
"solution of Cd(OH)2 at r.t.p. to increase the solubilityofCd(OH)2 to 8.00 x 10'2 mol
dm-3.
.

[
[

L
[

[3]
[Total: 10]

1,__

l
l
[

Prelim Exam 1006 Chem P l

\
r-

When aqueous iron(II) sulphate is boiled with an excess of sodium cyanide a yellow
solution is produced, from which pale yellow crystals can be obtained by evaporation.
These crystals contain 30.26% sodiU111, 18.42% iron, 23.68% carbon and 27.63%
nitrogen by mass.

Acidification of the yellow solution, followed by the passage of chlorine, gives a red
solution, from which dark red crystals can be obtained by evaporation.
(a)

(i)

i[

Calculate the empirical formula of the yellow crystals that were obtained by
evaporation.

il'L

c
[
(ii)

Write the balanced equation for the reaction between iron(II) sulphate and
sodium cyanide.

c
[

(iii)

(iv)

~'I

.State the role of chlorine in this reaction.

Write down the chemical formula of the dark red crystals that were obtained at
the end ofthe reaction .
.. w . . . . . . . . . . . . . . . . . . . . . . . . . . . . . . . . . . . . . . . . . . . . . . . . . . . . . . . . . . . . . . . . . . . . . . . . . . . . . . . . . . . . . . . . . . . . . . . . . . . . . . .

.(v)

- Hence, :write the balanced chemical equation for the reaction between the
yellow solution and chlorine gas.

....................................................... ' ......................................... .


[5]

c
lj

u
u
l

u
L
Prelim Exam 2006 Chem P 2

L:
r

L
[

(b)

By making reference to appropriate E6 values in the Data Booklet, discuss the relative
stabilities of the yellow and dark red complexes.

[
[

.......... 0

...... . . . . . . . . . . . . . . . . . . . .

............... 0

..................

..................................

0 oooo 000 0

o o o o 00

0 0000 ooooo oo 0

ooo ooo 000 oo 0

0 00 0

oooo oo 0

000

0 0 0 0 0 0 . Oo o o o o o

[2]

c
[

(c)

The oxidation of Fe3+ ions by NaOCl in a strongly alkaline solution produces a purple
solution from which a salt with the formula Na2Fe04 can be isolated. Chloride ions
are left in solution.
Calculate the oxidation number of iron in Na2Fe04 and suggest an ionic equation for
the formation of the Feo/ ion in the above reaction.

[
[

[
[

Equation: ........ .. .............


[Total: 1[3]
0]

l
L
Prelim Exam 2006 Chem P 2

l~

l
[
5 (a)

A Chemistry lecturer wanted his student to devise a synthetic method to convert


methylbenzene to 2-amino-3,5-dibromobenzoic acid. The following method was what
the student has suggested:

NH2

NH2

K2Cr207/H+

c.NH3
heat in
sealed tube

..

Br2 with c. HCI


H 20

Br

Br

His lecturer commented that there were errors to some of his suggested steps of the
synthesis.

(i)

Comment on the errors to his suggested method, stating clearly the reasons.

[
[
(ii)

Suggest a correct synthetic method, with the use of equations (need not be
balanced in this case), to show. how methyl benzene can be converted to
2-amino-3,5-dibromobenzoic acid, stating clearly the reagents and conditions
used in each step.

[
r~

r
t ..

[
[
[
[

[7]

l_
Prelim Exam 2006 Chem P 2

L
,-

\L.

r
[

(b) After synthesis, the student used NMR spectroscopy to confirm the identity of the
product. He obtained the 1H NMR spectrum of the compound as shown below.
-

[
[
[

12

10

PPM

(i)

From the NMR spectrum above, it was confirmed that 2-amino-3,5dibromobenzoic acid has been successfully synthesized. Using the splitting
patterns of the NMR spectrum above, explain how the synthesis of the product
is verified.

(ii)

What changes to the 1H NMR spectrum of the compound would you expect if
D20 solvent is used during the analysis?

[
[

c
[

l '.
-'

r
L

[3]
[Total: 10]

[_

L
['

Prelim Exam 2006 Chem P 2

l.
6 (a)

In the syntheses below, identify suitable reagents used in the different reaction stages
and detennine the structural formula of the intermediate I products formed.

c
[

Synthesis 1

[
CH3CN

Step 1

Step 2 ., CH3 C02CH2CH3

r
[

Step 1:

Reagents: .......................................................................................... .
1~

Conditions: ......................................................................................... .
Step 2:

Reagents: .......................................................................................... .
Conditions: ...... ................

'[J'J

Synthesis 2

o-OH

Step I

-ocoCH,

L,

r
[
[:

1. W(aq}, heat
.._

2. LiAIH4, r.t.

Compound
A&B

Step 1:

Reagents: ........................................................................... ' ............... ..

Conditions: ......................................................................................... .

L
Compound A

CompoundB
[3]

Prelim Exam 2006 Cltem P 2

10

L
l
l
L
\

t.

c
r
'

(b

Adrenalin is a hormone which raises blood pressure, increases the depth of breathing
and delays fatigue in muscles, thus allowing people to show great strength under
stress.

[
[
'

HQ

.-'

HO

CH(OH)CH2NHCH3

Adrenalin

Benzedrine is a pharmaceutical drug which stimulates the central nervous system in a


similar manner to adrenalin.

CH2CH(CH3)NH2

Benzedrine

f~
[

(i)

c
[
[

Suggest if adrenalin is more or less soluble in water than benzedrine. Give a


brief explanation.

....................................................................................... -. ... [2]


(c)

Give the full structural formula of the organic products formed when benzedrine
reacts with:
(ii) ethanoic acid

(i) ethanoyl chloride, CH3COC1

L
[

[2]
[Total: 10]

L
Prelim Exam 2006 Chem P 2

L
t

11

--

[~

~
'

~
,_
J

[
.r--,.

(b)

In recent developments of fuel cells, the direct methanol fuel cell (DMFC) has
successfully been demonstrated to be a better alternative compared to the
hydrogen-oxygen fuel cell, especially in applications such as the powering of
mobile phones and laptops. The DMFC has a similar setup as the hydrogen-oxygen
fuel cell except that it uses a solid polymer membrane as the electrolyte. The
overall cell reaction that occurs is:

2CH30H(l) + 30z(g) ..-. 2COz(g) + 4HzO(l)

(i)

If the same reaction occurs at the cathode for both the hydrogen-oxygen fuel
cell and DMFC, write the reaction that occurs at the anode for DMFC.

(ii)

List two advantages of DMFC over the conventional hydrogen-oxygen fuel


cell.
[3]

[
3

EITHER
0.010 mol ofhydrogen and 0.010 mol of iodine were placed in a 2.0 dm3 flask. The
mixture was heated to 400 C. The pressure was 1.1 atm. After an hour, the following
equilibrium was established.

(a)
"\

-"

H2(g) + I2(g) :::;:::::::: 2HI(g)

The flask was then rapidly cooled, the HI dissolved in water and made up to 100 cm
in a standard flask. 25.0 cm3 portions of the solution were pipetted and titrated against
O.lOmol dm-3_NaOH. The average titre value was 30.00 cm3

(i)

(ii)

Give an explanation for cooling the flask rapidly.


Calculate the number of moles of HI present at equilibrium. Hence calculate

Kc for the reaction.

;;:~~.

, (iii) Suggest with a reason what the value of Kp might be. (Do not carry out any
calculations to find the value ofKp)

[6]

[\
f'

t~

L
L
L
L

(b)

The experiment was repeated with the same amount of H 2 and h, in the same flask
but the temperature was increased to 600 C. The value ofKc was 0.36.
(i)

Calculate the number of moles ofHI at the new equilibrium.

(ii)

Based on your answer in (b)(i), state and explain briefly whether the enthalpy
change of the forward reaction is exothermic or endothermic.
[4]

Prelim Exam 2006 Chem P3

l'\

c
c

OR

0.10 mol dm-3 of methylamine is slowly added to 30.0 cm3 of 0.10 mol di11-3 of
hydrochloric acid and the change in pH is monitored and shown below:

pH

c
[
A

15

30

45

60

75

Volume of methylamine I cm3

(a)

Calculate the initial pH ofO.lO mol dm"3 solution of methylamine, given that the Kb
of methylamine is 4.37 x 10"4 mol dm"3
[2]

(b)

At point A,

L
L'
[
-.
f.
[
/

. (i)
(ii)

write an equation to show that the pH is less than 7,


calculate the concentration of the salt, CH3NH/Cr, formed,

[4]

f'

At a certain point in the titration, a solution of maximum buffer capacity can be


obtained.

(iii) determine the pH of the solution.

(c)

(i)

State the volume of methylamine used to reach this point.

(ii)

Calculate the pH of this solution.

lL '

(iii) Explain, with the help of an equation, how this solution is able to maintain pH
when a small amount of OH" is added.
[4]

L
Prelim Exam 1006 Chem P3

('

L
~

c
t

CATHOLIC JUNIOR COLLEGE


PRELIMINARY EXAMINATION 2006

r
[

catholic junior college catholic junior college catholic junior college catholic junior college catholic junior
college catholic junior college catholic junior college catholic junior college catholic junior college catholic
junior college catholic junior college catholic junior college catholic junior college catholic junior college
catholic junior college catholic junior college catholic junior college catholic junior college catholiC<. junior
college catholic junior college catholic junior college catholic junior college catholic junior college catholic
junior college catholic junior college catholic junior college catholic junior college catholic junior college cath

CHEMISTRY

9251/3

PAPER3
19 SEP 2006

2 HOURS 45 MINUTES

INSTRUCTIONS TO CANDIDATES

[
r-

Answer all questions in Sections A, B and C. The last question in each Section
is of the form either/or.

In Section D, you . are required to answer both questions, either from the
Environmental Chemistry Option or Biochemistry Option.

Hand in your answers to Sections A, B, C and D separately.

The number of marks is given in [ ] at the end of each question or part question.

['

A Data Booklet is provided.

You are reminded of the need for good English and clear presentation in your
answers.

L
L
L
f

L~

r
\

.'

L:
. ...

X is a gas having a certain mass, m. The table below shows the volume occupied by
X at different pressures.
Volume/ cm3
1000
31.2
14.9
-~

Temperature I K
273
273
273
-

---

---

----

(b)

An .organic compound, Y, contains carbon, hydrogen and oxygen only. When


vapourised at 101 kPa and 373 K, 0.100 g of Y occupied a volume of 66.7 cm3
Calculate the relative molecular mass ofY.
[3]

(c)

Diamond is one of the hardest natural substances. Before the production of


carborundum (silicon carbide, SiC), powdered diamond was most widely used as an
abrasive. Carborundum is made by heating coke with sand (silicon(IV) oxide) at2500

oc.

(i)

Explain, in terms of structure and bonding, why diamond and carborundum


are suitable as abrasives.

(ii)

"Diamonds are a girl's best friend". Suggest why carborundum has not
superseded diamonds in this context.
[4]

(ii)

Draw a labelled diagram of a hydrogen-oxygen fuel cell in an alkaline


electrolyte.
In the hydrogen-oxygen- fuel
according to the equation:

~cell,

C.

Using the information, deduce if X shows ideal behaviour. You may include relevant
calculations to explain your answer.
(3]

2(a) (i)

f.

Answer all the questions from this section.

Pressure I atm
1
30
60

Section A

l(a)

oxygen gas reacts at one ofthe.electrodes

02(g) + 2H20(l) + 4e- ~ 40K(aq)

-1

__J

u
L

u
[
[

L
[

Write the equation for the reaction of hydrogen at the other electrode.
(iii)

The hydrogen-oxygen fuel cell operates on the simple reaction:


2H2(g) + 02(g) ~ 2H20(l)
If the cell produces 1.5 A of cui-rent, and if hydrogen is contained in a 1 dm3
tank at 200 atm pressure at 25 C, how long can the fuel cell operate before
the hydrogen runs out? (Assume there is an unlimited supply of02.)
[1 atm = 1.01 x 105 Pa]
[7]

Prelim Exam 2006 Chem P3

L
t.
L

L
[

Section B

r
[

Answer all the questions from this section.


4 (a)

The elements A, Band C can be one of the following: Na, Mg, Al, Si and P.
Both the chloride and oxide of A can react with aqueous sodium hydroxide.
Furthermore, the oxide of A can also react with aqueous hydrochloric aCid.

Element B has a chloride and an oxide which react vigorously with water to form
solutions containing strong acids.

Element C has a crystalline solid oxide with a very high melting point. This oxide is
classified as an acidic oxide but it is not soluble in water.

,'

Identify the elements A, B and C. Explain the observations :with the help of relevant

[7]
balanced equations where necessary.

(b)

f'

A chloride of Group V element has the formula MCh. When 0.100 of MCh was
added to water and the resulting solution titrated with 0.0500 mol dm silver nitrate,
it was found that 26.30 cm3 of aqueous silver nitrate was needed to precipitate all the
chloride ions. Use these data to calculate the Ar ofM and hence identify M.
[3]

.,.

L
[

EITHER
(a) Magnesium occurs naturally as the mineral carnallite,. KCL:MgClt.6H20.

(i) State what is observed and give a balanced equation for the reaction which occurs
when a solution of carnallite is treated with sodium hydroxide solution.

(ii) Suggest a test for the presence of chloride ions in the carnallite solution, giving
details of the reagents added, observations and equations of any reactions
occurring.
[5]

L
[

Prelim Exam 2006 Chem P3

r
(b)

The table below gives some information about the hydroxides of elements in Group
II.

Lattice
Enthalpy of
Solubility of
2 x Enthalpy of
energy/ hydration ofM2+/
Hydroxide M(OH)v' moles
hydration of
per 100 g of water kJ mor1
kJmor1
Oir/kJmor1

Be(OH)2
Mg(OH)2

II

8.0 x 1o1.6 x 1o-s

I
I
!I Ca(OH)2 II
! Sr(OH)2 II

2.5 x 1o-

II

4.1 x 10-

I! Ba(OH)2

3.4 x 10-

II

_jl

II

II
_jl

-3120
-2993

II

-2644
-2467

II
II

-2320

-1100

-1414

II
II
II
II

-1273

II.

-1100

-1960
-1890
-1562

I
I
I
I

-1100
-1100
-1100

(i)

Write an equation to show how discrete ion hydration enthalpies and lattice
energy are related to the enthalpy change of solution for Group II hydroxides.

(ii)

From the information given in the table above and your answer in (b)(i),
explain qualitatively the trend in solubility of Group ll hydroxides down the
group.

(b)

Write balanced equations to show how the reaction with S2ol- ion can be
used to illustrate the difference in oxidizing powers of chlorine and iodine.
[4]

When chlorine is dissolved in cold water, a pale green solution, chlorine water, is
foimed. A piece of universal indicator paper, dipped into chlorine water, first turns
red.,and then becomes white.

(i)

Write an equation for the reaction between chlorine and cold water.

(ii)

Explain the colour changes observed when universal indicator paper is dipped
into chlorine water.

(iii) Using your answer in (b)(i), write a displacement equation between the agent
that turns the universal indicator paper white and iodide ions in an acidic
(4]
solution at low temperatures.
(c)

f"'
L

(i) Using relevant Ee values from the Data Booklet, state and explain the trend in
oxidizing power of the halogens frorri chlorine to iodine.
(ii)

OR .
(a)

(iii) Explain why there is a difference in trend in the solubility of Group II


hydroxides as compared to Group II sulphates.
[5]

--5

Given samples of chloride and iodide salts, how would you distinguish them other
than by using concentrated sulphuric acid?
[2]

L
L
L
[

L
Prelim Exam 2006 Chem P3

L.

L
[

Section C

Answer all the questions from this section.

6 (a)

Suggest how each of the following multi-step transfonnations can be achieved.


Indicate reagents and conditions you would use for each step, but do not give
experimental details.

(i)

/0
/o-c""CH3CHO

(ii)

~
u

(ii)

To increase the ability to withstand high-impact and increase tensile strength


of rubber tyres, butadiene is copolymerized with another monomer (called a
copolymer) to form the following polymer:
. H

- I

fH4--r--eH,--CH~
H!
IQin
Give the structure of the copolymer.
[2]

_,

H02CCH2CH2CH2CH2C02H

Draw 1 repeat unit of the polymer formed from butadiene.

L
L
L

CH,

(i)

L
L

c/

Butadiene CH2=CH-CH=CH2, is one of the monomers used in the making oftyres.

_fHCHJ

[8]
(b)

CH 3CH

""-c-o

CompoundS, .C 19H21 03N, when reacted with hpt aqueous NaOH, forms a salt of
compound T and U, C5HnON. After acidification, compound T produces
effervescence with aqueous sodium carbonate. The 1H NMR spectrum of compound
T only has 2 peaks, a multiplet and a singlet in the ratio of 5:1 respectively.
Compound U does not decolourise acidified potassium manganate(VII) solution but
reacts with both sodium metal and dilute hydrochloric acid. Compound U can be
synthesized from compound R in 2 steps. Organic compound R contains bromine
and oxygen and is optically active. Its mass spectrum shows peaks at mass numbers
15, 17, 29, 152 and 154. (Bromine has 2 isotopes: 79Br and 81 Br.)
(i)

Deduce the structures of compounds R, S, T and U, explaining the chemical


processes involved, and write appropriate equations.

(ii)

State the reagents, conditions and the intermediate needed for the synthesis of
U fromR.
[10]

Prelim Exam 2006 Chem P3

r
8

EITHER
(a)

Pentan-1-ol, C 5H 11 OH, is important in the synthesis of organic compounds.


(i)

Give the structural formula of another primary alcohol which is an isomer of


pentan-1-ol.

(ii)

Write a balanced equation for the preparation of 1-bromopentane from pentan1-ol.

(iii)

If this preparation gives a yield of 60%, what mass of pentan-1-ol is required


to produce 15.0 g of 1-bromopentane?
[5]

(b)

Dehydration of pentan-1-ol produces pent-1-ene. Sketch a diagram and label the cr


and n bonds between the two carbon atoms in the alkene functional group.
[2]

. . (c)

Compounds A and B are among many compounds secreted by insects to attract


members of the same species.

[
[
[

CH3(CH2)3C02H
CH3C02(CH2)4CH3
B
A
Such compounds are used in traps to control insect populations. They need to be
made synthetically. Outline how compounds A and B could be prepared from
pentan-1-ol. In each synthesis, give the necessary reagents and conditions.
[3]

CH3CH2CH2COCH3
D

CH3CH2COCH2CH3

Which one of these compounds would react with alkaline aqueous iodine?
Draw the structural fonnulae of the products tormed during these reactions.

[
[2]

(b) .' The reduction of.C with,NaBH.fproduces just one alcohol; but a similar reduction of
D produces two isomers in equal amounts.

Explain these observations. Write equations or draw structures where appropriate. [3]
Starting from a.suitable carbonyl compound of your choice, and using a cyanohydrin
as an intermediate, devise a 3-stage synthesis of compound E.
CH3-......._ /OH

/c.....___

CH3

C02CH2CH3

Suggest reagents and conditions for each step, and draw the structural formula of
every intermediate compound.
[5]

Prelim Exam 2006 Cltem P3

c
[

Compounds C and D are both ketones.

,- (c)

OR

(a)

[
[

L
l
l
L

L
r

r
[

Section D
Answer both questions on only one option.

Biochemistry Option

[
[

Answer both questions.


9 (a)

[
[
[

[
[~

(b)

At a given enzyme concentration, the maximum rate of an enzyme-catalysed


reaction is 80 J.tmol of substrate converted per minute when the substrate
concentration is 100 J.tmol dm-3 The reaction Rroceeds at half its maximum rate
when the substrate concentration is 25 J.tmol dm- .
(i)

Include all this information on a labelled sketch graph of rate against


substrate concentration. The axes of your sketch need to be labelled with the
correct units. State the value of the Michaelis constant, Km, for the reaction.
[1 ~-tmol = 1 x 10-6 mol.]

(ii)

What would be the maximum rate of reaction if the enzyme concentration


were to be doubled?
[5]

To determine Km values more precisely, an alternative graph known as a double


reciprocal plot, or the Lineweaver-Burke graph can be drawn instead. From this
graph, different types of inhibitors can be distinguished.

[
[

lN

.inhibitor A~-"'

~no

"

inhibitor

L
[
[

1/[S)

Using they- and x-intercepts on the graph above, identify, with reasons, the types
of inhibition displayed by inhibitors A and B.
[3]

L
L
L
(-'

Prelim Exam 2006 Chem P3

l~

[
(c)

As part of the citric cycle, succinate is dehydrogenated to fumarate. The reaction is


catalysed by the enzyme succinate dehydrogenase:

~?
I

__.,coo-

CI-t.L

-OOC

-2H
succinate dehydrogenase_

succinate

cooHe--"

c
[

II

CH
-OOC/

fumarate

In terms of structures, explain why succinate dehydrogenase is affected by


malonate.

L
r~

-j

H,C,...COO'I
COO-

malonate

[2]

[
[

10

Deep-sea divers may experience 'nitrogen narcosis' during a dive. At sea level,
nitrogen is inert and dissolves in the blood. As a diver goes deeper underwater, the
partial pressure of nitrogen increases and more nitrogen dissolves in the blood. As a
consequence, nitrogen can diffuse into the phospholipids bilayer.

(a)

Draw a diagram of the phospholipids bilayer. State the nature of the intermolecular
forces acting at each end of a phospholipid in a bilayer.
[3]

(b)

Nitrogen collects in the alkyl part of the lipid bilayer.


How would nitrogen interact with this section of the lipid bilayer?

(ii)

Suggest what effect a high nitrogen concentration might have on the lipid
bilayer.

.~.The high nitr9gen ,concentration ,impairs. the .conduction oLnerve .impulses. Nerve
impulses rely upon movement ofNa+ and K+ into and out of the cell.
Describe how Na+ and K+ move into and out of cells such as nerve cells, with
reference to the parts of the cell and the molecules involved.
[4]

Prelim Exam 2006 Chem P3

c
[

(i)

[3]
, (c)

10

L
[

L
L
L
L
L
f

L
[
[

Environmental Chemistry. Option

Answer both questions

11

During the early 1970s nearly one million tonnes of CFCs were being manufactured
annually. CFCs were used widely as blowing agents for making foam, cleaning
agents, propellants and as a component of air conditioning units ..

(a)

Give three physical or chemical properties of CFCs relevant to their use in these
applications.
[2]

(b)

Using appropriate equations, describe how CFCh (CFC-11) can be involved in the
destruction of the ozone layer. Explain why a single molecule of CFC may destroy

[4]
many molecules of ozone.

(c)

CFCs may be classified according to both their global warming potential (GWP)
relative to carbon dioxide and their ozone depletion potential (ODP) relative to
CFC/3. The global warming potential is usually quoted to cover a specified period
of time.

The table below summarises this information for CFC/3 and CFzC/z (CFC-12).

ODP

[
[
[

[
[

(d)

20yearGWP

100 year GWP

CFCl3

4500

3400

CF2Cl2

3.2

7100

7100

(i)

Suggest why CF2 Cl2 has a greater value for its ODP.

(ii)

Suggest why there is a difference in the 20 year and 100 year GWP figures for
CFCl3 but not for CF2 C[z.
[3]

The CFCs used in air conditioning units have now been almost entirely replaced by
compounds such as CF3CHC[z (HCFC-123).
Explain why this compound is considered to be preferable to CFC/3 .

[1]

[
r
L

L
L

Prelim Exam 1006 Chem P3

11

[
'

12(a)

Agricultural land often becomes acidic over a period of time and may then need
liming with calcium carbonate to raise the pH.
(i)

Explain why growing plants tend to make the soil acidic.

(ii)

Liming neutralises the immediate acidity of the soil solution but also has a
longer term effect. Suggest how liming is able to resist a reduction in pH over
a longer period of time.

(iii) Suggest three reasons why it is important to raise the pH of soil to maintain
effective growing conditions.
[6]

(b)

Rice grows in paddy fields where the acidic conditions cause nitrate ions to be
reduced to ammonium ions.
(i)

Give the half-equation for the reduction of nitrate ions to ammonium ions.

(ii)

Explain how the water-saturated conditions of rice paddy fields also favour
the formation of ammonium ions.

(iii) While ammonium ions are soluble in water, explain why these ions are not
completely washed out of soil as a result of heavy rainfall.
(iv) Acidic rainwater, however, may cause some loss of ammonium ions from the
soil. Suggest how this could occur.
(4]

(
[

c
[

L
[

L
[

l'
[
[

["

L
L

L
L
L
Prelim Exam 2006 Chem P3

12

,..---..
I

'

r---,

'!""----!,

,..._.._,

/~
'
J

'

'',

JC2 2006 Preliminary Exam Paper 1 Answer Scheme


(iv)

Qn
No

2
3
4
5
6
7
8
9
10

Answer

D
B
A
A
D

c
B

c.

Qn
No

11
12
13
14
15
16
17
18
19
20

Answer

Qn
No

Answer

21
22
23
24
25
2(3
27
28
29
30

c
c
A
A

c
D

c
B

Qn
No

Answer

31
32
33
34
35
36
37
38
39
40

c
c
c
A
B
A

D
D

D
D
A
D

0 252

= 0.1125 cm 3
2.24

0.1125
Length of tape used=---= 22.5 em
0.5x0.01
Original length of tape=

~x 22.5
12.5

= 180 em

(bf Phosphorus acid, H3P03, is a dibasic acid which can be obtained by the
hydrolysis of phosphorus(Ill) oxide with ice-cold water. In the molecule, one
of the hydrogen atoms is bonded directly to phosphorus and it is a tetrahedral
stmcture around the phosphorus.
(i)

JC2 2006 Preliminary Exam Papet 2 Answer Scheme


l(a)

Volume of tape used=

c
A
B
B
B
A

If 12.5% of the original length of tape was used for the experiment,
calculate the original length of the tape. (Width of the tape = 0.5 em;
thickness= 0.1 mm; coating density ofCr0 2 = 2.24 gem'\

H
+

Chromium oxide, CrOz !san unusual oxide of chromiitm. It is nonnally found


on the magnetic layer of cassette tapes. This compound is both highly
conductive and ferromagnetic, which provides a good audio response. When
tl~e oxide :rom .a particular !eng!? of tap_; is dissolvef i.t\ dilute sulphuric .a~id,
dtsproport!Onatton occurs,. formmg cr' and Cr201. ; The resultmg solu,tton
required 30.0 cm 3 of 0.10 mol dm-3 'Fe2+(aq) to completely reduce Cr2o,- to
j+
.
Cr .

(ii) The relative strengths of both the P=O bond and the P-0 .bond are similar to
each other due to resonance effect..

(i)

(iii)

Suggest an equation for the disproportionation of Cr02 in acid solution.

b+P +x)

++ +

H+ .0.

6Cr0 2 + 1OH+-+ 4Cr + + Cr20l + SH20.


(ii)

Write an equatio11 for the reaction ofCr2ol with Fe2+ in acid solution.
Cr20l + 6Fe2+ + 141-t-> 6Fe3 + 2Cr3+ + 1H20

(iii)

Calculate the mass of Cr02 in the length of cassette tape.


30 0
xO.IO = 3.00 x 10'3
No. ofmol ofFe2+=
.
1000
No. of mol of Cr2ot =

.!.. x 3.00 xI o-'


6

'

= 5.00 x 10

2 (a)
3

No. of mol of Cr0 2 = 6 x 5.00 x 104 = 3:00 x I o3


Mass of Cr02; (3.00

Pl'elim E.'l:atJi 21HJ6 An.n~o'<.>rS J>aperl & J

I 0'3 )

(52.0 + 2

The decomposition of hydrogen peroxide is a lirst order reaction.


21-bOl(aq) ~ 2H20(l) + 01(g)
The rate constant kfor the reaction is !.06 x 10'3 min' 1.
(i)

Write the rate equation for the decomposition of H202.


Rate; k [H202]

16.0) = 0.252 g

Prdim Exam 20(16 Answers Paper/ & 3

r....__....

(_:

L-

l__;

._._.....,

',

\...-..)

1....--'

LJ

----.J

----'

--

'

L__;

L__.

"-----!

,__)

t..__l

.__)

._.._)

...._j

____j

,.

(ii)

How long will it take for 15 %of a sample of lh02 to de.compose?


Time taken for 50% to decompose= tw = ln2/k = 653.9 min
C,IC0 = (I /2)" where n =no. Y, lives
0.85 /I = (l/2)"
n = 0.234
Therefore time= 0.234 x 653.9 = 153.3 min
OR use C1 = C0

3.
(a)

A saturated solution of cadmium hydroxide, Cd(OH),, has a pH of9.56 at


25~C.

(i)

(iii)

(ii)

The decomposition of hydrogen peroxide can be catalysed by


bromide ions. The rate equation is: rate = k[H20 2] [Br"]. Give a 2step mechanism involving Bro intermediate that is consistent with
this rate equation.

Detem1ine the K,p value for Cd(OI-!)2


pOH = 14 9.56 = 4.44
[OH'] = 10 4 .4 4 = 3.631 x 10'5 mol dm'3

141

Hi'5 + 2 = !.815

10'5 mol dm' 3

K,p ofCd(OH)2 = [Cd 2'1[0H-] 2


= (1.815 X 10'5)(_3.631 x 10'1) 2
:_ 2.39 X !0' 14 mo1 3 Qfll-6

H202 + Br" - t H20 + Bro


tast

BrO' + Hz02 - t lhO + 02 + Br"


f.odide ions are oxidized in acid solution by hydrogen peroxide.

(b) When hydrogen chloride gas is bubbled into a saturated Cd(OH)2 solution, the

Hz02(aq) + 2H+(aq) + 21"(aq)--+ Iz(aq) + 2H20(I)


(i)

solubility of Cd(OH)l is observed to increase. Explain this observation with


reference to a relevant equa,tion.
[21
Cd(OHh (s) + aq~
Cd 2\aq)+ 201-f(aq) .......... (I)

State two conditions that determine whether a reaction occurs when


molecules collide.

!'v1olecules collide with sufficient energy(<: Ea) and correct orientation.


(ii)

Assuming the rate of tlie above reaction is zero order with respect to
pH, determine the order of reaction with respect to [H 20z] and [!"]
based on the experimental results shown below. Hence, detennine the
rate constant for the reaction. (Assume order of reactions with respect
to H20 2 and r are positive integers.)
xpt
[Ih02j/mol dnt"3
number

Initial rate/
mmol dm' 3 s1

[f]/mol dm'3

0.01

0.02

0.08

0.02

0.03

0.24

0.03

0.04

0.48
[I mmol =

~am

2006 .4nswers Pupt:rl & J

W ions fonned from dissociation ofHCI will react with the OH' ions to form
H20. [OH'J"will decrease, tlius resulting in an increase in the solubility of
CdCOH)1 .as the position of equilibrium en will shift to the right to counteract
the decrease (must include equation).
(c)

Assume that hydrogen ch!or.ide gas dissolves completely into the Cd(OH)2
solution. Calculate the volume 6f hydrogen chloride gas that should be
bubbled into a2 dm3 solution of Cd(OH)z at r.t.p. to incraase the solubility of
Cd(OHh to 8.00 x 102 molctnf3
Ksp of Cd(OHh = [Cd 2'1[0ll'f
2.39 X 10' 14 = (8.00 X J0'2)(0H'f
[OH'] = ~2:39x w- 14 /S.OOxt0-2 = 5.46

Jo' mol]

f31
X

10'

mol dm'3

No._ofmoles ofHCI to be added to the 2 dm 3 container


= 2 x Decrease in no. of moles of OH' per dm 3
7
5
= 2 x (3.631 x 1o - 5.46 x , o )
-7.153 x 105 mol

Rate= k(H 202]m [I']"


Expt 3 I Expt I
(0.48/ 0.08) = (0.03]"' [0.04]" I [0.01]"' [0.02]"
'6=3"' X 2"
3 X 2 =3m X 2"
By inspection and m, n are. positive integers, m=l, n=l
Rate= k [H 202] (I']
Substitute values from expt I.
0.08 X 10' 3 = k (0.01) (0.02)
k = 0.4 mol' 1dm\'

Prelim

III

Solubility ofCd(O!ih = 3.631

slow

(b)

Write t11e Ksp expression of Cd(OH)2.


Ksp = [Cd 2'1[0H-f

:. Volume ofHCI gas needed= 7.153 x 105 x 24 = 1.72 x 10'3 dm 3 = 1.72 cm 3

Preli!tt E..wtm 211(16 Altswers Paper/ & .~

',11

,..----.

,---...
I

4.

I~

1.

When aqueous iron(!!) sulphate is boiled" with an excess of sodium cyanide a


yellow solution is produced, from which phle yellow crystals can be obtained by
evaporation. These crystals contain 30.26% sodium, 18.42% iron, 23.68% carbon
and 27.63% nitrogen by mass.
Acidification of the yellow solution, followed by the passage of chlorine, gives a
red solution, from which dark red crystals can be obtained by evaporation.

,....-,

(c) The oxidation ofFe3+ io.ns by NaOCI in a strongly alkaline solution produces a
purple solution from which a salt with the formula Na2Fe04 can be isolated.
Chloride ions are left in solution. Calculate the oxidatiotl number of iron in
Na2 FeO. and suggest an ionic equation for the formation of the FeO/ ion in
the above reaction.
Oxidation number of Fe in Na2Fe0 4 is..

(a) (i) Calculate the empirical formula of the yellow crystals that were obtained by
evaporation.

%
Mol
Mol ratio

Na
30.26

1.31
4

Fe
18.42
0.33

23.68

N
27.63

1.97

1.97

Thus, the empirical formula of the yellow crystals is Na4FefCN)

2 [Fe3 (aq) + 80R(aq)

= FeO/"(aq) + 4H 0(aq) + 3e"]

2Fe 3 + 3Cl0' + I OOH'

=2Feol + 3Cr + 51-lzO

3 [ClO'(aq) +1-hO(aq) + 2e =Cl""(aq) + 20H'(aq)]

5. (a) A Chemistry lecturer wanted his student to devise a synthetic method to convert
methylbenzene to 2-amino-3,5-dibromobenzoic acid. The following method
was what the student has suggested:
2

(ii) Write the balanced equation for the reaction between iron(II) sulphate and
sodium cyanide.

C0 H

c.NH 3

heat in
sealed tube

(iii) State the role of chlorine in this reaction.


Chlorine acts as an oxidizing agent in this reaction.
Or it oxidizes Fe(II) ions in NtLtFe(CN)G to Fe( III) ions to form Na3Fe(CN)r..
(iv) Write down the chemical formula of the dark red crystals that were

NHz

ACOzH NHz

Br2 with c. HCI


1-lzO
Br

Br

His lecturer commented that there were errors,to some of his suggested steps
of the synthesis.
(i) Comment on what are the enors to his suggested method. stating clearly the
reasons.

obtained from the second part of the reaction.

(v)

Hence, write the balanced chemical equation for the reaction between the
yellow solution and chlorine gas.

(b) (i) By making reference to appropriate E0 values on the Data Booklet, discuss
whethe.r the yellow complex is more or less stable than the dark red complex.
t.

Fe(CN)G 3"(aq) + e = Fe(CN)/


E0 = +036Y
3
As the half equation suggest, the Fe{Ill) in Fe{CN) 6 " can change
spontaneously to Fe(II) in Fe(CNli= since the E 0 vatue is positive,
4
implying that the vellow complex, Fe(CN)6 " is more stable as compared
to dark red complex, Fe(CN){.

Prelim '1:am 21HJ6Answers Pttperl & J

The first step from methylbenzene to benzoic acid, acidified K2Cr20 1 is


not strong enough an oxidizing agent to oxidize the methylbenzene to
benzoic acid. or n1ention use of acidified KMn0 4 instead.
For the second step there are 2 errors, the tirst error is electrophilic
substitution of the .benzei1e ring of benzoic acid cannot occur as NH3 is
not an electrophite, it is a nucleophile . So the. proposed second step is
unlikely to happen.
For the second step again, there is a -C0 1H gtoup on the benzene ring,
which is an electron-withdrawing group and should direct the incoming
electrophile to the me.ta positions, 3- and/or 5-positious with respect to

it.
(ii) Suggest the correct synthetic method, with the use of equations (need not
be balanced in this case), how the methylbenzene can be converted to 2amino-3,5-dibromobenzoic acid, stating clearly the reagents and
conditions used in each step.

Prebm E.wmt 1fHJ6 Answers. P~ttlei-1 & .t

:_-..j

c__

.____.....,

L...J

\.....__.

CH

c6rCH, NO,

c___;

I /NH,.
fQJ.

I) Sn with c. HCI

c. H,so,

2) NaOH(aq)

L-.J

L_i

:....____;

'
:...___;,

L-.

___J

;___)

:.._..;

:_j

___)

____)

(ii) What changes to the NMR spectra of both compounds would you expect if

IH/NH,

KMnO,;H

Br2 with c. HCI


H,o

D20 solvent is used during the analysis?


For 2-anlino-3,5-dibromobenzoic acid NMR spectrum, the peaks due to labile
protons of -CO~H and -NH~ groups will disappear .

6. In the syntheses below, identify suitable reagents used in the different reaction
stages and determine the structural fomwla of the intem1ediate I products
fonned.

( lm each for correct reagents and conditions used for each step.)

(Cannot oxidize methyl group to acid gp first)

LNH,

(Total; 4m)

(a) Syntbesis 1

Br/8\Br

Step 2

Step 1
CH,COOH

CI-!3CN

CH3C02CH2CH3

Step 1:

(b) After synthesis, the student used NMR spectroscopy to con finn the identity of
the product He obtained the ti-l NMR spectrum of the compound as shown
below.

Reagents: !-!+I H20


Conditions: reflux
Step 2:
Reagents: CH3CH20H
Conditions: cone. H2S04, reflux

(b) Synthesis 2

Q-oH

srepJ

Q-OCOCH3

ut (aq), heat
2. LiAIH,, r.t.

Step t:
Reagents: CH,COCI
Conditions: room temperature
12

(i)

10

PPM

Prom the NMR spectrum above, it was confinned that 2-amino-3,5dibrornobenzoic acid has been successfully synthesized. Using the splitting
patterns of the NMR spectrum above, explain how the synthesis of the product
is verified.
1 labile proton
/
Sinalet at 8 = 11 ppm

H*C02H NH2 " .


0 + I = I (Singlet)/
o = 8.0 ppm

Br

Compound A

----.._0 + 1 = 1 (Singlet)

Adrenal in

o = 7.5-7.9 ~pm

E.~.;(llu

1(}(16 Answers Paper/ & 3

Compound B

(b) Adrenal in is a honnone v,;hich raises blood pressure, increases the depth of
breathing and delays fatigue in muscles, thus allowing people to show
great strength under stre~s.

HO~C~OH)O>>NH'"

. Br
H

Prelim

Cl:-bCl-I20H

HO

""'2 labileprotons
Singlet at o= 4 ppm

I<0>- I
0

Prelim E.'Cttm 10(16 Amrwers Paper/ & .f

____..;

r'-"-1;

r--,
I

r---j

'

Benzedrine is a phanriaceutical which stimulates the central nervous


system in a similar manner to adrelialin.

g-b<,C"ICH,)NH,
Benzedrine
(i)

Suggest if adrenal in is more or less soluble in water than


benzedrine. Give a brief explanation.

Adrenatin is more soluble than benzedrine in water.


Adrenal in can fonn more extensive hydro!!en bonds per molecule with the
H20 molecules.
(c) Give the full stmctural formula ofthe organic products formed when
benzedrine reacts with:
(.i) ethanoic acid

(ii)

ethanoyl chloride, CH3COCI

Prdim \:nm 2fHJ6 Ailsh'ers Pupal & .-'

"

,----,
I
I

,---

r
' -

Pt Yt = (1)(1000) = 1000 atm cm


P2Y2 = (30)(31.2) = 936 atm cm3
P3V3 = (60)(14.9) = 894 atm cnl
Since PtY 1;t Pz Y2 ;t

(c)

?3 V3,

3.

(ii)

EITHER

(a) (i)

(ii)

The flask is rapidly cooled to slow down the rates of reaction in both directions.
No. of moles of NaOH = 0.10 x (30.00 /I 000) = 0.003 mol
No. of moles of HI in 25 cm3 =No. of moles ofNaOH = 0.003 mol
:. No. of moles of HI present at equilibrium= (100 + 25) x 0.003 = 0.012 mol

46.0

Structure: Giant covalent structure


,,
Strong covalent bonds between the atoms in the latticewhich require
to break. Therefore, they are hard and suitable for use as abrasives.

(i)

(,__..,

X does not show ideal gas behavior.

pV=nRT
pV =(m/Mr)RT
Mr = (mRT)/pV (1m)
(0.1 00)(8.31)(373)
(101 X 10 3 )(66.7 X I O"")

Compact, more portable, lightweight, minimize dangers of carrying H2 gas

(ii)

If gas exhibits ideal gas behavior, pV =constant


3

(b)

(b) (i)

JC2 2006 Preliminary Exam Paper 3 Answer Scheme


l(a)

,---....

Initial/ mol:
Eqmlmol:

a lot of energy

H2(g)
0.010
0.004

2HI(g)
0
0.012

K, = (HI] 2 I [H2][h] = (0.012 I 2)2 I (0.00412i = 9.00

(b) (i)

2(a)' (i)

H2(g)
+
Initial I mol: 0.010
Eqmlmol: 0.010-x

~Mfiti!ic
qJ~fcd

h(g)
0.010
0.004

(iii) Kp = K, = 9.00
The number of moles of gaseous reactants is the same as the no. of moles of gaseous
product.

Not aesthetically pleasing, shiny, sparkling, dazzfing


Not rare. (Either one; l m)

-===

12(&)
0.010
0.010- X

2Hl(g)
0
2-.:

K, = 0.36 = (2x) 2 / (O.QIO- x) 2

with

niddH

No. of moles of HI= 2(0.002308) = 0.00462 mol.


(i i)

The forward reaction is exothermic in nature.


As temperature increases, the amount of HI present at equilibrium is lower.

3. OR
(a)

(OH"] = ~Kb[CH3 NH 2 ] = J4.37x J0-4 (0.10)= 0.006611 mol dm-3


pOH=2.18
:.pH= 14-2.18 =ill

(iii) PV'=nRT
200 X 1.0 l X I OS X I X I 0"3 = n X 8.314 X 298 .
n (Moles ofH, gas}= 8.07
.
no. moles of electrons discharged= 211 = 16.14
:.charge Q= 16.14 x 96500 = 1557SIOC
Q=lt
.
t = 155751011.5 = I038340 s

(b)(i)
(ii)

.I

Total volume of solution= 60 cm 3


3
[CH3NH3i = 3011000 x 0.100 + 60/1000 = 0.05 mol dm"

Prelim .F-'I:am 20061tJS,'~r~ faf?er$

L-'

L__;

K
K,: ---..!!.:

Kb

K,

L__\

10 -14

I.--;

(___)

'---'

'

;
i.,.___:

1...-.-...:

L,.__)

[CH,NH/]

(iii)

Or

PCh<II + 3H20(1)-+ H3POJ(aqJ + 3HClcaqJ


PCls<sl + 4 H20 (I)-+ HJP04(oql + SHCI(aql

Or

P406(sJ + 6H20(I) -+ 4H3P03coqJ


P40IO(s) + 6H20{I)-+ 4H3P04(oq)

~K,[CHJNH/J = Jz.288x 10-11 (0.05): 1.07 X 10"6 mol dm' 3


:. pH= -lg(1.07

:.__J

i....---;

:.___)

__j

___!

Only phosphorous and silicon has both acidic chlorides and oxides. But only phosphorous
chloride and phosphorous oxide will react vigourously with water. Not silicon oxide and
silicon chloride.

:2.288xl0- 11

4.37 X IO-

=[Hp][cH 3NH2 ]

[H'l:

r
1
'------'

!0'6) = 5.97

Co.mpound B is P.
(c)(i) Vol of methylamine used= 60.0 cm

Si02 has an acidic nature as it reacts with alkalis but is insoluble in water and bas a high
melting point as it has a giant covalent structutc.

(ii)
pOH = pKb= -lg(4.37 x!0'4) = 3.36
:.pH= 14-3.36= 10.6

[or]

pH= pK, = -lg(2.288

I 0' 11 ) = I 0.6

Compound Cis Si.


(b)
No. of mol of Cl atoms in MCI) =No. of mol of AgCI formed
2630
=
X 0.0500
!000
= 1.32 x 10'3 mol

(iii) CH,NH/ + OH' -> CH3NH2 + H20


(added)
The small amount of excess OH' ions is removed by neutralisation with the large reservoir
of conjugate acid CH1M:L present in the buffer solution. Hence pH of the solution
remains fairly constant.

No. of mol of MCJ, used= .!. x No. of mol of AgCI


'
.
3
.
=

4(a)

Or
And

Al20J(s) + 2 NaOH(aq)+ 3 H20(!)-> 2 NaAI(OH)4(aq) (aluminate)

3
"'4.38

Since the oxide can react with both NaOH and HCI, it implies it is an amphoteric
compound. Among all the elements above, only Ah0 3 is amphoteric in nature.
Al20J(sl + 6 HCI<aql -+ 2 AICJ,,,ql + 3 H20m
AhOJ(s) + 6 HCI(oq)-+ AI2CI6faql + 3 H20(!)

.!..

1.32 X ] 0'3
x

10"' mol

Therefore, the A, of M = [(
,
,
4.38xlo=.121.8

0.1 00)- 3(35. 5)]

= 122

Thu.s, the identity of element M is Sb, Antimony.

AICI 3 when dissolves in H20, undergoes hydrolysis due to high polarizing power of AJ 3+ to
release H+ that causes solution to be acidic a1id can react with NaOH.
AICI) + 6 H20 _, [AI(H20)6] 3+<= [AI(H20)s(OH)f+ + H+ <= [Al(H20)4(0H)2] 2++ H+

EITHER
(a)

(i)

Whiteppt
Compound A is AI.
KCI.M~h.6H20

+ 2Na0H-+ Mg(OH) 2 + KCI + 2NaCI


OR Mg + 20H"-+ Mg(OHh '

(ii)
Reagent: aq. AgN03 \iith H+ added
Observation: white ppt
Equation: Ag\aq) + Cr(aq) _, AgCI(s)

Prelim E.t11m 2f~ll6 Answers Paper.J

Pr.e!im R-wm 20116 AllsHe~ PaperJ

II

r--l

/~

(.-----,

-~

:---iJ

6(a)(i)

(b) (i)
~Hsol = -LE + ~HhydM2+ + ~HhydoH-

(ii)

CH3CHO
HCN,little
KCN/NaOH,
i0-20"C

Solubility of M(OH)2 increases down Groltp


From (b)(i),_.lHsol dependent on LE & .lHhydM1+
Cationic size increases down the group. (1m)

CH3CH(OH)CN

heat

Down the Group, both LE & Ll.HhydM2+ decreases,

)0
UCH,

(ii)

But LE decreases more than Ll.HhydM2.. ,


Hence down the Group, a smaller +ve i-LE) value & larger -ve llHhydMl+ is
obtained, and overall the L\.Hsol becomes increasingly exothermic and hence more
soluble.
(iii)

KMnO,!H',

In Group II sulphaies, solubility decreases down the Group because


~Hsol becotries more endothennic down the Group.
This is because the so/ is larger than the OH'
The LE decreases less down the group compared to llHhydMz+,
LE a (qMz+ qanion I rM2+ + r onion)
If is Tanion is relatively larger value in so/, the LE is affected relatively less by
changes in the cation size.

reflux

H' (aq)

b,OH',
heat
(b)(i)
(ii)

OR
(i)

(a)

Ch +2e"-> 2Cr
Br2 + 2e' -> 2Br"
h +2e _, 2r

Ee
+1.36

+1.07V
+0.54 v

Oxidising powers of halogens decrease down the Group.


This is because Down the Group, E9 decreases. Hence ease of reduction of the
elements decreases.
CH2CH3

(ii)

4Ch + S20/' + SHzO-> 8Cr + 2SOt + IOH+


12 + 2Szo/ _, 2r +

U: HO----)-CH:r-NHz

s4ot

Ciz is more oxidizing than 12 as S is oxidized from +2 to +6 oxidation state by Clz


but only from +2 to + 2.5 by !2.

T:

Reaction with Na2C03 c:> has an acid/C02H gp.'(or Equation)

(b)(i)
(ii)
(iii)
(c)

Cl2(aq) + H20(I) "' HCI(aq) + HCIO(aq)

Add aq. AgN0 3 to salts, distinguish by observation of white ppt of AgCI and yellow
ppt of AgBr.

Prelim Exam 2flfJ6 Amwers Paper3

NMR Spectrum:
-- I, singlet: labile C0 2H proton

Universal indicator paper changes to red because of acid HCI present Subsequ.ently
it changes to white due to bleaching agent HCIO present
2I'(aq) + CIO'(aq) +2H(aq)-> lz(s) + Cl'(aq) + HzO(I)

R:

CH3

-- 5, multiplet: 5 H on monosubstituted benzene ring

S:

Reaction with Na

-DH gp present (or Equation)

No reaction with CrzOl'tH: tertiary ROH

Reaction with dil. HCI: basic /-NH 2 gp present

Prelim Exi1nt 2fHJ6 Afi.Slt-'er~ PaperJ

.__.._,

L-

L...J

R:

L_;

l__)

I_

(..__,;

L....J

...___.~

'

'-------'

[_J

l_j

[___:,

L....J

._._)

i_j

Mass Spectnun:
~

.... )

15- [CH3f

(Ill

0,

)'o

~ 17- [OHt

Vie

ld =

actual yield
theoretical yield

---~-

~=
15.0
I00 theoretical yield

~ 29 - (CH2CH3t

~ I 52 - [CH 3CH2C(OH/9BrCH3t

Therefore, theoretical yield= 15.0

~ 154- (CH3CH2C(OH) 81 BrCH3t

R ----+

KCN(alc~

reflux

;__]

L__j

'" \

;{iAIH4,
dry ether

Mass ofpentan-1-ohequired = n M,= 0.166 x 88


= 14.6g

HO-C-CN

(b)

\~bood

CH3

->

60

mol. ofpentan-1-o1 =mol. of 1-bromopentane


25.0
150.9
= 0.166

CH2CH3

Eq. S + 2 OH-

~ 1OO = 25.0 g

@-co2

+U

/~

(or hydrolysis)

[ !]Iabeii ing cr and


(1]sketch

1t

cr bond

8 Either
(c)

(a) (i)
. CHaCH2CHCH20H

I
CHa

OR

CHaCHCH2CH20H

CH3(CH2)aCH20H

OR

K2Cr2071!-1
CH3(CH2)aC02H
reflux

CHa
CH,

I
I
CHa

[I]

CHa--C-CH20H

CH3(CH2)aCH20H

CH3C02H
c. H2S04
reflux
[I]

CH3C02(CH2)4CH3
B

(ii)
CH 3CH2CH 2CH2CH20H + HBr - -

CH 3CH2CH 2CH2CH 2Br + H20

OR 3 CH3CH2CH2CH2CH20H + PBr3 - - 3 CH3CH2CH2CH2CH2Br + H3P03


[OR

CH3(CH2hCH20H

CH3COC!
r.t.

CH3C02~CH2)4CH3]

Prelim ~:am 2006 A.ft.fll.'ers Paper]

Prelim '1:am2(}06 AILfH.ers Paper]

___]

(~
\,
j

I~

r--1

_)

9 (a} (i)

80R

Rate or Vol

Correct shape of graph


with units & axes

fl!Uolmin'
(a)

Products: CHI 3

(b)

H
CHaCH2COCH2CHa

2IHJ

I
IOH

CHaCH2--c-CH2CHa

L_~--------------__J 1
25
100

[S] I pmol dm'

2IHJ

. .I
CH 3CH2CH,---C-CHa

Km = 25
(ii)

-~

Inhibitor A- non-competitive inhibitor

(b)

The 2 alcohol obtained from reduction of C has no chiral carbon, while that obtained from
reduction of D has one chiral carbon.

Therefore, 2 alcohol obtained from,reduction of D exists as a pair of optical isomers.

Reason: higher - -

Vm~

Vmax

Km

CH3

CHa

Km

Malonate- similar in structure as substrate, succinate.


:. malonate will compete for active site of enzyme succinate dehydrogenase.

(c)

(c)

(lower Vm~J, same - -- (same Km)

Inhibitor B --competitive inhibitor


I
I
Reason: same - - (same V max}, larger--,- (larger K.,)

\.c=o

~Lmol dm'

Max rate= 160 J.Lmol min'

l_....:oH

HCN; trace NaOH

10-2oc

IO(a)

cH("c""-cN

CHa

j :,:~ "

dil. H2so,
heat

Note: Diagram must


be labelled

At the hydrophilic heads, hydrogen bonds, ion-dipole or dipole-dipole forces.


At the hydrophobic tails, van der waals' intennolecular forces.
(b}(i}

(ii}
CH,

(c)

l~oH

/c""-.

CHa

C02CH2CH,

N2(g} is non-polar molecule,


Hence, it forms van der waals' forces with the hydrophobic alkyl tails.
High [N 2) will break/ disrupt the bilayer arrangement.
--Movement is facilitated by a pump;
--to go against concentration gradient, ie. from low concentration to high concentration.
-- Energy supplied from ATP hydrolysis is needed,
--to move K+ into the cell and Na+out of the cell,
(or High [K+] in the cell, low [Naj out of cell.

.. .Pr..eNm ''1:am1tJ06 A n~er:; PaperJ

10

'~

L,_

L-...:

L-.:,

r__;

L-.1

L.j

L...J

(......J

..

i__;

L......:

L._)

l.._...i

'

'---"""

i...----.1

:._j

____)

._j

..,_j

.____]
...

ll(a)

non-tlammable I inertness to combustion


volatile I easily liquefied
chemical inertness
non-toxic

(b) CFC/l
-+ CFCh' + C/'
C/'+0, -+ C/0' +02
C/0'+0 ~ C/' +02
Cl' is regenerated, and can thus destroy many ozone molecules.

(c) (i) it breaks down to form Cl' more easily.


(ii) CFC-11 have a shorter residence time than CFC-12.
CFC-12 have a ven' long residence time(> 100 years).
(d) HCFCs are more readily destroyed in the troposphere- since C-H bond is polarised and
this promotes the breakdown of the molecule.

Root respiration releases C02, thereby making the soil acidic.


or As nutrients are removed by growing plants, H+ ions occupying exchange sites are
released, thereby making the soil acidic.

12 (a) (i)

(ii). Ca2 ions from the liming displaces H+ ions from the exchange sites.
ci ions (being doubly charged) are held strongly at th.e cation exchange sites, and so
inhibits subsequent retention of W ions at the exchange site;
(iii) to prevent leaching of soil nutrients into soil solution.
to minimise release of A/ 3+ ions into soil solution.
to prevent development of reducing conditions in soil.
(b) (i) NOJ- + 10 W + 8e--+ NH/ + JH20
(ii) the water-saturated conditions restrict air circulation and this favours the reduction of
nitrate ions to NH/ ions.
(iii)because NH/ ions are held strongly at the cation exchange sites.
(iv)As [H+] increases, H+ ions displace other cations from the surface (at exchange sites)
and so, may cause some loss of N~ ions from the soil.

Prelim

'1:flnt

2006 A11swers Papcr.J

II
:4

.....,._

l
['"
[
millennia
institute

2006 Preliminary Examination


Pre-university 3

L
[
[
[

[
[
[
[
.

-'

CHEMISTRY

0?1:;1I I/1
'fiJ._.._,

Friday

15 September 2006

Additional materials:
OMR
Data Booklet
INSTRUCTIONS TO CANDIDATES
1.

Do not turn over this question paper until you are told to do so.

2.

Write your name, class and index number in the spaces provided at the top of this
page and on the OMR provided.

3.

Answer ALL questions and shade the correct answers on the OMR provided.

4.

Hand in the question paper and the OMR separately.

INFORMATION FOR CANDIDATES


Marks will not be deducted for wrong answers; your total score on this test will be the number of correct answers given.

FOR EXAMINER'S USE


TOTAL {40 marks)

[
[

L
L
L
[

1 hour

This paper consists of 16 printed pages

['

Section A (30 marks)


Answer all questions on the OMR form provided

1.

A sample of 10 dm 3 of polluted air is passed through lime water so that all the
carbon dioxide present is precipitated as calcium carbonate.

The mass of

calcium carbonate formed is 0.05g.

What is the percentage, by voiume, of carbon dioxide in the aii sample?


(1 mole of gas under experimental conditions has a volume of 24dm

2.

A.

0.03%

B.

0.05%

C.

0.12%

D.

0.30%

186

188

190

II

186

190

188

188

and

81

35 Br,

in equal

C2 H4Br2 is obtained. How would the

c
[

.-------

35 Br

190

79

186. 188

186

12

mle values above 184 appear?

c.

D.

I I I

B.

[
[

abundance. The mass spectrum for

A.

Bromine occurs naturally as two isotopes,

spectrum for

190

L
L
L
l
I~

L
[[

3.

Which. one of the following shows 2 molecules with the same molecular shape
relative to their constituent atoms?

[
[

4.

A.

BF3 and NH 3

B.

BeCI 2 and HCN

C.

HzO and HCN

D.

SOz and C0 2

The element radon, francium and radium are consecutive in the periodic table.
What is the order of their first ionisation energies?

Least endothermic

[
[
5.

Most endothermic

A.

Fr

Ra

Rn

B.

Fr

Rn

Ra

C.

Ra

Fr

Rn

D.

Rn

Ra

Fr

The formula of crystalline magnesium sulphate may be written MgS04.XHzO.

-,

Heating 4.3g of crystalline magnesium sulphate to constant mass produced

2.1 g of the anhydrous salt. The value of x is

[
[

A.
6.

B.

C.

D.

10

Ozone molecules in the stratosphere absorb much of the harmful radiation


from the sun.

Typically, the temperature and pressure of ozone in the

stratosphere are 250K and 1.0 x 1o-3 atm respectively.

How many ozone molecules are present in 1.0dm3 of air under these

conditions.
'

A.

L
L
r-

C.

1 X 10-3 X 6.02 X 10 23
8.31 X 250
1x101xl0-3
8.31 X 250
3

B.

1 X 101 X 6.02 X 10 23
8.31 X 250

D.

1 X 101 X 10-3 X 6.02 X 10 23


8.31 X 250

r
[
7.

The radius and charge of each of six ions are shown in the table below.

ion

radius/ nm

0.14

0.18

The ionic solids JX, LY and MZ are of the same lattice type. What is the
correct order of their lattice energies placing the one with the highest

8.

numericai vaiue first?

A.

JX > LY > MZ

B.

JX> MZ> LY

C.

LY>JX>MZ

D.

MZ> JX> LY

The enthalpy changes for two reactions ar~ given by the equations below.
2Fe (s) +

_% 02 (g)--..

~e

Fe203 (s)

=-822 kJ mor1

~e = -110 kJ mor1

C (s) + Yz02 (g)...,. CO (g)

What is the enthalpy change for the following reaction?

[
[
[

3C (s) + Fe203 (s)...,. 2Fe (s) + 3CO (g)

9.

+712 kJ mor 1

A.
B.

+492 kJ mor 1

C.

-492 kJ mor1

D.

-712 kJ mor1

[
[

A current of 8 A is passed for 100 minutes through molten aluminum oxide

using inert electrodeS; What will be the approximate volume of gas liberated,

measured at s.t.p?

A.

0.046dm 3

B.

2.8 dm3

C.

5.6 dm 3

D.

11.2 dm 3

L
L
L
L

l
[

10.

A substance has a high melting point and does not conduct electricity either as
a solid or when molten. The substance is likely to be

[
[
11.

A.

caesium chloride

B.

a ceramic

C.

silver

D.

astatine

A current is produced in the following cell.


Pt yz+ (aq), Y 3+ (aq)
X:o4- (aq), X 2+ (aq),

II

E at 298K/ V

Electrode
Pt Y2+ (aq), Y 3+ (aq)
Pt 1Xo4- (aq), X2+ (aq)

+0.77
+1.52

When a current is flowing, which one of the following species is oxidised?

A.

H+ (aq)

B.

xo4- (aq}

C.

y3+ (aq)

D.

y2+ (aq)

[
[
[
[

12.

Public swimming pools are often chlorinated to kill bacteria. As an alternative


. to chlorination, silver ions can be used in a concentration of not more than 1o-6
mol dm-3 and not less than 1o-7 mol dm-3 of silver ions.

..

Which of the following compounds would, in .saturated solution, provide the


necessary concentration of silver ion?

[
Compound

Solubility product
1

A.

Ag2C03

1 o-T mol dm-tl

B.

AgBr

10-l;l mol dm-o

C.

AgCI

10-10 mofl dm-1:i

D.

Agi03

1o-B mol 2 dm-o


j

L
L

L
f

w (aq) I Pt

L
13.

In the diagram, curve X was obtained by observing the decomposition of

~
[

100cm 3 of 1.0mol dm-3 hydrogen peroxide, catalysed by manganese (IV) oxide.


Volumeo
Ozformed

-v

..~

[
[

Which alteration to the original experimental conditions would produce curve

Y?

14.

A.

adding water

B.

adding some 0.1 mol dm-3 hydrogen peroxide

C.

using less manganese(IV) oxide

D.

lowering the temperature

[
[

During the industrial production of nitric acid, ammonia is oxidized to nitrogen


oxide,
4NH3(g) + 50z(g) ~ 4NO(g) + 6HzO(g)

=-909 kJ mor

Which of the following conditions favour the 'highest proportion of NO in the

[
r-

equilibrium mixture?

15.

A.

high pressure and high temperature

B.

h1ghpressure and' low terhper'ature

C.

low pressure and high temperature

D.

low pressure and low temperature

[
[

Which of the following elements is expected to show the greatest tendency to


form some covalent compounds?

A.

barium

B.

calcium

C.

magnesium

D.

potassium

[
L:

L
r

r-

16.

The table shows the results of experiments in which the halogens, X2 , Y 2 and

Zz were added to separate aqueous solutions containing

x-, y- and z ions.

Xz

no reaction

no reaction

no reaction

Yz

Xz formed

no reaction

Zz formed

Zz

Xz formed

no reaction

no reaction

[
Which set contains the ions

x-, y- and z in order of their decreasing strength

as a reducing agent?
Strongest

x
x

y-

Weakest

y-

z-

z
z
x-

y-

xy-

17.

r
L

[
[

. 18.

Which of the following ions is readily able to donate electrons?

A.

Al 3+

B.

Cu 2+

C.

Fe2+

D.

Mgz+

Water dissociates according. to the equation:

M-1 8 = +56 kJ mor 1

H20(I) <=> H+(aq) + OH-(aq)

At 25C, water has a pH of 7.

["

Which of the following occurs when water is heated to 30C?

['

A.

It remains neutral and its pH is 7.

B.
c.
D.

It remains neutral and its pH decreases.

l
L
L
r

It becomes acidic and its pH decreases.


It becomes acidic and its pH increases.

r
19.

In the conversion of compound P into compound R, it was found that reaction

proceeded by way of compound Q, which could be isolated. The steps

involved were

p -7 Q;

.6H is negative

Q-7 R;

.6H is positive

Which reaction profile fits these data?

R~;P-R
C

~-~

ProgreS$ of reacti<Mt

Progress of reaction

energy A\

/\

energy

[
[
[
~~,

Q
Progrcw 11freaetion

20.

''

Progn;ss of re!ction

The following reaction has a negative E 9 so that it does not occur under
standard conditions.

2N03-(aq) + 8H+(aq) + 6Cr(aq) -7 2NO(g) + 4Hz0(1) + 3Ciz{g)


Nevertheless, the reaction may be made to proceed under non-standard
-conditions.

[
[

L
[

Which of the following would not help the reaction to proceed?


A
8
C

increase in acidity
addition of potassium iodide
addition of potassium nitrate
addition of bromine

[
r
8

[
[

[-

21.

formed. This precipitate dissolves when excess NH3(aq) is added, forming a

deep blue solution. Which process does not occur in this sequence?

A
B
C
D

When drops of NH 3(aq) are added to Cu(N0 3h(aq), a pale blue precipitate is

22.

Dative bond formation.


Formation of a complex ion.
Precipitation of copper(ll) hydroxide.
Reduction of copper(ll) ions.

Consider the following four compounds:

i.

(CH3)3CH

ii.

CH3CHzCHzOH

iii.

CH3CHzCHzSH

iv.

CH3CHzCHzCH3

What is the order of increasing boiling point of the compounds (lowest first)?

[
[

23.

A.

i < iv< iii< ii

B.

ii < iii < iv < i

C.

iii <ii < iv< i

D.

iv < i < ii <iii

The-diagram below shows the structure of vitamin C.

o--H

H H
I

C-C-0

L
L
L
L
l
t

H/

I I
0 -H

How many chiral centre(s) is/ are there in one molecule?

A.
C.

1
3

B.
D.
9

2
4

L
(
24.

How many structural and cis-trans isomers are there for dichloropropene,
C3H4CI2?

25.

A.

B.

C.

D.

Which one of the following is a propagation step in the reaciion between


methane and chlorine?

26.

A.

Cl2 __,. 2CI

B.

CH3. + Cl. __,. CH3CI

C.

CH3. + HCI __,. CH3CI + H .

D.

CH3. + Cb __,. CH3CI + CJ"

r
r
[
[
[
[

The complete combustion of alkanes to produce carbon dioxide and water is


an important exothermic reaction.

Which line on the graph shows the relationship between the number of carbon
atoms in the alkane and the number of moles of oxygen gas needed for
complete combustion of the alkane?

c
[

[
A

L
Ooxygengas

~--------------D

L"

no. of carbon atoms In alkane

L
I

10

l
L
I-

27.

How many stereoisomers are possible for the compound Z shown below?

Qc(CN)CH3

OH

[
A

[
[

28.

Arrange the following in increasing pKa values:

f'A(OH

CH3CH2~

rA-fOH

CH3NH~

(Ill)

(II)

(I)

F-OOH JOH

[
[

(1), (II), (111), (IV)

(IV), (II), (Ill), (I)

(II), (1), (Ill), (IV)

(Ill), (IV), (1), (II)

r~

L~~

29.

A polymer has the following repeat unit.

-(CH2-CHCI-CH2-CH=CH-CH2)-

Which pair of monomers could be used to make this polymer?

A.

CH2=CHCI and CH2=CH2

B.

CH 2 =CHCI and CH2=CH-CH=CH2

C.

CH3-CH 2CI and CH3-CH=CH-CH3

D.

CH2=CCI-CH=CH2 and CH2=CH2

L
l
L
L

11

(IV}

l-,

c
30.

The H NMR spectrum of a naturally occurring compound is shown below:

l
[
[

....

14

14

.-----------------..

10

a/ppm

What can the compound be?

~COOH

:::"COOH

OH
B

8'COOH

~
0
0

[
[

Section B (10 marks)

[
For questions 31 - 40, in this section one or more of the three numbered statements
1 to 3, may be correct. Decide whether each of the statements is or is not correct
(you may find it helpful to put a tick against the statements which you consider to be
corr-ect). The responses A to D, should be selected on the basis of

1,2 and 3
"are correct

8
c
1 and 2
2and 3
onlyare correct only are .correct

D
1 only
is correct

L
I

[
r-

[
12

L
L
L

L
['

31.

[
[
[

32.

Which of the following is/are hydrogen bonded in the liquid state?

1.

CH2F2

2.

CH3NH2

3.

CH3CH20H

What is/are the condition(s) usually quoted for the standard electrode potentiai
of hydrogen to be 0.00 V?

"

1.

The concentration of H+ (aq) is 1 mol dm3 .

2.

The atmospheric pressure is 1 atm.

3.

The temperature is at 273K.

..,

[
I

c)

-~

33.

The table below contains data for three elements. Which of the element(s) is/
are transition metals?

~-

Element

melting point/ C

density/ g cm3

1.

1535

7.86

2.

660

2.70

3.

328

11.34 -

--,

-~

34.

Halothane (diagram below) is a widely used anaesthetic.

H F
I

L
L
r

L
f
L

l
c

CI-C-C-F

I I
Br F

Which statement(s) about halothane is/are correct?

1.

It is relatively unreactive.

2.

The molecule has a chiral centre.

. 3.

It may cause depletion of the ozone layer.


13

Li
[
35.

When bottles containing still (non-fizzy) wines are opened, the wine soon

becomes 'vinegary' even when the cork is replaced. Such wines may be kept

resealing the bottle.

How does this mixture prevent the 'vinegary' taste developing?

fresh by pumping in a mixture of carbon dioxide and nitrogen gases before

36.

1.

Oxidation of ethane! to ethanoic acid is prevented

2.

The acidity of the wine is decreased

3.

The amount of carbon dioxide dissolved in the wine increased

Acrolein is produced in photochemical smog. It has a strong smell, irritates


eyes and mucous membranes and is carcinogenic.

H
\
I

[
[
[

C=C,c 0
/

Acrolein
What can be deduced from this structure?

37.

1.

All bond angles are approximately 120.

2.

It will undergo electrophilic addition reactions.

3.

It will undergo nucleophilic addition readi()ns.

The ease with which fabrics become stained depends upon the nature of the
fibre used. Carpets made from wool, which is a protein, are relatively easily
, stained, whereas floor coverings made from poly(propene) are more stain

[
[

resistant.
Which

correct statement(s) explain why wool

poly( propene) is not?

1.

Poly(propene) is a saturated hydrocarbon.

2.

Wool contains side-chain hydroxy groups.

3.

Wool contains side-chain polar groups.


14

is easily stained and

L
l'
L
L
l

L
[

38.

In a bee hive, the queen bee secretes the substance below that will cause the
worker bees to construct the royal colony cells.

CH3CO(CH 2}sCH=CHCOzH

(Queen bee substance)

true?

1.

It gives a positive test with Fehling's solution.

2.

It gives a positive tri-iodomethane test.

3.

It could exist as cis-trans isomers.

From the structure shown above, which of the following statement(s) is/ are

[
[

39.

Drugs that are soluble in non-polar solvents accumulate in the fatty tissues of
the body and can be released into the blood stream over a period of 5 to 8
days. These drugs have a longer-lasting effect than water-soluble drugs.
The psychoactive drug in cannabis has the structure shown.

CHa

CH-CH2

I \ CH
\ C=CI HO
I \ C=CH2
H
CH2

[
[

I .

CH3
What functional groups in the molecular structure help it dissolve in fatty tissue?

1.
2.
3.

alkyl groups
alkene groups
hydroxy groups

[
15

L
L
[

CH2CH:tCH2CH2CH3

l'
L
40.

Which of these naturally-occurring compounds contain both a secondary

['

alcohol and an ester linkage?


CH3 -...

CH2
I

CH_...CH
3
1

o__a

CHsCCo fH2 0.

u2

OH

CH

lovastatln
OCH3 CH3

[
.,

HO

CHa

,'

CH
,,

CH3 CH3
bafifomycin A 1

[
3
HO

OH

'

,'

[~

gibberellic acid

L
[
--- End of paper --

L
[

16

L
L
L
r

f'

Millennia Institute
Preliminary Examination 2006

L
[

[
[
[
[

[
[
r.,
L

[
[
[

L
[

L
[

L
L

1
2
3
4
5
6
7
8

9
10
11
12
13
14
15
16
17
18
19
20

c
B
B
A

c
D
D
B
B
B

D
B
B

c
B

c
B

D
D

21
22
23
24
25
26
27
28
29
30
31
32
33
34
35
36
37
38
39
40

D
A
B

D
D
B

c
D
B
A

c
B

c
D
A
A

c
B
A

--

r-\

'

'

LCandidate Name:._ _ _ _ _ _ __

Class: 04_ __

f~

millennia
institute

[
[

Preliminary Examination 2006


Pre-university 3
CHEMiSTRY
PAPER 3

9251/3

[
Monday

[
[

[
[

11 Sept 2006

2 hours 45 minutes

Additional materials:
Data Booklet
Answer Paper
Graph Paper

READ THESE INSTRUCTIONS FIRST


Write your name, index number and class in the spaces at the top of this page and on all
the work you hand in.
Write in dark blue or black pen on both sides of the paper.

[
[

You may. use a pencil for any diagrams, graphs, or rough working.
Do not use staples, paper clips, highlighters, glue or correction fluid.

Answer all questions in Sections A, 8 and C. The last question in each section is of the
form either/or. In Section D, you are required to answer both questions on one option
topic.
At the end of the examination, fasten all your work securely together.

[
['

L
l
L
L
L

The number of marks is given in brackets [ ) at the end of each question or part question.

A Data Booklet is provided.


You are reminded of the need for good English and clear presentation in your answers.

This question paper consists of 10 printed pages.

(Turn over

t_

[
[

Section A: Physical Chemistry


1. The equation pV = nRT may be used in the determination of the relative molecular
mass of a volatile liquid. The diagram shows the apparatus that could be used in this
determination. A known mass of a volatile liquid is injected through the self-sealing
rubber cap into the gas syringe, which is then heated in a water bath for several
minutes before the volume of the vapour is noted.

[
[

//

[
[

Self-sealing
rubber cap-

(a) Suggest why it is essential to leave the syringe in the boiling water bath for several
[1]
minutes before reading the volume of the vapour.
(b) State three of the basic assumptions made in the kinetic theory of gases regarding
[3]
the behaviour of gaseous molecules.
(c) Using apparatus similar to that shown in the diagram, 0.167 g of ethanol, C2HsOH
was injected into a gas syringe.and the. syringe was then placed in a boiling water
bath for several minutes at 1 atm. The temperature of the water bath was 100C.
(i) Calculate the theoretical volume, in cm 3 , of ethanol vapour that would have been
[2]
produced under these conditions. Assume R 8.31 J K"1 mor1

{ii) The volume of vapour collected in the gas syringe was 70 cm 3 Comment on the
[2]
result.

(iii) Under what conditions of temperature and pressure would you expect the
[2]
behaviour of ethanol to be most like that of ideal gas? Explain your answer.

[
[
[

[
[

L
_-,
l
~

[Turn over

L
L
l
l
r

[
[

2. This question is about the following reaction:


H catalyst

[
[

[
[
[

[
[

Br2(aq) + HC02H(aq)

-7

28((aq) + 2W(aq) + C02(g)

Time/min
0.0
0.5
1.0
1.5
2.0

[Br2]/ mol dm-;j


0.0100
0.0090
0.0081
0.0073
0.0066

3.U

0.0053
0.0044
0.0028
0.0020
0.0013
0.0007

4.0
6.0
8.0
10.0
12.0

(a) These figures were obtained using a colorimeter to measure [Br2]. Give another way
by which the rate of reaction could have been followed.
[1]
(b) Plot a graph of [Br2] versus time.

[2]

(c) Determine the order of the reaction with respect to Br2.

[2]

(d) Given that the order of reaction with respect to methanoic acid is 1st order, write the
rate equation and state the units of its rate constant.
i2]

-~

r-

(e) In this experiment the [HC0 2H] was kept constant. How could this be achieved in
practice?
[1]
(f) The student wishes to modify the experiment by changing the concentration of

bromine. On the same axes, sketch another graph to show how the concentration of
bromine will change when the initial concentration of bromine is changed to 0.008
mol dm-3 .
[1]
(g) Find the initial rate of the reaction for the graph that you have sketched in (f).

[1]

[
[

L
L
L
L
L
L

[Turn over

L
[
[

3. either
(a) 4.6 g of ethanol and 12.0 g of ethanoic acid were mixed in a flask with 20 cm 3 of 1
mol dm-3 HCI as catalyst at room temperature. The contents of the flask were left for
a week to reach equilibrium, and then titrated with 1.0 mol dm-3 of sodium hydroxide.
137 cm 3 of NaOH was required for complete neutralisation of the reaction mixture.

[1]

for the reaction between


[1 J

,-

(i) Write an equation for the reaction of ethanol with ethanoic acid.
(ii) Write an expression for the equilibrium constant
ethanol and ethanoic acid.

Kc

(iii) Calculate the value of Kc at room temperature.

[4]

[
(b) 1.9 mol-of H2 and 1.9 mol of 12 were allowed to reach equilibrium at 710 Kin a 20
3
dm vessel. The equilibrium mixture contained 3.0 mol of HI. Determine the
equilibrium constant, Kp, of the reaction mixture given that the total pressure is 11.2
~-

[
[

Hz(g) + lz(g) -7 2HI(g)

[
f
[

--~

3. other
The ingredients of a typical lemonade are:

Ingredients: Carbonated water, Citric acid, Flavouring,


Acidity regulator: sodium citrate; artificial sweetener:
aspartame, Preservative: sodium benzoate; Stabilizer:
CMC

(a) Carbonated water is an aqueous solution of carbon dioxide. Some of the dissolved
carbon dioxide reacts to form carbonic acid, H2C03 .

The carbonic ,acid then ionise.s according to the following equation:


H2C03(aq) + HzO(I) ~

H30+(aq) + HC03- (aq)

pKa of carbonic acid =3. 7

(Turn over

L
l

L.

(i) Explain with the aid of an equation why HC03- is classified as a Lowry-Bronsted
base.
[1]

(ii) Write an expression for the acid dissociation constant, Ka, for carbonic acid.

[1]

(iii) Calculate the pH of 0.1 mol dm-3 carbonic acid. State any assumptions that you

have made.

[4]

(b) Let the formula of citric acid be HaA.


(l) Explain what is meant by the term buffer solution using a solution of citric acid
and sodium citrate as an example.
[2]

(ii) Calculate the pH of a buffer that contains 0.50 mol dm3 citric acid and 0.20 mol
dm-3 sodium citrate. (Ka of citric acid= 7.08 x 104 mol3 dm-9}
[11

"/.t:

(iii) For most weak acids, ionisation is an endothermic process. State how you will
expect Ka to change with an increase in concentration of the acid.
[1]

[
Section B: Inorganic Chemistry

r,
d

L
[

["
[

L
L
L
L
L

4. Iron is ad- block element. Its compounds are useful reagents in the laboratory.
(a) Using the s, p, d, f notation, write the electronic configuration of Fe and Fe3+.

[2]

(b) In an acidic aqueous solution, the dicromate(VI) ion, (Cr2 o/), is a powerful oxidising
agent. A pale green solution of iron(ll) ions can be oxidised by dicromate(VI) ions to
form-a yellow solution.
(i) By using <.:~ppropriate data from the Data booklet, construct a balanced equation
to illustrate the react15m between .Fe2+ and dicromate(VI) ion, (Cr2 0?") and
calculate the overall E .
[2]
(ii) Calculate the volume of acidified aqueous potassium dichromate (VI) of
concentration 0.0200 mol dm3 that will oxidise 25.00cm 3 of aqueous iron(ll)
sulphate of concentration 12.15 g dm3 .
[2]

(c) Compounds of iron are also good homogeneous catalysts. What feature of transition
metals make them suitable as homogeneous. catalysts? Illustrate your answer by
describing how compounds of iron can act as a homogeneous catalyst in the
following reaction.
[3]
21-(aq) + S 2ol(aq) -7 lz(aq) + 2Sol-(aq)
(d) Discuss the effect of cyanide ligands on the stability of Fe2+ and Fe3+ using
appropriate data from the Data booklet.
[1]

[Turn over

[
[
5. either

(a) The chlorides of the elements sodium to phosphorus all dissolve in or react with
water.
(i) State the formula of a chloride of each of these five elements.

[2]

(ii) Describe the reaction, if any, of the chlorides of sodium and silicon with water,
relating any differences to their bonding, Give equations for any reactions and
suggest the pH values of the resulting solutions.
[5]
(b) Sulphur and chlorine can react together to form S2CI2. When 1.00g of this sulphur
chloride reacted with water, 0.36g of a yellow precipitate was formed, together with a
solution containing a mixture of sulphurous acid, H2S03 , and hydrochloric acid.
Use the above data to deduce the equation for the reaction between S2CI2 and water.
[3]

l.
L
[

[
[

L
[
[

5. other
(a) State the formulae, "and describe the reactions with water, of the oxides of the
[6]
elements in the third period from sodium to sulphur.
(b) Campden tablets are composed of a mixture of sodium metabisulphite, Na2S20 5, and
various preservatives. They are used during wine making to sterilise equipment by
dissolving them in water and adding an organic acid, HA.
These,two components react together to produce an aqueous solution of sulphur
dioxide and the sodium salt of the organic acid. Write an- equation for the reaction of
sodium metabisulphite with the acid, and use it to predict the volume of sulphur
dioxide that will be produced at r.t.p from dissolving one tablet containing O.BOg of
Na2S20s.
[4]

['
r,
L.~

[
[

L
6

(Turn over

l
L
L
r-

L~

r
[
[
[

L
[

Section C: Organic Chemistry

6. A young researcher tried out a new synthetic route to synthesize an alcohol.


The alcohol contains the following elements with the given percentages by mass:
C, 64.9%,
H, 13.5%
0, 21.6%
(a) The peak with the highest m/e value in the mass spectrum of the alcohol was at m/e
[2]
74. Use these data to deduce the molecular formula of the alcohol.
Using 1H NMR spectroscopy, he confirmed that his synthetic pathway was successful.
The 1 H NMR spectrum of his product is shown below.
1

H NMR spectroscopy

9H

-,

[-[
[

1H

10

HPH-00-190

s
pprn

' 1

(b) Use the spectra to identify the alcohol, and draw its displayed formulae. Explain your
reasoning,
[3]

(c) Predict and explain the changes in the NMR spectra (if any) of adding D20 to the
[2]
samples.

(d) Explain why tetrachloromethane is widely-used as a solvent in n.m.r spectroscopy.[1]


(e)(i) Draw the structural formula of a possible isomer of the above alcohol.

[1]

(ii) Sketch a low resolution NMR spectra (ignore splitting pattern) for the isomer that
you have drawn in e(i).
[1]

L
L
L
[

[Turn over

r
7.

(a) Suggest reagents and conditions by which you could carry out the following
conversions:

(i) CH3CH2CH 20H to CH 3CH(CH 3)COOH

(ii) CH3CH2CH3 to CH3CH2CH20H

[6]

(b) The cyanide ion is a versatile species in organic reaction.


(i) Outline a mechanism for the reaction of hydrogen cyanide under suitable
conditions with butanone.
(ii) Draw the displayed formulae of all the possible isomers of the organic
product that can be formed in the reaction.
[4]

cL
[

[
[
[

8. either
(a) An organic compound H (C 2 H4 0 3) is oxidised to I (C 2 H2 0 3), which can be oxidised to
J (C2H204). H, I and J dissolve in water to give acidic solutions which decolourise
acidified potassium manganate(VII) on warming.
On treatment with phosphorus (V) chloride, all three react, with H and J giving 2
moles of hydrogen chloride per mole, and I giving 1 mole of hydrogen chloride per
mole. I gives a precipitate with2,4-dinitrophenylhydrazine.
Deduce the identity of H, I and J. Explain the chemistry of the reactions described,
writing equations where appropriate.
[6]
(b)-,For each of the following pairs ofcompounds, state the reagents and .conditions that
would enable you to distinguish between them. State clearly how each compound
behaves in the test and write balanced equations for the reactions involved.

[4]

[
[

L
[

(i) CH3GH2COOCH3 and Cr.J3CH2C.GNH2 . _


(ii) CH3COCH3 and CH3CH 2COCH2CH3

[Turn over

L
L
l
L
L
r

L
l~

[
[
[

[
[
[

8. other
(a) W is a colourless aliphatic liquid with a relative molecular mass of 123. W is insoluble
in water. When refluxed with aqueous sodium hydroxide, it gives a solution of X and
Y.
An acidified solution of X gives a creamy yellow precipitate with silver nitrate solution.
Y can be distilled from the solution. It gives a positive iodoform test and is oxidised
by acidified KzCr2 07 to Z. Z gives a positive iodoform test and reacts with 2,4dinitrophenylhydrazine but not with Tollens' reagent.
Identify W, X, Y and Z. Explain the chemistry of the reactions described, writing
equations where appropriate.
[6]

(b) For each of the following pairs of compounds, state the reagents and conditions that
would enable you to distinguish between them. State clearly how each compound
behaves in the test and write balanced equations for the reactions involved.

~
(i)

~CH 3

[
[

ococH3

(ii)

and

O-coocH3

and~

CHzCH3

[4]

[
[
[
[

L
[_~

L
l
L
[

[Tum over

Section D: Environmental Chemistry.

9. In hot summers, such as that of 1995, there may be significant increases in the
concentrations of tropospheric (low-level) ozone, Pollution from car exhausts is
considered to be a significant contributor to the problem.

[2}

(b) Nitrogen monoxide may be involved in the destruction of both tropospheric and
stratospheric ozone. Explain, using equations, how nitrogen monoxide is particularly
effective in this process.
[3]

(a) Give 2 examples of the adverse effects of ozone in the lower atmosphere.

(c) Both sulphur dioxide and oxides of nitrogen contribute to the formation of acid rain.

(i) Write a balanced equation for the reaction between nitrogen dioxide and water.
(ii) Write an equation forthe reaction between nitrogen dioxide and sulphur dioxide.

(iii) Explain why the reaction in (c)(ii) considerably enhances the likelihood of acid
rain being formed.
(iv) Briefly describe the use of limestone-based fluidised beds to control sulphur
dioxide emission. Give equations for the reactions involved.

[
[51

10. (a) Oxygen circulation within soils is essential if many of the ions present are to be
maintained in a form suitable for plant uptake.
(i) Use the Data Booklet to write the half-equation for the reaction responsible
for maintaining oxidation and quote its electrode p0tential under standard
conditions.
[1]
(ii) In practice, the redox potential within soils is considerably less than the
standard value. With reference to the half-equation you have quoted in (a)(i),
[2]
give two reasons why this is so.
(iii) State two consequences if the necessary ions (Fe3+, and W0 3-) are not
available in their oxidised form for uptake by the plant. Write balanced
chemical equations where applicable.
[4]

(b) The presence of aluminium ions in the soil solution enhances acidity and causes
lead ions to become more mobile.
(i) Using equations, explain how aluminium ions enhances acidity.

[2]

L
L
f'

[
[

(ii) Briefly explain why increased acidity causes lead ions to become more
mobile.
[11

[Turn over

10

L
r
I

l.
l.

Candidate Name:._ _ _ _ _ _ __

Class: 04_ __

millennia
institute

[
[

Preliminary Examination 2006 (Answer Scheme)


Pre-university 3

CHEMiSTRY
PAPER 3

9251/3

[
Monday

11 Sept 2006

2 hours 45 minutes

Additional materials:
Data Booklet
Answer Paper
Graph Paper

READ THESE INSTRUCTIONS FIRST


Write your name, index number and class in the spaces at the top of this page and on all

the work you hand in.


Write in dark blue or black pen on both sides of the paper.

You may use a pencil for any diagrams, graphs, or rough working.
Do not use staples, paper clips, highlighters, glue or correction fluid.

[
[

Answer all questions in Sections A, B and C. The last question in each section is of the
form either/or. In Section D, you are required to answer both questions on one option
..

~.~.~

topic.
At the end of the examination, fasten all yol.Jr work securely together.

['

L
L
l
L
~

The number of marks is given in brackets [ ] at the end of each question or part question.

A Data Booklet is provided.


You are reminded of the need for good English and clear presentation in your answers.

This question paper consists of 10 printed pages.


[Turn over

[
[

Section A: Physical Chemistry

1. The equation p V nRT may be used in the determination of the relative molecular
mass of a volatile liquid. The diagram shows the apparatus that could be used in this
determination. A known mass of a volatile liquid is injected through the self-sealing
rubber cap into the gas syringe, which is then heated in a water bath for several
minutes before the volume of the vapour is noted.

l.
!"

L
[

c
Selfsealing
rubber Clip

(a) Suggest why it is essential to leave the syringe in the boiling water bath for several
minutes before reading the volume of the vapour.
[1]

This is to ensure that all the volatile liquid has vapourised.

{b) State three of the basic assumptions made in the kinetic theory of gases regarding
[3]
the behaviour of g~seous molecules.
Collisions between the molecules or between a molecule and the wall of the
container are perfectly elastic. During collision, energy is exchanged but the
total kinetic energy is unchanged.
The average kinetic energy of the molecules is independent of the nature of
the gas and is directly proportional to the absolute temperature.
The molecules in a gas are widely spaced out. The gas molecules have
negligible volume compared with the volume of the whole gas i.e. volume of
the container.

_(c) Using apparatus similar to that shown in the diagram, 0.167 g of ethanol, C2H5 0H
-was injected into a gas syringe and the-syringe was then placed in-a boiling water
bath for several minutes at 1 atm. The temperature of the water bath was 100C.

[
[

[
[
[

,-

[Turn over

L
l
l
L
r

L"

(i) Calculate the theoretical volume, in cm 3 , of ethanol vapour that would have been
(2]
produced under these conditions. Assume R 8.31 J K-1 mor1

pV

=nRT

(1.01 X 105) V

6
= 2x12.0+5.0+16.0
O.l ?
+ 1.0

8.31

(273+100)

V= 111.4 cm 3

(ii) The voiume of vapour coiiected in the gas syringe was 70 cm 3 . Comment on the
result.
[2]

The vapour collected in the gas syringe is non-ideal as the volume


collected is smaller than the theoretical volume calculated in {i).

Volume collected is smaller than expected because of the presence of


hydrogen bonding/intermolecular forces between ethanol molecules.

(iii) Under what conditions of temperature and pressure would you expect the
behaviour of ethanol to be most like that
ideal gas? Explain your answers.
[2]

of

When the temperature is high, molecules have larger kinetic energy and
move furth~r apart i.e. volume of molecules negligible, intermolecular
forces less significant.
When pressure is low, volume of gas molecules is small and almost
negligible compared to volume occupied by gas. Molecules are further
apart i.e. intermolecular forces less significant.

[
[

[
[

~.;i)<::

l~

[
r -

l
L
l

[Turn over

L
f

2. This question is about the following reaction:


H catalyst

Brz(aq) + HC02H(aq)

r
r

28((aq) + 2H+(aq) + C02(g)

-7

[Brz]/ mol dm-3


0.0100
0.0090
0.0081
0.0073
0.0066
0.0053
0.0044
0.0028
0.0020
0.0013
0.0007

Time/ min
0.0
0.5
1.0
1.5
2.0
3.0
4.0
6.0
8.0
10.0
12.0

[
[~

(a) These figures were obtained using a colorimeter to measure [Br2]. Give another way
by which the rate of reaction could have been followed.
[1]
Measuring the volume of C0 2 evolved with the aid of a gas syringe.
(b) Plot a graph of [Br2] versus time.

[2]

Graph of [Br21 against time

0.011
0.01
0.009
0.008
0.007
'E' 0.006
~ 0.005
0.004
0.003
0.002
.. 0.001''

[
[

0
0

10

11

12

13

...time

1 mark for labelled axis; 1 mark for smooth curve.;


(c) Determine the order of the reaction with respect to Br2

[2]

1st order wrt Br2 ; constant T112 of 3.2 min; (+/-0.5 min is acceptable)

L
l
[

[Turn over

l
L
r-

l:
[
[

[
r

(d) Given that the order of reaction with respect to methanoic acid is
rate equation and state the units of its rate constant.
R

=k[Br2][HC0 H]; Units of k =mor dm min1

['
[

[
r-

order, write the


[2]

(e) In this experiment the [HC02 H] was kept constant. How could this be achieved in
[1]
practice?
By using excess volume of HC02 H.;
(f) The student wishes to modify the experiment by changing the concentration of
bromine. On the same axes, sketch another graph to show how the conc-entration of
bromine will change when the initial concentration of bromine is changed to 0.008
mol dm-3
[1]

1st

Graph of [Br2] against time

0.011
0.01
0.009
0.008
0.007
'N' 0.006
....
!!?.. 0.005
0.004
0.003
0.002
0.001
0
0

10

11

12 13

14

time

[
(g) Find the initial rate of the reaction for the graph that you have sketched in (f).

[1]

2 x 10~3 moldm3 min-1

L
f'

L
l
[

[Turn over

L
[

3. either
(a) 4.6 g of ethanol and 12.0 g of ethanoic acid were mixed in a flask with 20 cm 3 of 1
mol dm-3 HCI as catalyst at room temperature. The contents of the flask were left for
a week to reach equilibrium, and then titrated with 1.0 mol dm-3 of sodium hydroxide.
137 cm 3 of NaOH was required for complete neutralisation of the reaction mixture.

[1]

{i) Write an equation for the reaction of ethanol with ethanoic acid.
CH3COOH + CH3CH 20H 7 CH3COOCH 2CH3 + H20 ;

(ii) Write an expression for the equilibrium constant


ethanol and ethanoic acid.

Kc for the reaction between


[1]

= [H2 0][CH3 COOCH2 CH3 ]

[CH3 COOH][CH3CH 2 0H]

(iii) Calculate the value of Kc at room temperature.

Initial no of
moles;

CH3COOCH2CHa +

H20

'0

-X

0.1-x
= 0.017

=0.083

=0.083

CH3COOH +

CH 3CH20H

12.0 =0.2
60

4.6 = 0.1
46

Chan!=le
-X
Eqm no of 0.2-x
=0.083
moles;

>

-7

[4]

[
[

[
[

[0.083

---y----12

0.2-x = 0.117
X =0.083;

l0.083 0.017

No of moles of NaOH required for titration with the ethanoic acid remaining at equilibrium
=(137-20) X 10"3 X 1
= 0.117

Kc =

= 3.53;

---y----Uv--1

L
6

[Turn over

c
[

(b) 1.9 mol of H2 and 1.9 mol of h were allowed to reach equilibrium at 71 0 K in a 20
dm 3 vessel. The equilibrium mixture contained 3.0 mol of HI. Determine the
equilibrium constant, Kp. of the reaction mixture given that the total pressure is 11.2
~-

H2(g) + l2(g) -7 2HI(g)

H2(g) +

h(g)

-7

1.9

2HI(g)
I

[
[

Initial no of 1.9
moles
Change
-X
Eqm no of 1.9 -X
, ITl_Oies; _
=0.4
2x

=3.0

X=

1.5;

Kp

2x
2x
=3.0

-X
1.9-x
=0.4

=(3.8xll.2xl03)2
(0.4
;=56 2 .
3.8xll.2xl03)2
. 5.,

[
[

c
--,

-'

[
r~

3. other
The ingredients of a typical lemonade are:

Ingredients: Carbonated water, Citric acid, Flavouring,


Acidity regulator: sodium citrate; artificial sweetener:
aspartame, Preservative: sodium benzoate; Stabilizer:
CMC
(a) Carbonated water is an aqueous solution of carbon dioxide. Some of the dissolved
carbon dioxide reacts to form carbonic !3Cid, H 2C03.
The carbonic acid then ionises according to the following equation:
H2C03(aq) + H20(I) ~
pKa of carbonic acid

H30+(aq) + HC03- (aq)

=3. 7

L
L
L
L
L

(Turn over

r
(i) Explain with the aid of an equation why HC03- is Classified as a Lowry-Bronsted
base.
[1]
It is classified as a Lowry-Bronsted base because it accepts a proton to
form H2COJ.;
(ii) Write an expression for the acid dis::;ociation constant, Ka, for carbonic acid.
Ka

[1]

=[H30+][HC03-]

(iii) Calculate the pH of 0.1 mol dm-3 carbonic acid. State any assumptions that you
[4]
have made.

10-3.7

= (x)(x)

1 0

pH= 2.35;
(b) Let the formula of citric acid beH3A.
(i) Explain what is meant by thE:Herm buffer solution using a solution of citric acid
[2]
and sodium citrate as an example.

A buffer solution is a solutio.nwhos~ pH remains almost unchanged if a


little acid oralkali is added to it.;

H3 A

(Assumption 1: water is present in large excess. Concentration of water is


constant,)
(Assumption 2: x is so small, 0.1 - x ~ 0.1 );
1o-3.7 = (x)(x}'
(0.1)

=4.467

{0.1-x)

[
[

[H,CO'~l

PKa =3.7
Ka 1037

3W + A3- --(1)

. Na3A 7 3Na+ + A 3- --(2)


When acid is added, H+ will react with AJ- to form HA.
When base is added, OH- will react with H~ from to form AJ-.;
3

(ii) Calculate the pH of a buffer that contains 0.50 mol dm- citric acid and 0.20 mol
dm-3 sodium citrate. (Ka of citric acid= 7.08 x 10-4 mol3dm-9 )
[1]

[Turn over

L
L

L
L
L
r

[
[

[salt]
pH=pKa +log 10 - . - = 2.75
[ac1d]

(iii) For most weak acids, ionisation is an endothermic process. State how you will
expect Ka to change with an increase in concentration of the acid.
[1]
Ka will remain a constant since it is temperature dependant.;

L
[

Section B: Inorganic Chemistry

4. Iron is a d- block element. Its compounds are useful reagents in the laboratory.

(a) Using the s, p, d, f notation, write the electronic configuration of Fe and Fe3 +.

Fe: 1s2 2s2 2p6 3s 2 3p 6 4s 2 3d 6 ;


Fe3+: 1s2 2s2 2p6 3s 2 3p6 4s 0 3d 5

[2]

(b) In an acidic aqueous solution, the dicromate(VI) ion, (Cr2 o/). is a powerful oxidising
agent. A pale green solution of iron(ll) ions car be oxidised by dicromate(VI) ions to
form a yellow solution.

['

(i) By using appropriate data from the Data booklet, construct a balanced equation
to illustrate the react~m between Fe2 + and dicromate(VI) ion, (Cr20/-) and
[2]
calculate the overall E .

Fe2 +
-7 Fe3+ + e
-0.77V
Crzol- + 14H+ + 6e -7 2Cr3 + + 7Hz0 +1.33V

.6Fe2+ + Cr 0/- + 14H+-7 6 Fe3+ + 2Cr3+ + 7H 20;


Overall E = 1.33-0.77 = 0.56 V;

(ii) Calculate the volume of acidified aqueous potassium dichromate (VI) of


concentration 0.0200 mol dm-3 that will oxidise 25.00cm 3 of aqueous iron(ll)
sulphate if concentration 12. 15 g dm-3 .
[2]

No of moles of iron (II) sulphate used=

L
L
[

L
l
L

12 5
.1
x 25 00 = 2.00 x 10-3;
55:8 .f. 32.0 + 4 X 16.0 1000
.

2xlo-3 x_!_
6 =0.01667 dm 3 = 16.7 cm 3 ;
Volume of potassium dichromate required=
0.02

..

(c) Compounds of iron are also good homogeneous catalysts. What feature of transition
metals make them suitable as homogeneous catalysts? Illustrate your answer by
describing how compounds of iron can act as a homogeneous catalyst in the
following reaction.
[3

[Turn over

c
[
2r(aq) + Sz082-(aq) -7 l2(aq) + 280/-(aq)
The ability to exist in variable oxidation states enables transition metals to be
good homogeneous catalyst.;
The uncatalysed reaction is slow due to high activation energy (since 2
negative ions are involved and they repel each other);
Step I

2Fe3+ + 2r

-7

2Fe2+ +

Step II

2Fe2+ +
S20/"
21" + 520/"

-7

2Fe3+ +
2S042"
12+2SO/"

Overall

-7

E8
E8

=+0.77-0.54 > OV
=-0.77+2.01> OV

[
[
[

E 8 > OV

With the homogenous catalyst, step I and II will involve the interaction of
oppositely charged ions (Fe3+ and r in step I; Fe 2+ and 5 20/" in step II), which
attract one another strongly and hence, enhances the rate of reaction. ;
(d) Discuss the effect of cyanide ligands on the stability of Fe2+ and Fe3+ using
[1]
appropriate data from the Data booklet.

Fe3+(aq) + e
-7 Fe2+ (a<Q
E6 +0.77 V
0
[Fe (CN)6t
-7 [Fe (CN)6]E = +0.36V
0
When eN replaces H 2 0 as the ligand, the E value becomes less positive.
Hence, the +3 oxidation state of iron is stabilised relative to the +2
oxidation state by the eN-ligand. (Evidence: Low E0 value)

L
[
[

L
[

L
10

[Turn over

c
[

5. either

(a) The chlorides of the elements sodium to phosphorus all dissolve in or react with
water.

[
[
[
[

(i) State the formula of a chloride of each of these five elements.

[2]

NaCI, MgCh, AICI 3, SiCI4, PCis

(ii) Describe the reaction, if any, of the chlorides of sodium and silicon \Nith \AJater,

relating any differences to their bonding, Give equations for any reactions and
[5]
suggest the pH values of the resulting solutions.
NaCI {s) dissolves in water forming aquated ions, Na+ (aq) and cr (aq);
NaCI (s)

H20 (I}

Na+ {aq) + cr (aq)

pH ::::7;

NaCI is an ionic cpd & water is a polar solvent. The hydration energy
evolved is sufficient to compensate for the LE of NaCI.;

Na+ does not have a high charge density.


hydrolysis in water.;

SiCI4 undergo complete hydrolysis in water to give strongly acidic


solutions {pH~);

c
[
[

Hence it does not undergo

SiCI 4 {I) + 2H 2 0 (I)--'; Si02 (s) + 4W (aq) + 4Cr (aq);


SiCI4 is a covalent cpd.

SiC/;; is able to undergo hydrolysis because being in Period 3 it has


energetically accessible d-orbitals which allows H 2 0 molecules to form
dative bonds to it hence initiating the hydrolysis process.;
[max 5]

c.
[

L
L
l
L
l

11

[Turn over

t
r
(b) Sulphur and chlorine can react together to form S 2CI 2 When 1.00g of this sulphur
chloride reacted with water, 0.36g of a yellow precipitate was formed, together with a
solution containing a mixture of sulphurous acid, H2 S03 , and hydrochloric acid.
Use the above data to deduce the equation for the reaction between S2 Cb and water.
[3]
Mr(S2 CI2) 135.2

=0.00740 moles
Ami of S ppt = 0.36/32.1 =0.0112 moles
Amt of S2C/2

c
[
[

Ratio of S2CI2 : S = 0.00740 moles: 0.0112 moles ie 2:3;

2 S2C/2 + 3H20 7 3S + H~03 + 4HCI;

[
5. other
(a) State the formulae, and describe the reactions with water, of the oxides of the
elements in the third period from sodium to sulphur.
[6]
The formulae are Na 20, MgO, Al 20 3 , Si02, P40s or P401 0, 502 or 503.;

r-4

Both Na20 and MgO are alkaline. They react with water to give OH- (Na 20 is ..
more basic).
0 2-(s) + H20(1) -7 20H-(aq) ;
AI20 3 .~nd

L
-,

-~

5i02does not react with water.

The remaining oxides are all acidic, yielding W 'on dissolving in water.

l'

P401o (s) + 12NaOH (aq}-+ 4NaJP04 (aq) + 6H20 (I);


802 (g) + 2NaOH (aq)-+ Na2S03 (aq) + H20 (I) }

either one;

503 (g) + 2Na0H (aq)-+ Na2S04 (aq) + H20 (I)


[max 6]
(b) Campden tablets are composed of a mixture of sodium metabisulphite, Na2S20 5 , and
various preservatives; They are used during wine .making to sterilise equipment by
dissolving them in water and adding an organic acid, HA.
These two components react together to produce an aqueous solution of sulphur
dioxide and the sodium salt of the organic acid. Write an equation for the reaction of
sodium metabisulphite with the acid, and use it to predict the volume of sulphur
dioxide that will be produced at r.t.p from dissolving one tablet containing 0.80g of
Na2S20s.
[4]

12

[Turn over

[
['

L
L
I

L-

[
[

Na252os + 2HA ~ 2502 + 2NaA + H20 ;


No of moles of Na25 20 5 used

0 80

= 23.0 x2+2x32.0
=4.21x10"
+5x16.0

No of moles of 502 formed= 2 x 4.21x10"

;
3

Volume of 502 formed= 24 x 2 x 4.21x10-3 = 0.202 dm

L
[
[
[
[

[
[
[

L
L
L
r

13

[Turn over

r
[

Section C: Organic Chemistry

6. A young researcher tried out a new synthetic route to synthesize an alcohol.


The alcohol contains the following elements with the given percentages by mass:
C, 64.9%,
H. 13.5%
0, 21.6%
(a) The peak with the highest m/e value in the mass spectrum of the alcohol was at m/e
[2]
74. Use these data to deduce the molecular formula of the alcohol.

% composition

Ar
No of moles
Mole ratio - - - - - - -

L..-----~--

(C4H1oO)n
n=1

[
[

64.9
12.0
5.41
4

13.5
1.0
13.5
10

L.__

0
21.6
16.0
1.35
1

[
[

=74

Molecular formula of alcohol

=C4H

100;

Using 1H NMR spectroscopy, he confirmed that his synthetic pathway was successful.
The 1 H NMR spectrum of his product is shown below.
1

H NMR spectroscopy

9H

[
[
1H

10

HPM-Oo-190

1
z

ppm

[
14

[Turn over

l
L
L
[

[
[

(b) Use the spectra to identify the alcohol, and draw its displayed formulae. Explain your
reasoning.
[3]

2 peaks -7 2 types of chemically equivalent proton.;


8 at 2 ppm -Ot! proton ;
8 at 1.3 ppm = -C!h proton ;

H
I

H-C-H
I

M
II
I

H-C-C-0-H

~ I

H-C-H
I
H

(c) Predict and explain the changes in the NMR spectra (if any) of adding 0 2 0 to the
samples.
[2]
The peak at 8 at 2 ppm will disappear because proton in the -OH group is a
labile proton and it can be exchanged with 2H.;

[
[

Since 2H does not absorb energy in this region of the spectrum, the peak is not
observed.;

(d) Explain why tetrachloromethane is widely-used as a solvent in n.m.r spectroscopy.[1]

-~

There are no protons in CCI 4 Therefore it does no' give a signal in n.m.r
spectrum and thus no interference with signals of the sample.
CCI 4 is inert and volatile, solvent can be easily evaporated away to recover
sample compound.

(e) (i) Draw the structural formula of a possible isomer of the above alcohol.

[1]

H
I

H-C-H

[\)

H
I

(ii) Sketch a low resolution NMR spectra (ignore splitting pattern) for the isomer that
you have drawn in e(i).
[1]

L
l
L

H H H H
I I I I
H-C-C-C-0-H
H-C-C-C-C-0-H
I I
I
I I I I
H H H
OR C\t}H H H H
H
I

6~

(\i)

tl)

.)yt

6\1

r \H
J~.--~1
IH

3H "[Turn over

15

.'

d-rt

.?f1

IH

'

-.,.,.-__.:.__

__!_,-

C)

c
7.

(a) Suggest reagents and conditions by which you could carry out the following
conversions:
(i} CH3CH 2CH 2 0H to CH 3 CH(CH3)COOH

[6]

(i) Outline a mechanism for the reaction of hydrogen cyanide under suitable
conditions with butanone.

I I )I

H H

..

H H
I I

o-

H
I I'
H-C-C-C-C-H
I I I I
H H CNH

+H+

(b) The cyanide ion is a versatile species in organic reaction.

H H 0 H
I I llJ I
H-C-C-C-C-H

[
[

CH 3CH 2CH 2 0H + cone HzS04 (reflux}7 CH3CH=CHz;


CH3CH=CH 2 + HCI 7 CH3CHCICH 3 ;
CH3CHCICH 3 + KCN (alcoholic, reflux) 7 CH3CHCNCH 3 + KCI;
CH3 CH(CN)CH 3 + dil H+ 7 CH 3CH(CH 3 )COOH;

(ii) CH3CHzCH3 to CH3CHzCHzOH


CH3CHzCH3 + Cl 2 (UV light) 7 CH 3CHzCHzCI + HCI;
CH3CHzCHzCI + NaOH (reflux)7 CH3CH 2CHzOH + NaCI;

H OHH
I I I I
H-C-C-C-C-H
I I I I
H H CNH

eN-

L
[

"

(ii) Draw the displayed formulae of all the possible isomers of the organic
[4]
product that can be formed in the reaction.

CN

CN

C-CH3
H3CH2C', ~OH

H3C-C 'CH2CH3

HO~

r,
[

L
16

[Turn over

L
L.

['
[

8. either

(a) An organic compound H (C2 H40 3) is oxidised to I (CzHz03), which can be oxidised to
J {C2 H20 4 ). H, I and J dissolve in water to give acidic solutions which decolourise
acidified potassium manganate(VII) on warming.

On treatment with phosphorus (V) chloride, all three react, with H and J giving 2
moles of hydrogen chloride per mole, and I giving 1 mole of hydrogen chloride per
mole. I gives a precipitate with 2,4-dinitrophenylhydrazine.

[
[

[
[
-,
r
-~

Deduce the identity of H, ! and J. Exp!ain the chemistry of the reactions described,
writing equations where appropriate.
[6]
H, I and J contains -COOH group.;

H, I and J dissolve in water to give


acidic solutions
On treatment with phosphorus (V)
chloride, all three react, with H and J
giving 2 moles of hydrogen chloride per
mole, and I giving 1 mole of hydrogen
chloride per mole.
I gives a precipitate with 2,4dinitrophenylhydrazine.
H, I and J dissolve in water to give
acidic solutions which decolourise
acidified potassium manganate(VJI) on
warming.
'

H and J have 2 -OH groups while I has


one -OH group.;

I is a carbonyl compound.;
C2H204 +(0] 7 2C02 + H20;

'

-------------------

0
II

H
I

0
II

H
I

0 0
II

II

HO-C-C-OH

HO-C-C=O

HO-C-C-OH

H;

I;

J;

I
H

[Max 6]

(b) For each of the following pairs of compounds, state the reagents and conditions that
would enable _you t9 distioguish between them. State clearly how each compound
behaves in the test and write balanced equations for the reactions involved.

L
L
L

L
L
L

(i) CH3CHzCOOCH3 and CH 3CHzCONH 2


3
Add 5 cm of NaOH to 1 cm 3 of the organic compound, heat, reflux.
}
Test the gas evolved using moist litmus paper. CH3CH 2CONH 2will turn moist red
litmus paper blue.
CH3CHzCONH 2 + NaOH 7 CH3CH2CooNa+ + NH3;

17

[Turn over

n
u

[4]

(ii) CH3COCH3 and CH3CH2COCH2CH3

I,

Add aqueous alkaline 12 to equal volume of the organic compound. Heat and reflux.
CH3COCH 3will give a yellow ppt.
CH3COCH3 + 3/2 12 + NaOH -7 CH3COO-Na+ + CHI3

r
[

8. other
(a) W is a colourless aliphatic liquid with a relative molecular mass of 123. \AJ is insoluble
in water. When refluxed with aqueous sodium hydroxide, it gives a solution of X and

Y.

An acidified solution of X gives a creamy yellow precipitate with silver nitrate solution.
Y can be distilled from the solution. It gives a positive iodoform test and is oxidised
by acidified K2Cr20 7 to Z. Z gives a positive iodoform test and reacts with 2,4dinitrophenylhydrazine but not with Tollens' reagent.

Identify W, X, Y and Z. Explain the chemistry of the reactions described, writing


equations where appropriate.
[6]

X contains sr;

An acidified solution of X gives a


creamy yellow precipitate with silver
nitrate solution.
Y gives a positive ioqoform test and is
oxidised by acidified K2 Cr2 0 7 to Z.

'

Y is an alcohol that has the structure.

H
I
H3C-CI
OH

[
;

Z gives a positive iodoform test

II

H c-cZ has the structure 3


.;
Z reacts with 2,4-dinitrophenylhydrazine Z is a ketone.;

but not with Tollens reagent.


-

NaBr

H
I

H3C-C-CH3

H3C-C-CH3
I

Br

OH
y

II
H3C-C-CH3

l
[

18

[Turn over

l
l
l
l
\

r
[

[
[

L
[

(b) For each of the following pairs of compounds, state the re~gents and conditions that
would enable you to distinguish between them. State clearly how each compound
behaves in the test and write balanced equations for the reactions involved.

(i)

OCOCH3

~COOCH3

and~-

Add 5 cm 3 of NaOH to 1 cm 3 of the organic compound, heat, reflux, distill.

Add 1 cm3 of FeCh solution to the acidified distillate.

<~r-ococH3
~
will

give a purple colouration.

[
[

(ii)

~OH + CH3COOH

OCOCH3
+ NaOH +acidification-?

~--

CH3 OCH3
"'!d

[4]

[
[
[

L
[

Add equal volume of acidified KMn0 4 to the organic compound.

CH2CH3

will decolourise acidified KMnO,.

CH2CH3

COOH

+6[0]

+C0,+2H,O;

L
L
L
l
[

19

[Turn over

[
[
['

Section D: Environmental Chemistry

9. In hot summers, such as that of 1995, there may be significant increases in the
concentrations of tropospheric (low-level) ozone, Pollution from car exhausts is
considered to be a significant contributor to the problem.
(a) Give 2 examples of the adverse effects of ozone in the lower atmosphere.
Formation of photochemical smog;
Formation of peroxyacetylnitrate (PAN) which contributes to smog
Respiratory diseases.;
It's a greenhouse gas which contributes to global warming

- [2]

[
[

NO(g) + 03(g) 7 N02(g} + 02(g) --(1);


N02(g) + O(g) 7 NO(g) + 02(g) --(2};
It can be seen from these equations that because NO is regenerated in the
second reaction the nitrogen oxides can destroy many ozone molecules and
can be seen to catalyse ozone destruction.;

(i) Write a balanced equation for the reaction between nitrogen dioxide and water.
2N02 + H20 7 HN03 + HN02;
{ii) Write an equation for the reaction between nitrogen dioxide and sulphur dioxide.
. SOz(g} + N02(g) 7 S03(g) + NO(g) ;

[
[5]

By bun1ing-coalin abed offluidised limestone, a high proportion of the


sulphur dioxide that have been produced will be removed via the following
equation:
S02(g) + CaC03(s) 7 C02(g} + CaS03(s}
2CaS03(s} + 02(g) 7 2CaS04(s}

20

[
[

(iii) Explain why the reaction in (c)(ii) considerably enhances the likelihood of acid
rain being formed.
S03(g) + H20(I) 7 H2S04 (aq) (Strong acid);
(iv) Briefly describe the use of limestone-based fluidised beds to control sulphur
dioxide emission. Give equations for the reactions involved.

L
[

(c) Both sulphur dioxid~_and oxides of nitrogen contribute to the formation of acid rain.

[
[

(b) Nitrogen monoxide may be involved in the destruction of both tropospheric and
stratospheric ozone. Explain, using equations, how nitrogen monoxide is particularly
effective in this process.
[3]

}
[Turn over

~~

_o

L
L
L
[
r

[
[

r
[

[
[

[
[
[
[

10. (a) Oxygen circulation within soils is essential if many of the ions present are to be
maintained in a form suitable for plant uptake.
(i) Use the Data Booklet to write the half-equation for the reaction responsible
for maintaining oxidation and quote its electrode potential under standard
conditions.
[1]
02(g) + 4W(aq) + 4e -7 2H20(1), +1.23V;
(ii) In practice, the redox potential within soils is considerably less than the
standard value. With reference to the half-equation you have quoted in (a)(i),
give two reasons why this is so.
(2]
Decrease in acidity
Lower oxygen concentrations;
A decrease in acidity and a lowering of oxygen concentration will shift the
equilibrium of the equation in (i) to the left hand side and hence decrease
the redox potential.;

(iii) State two consequences if the necessary ions (Fe3 + and N03-) are not
available in their oxidised form for uptake by the plant. Write balanced
. (4]
chemical eq'::'ations where applicable.
Fe2+(aq) + 5 2-(aq) -7 FeS(s);
The presence of iron(ll) and sulphide ions together results in the
loss of available iron in solution due to formation of the very
insoluble iron(ll) sulphide.;
N03-(aq) + 10H+(aq} +Be -7 NH/{aq} +'3H20(1);
Ammonium ions will be absorbed less efficiently.;

c
[
[
[

l
..

'

-~

(b) The presence of aluminium ions in the soil solution enhances acidity and causes
lead ions to become more mobile.
(i) Using equations, explain how aluminium ions enhances acidity.

[2]

Due to the high charge density of the central Al 3 +, the water molecules
. ,.coordinated to it are greater polarised so that hydrolysis occurs, .
producing H30+;
[AI(H20}s]3+ (aq) + H20(1) -7 [AI(H20}s(OH)]2+ (aq) + H30+;

(ii) Briefly explain why increased acidity causes lead ions to become more
mobjle.
[1)

21

[Turn over

'l
Increased acidity causes sulphide and carbonate to be converted to
gaseous hydrogen sulphide and carbon dioxide respectively so that
solubilities of lead(ll) sulphide and lead{ll) carbonate are increased.

r
r
L
[
[

[
[
['.
[

',
'

['

[
[
[

L
L
22

[Turn over

L
.J

S-ar putea să vă placă și